Sei sulla pagina 1di 232

B

BA
1
Pengukuran
Measurement
Analisis Soalan SPM
Kertas 2011 2012 2013 2014 2015 2016 2017 2018
1 ✔ ✔ ✔ ✔ ✔ ✔ ✔ ✔

PETA Konsep 2
3


✔ ✔

.
hd
PENGUKURAN
MEASUREMENT

.B
dn
Kuantiti fizik Kuantiti skalar Penyiasatan saintifik
Physical quantity Scalar quantity Scientific investigations

iS
Tafsiran bentuk-bentuk graf
Kuantiti vektor Interpretation of the shapes of
Vector quantity
ng graphs

Analisis graf untuk


rumusan saintifik
la
Kuantiti asas Kuantiti terbitan Analysis of graphs for
Base quantity Derived quantity scientific summarisation
Pe

Penulisan laporan lengkap


• Panjang / length, l Contoh / Example: Complete report writing
• Jisim / Mass, m • Luas / Area, A = l × l
n

• Masa / Time, t • Isi padu / Volume, V = l × l × l


ita

• Arus elektrik / Electric current, I • Ketumpatan / Density, • Hubungan antara dua kuantiti
• Suhu / Temperature, T m m fizikal
ρ= = Relationship between two
V l×l×l
• Keamatan berluminositi, Iv
rb

physical quantities
Luminous intensity, Iv l
• Halaju / Velocity, v =
t • Kecerunan graf
• Kuantiti jirim, n Gradient of graph
ne

Quantity of matter, n • Pecutan / Acceleration,


v l • Luas di bawah graf
a= =
t t×t Area under the graph
m×l • Ekstrapolasi graf
Pe

• Daya / Force, F = ma =
t×t Graph extrapolation

1
  Fizik  Tingkatan 4  Bab 1 Pengukuran

Kuantiti Fizik
1.1
BAB

Physical Quantities

1. Pengukuran yang tepat merupakan satu bidang Fizik yang penting. 1 2


The correct measurement is an important Physics field.
1 kaedah
2. Pengukuran ialah untuk menentukan nilai kuantiti fizik.
Measurement is a method to determine the value of a physical quantity.

3. Kuantiti yang boleh diukur dikenali sebagai kuantiti fizik .


physical quantity .

.
Quantities that can be measured are known as

hd
4. Kuantiti fizik terbahagi kepada kuantiti asas dan kuantiti terbitan .
The physical quantity is divided into base quantity and derived quantity .

.B
Kuantiti Asas dan Kuantiti Terbitan
Base Quantities and Derived Quantities

dn
1. Kuantiti asas ialah kuantiti fizik yang tidak boleh diterbitkan daripada kuantiti fizik yang lain.
The base quantity is physics quantity which cannot be derived from other physical quantities.

iS
2 Jadual yang berikut menunjukkan tujuh kuantiti asas dan unit SI masing-masing.
The following table shows the seven base quantities and their respective SI units.
ng
Kuantiti asas Simbol Unit SI Simbol unit
Base quantity Symbol SI unit Unit symbol

(a) Panjang / Length l meter / meter m


la

(b) Jisim / Mass m kilogram / kilogram kg


Pe

(c) Masa / Time t saat / second s

(d) Arus elektrik / Electric current I ampere / ampere A


n

(e) Suhu / Temperature T kelvin / kelvin K


ita

(f) Keamatan cahaya / Luminous intensity Iv candela / candela cd

(g) Kuantiti jirim / Quantity of matter n mol / mol mol / mol


rb

3. Kuantiti terbitan ialah kuantiti fizik yang diterbitkan daripada kombinasi kuantiti asas melalui hasil
ne

darab atau hasil bahagi atau kedua-duanya sekali.


Derived quantities are physical quantities derived from the combinations of base quantities through multiplication
or division or both of these operations.
Pe

4. Contoh-contoh kuantiti terbitan dengan simbol dan unit ditunjukkan dalam jadual di bawah.
Examples of derived quantities with symbols and units are shown in the table below.

Kuantiti terbitan Simbol Unit Kuantiti terbitan Simbol Unit


Derived quantity Symbol Unit Derived quantity Symbol Unit

(a) Luas / Area A m2 (d) Momentum / Momentum p kg m s–1 / N s

(b) Isi padu / Volume V m3 (e) Daya / Force F kg m s–2 / N

(c) Laju / Speed v m s–1 (f) Tenaga / Energy E kg m2 s–2 / J

2
Fizik  Tingkatan 4  Bab 1 Pengukuran 

Memerihalkan Kuantiti Terbitan dalam Sebutan Kuantiti Asas dan Unit Asas S.I.
Describing Derived Quantities in Terms of Base Quantities and Basic S.I. Units

BAB
1. Berdasarkan pengetahuan anda dan ungkapan yang diberikan, lengkapkan jadual di bawah.
Based on your knowledge and the expression given, complete the table below.
1
Kuantiti terbitan
Sebutan dalam Sebutan dalam unit
(simbol) Rumus
Formula
kuantiti asas asas S.I.
Derived quantity
Term in base quantity Term in S.I. base units
(symbol)

Luas (A) panjang (l) × lebar (l)


l × l = l 2 m × m = m2

.
Area (A) length (l) × width (l)

hd
Isi padu (V) (a) panjang (l) × lebar (l) × tinggi (l)
l × l × l = l 3 m × m × m = m3
Volume (V) length (l) × width (l) × height (l)

.B
Ketumpatan (ρ) (b) jisim / mass (m) m m kg
= = kg m–3
Density (ρ) isi padu / volume (v) l × l × l l 3 m3

dn
Laju (v) (c) jarak / distance (l) l m = m s–1
(e)
Speed (v) masa / time (t) t s

iS
Halaju (v) (d) sesaran / displacement (l) l m = m s–1
(f)
Velocity (v) masa / time (t) t s

v–u
a=
ng
t
Pecutan (a) l l m s–1 = m s–2
u = halaju awal / initial velocity = (g)
Acceleration (a) t × t t 2 s
v = halaju akhir / final velocity
la
t = masa diambil / time taken
Pe

F = ma
Daya (F) l ml (h) kg m s–2
m = jisim / mass m× = 2
Force (F) t×t t 
newton (N)
a = pecutan / acceleration

Ip = F × t
n

Impuls (Ip) ml ml (i) kg m s–1


F = daya / force ×t=
Impulse (Ip) t 2 t (N s)
ita

t = masa / time

p = mv
Momentum (p) l ml
rb

m = jisim / mass m× = (j) kg m s–1


Momentum (p) t t
v = halaju / velocity
ne

P= F (k) kg m s–2 = kg m–1 s–2


Tekanan (P) A l 1 m
m× × = 2 m2
Pressure (P) F = daya / force t×t l×l t  l
pascal (Pa)
Pe

A = luas / area

Tenaga (E) atau W=F×s


Kerja (W) ml ml 2 (l) kg m s–2 × m = kg m2 s–2
F = daya / force ×l= 2
Energy(E) or t 2 t  joule (J)
Work (W)
s = sesaran / displacement

Q = It
Cas (Q) (m) As
I = arus elektrik / electric current I × t = It
Charges (Q) Coulomb (C)
t = masa / time

3
  Fizik  Tingkatan 4  Bab 1 Pengukuran

Kuantiti Skalar dan Kuantiti Vektor


Scalar and Vector Quantities
BAB

1. Kuantiti fizik dapat juga dikelaskan kepada kuantiti skalar dan kuantiti vektor .
scalar quantities vector quantities
1 Physical quantities can also be classified into and .

2. Kuantiti skalar ialah kuantiti fizik yang mempunyai magnitud sahaja. Contoh kuantiti skalar ialah
jisim, suhu, masa, jarak , laju dan ketumpatan .
Scalar quantities are physical quantities that only has magnitude . Examples of scalar quantities are mass,
distance speed density

.
temperature, time, , and .

hd
3. Contoh aktiviti harian yang melibatkan kuantiti skalar ialah:
Examples of daily activities involving scalar quantities are:
(a) Menyejukkan ais daripada 0 °C sehingga –5 °C

.B
Cooling ice from 0 °C to –5 °C
(b) Menentukan masa yang diambil untuk perlumbaan 100 m
Determining the time taken for 100 m race

dn
4. Kuantiti vektor ialah kuantiti fizik yang mempunyai magnitud dan arah . Contoh
kuantiti vektor ialah sesaran, halaju, pecutan, momentum dan daya.

iS
Vector quantities are physical quantities that possess both magnitude and direction . Examples of
vector quantities are displacement, velocity, acceleration, momentum and force.
5. Contoh aktiviti harian yang melibatkan kuantiti vektor ialah:
Examples of daily activities involving vector quantities are:
ng
(a) Memandu 300 km ke arah Utara (c) Menerangkan arah sekolah kamu
Driving 300 km towards North Describing direction of your school
la
(b) Melambung bola ke atas dengan halaju 10 m s–1
Throwing a ball upwards with velocity of 10 m s–1
Pe

Tugasan 1
Kelaskan kuantiti fizik yang berikut kepada kuantiti vektor dan kuantiti skalar.
n

Classify the following physical quantities into vector and scalar quantities.
ita

Jarak Masa Laju Isi padu Sesaran Halaju Pecutan


Distance Time Speed Volume Displacement Velocity Acceleration
rb

Momentum Daya Jisim Suhu Luas Ketumpatan Berat


Momentum Force Mass Temperature Area Density Weight
ne

Kuantiti skalar / Scalar quantity Kuantiti vektor / Vector quantity


Jarak / Distance Sesaran / Displacement
Pe

Masa / Time Halaju / Velocity


Laju / Speed Pecutan / Acceleration
Isi padu / Volume Daya / Force
Jisim / Mass Momentum / Momentum
Suhu / Temperature Berat / Weight
Luas / Area
Ketumpatan / Density

4
Fizik  Tingkatan 4  Bab 1 Pengukuran 

Penyiasatan Saintifik
1.2

BAB
Scientific Investigation

Tafsiran Bentuk-bentuk Graf


Interpretation of Shapes of Graph
1
1. Graf ialah satu cara untuk menunjukkan hubungan antara dua pemboleh ubah. Dalam suatu eksperimen,
graf diplotkan untuk melihat bagaimana pemboleh ubah bergerak balas berubah dengan pemboleh ubah
dimanipulasi.
Graph is a tool used to represent the relationship between two variables. In an experiment, graphs are plotted to see how a
responding variable changes with a manipulated variable.

.
hd
Graf / Graph Penerangan / Explanation
(a) y
y berkadar terus dengan x.

.B
y is directly proportional to x.
a
a
Kecerunan / Gradient, m =

dn
b b
x Persamaan graf / Equation of graph, y = mx
0

iS
(b) y
y bertambah secara linear dengan x.
y increases linearly with x.
a
ng
Kecerunan / Gradient, m = a
c b b
a
Persamaan graf / Equation of graph, y = mx + c atau / or y = x + c
la
0 x b
di mana c ialah pintasan-y. / where c is the y-intercept.
Pe

(c) y berkurang secara linear


y dengan x.
c y decreases linearly with x.

Kecerunan / Gradient, m = – a


n

a
b
b
ita

Persamaan graf / Equation of graph, y = –mx + c atau / or y = – a x + c


0 x b
di mana kecerunan adalah negatif. / where the gradient is negative.
rb

(d) y

y berkadar songsang dengan x (atau berkadar terus dengan 1 ).


ne

x
inversely proportional directly proportional 1
y is to x (or to ).
x

( )
Pe

Persamaan graf / Equation of graph, y = m 1


x
0 x

(e) y

y bertambah secara tidak linear dengan x.


y increases non-linearly with x.

Persamaan graf / Equation of graph, y = mx2


0 x

5
  Fizik  Tingkatan 4  Bab 1 Pengukuran

(f) y
BAB

y berkurang secara tidak linear dengan x.


decreases non-linearly
1 y with x.

0 x

Menganalisis Graf untuk Mendapatkan Rumusan Siasatan


Analysing Graphs to Summarise an Investigation

.
hd
1. Lima perkara yang dilakukan dalam menganalisis graf ialah:
Five steps to be carried out in analysing graphs are:
• menyatakan hubungan antara dua pemboleh ubah yang diberi

.B
state the relationship between two variables given
• menentukan kuantiti fizik yang diwakili oleh kecerunan graf
determine the physical quantity that represented by the gradient of graph

dn
• menentukan luas di bawah graf yang mewakili suatu kuantiti fizik
determine the area under the graph that represents a physical quantity
• menentukan nilai kuantiti fizik secara interpolasi

iS
determine the value of physical quantity from interpolation
• membuat ramalan melalui ekstrapolasi
make prediction through extrapolation ng
2. Rajah 1 menunjukkan keputusan dalam bentuk graf yang diperolehi dalam satu eksperimen untuk mengkaji
hubungan antara isi padu dengan suhu suatu gas yang tertentu. Isi padu gas diukur apabila suhunya diubah
daripada 50°C hingga 150°C.
la
Diagram 1 shows the results in the form of graph obtained in an experiment that studies the relationship between the volume
and the temperature of a particular gas. Gas volume is measured when the temperature is changed from 50°C to 150°C.
Pe

V (cm3)

80
n
ita

60

40
rb

20
ne

T (°C)
–250 –200 –150 –100 –50 50 100 150
Pe


Rajah 1 / Diagram 1

(a) Daripada graf di atas, kita boleh nyatakan bahawa isi padu gas bertambah secara linear dengan suhu.
From the graph above, we can state the volume of the gas increases linearly to the temperature.

(b) Daripada graf di atas, kita boleh menentukan nilai isi padu gas pada suhu tertentu dengan melukiskan
garisan sepadan pada graf tersebut.
From the graph above, we can determine the value of gas volume at certain temperatures by drawing a corresponding
line on the graph.

6
Fizik  Tingkatan 4  Bab 1 Pengukuran 

Contoh 1

BAB
Berapakah nilai isi padu gas apabila suhunya pada 25°C dan 115°C?
What are the values of gas volume when the temperature is at 25°C and 115°C?

V (cm3) 1

80

.
60

hd
40

.B
20

dn
T (°C)
–250 –200 –150 –100 –50 50 100 150

iS
Berdasarkan graf di atas,
Based on the graph above,
46 cm3
Apabila suhu ialah 25°C, isi padu ialah
ng .
When the temperature is 25°C, the volume is 46 cm3 .
60 cm 3
la
Apabila suhu ialah 115°C, isi padu ialah .
When the temperature is 115°C, the volume is 60 cm 3
.
Pe

Daripada graf di atas, kita boleh menentukan nilai isi padu gas pada suhu tertentu dengan melukis garis
ekstrapolasi pada graf.
From the graph above, we can determine the value of gas at certain temperatures by drawing an extrapolation line
on the graph.
n


ita

Contoh 2
Berapakah suhu gas apabila isi padunya menjadi sifar?
rb

What is the temperature of the gas when its volume is zero?

Apabila isi padu ialah 0 cm3, suhu ialah –270°C .


ne

When the volume is 0 cm3, the temperature is –270°C .



Pe

v (m s–1)
3. Graf di bawah menunjukkan bagaimana halaju satu objek, v berubah
dengan masa gerakannya, t.
The graph below shows how the velocity of an object, v varies with the time of 30
its motion, t.

(a) Daripada graf halaju melawan masa, kita boleh menentukan


pecutan suatu objek dengan mendapatkan kecerunan graf.
From the graph of velocity against time, we can determine the acceleration
of an object by obtaining gradient of the graph.

0 t (s)
5 10 15
Rajah 2 / Diagram 2

7
  Fizik  Tingkatan 4  Bab 1 Pengukuran

Contoh 3
BAB

Apakah pecutan objek untuk masa berikut?


What is the acceleration of the object for the following time?

1 (i) 0 s ke 5 s / 0 s to 5 s
Pecutan pada masa 0 s ke 5 s = kecerunan graf = 30 = 6 m s–2
Acceleration at 0 s to 5 s = gradient of graph 5

(ii) 5 s ke 10 s / 5 s to 10 s
Pecutan pada masa 5 s ke 10 s = kecerunan graf = 0 = 0 m s–2
5

.
Acceleration at 5 s to 10 s = gradient of graph

hd
(iii) 10 s ke 15 s / 10 s to 15 s
Pecutan pada masa 10 s ke 15 s = kecerunan graf = – 30 = – 6 m s–2
5

.B
Acceleration at 10 s to 15 s = gradient of graph

(b) Daripada graf halaju melawan masa di atas, kita boleh menentukan sesaran objek dengan mendapatkan

dn
nilai luas di bawah graf.
From the graph of velocity against time, we can determine the displacement of an object by obtaining the area below a
graph.

iS
Contoh 4
Apakah sesaran objek untuk masa berikut?
ng
What is the displacement of the object for the following time?
(i) 0 s ke 5 s / 0 s to 5 s
1
Sesaran pada masa 0 s ke 5 s = luas di bawah graf = × 5 × 30 = 75 m
la
2
The displacement at the time of 0 s to 5 s = area below the graph

(ii) 5 s ke 10 s / 5 s to 10 s
Pe

Sesaran pada masa 5 s ke 10 s = luas di bawah graf = 5 × 30 = 150 m


The displacement at the time of 5 s to 10 s = area below the graph

(iii) 10 s ke 15 s / 10 s to 15 s
1
n

Sesaran pada masa 10 s ke 15 s = luas di bawah graf = × 5 × 30 = 75 m


2

ita

The displacement at the time of 10 s to 15 s = area below the graph



4. Graf di bawah menunjukkan bagaimana laju satu objek, v berubah dengan masa gerakannya, t.
rb

The graph below shows how the speed of an object, v varies with the time of its motion, t.
ne

v (m s–1)

35
Pe

30
25
20
15
10
5
t (s)
0 0.5 1.0 1.5 2.0 2.5

Rajah 3 / Diagram 3

8
Fizik  Tingkatan 4  Bab 1 Pengukuran 

(a) Daripada graf di atas, nyatakan hubungan antara halaju dan masa pergerakan objek tersebut.
From the graph above, state the relationship between velocity and a motion of the object.

BAB
Halaju objek berkadar terus dengan masa yang diambil.
Velocity of object is directly proportional to time taken.
1
(b) (i) Kirakan kecerunan graf dan nyatakan kuantiti fizik yang diwakili oleh kecerunan.
Calculate the gradient of the graph and state the physical quantity represented by the gradient.

Kecerunan graf / Gradient of the graph = 20 = 10 m s–2


2
Kecerunan graf / Gradient of the graph = pecutan objek / acceleration of object

.
hd
(ii) Tuliskan persamaan untuk mengaitkan v dan t.
Write an equation to relate v and t.
Persamaan untuk mengaitkan v dan t / Equation to ralate v and t = v = 10t

.B
(iii) Berapakah nilai v apabila t = 2.5 s?
What is the value of v when t = 2.5 s?

dn
Nilai v apabila t / The value of v when t = 2.5 s
Daripada graf, v / From the graph, v = 25 m s
–1

iS
Penyiasatan Saintifik dan Laporan Eksperimen Lengkap
Scientific Investigation and Complete Report Experiment ng
1. Suatu penyiasatan saintifik bermula dengan memerhati sesuatu situasi.
A scientific investigation begins by observing a situation.
la
2. Berdasarkan pemerhatian situasi, suatu inferens dibuat dan beberapa soalan dikemukakan. Salah satu daripada
soalan-soalan itu akan membawa kepada proses suatu penyiasatan saintifik.
Based on the observation of the situation, an inference is made and a few questions are suggested. One of those questions
Pe

will lead to the process of a scientific investigation.

3. Selepas membuat inferens, suatu eksperimen perlu dijalankan setelah perancangan eksperimen yang sesuai
telah diputuskan.
n

After making the inference, an experiment needs to be carried out after the proper planning of the experiment has been
decided.
ita

4. Langkah-langkah yang terlibat dalam merancang dan menjalankan suatu eksperimen ialah:
The steps involved in designing and carrying out an experiment are:
rb

(a) Nyatakan tujuan eksperimen .


State the aim of experiment .
ne

(b) Senaraikan pemboleh ubah dimanipulasikan , pemboleh ubah bergerak balas dan pemboleh ubah
dimalarkan
Pe

.
List the manipulated variables, responding variables and constant / fixed variables.

(c) Senaraikan radas dan bahan .


List the apparatus and materials .

(d) Lukis susunan radas.


Draw the arrangement of the apparatus.

9
  Fizik  Tingkatan 4  Bab 1 Pengukuran

(e) Tulis prosedur yang digunakan dalam eksperimen termasuk kaedah mengawal dan mengukur
BAB

pemboleh ubah.
Write the procedure used in the experiment including the method to control and measure the variables.

1 (f) Kumpul dan jadualkan data.


Collect and tabulate the data.

(g) Analisis data (menggunakan graf).


Analyse the data (using graphs).

.
hd
(h) Buat kesimpulan .
Draw a conclusion .

.B
5. Contoh penyiasatan saintifik dan penulisan laporan penyiasatan adalah seperti yang berikut. 4
Example of a scientific investigation and the writing of a report of investigation is as follows.

dn
Hubungan antara Tempoh Ayunan dan Panjang Bandul
Eksperimen 1.1 Relationship between Period of Oscillation and Length of Pendulum

iS
Pernyataan masalah / Problem statement:
Dua orang kanak-kanak sedang bermain buaian yang berbeza panjang di taman permainan. Didapati bahawa
ng
tempoh ayunan bagi setiap buaian itu adalah berbeza.
Two children are playing on swings with different lengths at a playground. It is found that the period of oscillation for each swing
is different.
la
Pe

Tali pendek
Short string
Tali panjang
Long string
n
ita
rb

Rajah 4 / Diagram 4
ne

Bagaimanakah panjang suatu bandul mempengaruhi tempoh ayunan bandul itu?


How does the length of a pendulum affect its period of oscillation?
Pe

Inferens / Inference:
Tempoh ayunan suatu bandul bergantung pada panjang bandul itu.
The period of oscillation of a pendulum depends on the length of the pendulum.

Hipotesis / Hypothesis:
Semakin bertambah panjang bandul, semakin bertambah tempoh ayunan.
As the length of a pendulum increases, the period of the oscillation increases.

10
Fizik  Tingkatan 4  Bab 1 Pengukuran 

(a) Tujuan / Aim:

BAB
Mengkaji bagaimana panjang suatu bandul mempengaruhi tempoh ayunan bandul itu .

To investigate how the length of a pendulum affects its period of oscillation .

(b) Pemboleh ubah / Variables: 1


(i) Pemboleh ubah dimanipulasi: Panjang bandul

Manipulated variable: Length of pendulum

(ii) Pemboleh ubah bergerak balas: Tempoh ayunan

.
hd
Responding variable: Period of oscillation

(iii) Pemboleh ubah dimalarkan: Jisim ladung

.B
Constant variable: Mass of pendulum bob

(c) Radas dan bahan / Apparatus and materials:

dn
Kaki retort dengan pengapit, jam randik, ladung, pembaris meter dan benang dengan panjang 100 cm – 120 cm.
Retort stand with clamp, stopwatch, pendulum bob, meter rule and thread with length of 100 cm – 120 cm.

iS
(d) Susunan radas / Arrangement of apparatus:

Pengapit
ng Clamp

Kaki retort Benang Panjang bandul


la
Retort stand Thread Length of pendulum

Ladung
Pe

Bob

Rajah 5 / Diagram 5
n

(e) Prosedur / Procedure:


1. Ladung diikat pada benang dan diapit pada kaki retort seperti yang ditunjukkan dalam rajah di atas.
ita

A bob is tied to a thread and clamped to a retort stand as shown in the above diagram.
2. Benang itu diselaraskan dengan panjang 40.0 cm.
rb

The thread is adjusted to a length of 40.0 cm.


3. Bandul diayun pada sudut yang kecil.
The pendulum is oscillated at a small angle.
ne

4. Masa untuk 10 ayunan lengkap diambil. Bacaan dicatatkan sebagai t1 dalam jadual yang disediakan.
Time for 10 complete oscillations is taken. The reading is recorded as t1 in the table provided.
Pe

5. Langkah 3 dan 4 diulang dan masa dicatatkan sebagai t2. Purata untuk dua bacaan dihitung dan tempoh
ayunan, T ditentukan.
Steps 3 and 4 are repeated and the time is recorded as t2. The average of the two readings is calculated and the period of
oscillation, T is determined.
6. Langkah 3 hingga 5 diulang untuk panjang, l = 50.0 cm, 60.0 cm, 70.0 cm dan 80.0 cm.
Steps 3 to 5 are repeated for lengths, l = 50.0 cm, 60.0 cm, 70.0 cm and 80.0 cm.
7. Keputusan direkodkan dalam jadual yang berikut dan graf bagi tempoh, T melawan panjang, l diplot.
The results are recorded in the following table and a graph of period, T against length, l is plotted.

11
  Fizik  Tingkatan 4  Bab 1 Pengukuran

(f) Penjadualan data / Tabulation of data:


BAB

Masa diambil untuk 10 ayunan


Panjang bandul, Time taken for 10 oscillations Tempoh,
Length of pendulum Period,
1 t
l/cm t1 + t2 T = 10 (s)
t1 (s) t2 (s) t= (s)
2

40.0

.
50.0

hd
60.0

.B
70.0

dn
80.0

(g) Analisis data / Analysis of data:

iS
T (s)
ng
la
0 l (cm)
Pe

(h) Kesimpulan / Conclusion:


Tempoh ayunan bertambah apabila panjang bandul bertambah .
increases increases
n

The period of the oscillation when the length of the pendulum .


ita
rb

PRAKTIS SPM 1
ne

Soalan Objektif
Pe

1. Kuantiti manakah adalah kuantiti asas? 2. Antara yang berikut, yang manakah suatu kuantiti
Which quantity is base quantity? terbitan?
2014 2018
A Pecutan C Laju Which of the following is a derived quantity?
Acceleration Speed A Masa C Daya
B Halaju D Panjang Time Force
Velocity Length B Arus D Suhu
Current Temperature

12
Fizik  Tingkatan 4  Bab 1 Pengukuran 

3. Antara yang berikut, unit SI manakah yang salah bagi


setiap kuantiti?

BAB
2015
Which of the following SI unit is wrong for each quantity?

Kuantiti Unit SI
Quantity SI unit 1
A Jisim Kilogram (kg)
Mass Kilogram (kg) Tali pendek
Short string
B Masa Saat (s) Tali panjang
Time Second (s) Long string

Rajah 2 / Diagram 2

.
C Panjang Meter (m)

hd
Length Meter (m) Pemboleh ubah manakah yang betul?
D Suhu Celsius (°C) Which variables are correct?
Temperature Celcius (°C)
Pemboleh Pemboleh Pemboleh

.B
ubah ubah bergerak ubah
4. Kuantiti manakah adalah kuantiti vektor?
dimanipulasi balas dimalarkan
Which quantity is a vector quantity?
2015 Manipulated Responding Constant

dn
A Panjang C Momentum variable variable variable
Length Momentum
B Kerja D Jarak A Jisim budak Tempoh ayunan Panjang tali
lelaki Period of oscillation Length of the

iS
Work Distance
Mass of the boys string
5. Antara yang berikut, yang manakah suatu kuantiti B Tempoh ayunan Panjang tali Jisim budak
skalar?
2012
Which of the following is a scalar quantity?
ng Period of Length of the string lelaki
oscillation Mass of the boys
A Momentum C Arus
Momentum Current C Panjang tali Jisim budak lelaki Tempoh
la
B Halaju D Daya Length of the Mass of the boys ayunan
string Period of
Velocity Force
Pe

oscillation
6. Rajah 1 menunjukkan satu kaedah mengukur D Panjang tali Tempoh ayunan Jisim budak
diameter sebiji bola pingpong. Length of the Period of oscillation lelaki
2013
Diagram 1 shows one method of measuring the diameter of a string Mass of the boys
n

ping-pong ball.
ita

8. Sebab utama suatu graf diplotkan dalam laporan


eksperimen adalah KBAT Menganalisis
The main reason for plotting a graph in the report of an
rb

experiment is
A untuk memplotkan titik-titik yang diperlukan.
Pembaris
to plot the necessary points.
ne

Ruler
4 5 6 cm
B untuk melukis satu garis yang menyambungkan
Rajah 1 / Diagram 1 semua titik.
to draw a connecting line to all the points.
Pe

Berapakah jejari bola pingpong itu?


What is the radius of the ping-pong ball? C untuk menunjukkan hubungan antara pemboleh
ubah dimanipulasikan dengan pemboleh ubah
A 0.30 cm C 0.60 cm
bergerak balas.
B 0.45 cm D 0.90 cm
to show the relationship between the manipulated and
7. Rajah 2 menunjukkan satu kajian tentang panjang responding variables.
tali pada buaian. Jisim budak-budak lelaki itu adalah D untuk menunjukkan hubungan antara pemboleh
sama untuk kedua-dua buaian. KBAT Mengaplikasi ubah dimanipulasikan dengan pemboleh ubah
Diagram 2 shows an investigation on the length of the string of dimalarkan.
a swing. The mass of the boys is the same for both the swings. to show the relationship between manipulated and
constant variables.

13
  Fizik  Tingkatan 4  Bab 1 Pengukuran

9. Pasangan kuantiti manakah adalah benar? 12. Antara berikut, graf yang manakah mematuhi
Which pair of quantities is correct? persamaan PV = k, di mana k ialah satu pemalar?
BAB

2016 2017
Which of the following graphs obeys the equation PV = k,
Kuantiti skalar Kuantiti vektor where k is a constant?
Scalar quantity Vector quantity A P
1
A Jarak, masa Sesaran, daya
Distance, time Displacement, force

B Halaju, momentum Pressure, voltage


1
Velocity, momentum Tekanan, voltan 0 V

.
hd
C Halaju, daya Momentum, voltage B P
Velocity, force Momentum, voltan

.B
Momentum, daya Impulse, pressure
Momentum, force lmpuls, tekanan

0 V

dn
10. Antara berikut, yang manakah perlu dilakukan apabila
memplot suatu graf? KBAT Mengaplikasi
Which of the following should be done when plotting a graph? C P

iS
A Label semua paksi dengan simbol dan unit.
Label all axes with symbols and units.
B Semua pemboleh ubah mesti ditunjukkan pada ng
paksi.
All the variables must be represented in the axis. 0 V

C Garisan mesti melalui semua titik.


D
la
The line must pass through all the points. P
D Semua paksi mesti bermula dari nilai satu.
Pe

All the axes must start from one.

11. Antara berikut, yang manakah merupakan salah satu


langkah dalam penyiasatan saintifik? 0 V
Which of the following is one of the steps in a scientific
n

investigation?
13. Joule (J) ialah unit bagi tenaga. Unit lain bagi tenaga
ita

A Mencari radas
ialah kg m2 s–2.
Searching apparatus 2017
Pernyataan manakah yang betul?
B Membentuk satu prinsip fizik Joule (J) is a unit for energy. Another unit for energy is kg m2 s–2.
rb

Forming a physical principle Which statement is correct?


C Membersihkan alat A 1000 J = 1 kg m2 s–2
ne

Cleaning equipment
B 100 J =1 kg m2 s–2
D Membuat hipotesis
C 10 J =1 kg m2 s–2
Forming a hypothesis
D 1 J = 1 kg m2 s–2
Pe

14
Fizik  Tingkatan 4  Bab 1 Pengukuran 

Soalan Struktur

BAB
Bahagian A

1. Rajah 1.1 menunjukkan spring berbeban sedang berayun.


Diagram 1.1 shows a loaded spring oscillating. C
1
(a) Berdasarkan Rajah 1.1, tulis lintasan yang betul untuk satu ayunan lengkap.
Based on Diagram 1.1, write the correct path for one complete oscillation. B

B→A→B→C→B A
[1 markah / mark ]

.
Rajah 1.1 / Diagram 1.1

hd
(b) Jam randik dalam Rajah 1.2 menunjukkan masa yang diambil untuk spring berbeban itu
membuat 10 ayunan lengkap.
The stopwatch in Diagram 1.2 shows the time taken for the loaded spring to make 10 complete oscillations.

.B
Berdasarkan Rajah 1.2, nyatakan
60
Based on Diagram 1.2, state
(i) senggatan terkecil bagi jam randik itu.

dn
45 15
the smallest division of the stopwatch.
30
1 s

iS
[1 markah / mark] Rajah 1.2 / Diagram 1.2

(ii) masa yang diambil untuk 10 ayunan lengkap.


the time taken for 10 complete oscillations.
ng
56 s
[1 markah / mark]
la
(c) Apakah tempoh ayunan (masa yang diambil untuk satu ayunan lengkap) bagi spring berbeban itu?
Pe

What is the period of oscillation (time taken for 1 complete oscillation) for the loaded spring?

5.6 s
[1 markah / mark]
n

2. Rajah 2 menunjukkan skala pada sebuah voltmeter yang digunakan untuk


ita

mengukur beza keupayaan. 0.5


Diagram 2 shows the scale of a voltmeter that is used to measure potential difference.
0 1.0
(a) Namakan kuantiti fizik yang diukur dengan voltmeter.
rb

Name the physical quantity measured by the voltmeter. V


Cermin
Mirror
Beza keupayaan / Potential difference
ne

[1 markah / mark] Rajah 2 / Diagram 2

(b) Nyatakan bacaan voltmeter dalam Rajah 2.


Pe

State the reading of the voltmeter in Diagram 2.

0.80 V
[1 markah / mark]
(c) Nyatakan jenis kuantiti fizik yang diukur oleh voltmeter.
State the type of physical quantity measured by the voltmeter.

Kuantiti terbitan / Derived quantity


[1 markah / mark]

15
  Fizik  Tingkatan 4  Bab 1 Pengukuran

(d) Pada ruang di bawah, lukis simbol bagi voltmeter.


In the space below, draw the symbol for the voltmeter.
BAB

v
1
[1 markah / mark]

Bahagian B

.
3. Rajah 3.2 menunjukkan dua jenis voltmeter, P dan Q, yang berbeza sebelum sebarang arus disambungkan kepada

hd
kedua-dua voltmeter itu.
Diagram 3.2 shows two different types of voltmeters, P and Q before any current is connected to both voltmeters.

.B
0.5 2 3 4
1 5

dn
0 1.0 0 6
V V

iS
P Q
Rajah 3.2 / Diagram 3.2
ng
(a) Apakah bacaan senggatan terkecil untuk voltmeter P dan voltmeter Q?
What is the smallest readings for voltmeter P and voltmeter Q? [2 markah / marks]
la
(b) Jika satu litar elektrik diperlukan untuk mendapatkan bacaan voltan dalam julat 0.20 V ke 0.80 V, voltmeter yang
manakah lebih sesuai digunakan? Nyatakan satu sebab bagi jawapan anda.
Pe

If an electrical circuit is required to get a reading voltage in the range of 0.20 V to 0.80 V, the voltmeter is more suitable to be used?
State one reason for your answer. [3 markah / marks]
(c) Rajah 3.1 menunjukkan satu litar elektrik.
Diagram 3.1 shows an electric circuit.
n
ita
rb

R
V A
ne
Pe


Rajah 3.1 / Diagram 3.1

(i) Namakan alat A dan alat V.


Name instrument A and V. [2 markah / marks]
(ii) Namakan alat R dan nyatakan fungsinya dalam litar?
Name the device R and state its function in the circuit? [2 markah / marks]

16
Fizik  Tingkatan 4  Bab 1 Pengukuran 

(d) Encik Kasim membeli sebuah peti keselamatan yang diimport dari England. Bagaimanapun, dia mendapati
bahawa dimensi peti keselamatan itu diberikan dalam inci sebagai 30 inci × 40 inci × 60 inci. Encik Kasim ingin

BAB
mengukur dimensi itu semula dalam unit sentimeter.
Mr. Kasim bought a safety box imported from England. However, he found that the dimensions of the safety box were given in inches
as 30 inches × 40 inches × 60 inches. Mr Kasim wants to measure the dimension back in centimeters.
1
Dengan menggunakan kaedah-kaedah pengukuran yang sesuai dan diberikan 1 inci bersamaan dengan 0.025 m,
cadangkan dan terangkan:
Using appropriate measurement methods and given 1 inch equivalent to 0.025 m, suggest and explain:
(i) Kaedah untuk menyatakan isi padu peti keselamatan itu dengan menggunakan dimensi yang diberikan.
The method to state the volume of the safe by using the given dimensions.

.
hd
(ii) Alat yang sesuai digunakan untuk mengukur dimensi peti keselamatan.
Appropriate tool used to measure the dimensions of the safety box. [7 markah / marks]
(e) Rajah 1 menunjukkan jejari bola ping pong diukur menggunakan pembaris meter dan dua pembaris segi tiga.

.B
Diagram 1 shows the radius of a ping pong ball measured using a metre rule and two triangle rulers.

dn
iS
ng
X Y
0 cm 1 2 3

la
Rajah 3.2 / Diagram 3.2

(i) Mengapakah kita perlu menggunakan dua pembaris segi tiga?


Pe

Why do we have to use two triangle rulers? [1 markah / mark]


(ii) Nyatakan bacaan X dan Y yang ditunjukkan dalam Rajah 1.
State the reading of X and Y shown in Diagram 1. [2 markah / marks]
n

(iii) Nyatakan jejari bola pingpong.


State the radius of the ping-pong ball. [1 markah / mark]
ita

Bahagian C
rb

4. (a) Apakah yang dimaksudkan dengan / What is meant by


(i) kuantiti skalar? / scalar quantities? [1 markah / mark]
ne

(ii) kuantiti vektor? / vector quantities? [2 markah / marks]


(b) Jadual 5.1 menunjukkan kuantiti-kuantiti fizik. Kumpulkan kuantiti-kuantiti fizik itu kepada kuantiti skalar dan
Pe

kuantiti vektor.
Table 5.1 shows physical quantities. Group the physical quantities into scalar and vector quantities.

Berat Daya Panjang Laju Momentum Halaju


Weight Force Length Speed Momentum Velocity

Pecutan Tenaga Isi padu Suhu Sesaran Ketumpatan


Acceleration Energy Volume Temperature Displacement Density
Jadual 4.1 / Table 4.1
[3 markah / marks]

17
  Fizik  Tingkatan 4  Bab 1 Pengukuran

(c) Jadual 4.2 menunjukkan ciri-ciri empat termometer P, Q, R dan S. Anda dikehendaki menyiasat ciri-ciri setiap
termometer untuk digunakan dalam suatu projek penyelidikan.
BAB

Table 4.2 shows the features of four types of thermometers P, Q, R and S. You are required to investigate characteristics of each
thermometer to be used in a research project.

1 Termometer Skala Julat suhu Cecair dalam termometer Warna cecair


Thermometer Scale Temperature range Liquid in the thermometer Liquid colour

Celsius Merkuri Lut sinar


P -10°C – 125°C Transparent
Celsius Mercury

Fahrenheit Alkohol Legap

.
Q 0°F – 500°F
Farenheit Alcohol Opaque

hd
Celsius Merkuri Legap
R -10°C – 200°C Opaque
Celsius Mercury

.B
Fahrenheit Alkohol Lut sinar
S 30°F – 214°F
Farenheit Alcohol Transparent

dn
Jadual 4.2 / Table 4.2

Ciri-ciri termometer yang digunakan dalam projek penyelidikan ini disenaraikan di bawah:
The criteria of the thermometer to be used in this research project is listed below:

iS
• Termometer mesti boleh mengukur suhu dalam julat 10°C ke 125°C.
The thermometer must be able to measure temperatures within the range of 10°C to 125°C.
• Termometer itu mesti menunjukkan perubahan suhu pada masa yang singkat.

ng
The thermometer must show temperature changes in a short time.
• Termometer itu mesti senang digunakan, terutamanya semasa mengambil pengukuran.
The thermometer must be user friendly, especially when taking measurements.
la

Terangkan kesesuaian setiap ciri bagi termometer dalam Jadual 4.2 untuk digunakan dalam projek penyelidikan
Pe

ini. Tentukan termometer yang paling sesuai digunakan.


Explain the suitability of each feature of thermometer in Table 4.2 for use in the research project. Determine the most suitable
thermometer to be used.
Berikan sebab-sebab untuk pilihan anda.
n

Give reasons for your choice. [10 markah / marks]


ita

(d) Rajah 4.1 menunjukkan bacaan suhu apabila satu


100

100

termometer direndamkan di dalam ais. Rajah 4.2 10 90


90

90

menunjukkan bacaan suhu suatu cecair dengan


rb

80

80

menggunakan termometer yang sama.


70

70

0 80
Diagram 4.1 shows the reading of temperature when a
60

60
ne

50

50

thermometer is dipped in ice. Diagram 4.2 shows the reading


40

40

of temperature of a liquid using the same thermometer.


30

30
20

20

(i) Berdasarkan Rajah 4.1 dan Rajah 4.2, dapatkan


Pe

10

10

nilai bacaan suhu yang direkod oleh kedua-dua


0

rajah tersebut.
–10

–10

Based on Diagram 4.1 and Diagram 4.2, find the


temperature reading values recorded by both diagrams. Rajah 4.1 / Diagram 4.1 Rajah 4.2 / Diagram 4.2

(ii) Kira perbezaan antara nilai suhu teori untuk ais dengan nilai suhu eksperimen untuk ais yang ditunjukkan
dalam Rajah 4.1. Berikan satu cadangan mengapa termometer pada Rajah 4.1 tidak memberi bacaan teori.
Calculate the difference in temperatures between the theoretical value of temperature and the experimental value of
temperature of ice shown in Diagram 4.1. Give one suggestion why the thermometer in Diagram 4.1 does not give a theoretical
reading. [4 markah / marks]

18
PRAKTIS SPM 1
JAWAPAN (b)

Soalan Struktur Kuantiti Skalar Kuantiti Vektor


Scalar Quantitiy Vector Quantity
Bahagian B
Panjang Sesaran
3. (a) P: 0.05 V Length Displacement
Q: 0.2 V

.
(b) Voltmeter P. Isi padu Halaju

hd
Volume Velocity
– Julat voltmeter P boleh memenuhi julat 0.20 V ke
0.80 V yang diperlukan. Tenaga Daya
Voltmeter P can meet the required range of 0.20 V to 0.80 V. Energy Force
– Voltmeter P dapat memberi bacaan dengan titik

.B
Laju Momentum
perpuluhan yang lebih banyak. Speed Momentum
Voltmeter P can give the reading to more decimal places.
(c) (i) A = ammeter, V = voltmeter Suhu Pecutan
Temperature Acceleration
(ii) R = Reostat / Rheostat

dn
Fungsi R ialah mengawal magnitud arus. Ketumpatan Berat
The function of R is to control the magnitude of the current. Density Weight
(d) (i) Diberi 1 inci / Given 1 inches
= 0.025 m, maka / so (b) – Skala suhu harus dalam darjah Celsius.

iS
The temperature scale should be in degree Celsius.
30 inci / inches = 30 × 0.025 – Menggunakan skala Celsius adalah lebih mudah
= 0.75 m dalam projek penyelidikan ini.
40 inci / inches = 40 × 0.025 Using a Celsius scale is more convenient in this research
= 1.00 m
60 inci / inches = 60 × 0.025
ng project.
– Julat suhu harus mempunyai suatu nilai minimum
= 1.50 m yang kurang daripada 10°C dan suatu nilai
Maka dimensi adalah maksimum yang lebih besar daripada 125°C.
la
0.75 m × 1.0 m × 1.5 m. The temperature range should have a minimum value
So the dimensions are which is smaller than 10°C and a maximum value which is
0.75 m × 1.0 m × 1.5 m. bigger than 125°C.
Pe

Oleh itu, isi padu tangki – Julat suhu sedemikian dapat memenuhi julat yang
Hence, volume of the tank diperlukan untuk 10°C hingga 125°C berdasarkan
= 0.75 m × 1.0 m × 1.5 m kriteria itu.
= 1.125 m3 Such a temperature range can accommodate the required
(ii) – Pembaris meter digunakan untuk mengukur range of 10°C to 125°C based on the criteria.
n

dimensi peti keselamatan. – Cecair dalam termometer adalah merkuri.


A metre rule is used to measure the dimensions of the The liquid in the thermometer is mercury.
ita

safety box. – Merkuri ialah pengalir haba yang baik dan cepat
– Pembaris meter juga cukup panjang untuk berubah apabila suhu persekitaran berubah.
Mercury is a good conductor of heat and quickly changed
mengukur ketinggian, panjang dan keluasan peti when the environmental temperature change.
rb

keselamatan. – Warna cecair di dalam termometer harus legap.


A metre rule is long enough to measure the height, the
The colour of the thermometer liquid should be opaque.
length and the area of the safety box.
– Hal ini akan menolong pemerhati mengambil
(e) (i) Untuk meningkatkan ketepatan dalam menentukan
ne

bacaan pada termometer dengan lebih mudah.


diameter bola ping pong. This will make it easier for observers to take readings on
To increase the accuracy in determining the diameter of
the thermometer.
the ping pong ball.
– Termometer R dipilih.
Pe

(ii) X = 1.1 cm Y = 2.1 cm Thermometer R is chosen.


(f) Jejari / Radius = (2.1 – 1.1) ÷ 2 – Termometer R mempunyai skala Celsius, julat suhu
= 0.5 cm daripada –10°C hingga 200°C, diisikan dengan
merkuri, dan ia adalah legap.
Bahagian C Thermometer R has a Celsius scale, a temperature range
4. (a) (i) Kuantiti skalar ialah kuantiti fizik yang mempunyai of –10°C to 200°C, is filled with mercury and is opaque.
magnitud sahaja. (d) (i) 2°C dan 85°C
Scalar quantities are physical quantities that have (ii) 2°C – 0°C = 2°C
magnitude only. Terdapat bendasing di dalam ais yang digunakan.
(ii) Kuantiti vektor ialah kuantiti fizik yang mempunyai There are impurities in the ice used.
magnitud dan arah.
Vector quantities are physical quantities that have
magnitude and direction.
B
BA
2
Daya dan Gerakan
Forces and Motion
Analisis Soalan SPM
Kertas 2011 2012 2013 2014 2015 2016 2017 2018
1 ✔ ✔ ✔ ✔ ✔ ✔ ✔ ✔

PETA Konsep 2
3
✔ ✔

✔ ✔





✔ ✔

.
hd
DAYA DAN GERAKAN I
FORCE AND MOTION I

.B
Graf / Graph

dn
Gerakan linear Momentum Impuls
Linear motion • Sesaran – masa Momentum Impulse
Displacement – time
• Halaju – masa

iS
Velocity – time Prinsip Keabadian
Keadaan gerakan: • Pecutan – masa Momentum
State of motion: Acceleration – time Principle of Conservation
• Pegun
ng of Momentum
Stationary m1u1 + m2u2 = m1v1 + m2v2
• Halaju seragam • Mentafsir jenis gerakan
la
Uniform velocity Interpret type of motion
• Halaju tak seragam • Menganalisis graf
Analyse graph Perlanggaran dan letupan
Pe

Non-uniform velocity Collision and explosion


• Menterjemah dan melakar
Interpreting and sketching
– graf sesaran – masa kepada
halaju – masa dan sebaliknya Daya / Force Daya impuls
n

• Jarak dan sesaran displacement – time graph to F = ma Impulsive force


Distance and displacement velocity – time graph and vice versa Ft = (mv – mu)
ita

• Laju dan halaju – graf halaju – masa kepada


Speed and velocity pecutan – masa dan sebaliknya
• Pecutan velocity – time graph to
rb

Acceleration acceleration – time graph and vice


versa
ne

• Gerakan jatuh bebas • Kekuatan medan Daya tindakan dan


Persamaan gerakan linear: Free fall motion graviti daya tindak balas
Pe

Linear motion equations: • Nilai pecutan graviti Gravitational Action and Reaction
Value of gravitational acceleration strength field force
• v = u + at
• Berat
• v2 = u2 + 2as Weight
1
• s = 2  (u + v)t Inersia and jisim
Inertia and mass
1
• s = ut + 2  at2
Hukum Gerakan Hukum Gerakan
Hukum Gerakan Newton Kedua Newton Ketiga
Newton Pertama Newton's Second Newton's Third Law
Newton's First Law of Motion Law of Motion of Motion

19
  Fizik  Tingkatan 4  Bab 2 Daya dan Gerakan

Gerakan Linear
2.1 Linear Motion

Pergerakan linear melibatkan pergerakan dalam garis lurus dan dikenali sebagai kinematik.
Linear motion involves motion in a straight line and is also known as kinematics.

Menghuraikan Jenis Gerakan Linear bagi Objek yang Berada dalam Keadaan Pegun, Halaju Seragam
dan Halaju Tidak Seragam
BAB

Describing the Type of Linear Motion of an Object in Stationary, Uniform Velocity and Non-uniform Velocity

garis lurus

.
2 1. Pergerakan linear melibatkan pergerakan dalam .

hd
Linear motion involves motion in a straight line .

2. Keadaan pegun ialah keadaan di mana laju adalah sifar . Hal ini bermakna objek tidak

.B
bergerak dan berada tetap pada satu titik.
The stationary state is the state where the speed is zero . This means the object is at rest and
is fixed at one point.

dn
3. Suatu objek dikatakan bergerak dengan halaju seragam jika objek mengalami perubahan sesaran
yang tetap per unit masa.

iS
An object is said to be moving at a uniform velocity if the object experiences constant displacement changes per unit
of time. ng
4. Rajah di bawah menggambarkan gerakan halaju seragam.
The diagram below illustrates constant velocity motion.
la
Pe
n


ita

5. Suatu objek dikatakan bergerak dengan halaju tidak seragam jika objek mengalami perubahan sesaran yang
berbeza per unit masa.
rb

An object is said to be non-uniform velocity if the object experiences different displacement changes per unit of
time.
ne

6. Rajah di bawah menggambarkan gerakan halaju tidak seragam.


The diagram below illustrates non-uniform velocity motion.
Pe

20
  Fizik  Tingkatan 4  Bab 2 Daya dan Gerakan 

Menentukan Jarak, Sesaran, Laju, Halaju dan Pecutan


Determining Distance, Displacement, Speed, Velocity and Acceleration

A Jarak dan Sesaran / Distance and Displacement

1. Jarak ialah jumlah panjang lintasan yang dilalui oleh suatu objek dari satu tempat ke tempat yang lain.
Jarak ialah suatu kuantiti skalar .

BAB
Distance is the total length of the path travelled by an object from one location to another. It is a scalar quantity.

2. Sesaran suatu objek ialah jarak yang diukur dalam lintasan paling pendek dari titik mula ke titik akhir

.
gerakan objek itu. Sesaran ialah suatu kuantiti vektor . 2

hd
Displacement of an object is the distance measured in the shortest path from the starting point to the ending point
of the motion of the object. It is a vector quantity.

.B
Contoh 1
Rajah 1 menunjukkan lintasan yang dilalui oleh sebuah kereta

dn
B 20 km A
dari A ke B, dari B ke C dan kemudian dari C ke D. Hitungkan
Diagram 1 shows the path travelled by a car from A to B, from B to C, then
from C to D. Calculate

iS
(a) jarak yang dilalui, s oleh kereta, 12 km

distance travelled, s, by the car, ng


(b) sesaran, s dari A ke D.
displacement, s, from A to D. C D
15 km
la
Penyelesaian / Solution: Rajah 1 / Diagram 1
(a) 20 + 12 + 15
Pe

= 47 km
(b) Sesaran, s dari A ke D / Displacement, s, from A to D
= 52 + 122 = 13 km
n
ita

3. Kedua-dua jarak dan sesaran mempunyai unit SI yang sama iaitu meter (m) .
Both distance and displacement have the same SI unit, which is metre (m) .
rb

B Laju dan Halaju / Speed and Velocity


ne

1. Laju ialah jarak yang dilalui per unit masa . Laju ialah suatu kuantiti skalar .
Speed is the distance travelled per unit time . It is a scalar quantity.
Pe

jarak (m)
Laju, v = / Speed, v = distance (m)
masa (s) time (s)

2. Halaju ialah laju suatu objek dalam arah tertentu iaitu kadar perubahan sesaran . Halaju ialah suatu
kuantiti vektor .
Velocity is the speed of an object in a specified direction, that is, the rate of change of displacement . Velocity is a
vector quantity.
sesaran (m)
Halaju, v = / Velocity, v = displacement (m)
masa (s) time (s)

21
  Fizik  Tingkatan 4  Bab 2 Daya dan Gerakan

3. Kedua-dua laju dan halaju mempunyai unit S.I. yang sama iaitu meter per saat (m s–1).
Both speed and velocity have the same S.I. unit, which is meter per second (m s–1).

Contoh 2
Rajah 2 menunjukkan lintasan yang dilalui oleh sebuah kereta dari titik A ke D. Masa yang diambil ialah
45 minit.
Figure 2 shows the path taken by a car from point A to D. The time of travel is 45 minutes.
BAB

Kira / Calculate
B 20 km A
(a) laju kereta itu, (b) halaju kereta itu,

.
2 speed of the car velocity of the car

hd
Penyelesaian / Solution:
12 km
jarak 47 km
(a) Laju, v = = = 62.67 km j–1

.B
masa 0.75 j
distance 47 km
Speed, v = = = 62.67 km h–1
time 0.75 h C D
15 km
13 km

dn
(b) Halaju, v = sesaran = = 17.33 km j–1 Rajah 2 / Diagram 2
masa 0.75 j
displacement 13 km
Velocity, v = = = 17.33 km h–1

iS
time 0.75 h

C Pecutan / Acceleration
ng
1. Pecutan ditakrifkan sebagai kadar perubahan halaju.
Acceleration is defined as the rate of change in velocity.
la
halaju halaju akhir, v – halaju awal, u
Pecutan = =
masa
Pe

masa diambil, t
velocity final velocity, v – initial velocity, u
Acceleration = =
time time taken, t
v–u

n

Maka / Therefore, a =
t
ita

2. Pecutan ialah suatu kuantiti vektor dan unit S.I. ialah meter per saat per saat (m s–2) .

Acceleration is a vector quantity and its S.I. unit is metre per second per second (m s ) –2
.
rb

Penerangan gerakan Jenis gerakan


ne

Explanation of motion Types of motion

(a) Suatu objek memecut jika halaju objek itu Pecutan seragam / Constant acceleration
bertambah
Pe

dengan masa. t=0s


increasing
An object is accelerating if its velocity is t=1s
u = 0 m s–1
with time.
t=2s
10 m s–1
t=3s
20 m s–1

v = 30 m s–1

v–u 30 – 0
a= = = 10 m s–2
t 3

22
  Fizik  Tingkatan 4  Bab 2 Daya dan Gerakan 

(b) Suatu objek yang bergerak dengan halaju seragam Halaju seragam (Pecutan = 0 m s–2)
Constant velocity (Acceleration = 0 m s–2)
adalah tidak memecut.
t=0s
An object travelling at a constant velocity is not
accelerating. u = 10 m s–1 t=1s

t=2s
10 m s–1

BAB
t=3s
10 m s–1

v = 10 m s–1

.
v–u 10 – 10 2

hd
a= = = 0 m s–2
t 3

(c) Apabila suatu objek diperlahankan , objek Pecutan seragam

.B
Constant acceleration
itu mengalami pecutan. t=0s
When an object is slowing down , the object
u = 40 m s–1 t=1s

dn
experiences acceleration.
t=2s
30 m s–1
t=3s

iS
20 m s–1

v = 10 m s–1
ng a=
v–u
=
10 – 40
= –10 m s–2
t 3
la
Analisis Gerakan / Motion Analysis
Pe

1. Sebuah jangka masa detik akan membuat 50 titik per saat pada suatu pita detik (Masa / Time = 0.02 s)
jika disambungkan pada bekalan arus ulang alik berfrekuensi 50 Hz. 1 detik / tick
A ticker timer will make 50 dots per second on a ticker tape if connected to an alternating
current (a.c.) power supply of frequency 50 Hz.
n

2. Masa di antara dua titik yang berturutan (1 detik) ialah 0.02 s .


ita

The time interval between two adjacent dots (1 tick) is 0.02 s . 2 titik / dots

3. Analisis gerakan pada pita detik. / Analysis of motion on a ticker tape.


rb

Pengiraan halaju/pecutan
Pita detik atau carta Jenis gerakan
Calculation of velocity/
Ticker tape or chart Type of motion
acceleration
ne

(a) Bergerak dengan halaju seragam yang kecil Halaju / Velocity


= 20
small constant velocity 10 × 0.02
Pe

Moving with
20 cm = 100 cm s–1 = 1 m s–1

(b) Bergerak dengan halaju seragam yang Halaju / Velocity


= 20
sederhana 6 × 0.02
20 cm medium constant velocity = 166.7 cm s–1 = 1.667 m s–1
Moving with

(c) Bergerak dengan halaju seragam yang tinggi Halaju / Velocity


= 20
Moving with high constant velocity 3 × 0.02
20 cm = 333.3 cm s–1 = 3.333 m s–1

23
  Fizik  Tingkatan 4  Bab 2 Daya dan Gerakan

(d) Pecutan atau halaju bertambah Halaju awal / Initial velocity,

Accelerating u = 2 = 100 cm s–1


or increasing velocity 0.02
= 1 m s–1
Halaju akhir / Final velocity,
v = 6 = 300 cm s–1
2 cm 6 cm 0.02
BAB

= 3 m s–1
Pecutan / Acceleration,
3–1

.
a = = 20 m s–2
2 (6 – 1) × 0.02

hd
(e) Halaju berkurang Halaju awal / Initial velocity,
u = 4.5 = 225 cm s-1

.B
Decreasing velocity 0.02
= 2.25 m s-1
Halaju akhir / Final velocity,
v = 1.5 = 75 cm s–1

dn
4.5 cm 1.5 cm 0.02
= 0.75 m s–1
Pecutan / Acceleration,

iS
a = 0.75 – 2.25 = –12.5 m s–2
ng (7 – 1) × 0.02
Panjang (cm) (f) Pecutan atau halaju bertambah Halaju awal / Initial velocity,
Length (cm)
u = 4
Accelerating 10 × 0.02
6.5 or increasing velocity
6.0 = 20 cm s –1
la
5.5 = 0.2 m s–1
5.0
Halaju akhir / Final velocity,
Pe

4.5
4.0 v = 6.5
3.5 10 × 0.02
3.0 = 32.5 cm s–1
2.5 = 0.325 m s–1
n

2.0
1.5 Pecutan / Acceleration,
ita

1.0 a = 0.325 – 0.2


0.5 5 × 0.2
0 Masa (s) = 0.125 m s–2
Time (s)
rb

Panjang (cm) (g) Halaju berkurang Halaju awal / Initial velocity,


Length (cm)
ne

u = 9
9.0
Decreasing 10 × 0.02
velocity
= 45 cm s–1
7.5 = 0.45 m s–1
Pe

6.0 Halaju akhir / Final velocity,

4.5 v = 1.5
10 × 0.02
= 7.5 cm s–1
3.0
= 0.075 m s–1
1.5 Pecutan / Acceleration,
0 Masa (s) a = 0.075 – 0.45
Time (s) 5 × 0.2
= –0.3 m s–2

24
  Fizik  Tingkatan 4  Bab 2 Daya dan Gerakan 

4. Selain daripada jangka masa detik, sistem photogate dan pemasa elektronik boleh digunakan untuk
mengkaji pergerakan linear dengan lebih jitu. Photogate adalah peranti yang mengukur masa

antara pergerakan yang dikesan oleh sinar infra-merah .


Other than ticker timer, photogate system and an electronic timer can be used to assess a more precise linear movement.
Photogate is a device that measures the time between events that are detected by an infra-red beam .

5. Rajah 3 menunjukkan sistem photogate dan pemasa elektronik yang digunakan bersama troli.

BAB
Diagram 3 shows a photogate system and an electronic timer used in with a trolley.

Pemberat Photogate pertama

.
Pemasa elektronik
2

hd
Weight First photogate Electronic timer

.B
Troli Photogate kedua
Trolley Second photogate

dn
Landasan
Plane

iS

Rajah 3 / Diagram 3
ng
6. Susunan radas di atas boleh menggunakan sistem photogate dan pemasa elektronik untuk menentukan halaju
dan pecutan pergerakan troli. Apabila troli bersama kadbod bergerak melalui photogate pertama, halaju awal
la
troli (u) akan dikira. Halaju akhir troli (v) akan dikira apabila troli bergerak melalui photogate kedua dan masa
antara halaju akhir dan halaju awal akan dicatatkan oleh pemasa elektronik.
The above apparatus can use photogate system and an electronic timer to determine the velocity and acceleration of movement
Pe

of the trolley. When the trolley with the cardboard moves through the first photogate, the initial velocity of the trolley (u) will
be calculated. The final velocity of the trolley (v) will be calculated when the trolley moves through the second photogate
and the time between the final velocity and the initial velocity will be recorded by the electronic timer.

7. Maklumat di bawah menunjukkan data yang diperolehi oleh sistem photogate dan pemasa elektronik untuk
n

troli di atas.
ita

The information below shows the data obtained by the photogate system and the electronic timer for the above trolley.

Halaju awal, u = 1.5 m s–1 dan halaju akhir, v = 3.5 m s–2. dan masa di antara halaju akhir dan halaju awal ialah
4 s.
rb

Initial velocity, u = 1.5 m s–1 and final velocity, v = 3.5 m s–2 and the time between the final velocity and the initial velocity
is 4 s.
ne

Maka pecutan / Then acceleration, a = v – u = 3.5 – 1.5 = 0.5 m s–2.


t 4

3
Pe

Contoh

Pita detik atau carta Pengiraan halaju/pecutan


Ticker tape or chart Calculation of velocity/acceleration

1. Halaju / Velocity
v = 8
6 × 0.02
8 cm = 66.67 cm s–1 = 0.667 m s–1

25
  Fizik  Tingkatan 4  Bab 2 Daya dan Gerakan

2. Halaju / Velocity
v = 16
15 × 0.02
16 cm = 53.33 cm s–1

= 0.533 m s–1

3. Halaju awal / Initial velocity,


BAB

Arah gerakan
Direction of motion
u = 2
0.02
= 100 cm s–1
= 1 m s–1

.
2

hd
Halaju akhir / Final velocity,
5 cm 2 cm
v = 5
0.02

.B
= 250 cm s–1

= 2.5 m s–1
Pecutan / Acceleration,

dn
a = 2.5 – 1
5 × 0.02
= 15 m s–2

iS
4. Panjang (cm) Halaju awal / Initial velocity,
Length (cm) 2
u =
10 × 0.02
8
ng
= 10 cm s-1
7
= 0.1 m s-1
6
la
5 Halaju akhir / Final velocity,
4 v = 8
Pe

10 × 0.02
3
= 40 cm s–1
2
= 0.4 m s–1
1
0 Masa (s) Pecutan / Acceleration,
n

Time (s)
a = 0.4 – 0.1
6 × 0.2
ita

= 0.25 m s–2
rb

5. Panjang (cm) Halaju awal / Initial velocity,


Length (cm) 20
u =
10 × 0.02
ne

20
= 100 cm s-1
= 1.0 m s-1
16
Halaju akhir / Final velocity,
Pe

12
v = 4
10 × 0.02
8 = 20 cm s-1
= 0.2 m s-1
4
Pecutan / Acceleration,
0 Masa (s) a = 0.2 – 1.0
Time (s) 4 × 0.2
= –1.0 m s-2

26
  Fizik  Tingkatan 4  Bab 2 Daya dan Gerakan 

Contoh 4
1. Sebuah kereta bergerak sejauh 30 km ke Selatan, kemudian 40 km ke Barat dan akhir sekali 60 km
ke Utara sebelum berhenti untuk rehat. Jumlah masa yang diambil oleh kereta itu untuk keseluruhan
perjalanan ialah 2 jam.
A car travels 30 km to the South, then 40 km to the West and finally 60 km to the North before stopping for rest.
The total time taken by the car for the whole journey is 2 hours.
Titik akhir

BAB
(a) Berapakah jumlah jarak yang dilalui? Ending point
What is the total distance travelled?

Jumlah jarak / Total distance = 30 + 40 + 60 = 130 km

.
2

hd
(b) Berapakah sesaran yang dilalui oleh budak lelaki itu? Titik mula
What is the displacement of the boy? 60 km Starting point

Sesaran / Displacement: 302 + 402 = 50 km

.B
30 km
(c) Berapakah laju purata kereta itu?
What is the average speed of the car?

dn
Laju purata / Average speed = 130 = 65 km j–1 / km h–1 40 km
2
(d) Berapakah halaju purata kereta itu? / What is the average velocity of the car?

iS
Halaju purata / Average velocity = 50 = 25 km j–1 / km h–1, 36.86° barat laut / north-west
2
2. Sebuah kereta bergerak dengan halaju seragam 50 m s–1 untuk 5 saat ke arah Utara dan berubah ke
Timur dengan halaju malar 40 m s–1 untuk 15 s.
ng
A car moving with a constant velocity of 50 m s–1 for 5 second towards the North and turn to the East and travel
with constant velocity of 40 m s–1 for 15 s.
la
(a) Berapakah jumlah jarak yang dilalui? / What is the total distance travelled?
Jumlah jarak ke arah Utara / Total distance toward North = s = vt = 50 × 5 = 250 m
Pe

Jumlah jarak ke arah Timur / Total distance toward East = s = vt = 40 × 15 = 600 m


Jumlah jarak / Total distance = 250 m + 600 m = 850 m
(b) Berapakah sesaran yang dilalui oleh kereta? / What is the displacement of the car?
n

Sesaran / Displacement: 6002 + 2502 = 650 m


ita

600 m Titik akhir


Ending
point
rb

250 m
ne

Titik mula
Pe

Starting point

(c) Berapakah laju purata kereta tersebut? / What is the average speed of the car?

Laju purata / Average speed = 850 = 42.5 m s–1


20
(d) Berapakah halaju purata kereta tersebut? / What are the average velocity of the car?

Halaju purata / Average velocity = 650 = 32.5 m s–1


20

27
  Fizik  Tingkatan 4  Bab 2 Daya dan Gerakan

Menyelesaikan Masalah Gerakan Linear dengan Menggunakan Persamaan


Solve Linear Motion Problems by Using Equations

1. Persamaan yang digunakan dalam gerakan linear yang melibatkan dua jenis gerakan dengan pecutan seragam
(perubahan halaju) ialah:
The equations used in linear motion which involves two kinds of motion with constant acceleration (change in velocity) are:
(u = halaju awal / initial velocity, v = halaju akhir / final velocity, t = masa / time, a = pecutan / acceleration,
s = sesaran / displacement)
BAB

Persamaan Pemboleh ubah yang terlibat Pemboleh ubah yang tidak terlibat
Equation Variables involved Variables not involved

.
2

hd
(a) v = u + at v, u, a, t s
(b) v2 = u2 + 2as v, u, a, s t

.B
1 2 s, u, t, a v
(c) s = ut + at
2
1 s, u, v, t a
(d) s = (u + v)t

dn
2
s
2. Untuk gerakan linear dengan halaju seragam (pecutan sifar), rumus ialah v = .
t

iS
s
For motion with constant velocity (zero acceleration), the formula is v =
t
.

Contoh 5
ng
1. Sebuah kereta bergerak dengan 30 m s–1 berhenti selepas melalui jarak 15 m. Apakah pecutannya?
A car travelling at 30 m s–1 stops after a distance of 15 m. What is its acceleration?
la
Penyelesaian / Solution:
v2 = u2 + 2as
Pe

0 = 302 + 2a(15)
a = − 900 = −30 m s−2
30
n

2. Suatu objek yang berada dalam keadaan rehat pada mulanya, bergerak dalam suatu garis lurus dengan
pecutan seragam dan melalui jarak sejauh 200 m dalam masa 10 s. Kira (a) pecutan (b) halaju akhir
ita

objek itu.
An object, initially at rest, moves in a straight line with constant acceleration and covers a distance of 200 m in
10 s. Calculate (a) the acceleration (b) the final velocity of the object.
rb

Penyelesaian / Solution:
(a) s = ut + 1  at2 (b) s = 1  (u + v)t
ne

2 2
200 = 0(10) + 1  a(100) 200 = 1  (0 + v)10
2 2
Pe

a = 4 m s–2 400 = 10 v
v = 40 m s–1
3. Sebuah kereta bermula dengan 10 m s–1 dan memecut dengan 8 m s–2 untuk 5 s. Apakah halaju akhirnya?
A car starts off at 10 m s–1 and accelerates at 8 m s–2 for 5 s. What is its final velocity?

Penyelesaian / Solution:
v = u + at
v = 10 + 8(5)
= 50 m s–1

28
  Fizik  Tingkatan 4  Bab 2 Daya dan Gerakan 

Tugasan 1
1. Lengkapkan jadual di bawah. / Complete the table below.

Rumus Jawapan
u (m s–1) v (m s–1) t (s) a (m s–2) s (m)
Formula Answer

BAB
2 4 1 ? 1 2 (a) 16 m
s = ut + at
2
0 4 ? 5 v2 = u2 + 2as (b) 1.6 m s–2

.
2

hd
? 4 4 16 1 (c) 4 m s–1
s= (u + v)t
2
3 ? 2 5 v = u + at (d) 13 m s–1

.B
8 12 ? 1 v = u + at (e) 4s

3 4 2 ? 1 (f) 7m

dn
s= (u + v)t
2
4 8 ? 10 v2 = u2 + 2as (g) 2.4 m s–2

iS
? 2 4 20 1 2 (h) 6 m s-1
s = ut + at
2
ng
2. Sebuah kereta yang bergerak dengan halaju 10 m s–1 berhenti selepas masa 20 s. Berapakah pecutan kereta itu?
A car travelling at 10 m s–1 stops after a time of 20 s. What is its acceleration?
Diberi / Given : u = 10 m s–1, v = 0 m s–1, t = 20 s, a = ?
la
Menggunakan rumus / Using formula , v = u + at
0 = 10 + (a)(20)
Pe

a = -0.5 m s–2

3. Sebuah kereta mula bergerak dengan 2 m s–1 dan memecut dengan 3 m s–2 selama 5 s. Berapakah halaju akhir kereta
itu?
n

A car starts off at 2 m s–1 and accelerates at 3 m s–2 for 5 s. What is its final velocity?
ita

Diberi /Given: u = 2 m s–1, a = 3 m s–2, t = 5 s, v = ?


Menggunakan rumus / Using formula , v = u + at
v = 2 + 3(5)
rb

v = 17 m s–1
ne

4. Suatu objek dalam keadaan rehat pada mulanya, bergerak dalam satu garis lurus dengan pecutan seragam dan
bergerak sejauh 250 m dalam masa 10 s. Hitung
An object, initially at rest, moves in a straight line with constant acceleration and covers a distance of 250 m in 10 s. Calculate
Pe

(a) Pecutan / Acceleration (b) Halaju akhir / Final velocity


Diberi / Given: u = 0 m s , s = 250 m, t = 10 s, a = ?
–1 Diberi / Given: u = 0 m s–1, s = 250 m, t = 10 s, v = ?
1 1
Menggunakan rumus / Using formula , s = ut + at2 Menggunakan rumus / Using formula, s = (u + v)t
2 2
1 1
250 = 0(10) + (a)(10)2 250 = (0 + v)10
2 2
a = 5 m s–2 v = 50 m s–1

29
  Fizik  Tingkatan 4  Bab 2 Daya dan Gerakan

Graf Gerakan Linear


2.2 Linear Motion Graphs

Menyelesaikan Masalah Gerakan Linear dengan Menggunakan Persamaan


Solve Linear Motion Problems by Using Equations

A Graf Sesaran-Masa / Displacement-Time


BAB

1. Graf sesaran-masa ialah graf yang menunjukkan bagaimana


A displacement-time graph is a graph showing how
(a) kedudukan suatu objek dari titik asal berubah dengan masa

.
2 the position of an object from its original position changes with time

hd
(b) sesaran suatu objek berubah dengan masa
the displacement of the object changes with time

.B
2. Kecerunan suatu graf sesaran-masa memberikan halaju .

The slope or gradient of a displacement-time graph gives the velocity .

dn
Sesaran / m
Displacement / m Daripada graf sesaran-masa, kita boleh tentukan
From the displacement-time graph, we can determine
100

iS
(a) sesaran objek pada masa
displacement of the object at the time
(i)
t = 10 s , s = 50 m (ii) t = 20 s, s = 100 m
50
ng
(b) halaju objek = kecerunan graf / object velocity = gradient of graph
= 100 = 5 m s–1
Masa / s 20
la
Time / s
10 20
Pe

B Graf Halaju-Masa / Velocity-Time Graphs


1. Graf halaju-masa ialah graf yang menunjukkan bagaimana halaju suatu objek berubah dengan masa.
The velocity-time graph is a graph showing how the velocity of an object changes with time.
n

2. Kecerunan suatu graf halaju-masa memberikan pecutan .


ita

The gradient of a velocity-time graph gives the acceleration .

3. Pada graf halaju-masa, luas di bawah graf adalah sama dengan sesaran .
rb

On a velocity-time graph, the area under the graph is equal to the displacement .
ne

Dari graf halaju-masa, kita boleh tentukan


From the velocity-time graph, we can determine
Halaju / m s–1
Pe

Velocity / m s–1
(a) halaju objek pada masa
velocity of the object at the time
20
(i)
t = 2 s , v = 10 m s–1 (ii)
t = 4 s, v = 20 m s–1
(b) pecutan objek = kecerunan graf / object acceleration = gradient of graph
10 = 20 = 5 m s–2
4
(c) sesaran objek dari t = 0 s ke t = 4 s = luas di bawah graf
displacement of the object from t = 0 s to t = 4 s = area below the graph
Masa / s
2 4
Time / s = 1 × 4 × 20 = 40 m
2

30
  Fizik  Tingkatan 4  Bab 2 Daya dan Gerakan 

C Graf Pecutan-Masa / Acceleration-Time Graphs


1. Graf pecutan-masa ialah graf yang menunjukkan bagaimana pecutan suatu objek berubah dengan masa.
An acceleration-time graph is a graph showing how the acceleration of an object changes with time.

2. Pada graf pecutan-masa, kecerunan tidak mewakili sebarang kuantiti fizik.


In the acceleration-time graph, the slope does not represent any physical quantity.

3. Objek yang mengalami pecutan seragam diwakili oleh garis mendatar.

BAB
An object undergoing constant acceleration is represented by a horizontal line.

Pecutan / m s–2 Dari graf pecutan-masa, kita boleh tentukan

.
Acceleration / m s–2
From the accleration-time graph, we can determine 2

hd
4
(a) pecutan objek pada masa
acceleration of the object at the time
3
(i) t = 5 s , a = 3 m s–2

.B
2 (ii)
t = 10 s, a = 4 m s–2
1
Masa / s (b) Pintasan pada paksi-y mewalili pecutan awal.
0

dn
5 10
Time / s Intercept on the y-axis represent the initial acceleration.
Pecutan awal / the initial acceleration = 2 m s–2

iS
Menganalisis Graf Sesaran-Masa untuk Menentukan Jarak, Sesaran dan Halaju
Analysing Time-Displacement Graphs to Determine Distance, Displacement and Velocity
ng
1. Pelbagai graf sesaran-masa ditunjukkan dalam jadual yang berikut.
The various displacement-time graphs are shown in the following table.
la
Graf sesaran-masa Kedudukan dan gerakan objek Sesaran dan halaju
Displacement-time graph Position and motion of the object Displacement and velocity
Pe

(a) s (m) Objek dalam keadaan rehat Sesaran objek adalah malar . Halaju
untuk suatu jangka masa yang objek adalah sifar.
tertentu.
n

Its displacement is constant . Its velocity is


rest
Objek dalam keadaan rehat The object is at over a zero.
ita

Object is at rest period of time.

0 t (s)
rb

(b) s (m) Objek bergerak dalam arah positif Sesaran bertambah secara linear (pada kadar
ne

dari kedudukan asal. malar). Kecerunan graf itu positif


The object moves in positive direction
from its original position. dan malar . Halaju objek adalah
Pe

Halaju seragam positif seragam


Constant velocity dan .
Its displacement is increasing linearly (at
t (s) constant rate). The gradient of the graph is
0
positive and constant . Its velocity is
positive and constant .

31
  Fizik  Tingkatan 4  Bab 2 Daya dan Gerakan

(c) s (m) Objek bergerak dalam arah negatif Sesaran berkurang secara linear (pada kadar
dan kembali ke kedudukan asal. malar). Kecerunan graf itu negatif
The object moves in negative direction
Halaju seragam back to its original position. dan malar . Halaju objek adalah
Constant velocity
negatif dan seragam .
Its displacement is decreasing linearly (at constant
t (s)
BAB

0 rate). The gradient of graph is negative and


constant . Its velocity is negative and
constant

.
.
2

hd
Contoh 6

.B
Graf sesaran-masa di bawah menunjukkan pergerakan suatu objek.
The displacement-time graph below shows the movement of an object. s/m
(a) Terangkan secara ringkas pergerakan objek yang diwakili oleh

dn
garis OB, BC, CD dan DE. D
10
Briefly describe the motion of the object represented by OB, BC, CD
B C
and DE. 5

iS
(b) Cari / Find E
0 t/s
(i) sesaran selepas 7 s, 5 10 18 22
the displacement after 7 s,
ng
(ii) masa yang diambil oleh objek itu untuk sampai pada 10 m
dari titik asal.
la
the time taken to reach at 10 m from the origin.
(c) Kira halaju purata dalam setiap jangka masa ini:
Pe

Calculate the average velocity in each of these time intervals:


(i) 0 s – 5 s (iii) 10 s – 18 s
(ii) 5 s – 10 s (iv) 18 s – 22 s

Penyelesaian / Solution:
n

(a) OB: Objek itu bergerak 5 m ke depan dengan halaju positif.


ita

The object moved 5 m forward with positive velocity.


BC: Objek itu berada dalam keadaan rehat.
rb

The object is at rest.


CD: Objek itu bergerak 8 m ke depan dengan halaju positif.
ne

The object moved 8 m forward with positive velocity.


DE: Objek itu bergerak 10 m ke belakang dengan halaju negatif dan kembali ke titik permulaan.
The object moved 10 m backward with negative velocity and returned to its starting point.
Pe

(b) (i) Apabila / when t = 7 s, s = 5 m


(ii) Apabila / When s = 10 m, t = 18 s

(c) (i) Halaju = kecerunan graf / Velocity = gradient of the graph (iii) v = 10 – 5 = 5 = 0.625 s–1
18 – 10 8
v = 5 = 1 m s–1
5
(ii) Objek itu adalah pegun. Maka, halaju = 0 m s–1 (iv) v = 0 – 10 = – 10 = –2.5 m s–1
22 – 18 4
The object is stationary Therefore, v = 0 m s–1

32
  Fizik  Tingkatan 4  Bab 2 Daya dan Gerakan 

Menganalisis Graf Halaju-Masa untuk Menentukan Jarak, Sesaran, Halaju dan Pecutan
Analysing the Velocity-Time Graph to Determine the Distance, Displacement, Velocity and Acceleration

1. Pelbagai graf halaju-masa ditunjukkan dalam jadual yang berikut.


The various velocity-time graphs are shown in the following table

Graf halaju-masa Halaju dan pecutan


Velocity-time graph Velocity and acceleration

BAB
(a) v (m s–1) Kecerunan graf adalah sifar maka objek bergerak dengan halaju
seragam . Maka, pecutan adalah sifar .

.
2

hd
The gradient of graph is zero  , thus object is moving with uniform
Halaju seragam
Constant velocity zero
velocity. Hence, its acceleration is .
0 t (s)

.B

Kecerunan graf adalah positif dan malar, maka halaju objek


(b) v (m s–1)

dn
bertambah secara linear (pada kadar malar). Objek itu bergerak
dengan pecutan seragam .

iS
Pecutan seragam positive
Constant acceleration The gradient of graph is and constant, thus the velocity of
object increases linearly (at constant rate). The object is moving with
t (s)
0
constant acceleration
ng .

Kecerunan graf adalah negatif dan malar, maka halaju objek


(c)
la
v (m s–1) berkurang secara linear (pada kadar malar). Objek itu bergerak
pecutan seragam
Pe

Pecutan seragam dengan .


Constant acceleration
The gradient of graph is negative and constant, thus the velocity of
object decreases linearly (at constant rate). The object is moving with
0 t (s)
n

constant acceleration .

ita

Contoh 7
rb

Graf halaju-masa di bawah menunjukkan pergerakan suatu objek.


The velocity-time graph shows the movement of an object.
ne

v / m s–1
(a) Cari / Find
5
(i) halaju awal / the initial velocity,
(ii) halaju apabila / the velocity when t = 2 s,
Pe

(iii) halaju apabila / the velocity when t = 4 s.


0 t/s
2 4
(b) Cari sesaran pada jangka masa yang diberikan berikut.
Find the displacement at the given time intervals.
(i) 0 – 2 s –5
(ii) 2 – 4 s
(iii) 0 – 4 s
(c) Apakah pecutan objek ini pada t = 1 s?
What is the acceleration of the object at t = 1 s?

33
  Fizik  Tingkatan 4  Bab 2 Daya dan Gerakan

Penyelesaian / Solution:
(a) (i) u = 5 m s−1
(ii) v = 0 m s−1
(iii) v = −5 m s−1

(b) (i) Luas di bawah graf / Area under the graph = 1 (5 × 2) = 5 m


2
(ii) Luas di bawah graf / Area under the graph = 1 (–5 × 2) = –5 m
BAB


2
(iii) 5 + (−5) = 0 m

.
2 (c) a = –5 = –2.5 m s–2 / 2.5 – 5 = –2.5 m s–2

hd
2 1–0

.B
Menterjemah dan Melakar Graf
Interpreting and Sketching Graph

dn
1. Jadual di bawah menunjukkan perbandingan antara graf sesaran-masa, graf halaju-masa dan graf pecutan-
masa untuk gerakan sepanjang satu garis lurus dalam satu arah.
Table below shows a comparison between displacement-time graph, velocity-time graph and acceleration-time graph for

iS
motion along a straight line in one direction.

Graf Sesaran-Masa Graf Halaju-Masa Graf Pecutan-Masa


Displacement-Time Graph
ng
Velocity-Time Graph Acceleration-Time Graph

(a) s/m (b) v / m s–1 (c) a / m s–2


la
Pe

t/s t/s t/s


n

Sesaran kekal sama. Objek berada Oleh kerana objek tidak bergerak, Oleh kerana objek tidak bergerak,
pada rehat (tidak bergerak). halaju adalah sifar . pecutan adalah sifar .
ita

The displacement stays the same. The Since the object is not moving, the Since the object is not moving, the
object is at rest (not moving). zero acceleration is zero .
velocity is .
rb

(d) s/m (e) v / m s–1 (f) a / m s–2


ne
Pe

t/s t/s t/s

Sesaran bertambah dengan seragam Halaju tetap sama . Objek Halaju malar memberikan pecutan
dengan masa. Objek itu bergerak malar sifar .
bergerak dengan halaju .
dengan halaju yang tetap .
same Constant velocity gives zero
The velocity stays the .
The displacement is increasing acceleration.
The object is moving with constant
uniformly with time. The object is moving
velocity.
with constant velocity.

34
  Fizik  Tingkatan 4  Bab 2 Daya dan Gerakan 

(g) s/m (h) v / m s–1 (i) a / m s–2

t/s t/s t/s

BAB
positif .
Kecerunan semakin meningkat Halaju bertambah seragam dengan Pecutan adalah
dengan masa. Oleh itu, halaju masa. Objek bergerak dengan Objek bergerak dengan pecutan
seragam .

.
semakin meningkat dengan masa. pecutan seragam. Sesaran
2

hd
increasing adalah sama dengan luas di positive
The gradient is with time. The acceleration is . The
increasing bawah graf. object is moving with constant
Therefore, the velocity is

.B
with time. The velocity is increasing uniformly acceleration.
with time. The object is moving with
constant acceleration . Displacement is

dn
equal to the area under the graph.

(j) s/m (k) v / m s–1 (l) a / m s–2

iS
ng t/s

t/s t/s
la
negatif
Kecerunan semakin berkurang Halaju berkurang dengan Pecutan adalah . Objek
seragam dengan masa. Objek yang bergerak dengan pecutan
Pe

dengan masa. Oleh itu, halaju


pecutan seragam.
semakin menurun dengan masa. yang bergerak dengan
negative .
decreasing
seragam. Sesaran adalah sama The acceleration is
The gradient is with time. luas The object is moving with constant
decreasing dengan di bawah graf.
n

Therefore, the velocity is acceleration .


Kecerunan graf negatif.
with time.
ita

The velocity is decreasing constantly


with time. The object is moving with
constant acceleration . The displacement
rb

is equal to the area under


the graph. The gradient of the graph is
ne

negative.

Contoh 8
Pe

Rajah 4 menunjukkan graf sesaran-masa suatu objek yang bergerak secara linear. s/m
Diagram 4 shows displacement-time graph for an object moving in linear motion.
(a) Tentukan halaju pergerakan objek ini untuk setiap satu peringkat 10
pergerakannya.
Determine the velocity of the object for each level of motion.
(b) Terjemahkan graf sesaran-masa dalam Rajah 4 untuk melakarkan graf
halaju-masa. 0 t/s
2 6 8
Interpret the displacement-time graph in diagram to sketch velocity-time graph.
Rajah 4 / Diagram 4

35
  Fizik  Tingkatan 4  Bab 2 Daya dan Gerakan

Penyelesaian / Solution:
(a) Halaju = Kecerunan graf sesaran-masa (b) v / m s–1
Velocity = Gradient of displacement-time graph
5
0 – 2 s, v = 10 = 5 m s–1
2
0 t/s
2 6 8
2 – 6 s, v = 0 = 0 m s–1
BAB

24
–5
6 – 8 s, v = – 10 = –5 m s–1
2

.
2

hd

Contoh 9

.B
Rajah 5 menunjukkan graf halaju-masa yang diplot berdasarkan gerakan linear sebuah kereta.
Diagram 5 shows velocity-time graph plotted based on the linear motion of a car.

dn
v / m s–1

iS
A B
6
ng
2 C

0 t/s
la
O 4 8 10

Rajah 5 / Diagram 5
Pe

Terjemahkan graf halaju-masa bagi pergerakan kereta dan lakarkan


Interpret the velocity-time graph for the car motion and sketch.
(a) graf sesaran-masa
n

displacement-time sketch
ita

(b) graf pecutan-masa


acceleration-time sketch
rb

Penyelesaian / Solution:

(a) s/m a / m s–2


(b)
ne

36

28 1.5
Pe

0 t/s
4 8 10
12
–2
0 t/s
O 4 8 10

36
  Fizik  Tingkatan 4  Bab 2 Daya dan Gerakan 

Menyelesaikan Masalah Melibatkan Graf Gerakan Linear


Solving Problems Involves Linear Motion Graphs

1. Graf sesaran-masa menunjukkan gerakan suatu objek. s (m)


The displacement-time graph shows the motion of an object. D
45
(a) Terangkan secara ringkas gerakan objek yang diwakili oleh
35
AB, BC, CD dan DE. C

BAB
Briefly describe the motion of the object represented by AB, BC, CD A
and DE. 20
B
(b) Cari / Find

.
2

hd
E
(i) sesaran selepas 20 s, 0 t (s)
5 10 20 28
the displacement after 20 s,
(ii) masa yang diambil untuk bergerak 35 m dari titik asal.

.B
the time taken to move 35 m from the origin.

(c) Hitung halaju purata dalam setiap selang masa ini:


Calculate the average velocity in each of these time intervals:

dn
(i) 0 s – 5 s (ii) 5 s – 10 s (iii) 10 s – 20 s (iv) 20 s – 28 s

Penyelesaian / Solution:

iS
(a) AB: Objek dalam keadaan rehat 20 m dari titik mula .
20 starting point
AB: The object is at rest m from the .
ng
BC: Objek bergerak 15 m ke hadapan dengan halaju positif .
15 positive
la
BC: The object moved m forward with velocity.

10 positif
Pe

CD: Objek bergerak m lagi ke hadapan dengan halaju .


10 positive
CD: The object moved another m forward with velocity.

DE: Objek bergerak 45 m ke belakang dengan halaju negatif dan kembali ke titik mula.
n

DE: The object moved 45 m backwards with negative velocity and returned to its starting point.
ita

(b) (i) Apabila / When t = 20 s, s = 45 – 20 = 25 m


(ii) Apabila / When s = 35 m, t = 10 s
rb

(c) (i) Objek dalam keadaan rehat , maka halaju = 0 m s–1


ne

Object is at rest , hence the velocity = 0 m s–1

(ii) Halaju = kecerunan graf


Pe

Velocity = gradient of the graph


15
v= = 3 m s–1
5
45 – 35 10
(iii) Halaju / Velocity, v = = = 1.0 m s–1
20 –10 10
0 – 45 45
(iv) Halaju / Velocity, v = = – = –5.63 m s–1
28 –20 8

37
  Fizik  Tingkatan 4  Bab 2 Daya dan Gerakan

Tugasan 2
1. Graf halaju-masa menunjukkan gerakan suatu objek. v (m s–1)
The velocity-time graph shows the motion of an object. D
18
(a) Cari halaju apabila / Find the velocity when
(i) t = 5 s (ii) t = 18 s
BAB

A B
Halaju, v = bacaan pada paksi-y 10
Velocity, v = reading at y-axis C
5
(ii) v = 18 m s

.
2 (i) v = 10 m s –1 –1

hd
E
0 t (s)
5 10 14 18 24
(b) Cari sesaran pada selang masa yang diberikan.

.B
Find the displacement at the given time intervals.
(i) 0 s – 5 s (ii) 5 s – 10 s (iii) 18 s – 24 s

Sesaran, s = kawasan di bawah graf

dn
Displacement, s = area below the graph

(i) s = 1 (5 × 10) = 25 m (ii) s = 5 × 10 = 50 m (iii) s = 1 (6 × 18) = 54 m


2 2

iS
(c) Cari pecutan objek pada
Find the acceleration of the object at
(i) t = 1 s (ii) t = 7 s (iii) t = 12 s
(iv) t = 16 s (v) t = 20 s
ng
Pecutan, a = kecerunan graf
la
Acceleration, a = gradient of the graph
a = 10 = 2 m s–2 (ii)
(i) a = 0 = 0 m s–2 (iii) a = – 5 = –1.25 m s–2
5 5 4
Pe

a=
(iv) 13 = 3.25 m s–2
(v) a = – 18 = –3 m s–2
4 6

2. Berdasarkan graf yang berikut, terangkan gerakan objek dalam setiap bahagian.
n

Based on the following graphs, describe the motion of an object in each section.
ita

(a) Graf sesaran-masa (b) Graf halaju-masa


Displacement-time graph Velocity-time graph
Sesaran, s (m) Halaju, v (m s–1)
rb

Displacement, s (m) Velocity, v (m s –1)


ne

B C B C
Pe

D D
A Masa, t (s) A Masa, t (s)
Time, t (s) Time, t (s)

AB: Halaju seragam / Constant velocity AB: Pecutan seragam / Constant acceleration
BC: Halaju sifar / Zero velocity BC: Pecutan sifar / Zero acceleration
CD: Halaju seragam, objek kembali ke kedudukan CD: Pecutan seragam / Constant acceleration
asal / Constant velocity, object returns to the original
position

38
  Fizik  Tingkatan 4  Bab 2 Daya dan Gerakan 

Gerakan Jatuh Bebas


2.3 Free Fall Motion

1. Semua objek yang berlainan jisim, apabila dijatuhkan dari ketinggian yang sama , akan sampai ke tanah
pada masa yang sama jika rintangan udara diabaikan.
All objects of different masses when dropped from the same height, will reach the ground at the same time if

BAB
air resistance is negligible.

2. Semua objek jatuh kerana ditarik ke Bumi oleh daya graviti . Jadi, objek-objek akan jatuh dengan pecutan

.
seragam iaitu 9.8 m s
–2
2

hd
.
All objects fall because they are pulled towards the Earth by gravitational force . So, objects will fall with a constant

acceleration of 9.8 m s–2 .

.B
3. Di dalam tiub kaca berisi udara, bola logam akan sampai ke dasar
lebih cepat berbanding dengan bulu pelepah. Hal ini berlaku

dn
Udara Vakum
Air Vacuum demikian kerana rintangan udara yang bertindak ke atas bulu pelepah
Bola logam
Metal ball Bulu adalah lebih besar daripada bola logam. Di dalam tiub kaca vakum,

iS
pelepah masa yang sama  . Di
Feather
kedua-dua objek itu sampai ke dasar pada
dalam ketiadaan rintangan udara dalam vakum, kedua-dua bola
logam dan bulu pelepah sampai ke dasar dengan pecutan yang sama
Tiub kaca
Glass tubes
ng
tanpa dipengaruhi oleh jisim masing-masing. Maka, kedua-dua objek
itu sampai ke dasar pada masa yang sama .
la
In the air-filled glass tube, the metal ball will reach the base faster compared to the feather. This is because the air
Pe

resistance acting on the feather is bigger than the metal ball. In the vacuum glass tube, both the objects reach the
base at the same time . In the absence of air resistance in vacuum, both the ball and the feather fall to the base with
equal acceleration same time
regardless of their masses. Therefore, both the objects will reach the base at the .
n

4. Sebarang objek, jika dibenarkan jatuh bebas berdekatan dengan permukaan Bumi, objek itu akan mempunyai
ita

pecutan graviti, g .
Any object, if allowed to fall freely near the Earth’s surface, will have a gravitational acceleration, g .
rb

5. Semua objek berdekatan Bumi mengalami tarikan graviti yang bertindak ke arah pusat bumi. Kawasan
medan graviti Bumi.
ne

di mana daya tarikan bertindak dikenal sebagai


All objects near the Earth experience gravitational attraction acting towards the centre of the earth. The region
gravitational field
Pe

where the pulling force acts is called the of the Earth.

6. Kekuatan medan graviti ialah daya graviti yang bertindak ke atas suatu jisim 1 kg yang diletakkan
pada suatu titik dalam medan graviti itu.
Gravitational field strength is the gravitational force acting on a mass of 1 kg which is placed at a point in the
gravitational field .

F
g= m

39
  Fizik  Tingkatan 4  Bab 2 Daya dan Gerakan

7. Unit SI bagi kekuatan medan graviti ialah N kg–1 atau m s–2 .


The SI unit for gravitational field strength is N kg–1 or m s–2 .

Menentukan nilai pecutan graviti


Eksperimen 2.1 Determine the value of gravity acceleration
BAB

Tujuan / Aim:

.
nilai pecutan graviti
2 Menentukan disebabkan oleh .

hd
To determine the acceleration due to gravity .

Radas / Apparatus:

.B
Sistem photogate dan pemasa elektronik, pelepas elektromagnet dan bekas bola keluli.
Photogate system and electronic timer, electromagnetic release and steel ball container.

dn
Susunan radas / Arrangement of apparatus:

iS
ng
la
Pe
n
ita

Rajah 6 / Diagram 6
rb

Prosedur / Procedure:
ne

1. Radas disusun seperti yang ditunjukkan dalam Rajah 6.


The apparatus is set up as shown in the Diagram 6.

2. Laraskan photogate kedua supaya jarak pemisahan dari photogate pertama ialah 40.0 cm.
Pe

Adjust the second photogate so that the separation distance from first photogate is 40.0 cm.

3. Pastikan bola keluli boleh jatuh melalui kedua-dua photogate ke dalam bekas.
Make sure the steel ball can fall through the both photogates into the container.

4. Lepaskan bola keluli yang dipegang oleh pelepas elektromagnet.


Release the steel ball held by the electromagnetic release.

5. Catatkan masa yang diambil untuk bola keluli melalui kedua-dua photogate dalam Jadual 2.1.
Record the time taken for the steel ball through both photogates in a Table 2.1.

6. Ulang untuk jarak pemisahan photogate 50.0 cm, 60.0 cm, 70.0 cm dan 80.0 cm.
Repeat for photogates separation distances of 50.0 cm, 60.0 cm, 70.0 cm and 80.0 cm.

40
  Fizik  Tingkatan 4  Bab 2 Daya dan Gerakan 

Penjadualan data / Tabulation of data:

Jarak pemisahan di antara Masa yang diambil oleh bola keluli untuk
dua photogate, melalui dua photogate Pecutan graviti,
Separation distance between Time taken by the steel ball to go through two photogates Gravitational acceleration,
two photogates, g/ m s–2
x/cm t1 (s) t2 (s)

BAB
40.0

50.0

.
2

hd
60.0

70.0

.B
80.0
Jadual 2.1 / Table 2.1

dn
Analisis data / Analysis of data:
1. Tentukan nilai g menggunakan rumus g = 2x
t22 – t12

iS
Determine the value of g using formula

2. Daripada lima nilai g yang anda peroleh, hitungkan nilai puratanya


From the five values of g obatined, calculate the average value.
ng
g / m s–2
la
Pe

x / cm

n

Perbincangan / Discussion:
ita

1. Nilai g daripada eksperimen adalah hampir sama bagi nilai jarak yang berlainan.
almost identical distance
rb

The values of g from experiments are for different values of .

2. Nilai g adalah mungkin lebih kecil


daripada 9.8 m s–2. Perkara ini disebabkan oleh geseran
ne

dialami oleh bola keluli dan rintangan udara .


The values of g maybe smaller than 9.8 m s–2. This is due to the friction experienced by the steel
Pe

ball and also air resistance .

Kesimpulan / Conclusion:
Pecutan graviti ialah kira-kira 9.8 m s–2 dan tidak bergantung pada jarak objek.
The gravitational acceleration is about 9.8 m s–2 and does not depend on the distance of the object.

41
  Fizik  Tingkatan 4  Bab 2 Daya dan Gerakan

Menyelesaikan Masalah yang Melibatkan Objek yang Jatuh Bebas


Solve Problems Related to Free Falling Objects

Contoh 10
Sebiji batu dijatuhkan dari sebuah bangunan tiga tingkat dengan ketinggian 15 m. Jika rintangan udara boleh
diabaikan dan kekuatan medan graviti Bumi ialah 9.8 N kg–1, hitung
A stone is dropped from a three-storey building with a height of 15 m. If air resistance is negligible and gravitational field
BAB

strength of the Earth is 9.8 N kg–1, calculate


(a) laju batu sebelum mencecah tanah
the speed of the stone just before it strikes the ground

.
2

hd
Diberi / Given: s = 15 m, g = 9.8 m s–2, u = 0 m s–1, v = ?
v2 = u2 + 2gs
= 0 + (2)(9.8)(15)

.B
v = 17.15 m s–1
(b) masa yang diambil untuk batu itu mencecah tanah
the time taken for the stone to strike the ground

dn

Diberi / Given: s = 15 m, g = 9.8 m s–2, u = 0 m s–1, t = ?
s = ut + 1 gt2

iS
2
1
15 = 0 + (9.8)t2
2
t = 1.75 s
ng
3
la
Tugasan
Pe

1. Seorang angkasawan melompat dari ketinggian 10 m di atas permukaan Bulan. Berapakah masa yang diambil untuk
angkasawan itu sampai ke permukaan bulan?
An astronaut jumps from a height of 10 m above the surface of the Moon. What is the time taken for the astronaut to reach the surface of
the moon? [gM = 1.6 m s–2]
n

Penyelesaian / Solution:
ita

Diberi / Given: u = 0 m s–1, s = 10 m, g = 1.6 m s–2, t = ?


1
s = ut + gt2
2
rb

1
10 = 0 + (1.6)t2
2
t = 3.54 s
ne

2. Sebiji batu dibaling ke atas dengan halaju 10 m s–1. Jika rintangan udara boleh diabaikan dan kekuatan medan graviti
Bumi ialah 9.8 m s–2, hitung masa yang diambil oleh batu itu untuk jatuh kembali ke kedudukan asal.
Pe

A stone is thrown upwards with an initial velocity of 10 m s–1. If air resistance is negligible and gravitational field strength of the earth is
9.8 N kg–1, calculate the time taken for the stone to fall back to the initial position.
Penyelesaian / Solution:
Diberi / Given: u = 10 m s–1, s = 0 m, g = –9.8 m s–2, t = ?
1
s = ut + gt2
2
1

0 = 10t + (–9.8)t2
2

0 = t(10 – 4.9t)
t = 0 and / dan t = 2.04 s ∴ t = 2.04 s

42
  Fizik  Tingkatan 4  Bab 2 Daya dan Gerakan 

Menerangkan Konsep Inersia Melalui Contoh


2.4 Explain the Concept of Inertia Through Examples

1. Inersia ialah kecenderungan suatu objek untuk mengekalkan keadaan rehat , atau jika sedang bergerak
untuk meneruskan gerakan dalam satu garis lurus.
Inertia is the tendency of an object to maintain its state of re­st , or if moving to continue its motion in a

BAB
straight line.

2. Hukum gerakan Newton pertama menyatakan bahawa suatu objek yang berada dalam keadaan
rehat cenderung untuk kekal dalam keadaan rehat, dan jika dalam gerakan cenderung untuk

.
2

hd
kekalkan gerakan melainkan dikenakan ke atasnya suatu daya luar .
Newton’s first law of motion states that an object at re­st tends to stay at rest, and if in motion tends to stay in

.B
motion unless acted upon by an external force .

3. Situasi yang berikut menunjukkan contoh inersia: 3

dn
The following situations show examples of inertia:
(a) Pada mulanya, bas dalam keadaan pegun .
Penumpang berada dalam keadaan rehat dan tidak

iS
bergerak. Apabila bas mula bergerak ke hadapan
dengan tiba-tiba, inersia penumpang cenderung
ng untuk mengekalkan badannya pada kedudukan
Pegun Bas bergerak ke hadapan asal. Hal ini menyebabkan penumpang itu terdorong
Stationary Bus moves forward ke belakang .
stationary
la
Initially, the bus is . The passenger is at rest and not moving. When the bus starts to move forward
suddenly, the inertia of the passenger tends to keep his body in the original position. This causes the passenger to fall
Pe

backwards .
(b) Pada mulanya, penumpang bergerak bersama-
sama bas yang bergerak dengan laju seragam.
Apabila bas berhenti dengan tiba-tiba, inersia
n

penumpang cenderung untuk meneruskan


ita

gerakannya ke hadapan. Hal ini menyebabkan


Bas bergerak ke hadapan Bas membrek penumpang itu terdorong ke hadapan .
Bus moves forward Bus brakes
rb

Initially, the passenger is moving together with the bus,


which moves at constant speed. When the bus stops suddenly, the inertia of the passenger tends to continue his forward
forward
ne

motion. This causes the passenger to fall .

Hubungan antara Jisim dengan Inersia



Pe

Relationship between Mass and Inertia

1. Rajah 7 menunjukkan dua baldi, satu diisi dengan pasir dan satu lagi
kosong.
Diagram 7 shows two buckets, one bucket filled with sand while the other bucket
is empty.

2. Adalah lebih sukar untuk menggerakkan baldi yang berisi pasir berbanding Pasir
Sand
dengan baldi yang kosong apabila kedua-duanya dalam keadaan rehat
pada awalnya.
It is more difficult to move the bucket filled with sand compared to the empty bucket
when both are initially at rest. Rajah 7 / Diagram 7

43
  Fizik  Tingkatan 4  Bab 2 Daya dan Gerakan

3. Apabila kedua-dua baldi itu sedang bergerak, adalah lebih sukar juga untuk menghentikan baldi yang berisi
pasir berbanding dengan baldi yang kosong.
When both the buckets are moving, it is also more difficult to stop the bucket filled with sand compared to the empty bucket.

4. Baldi yang berisi pasir mempunyai kecenderungan yang lebih besar untuk berada dalam keadaan rehat

dan juga untuk terus berada dalam gerakan berbanding dengan baldi yang kosong.
The bucket filled with sand has a greater tendency to be at rest and also to continue to be in motion
BAB

compared with the empty bucket.

5. Baldi yang berisi pasir mempunyai lebih inersia daripada baldi yang kosong. Baldi yang berisi pasir

.
2 jisim

hd
mempunyai yang lebih besar daripada baldi yang kosong. Secara kuantitatif, inersia sesuatu objek
diukur dengan jisim .

.B
The bucket filled with sand has more inertia than the empty bucket. The bucket filled with sand has a bigger
mass than the empty bucket. Quantitatively, the inertia of an object is measured by its mass .

dn
Hubungan antara inersia dengan jisim
Eksperimen 2.2 Relationship between inertia and mass

iS
Pernyataan masalah / Problem statement: ng
Dalam situasi yang dinyatakan dalam halaman sebelum, baldi yang berisi pasir lebih sukar untuk diayunkan
berbanding dengan baldi yang kosong.
Apakah hubungan antara inersia dengan jisim?
la
In the situation stated in previous page, the bucket filled with sand is more difficult to swing compared to the empty bucket.
What is the relationship between inertia and mass?
Pe

Inferens / Inference:
Inersia suatu objek bergantung pada jisim objek itu.
n

The inertia of an object depends on its mass .


ita

Hipotesis / Hypothesis:
Apabila jisim suatu objek bertambah , inersia objek itu juga bertambah .
rb

When the mass of an object increases , the inertia of the object increases .

Tujuan / Aim:
ne

Untuk mengkaji hubungan antara inersia dengan jisim / To investigate the relationship between inertia and mass.

Pemboleh ubah / Variables:


Pe

(a) Pemboleh ubah dimanipulasi: Jisim plastisin


Manipulated variable: Mass of plasticine

(b) Pemboleh ubah bergerak balas: Tempoh ayunan


Responding variable: Period of oscillation

(c) Pemboleh ubah dimalarkan: Kekerasan bilah gergaji


Constant variable: Stiffness of hacksaw blade

44
  Fizik  Tingkatan 4  Bab 2 Daya dan Gerakan 

Bahan / Materials: Plastisin / Plasticine

Radas / Apparatus: Bilah gergaji, pengapit-G dan jam randik / Hacksaw blade, G-clamp and stopwatch

Susunan radas / Arrangement of apparatus:

BAB
Pengapit-G Bilah gergaji
G-clamp Hacksaw blade

.
Plastisin
2

hd
Plasticine

Prosedur / Procedure:

.B
1. Radas disusun seperti yang ditunjukkan.
The apparatus is set up as shown.
2. 25 g plastisin diletakkan pada hujung bilah gergaji yang bebas.

dn
A 25 g of plasticine is fixed to the free end of the hacksaw blade.
3. Masa untuk 20 ayunan lengkap diukur menggunakan jam randik dan dicatatkan. Langkah ini diulang sebanyak
dua kali untuk menghitung masa purata yang diambil. Kemudian, tempoh, T ditentukan.

iS
The time for 20 complete oscillations is measured using a stopwatch and is recorded. The step is repeated twice to calculate
the average time taken. Then, the period, T is determined.
4. Langkah 3 diulang dengan plastisin berlainan jisim, m = 50 g, 75 g, 100 g dan 125 g.
ng
Step 3 is repeated with different mass of plasticine, m = 50 g, 75 g, l00 g and 125 g.

Penjadualan data / Tabulation of data:


la
Masa diambil untuk 20 ayunan
Analisis data / Analysis of data:
Jisim plastisin, Tempoh,
Pe

Time taken for 20 oscillations T (s)


Mass of Period,
plasticine, m t
t1 (s) t2 (s) t1 + t2 T = 20 (s)
(g) t= 20 (s)
n

25.0
m (g)
ita

50.0 0

75.0
rb

100.00
125.00
ne

Perbincangan / Discussion:
Pe

Daripada eksperimen, kita perhatikan bahawa lebih besar jisim plastisin, lebih lama tempoh ayunan.
Maka, lebih besar jisim suatu objek, lebih besar inersia objek itu.
From the experiment, we observed that the bigger the mass of plasticine, the longer the period of oscillation.
Hence, the bigger the mass of an object, the bigger the inertia of the object.

Kesimpulan / Conclusion:
Apabila jisim suatu objek bertambah, inersia objek itu juga bertambah . Hipotesis diterima.
When the mass of an object increases, the inertia of the object increases . The hypothesis is accepted.

45
  Fizik  Tingkatan 4  Bab 2 Daya dan Gerakan

Kesan Inersia dalam Kehidupan Seharian


The Effects of Inertia in Daily Life

1. Kesan positif inersia / The positive effect of inertia: 3

Kepala tukul boleh diketatkan pada pemegang kayu dengan mengetuk


hujung pemegang itu pada permukaan keras. Kepala tukul mempunyai jisim
besar gerakan
BAB

yang dan akan kekal dalam , dengan itu kepala


tukul masuk lebih ketat pada pemegang.
The head of a hammer can be tightened onto the wooden handle by knocking the

.
2

hd
end of the handle against a hard surface. The head of the hammer has a big

mass and will remain in its motion  , thus fitting it tighter on the handle.

.B
2. Kesan negatif inersia / The negative effects of inertia: 3

(a) Jika sebuah kereta mengalami kemalangan semasa dalam perjalanan,


pemandu kereta itu masih dalam gerakan ke hadapan

dn
walaupun kereta sudah berhenti. Pemandu mungkin
terhumban ke cermin depan dan mengalami kecederaan.

iS
Perkara ini dapat dicegah jika pemandu itu memakai
tali pinggang keledar yang akan menariknya kembali daripada
gerakan ke hadapan itu.
ng
If a car crashes while travelling, the driver of the car is still in forward motion
even though the car has stopped. The driver may be hurled against the
la
windscreen and suffer injuries. This can be prevented if the driver is wearing a
seat belt
that will pull him back from the forward motion.
Pe

(b) Tangki lori yang membawa cecair dibahagikan kepada beberapa


ruangan lebih kecil yang terpisah . Pembahagian ini akan
n

mengurangkan kesan inersia daripada cecair sekiranya lori itu


ita

berhenti secara tiba-tiba.


The tank of a lorry carrying liquid is divided into several separated
smaller compartments . This will reduce the impact of the
rb

inertia of the liquid if the lorry stops suddenly.


ne

Momentum
2.5 Momentum
Pe

1. Momentum suatu objek ditakrifkan sebagai hasil darab jisim dengan halaju objek itu.
The momentum of an object is defined as the product of its mass and the velocity of the object.

Momentum / Momentum = jisim / mass × halaju / velocity


p = mv

2. Momentum ialah kuantiti vektor dengan unit kg m s–1 atau Ns . Arah momentum
ditandakan dengan tanda positif (+) atau negatif (–).
Momentum is a vector quantity with unit kg m s–1 or Ns . The direction of momentum is indicated
with a positive (+) or a negative (–) sign.

46
  Fizik  Tingkatan 4  Bab 2 Daya dan Gerakan 

Contoh 11
Rajah 8 menunjukkan seorang pemain besbol memukul sebiji bola. Jisim bola itu
ialah 200 g. Apakah momentum bola yang melayang dengan halaju 50 m s–1?
Diagram 8 shows a baseball player hitting a ball. The mass of the ball is 200 g. What is the
momentum of the ball flying with a velocity of 50 m s–1?

Penyelesaian / Solution:

BAB
Momentum, p = mv
= 0.2 × 50

.
Rajah 8 / Diagram 8
= 10 kg m s–1 2

hd
Aplikasi Prinsip Keabadian Momentum dalam Pelanggaran dan Letupan

.B
Application of the Principle of Conservation of Momentum in collision and Explosion

1. Prinsip keabadian momentum menyatakan bahawa dalam suatu perlanggaran, jumlah momentum dalam
sistem diabadikan jika tidak ada daya luar yang bertindak ke atas sistem itu.

dn
The principle of conservation of momentum states that in a collision, the total momentum in the system is conserved if no
external force acts on the system.

iS
Jumlah momentum objek sebelum perlanggaran sama dengan jumlah momentum objek
selepas perlanggaran jika tidak ada daya luar yang bertindak ke atas kedua-dua objek tersebut.
ng
The total momentum of the objects before collision equals the total momentum of both objects
after collision if no external force acts on both objects.
la

2. Prinsip keabadian momentum dapat digunakan dalam situasi yang berikut.


Pe

The principle of conservation of momentum can be used in the following situations.


(a) Perlanggaran dua objek / Collision of two objects
(b) Letupan / Explosion
n

Contoh 12
ita

Rajah 9 menunjukkan sebiji peluru berjisim 20 g u = 0 m s –1


ditembak secara mengufuk ke arah sebuah bongkah 100 m s–1
rb

kayu berjisim 4 kg. Halaju peluru itu ialah 100 m s–1. v


Selepas peluru terbenam pada bongkah kayu, kedua- 20 g 4 kg
duanya bergerak bersama. Apakah halaju sepunya
ne

bongkah kayu dan peluru selepas perlanggaran itu? Rajah 9 / Diagram 9


Diagram 9 shows a 20 g bullet fired horizontally towards a
Pe

wooden block with a mass of 4 kg. The velocity of the bullet is 100 m s–1. After the bullet has embedded itself in the wooden
block, both of them move together. What is their common velocity after the impact?

Penyelesaian / Solution:
Diberi / Given: m1 = 0.02 kg, u1 = 100 m s–1, m2 = 4 kg, u2 = 0 m s–1, v = ?
Menggunakan / Using: m1u1 + m2u2 = (m1 + m2)v
(0.02)(100) + (4)(0) = (0.02 + 4)v
v = 2
4.02
= 0.498 m s–1

47
  Fizik  Tingkatan 4  Bab 2 Daya dan Gerakan

Contoh 13
Rajah 10 menunjukkan dua buah troli yang berbeza 4 m s–1 2 m s–1 3 m s–1 v m s–1
jisim sebelum dan selepas satu perlanggaran. Cari
halaju troli 3 kg selepas perlanggaran itu. 5 kg 3 kg 5 kg 3 kg
Diagram 10 shows two trolleys of different masses before
and after a collision. Find the velocity of the 3 kg trolley Sebelum perlanggaran Selepas perlanggaran
after the collision.
BAB

Before collision After collision

Penyelesaian / Solution: Rajah 10 / Diagram 10

Diberi / Given: m1 = 5 kg, u1 = 4 m s–1, m2 = 3 kg, u2 = –2 m s–1, v1 = 3 m s–1, v2 = ?

.
2

hd
Menggunakan / Using: m1u1 + m2u2 = m1v1 + m2v2
(5)(4) + (3)(–2) = (5)(3) + 3v2

.B
–1
v2 = = –0.33 m s–1
3

dn
3. Dalam suatu letupan, dua objek bergerak dalam arah bertentangan selepas letupan.
In an explosion, two objects move in opposite directions after the explosion.

iS
m1 m2 m1
ng m2

u1 = 0 u2 = 0 v1 v2
la

tidak diabadikan
Pe

(a) Tenaga kinetik . Jumlah tenaga kinetik objek sebelum dan selepas letupan
tidak sama .
Kinetic energy is not conserved . Total kinetic energy of objects before and after explosion is not equal .
n

(b) Jumlah momentum objek sebelum dan selepas letupan adalah sama .
ita

Total momentum of objects before and after explosion is equal .

(c) Jumlah tenaga kedua-dua objek diabadikan .


rb

Total energy of both objects is conserved .


Rumus / Formula: m1u1 + m2u2 = m1v1 + m2v2 atau / or m1v1 + m2v2 = 0
ne

(d) Contoh letupan: Tembakan meriam


Example of explosion: Firing of cannon
Pe

48
  Fizik  Tingkatan 4  Bab 2 Daya dan Gerakan 

Contoh 14
Rajah 11 menunjukkan seorang budak lelaki
berjisim 60 kg dan seorang budak perempuan
berjisim 50 kg bermain kasut roda. Pada
mulanya, mereka saling berpegangan. Budak Budak Budak Budak
Kemudian, budak lelaki itu menolak lelaki perempuan lelaki perempuan

BAB
budak perempuan dan kedua-dua mereka Boy Girl Boy Girl
melepaskan tangan masing-masing. Jika
halaju budak perempuan ialah 3 m s–1 selepas Rajah 11 / Diagram 11

.
ditolak, cari halaju budak lelaki itu. (Anggap bahawa tiada daya geseran.) 2

hd
Diagram 11 show a boy of 60 kg and a girl of 50 kg on roller skates. Initially, they hold each other’s hand. Then, the boy
pushes the girl and both of them release their hands. If the velocity of the girl is 3 m s–1 after being pushed, find the velocity
of the boy. (Assume that there is no frictional force.)

.B
Penyelesaian / Solution:
Diberi / Given: mb = 60 kg, mg = 50 kg, u(b + g) = 0, vg = 3 m s–1, vb = ?

dn
0 = mbvb + mgvg
= 60vb + (50)( 3)

iS
vb = – 150 = –2.5 m s–1
60
ng
Tugasan 5
la

1. Hitung momentum sebuah kereta berjisim 2 000 kg yang Momentum, p = mv


Pe

bergerak dengan halaju 30 m s–1. = 2 000 × 30


Calculate the momentum of a car of mass 2 000 kg moving with a = 60 000 kg m s-1
velocity of 30 m s–1.
n

2. Sebiji peluru berjisim 50 g ditembak daripada sepucuk pistol Dalam letupan, jumlah momentum sebelum letupan
ita

berjisim 2 kg dengan halaju 120 m s–1. Berapakah jumlah ialah 0 kg m s–1.


momentum peluru dan pistol selepas tembakan? Jumlah momentum sebelum = Jumlah momentum
A bullet with a mass of 50 g is fired from a 2 kg pistol with a velocity selepas
rb

of 120 m s–1. What is the total momentum of the bullet and the pistol Maka, jumlah momentum peluru dan pistol selepas
after the shot? letupan ialah 0 kg m s–1.
In explosion, total momentum before explosion is 0 kg m s–1.
ne

Total momentum before = Total momentum after


Hence, total momentum of the bullet and the pistol after the
explosion is 0 kg m s–1.
Pe

3. Sebuah kereta berjisim 500 kg bergerak dengan halaju Jumlah momentum = Jumlah momentum
30 m s–1 berlanggar dengan sebuah lori berjisim 3 000 kg sebelum selepas
yang dalam keadaan rehat. Selepas perlanggaran itu, kedua- Total momentum before = Total momentum after
dua kereta dan lori bergerak bersama-sama. Berapakah m1u1 + m2u2 = (m1 + m2)v
halaju sepunya selepas perlanggaran? (500)(30) + (3 000)(0) = (500 + 3 000)v
A 500 kg car travelling at 30 m s–1 collides with a 3 000 kg lorry which is v = 4.286 m s–1
at rest. After the collision, both the car and lorry move together. What
is the common velocity after the collision?

49
  Fizik  Tingkatan 4  Bab 2 Daya dan Gerakan

4. Troli A berjisim 3 kg bergerak dengan halaju 4 m s –1 Jumlah momentum = Jumlah momentum


berlanggar dengan troli B berjisim 4 kg yang bergerak sebelum selepas
dalam arah bertentangan. Jika kedua-dua troli bergerak Total momentum before = Total momentum after
bersama dengan halaju 3 m s–1 dalam arah troli B selepas m1u1 + m2u2 = (m1 + m2)v
perlanggaran, cari halaju awal troli B. (3)(4) + 4u2 = (3 + 4)(-3)
Trolley A of mass 3 kg moving at 4 m s–1 collides with trolley B of mass 4 u2 = –8.25 m s–1
kg which is moving in the opposite direction. If both the trolleys move
BAB

together at 3 m s–1 in the direction of trolley B after the collision, find


the initial velocity of trolley B.

.
2 5. Sepucuk senapang berjisim 6 kg yang tersentak ke belakang Jumlah momentum = Jumlah momentum

hd
dengan halaju 2 m s–1 apabila sebiji peluru berjisim 0.05 kg sebelum selepas
ditembak. Cari nilai v. Total momentum before = Total momentum after
A 6 kg rifle a recoils backwards with a velocity of 2 m s–1 when a bullet 0 = mrvr + mbvb

.B
of 0.05 kg is fired. Find the value of v. = (6)(–2) + 0.05vb
v vb = 240 m s-1
2 m s–1

dn
iS
Daya
2.6 Force ng
1. Daya ialah suatu kuantiti vektor . Daya mempunyai magnitud dan arah . Unit SI bagi daya
ialah newton (N) .
la
Force is a vector quantity. It has magnitude and direction  . Its SI unit is newton (N) .
Pe

2. Tindakan menolak atau menarik dapat mengubah bentuk dan saiz objek, laju objek yang bergerak, dan
arah gerakan.
The action of pushing or pulling can change the shape and size of an object, the speed of a moving object and the direction
of motion.
n
ita

Hubungan antara jisim dan pecutan untuk keadaan daya tetap


Aktiviti 2.3 Relationship between mass and acceleration at a constant force
rb

Situasi / Situation:
ne
Pe

Seorang porter di stesen kereta api menarik sebuah kereta sorong. Daya bersih yang bertindak pada kereta sorong
menyebabkannya memecut. Apakah yang berlaku kepada pecutan kereta sorong itu jika dia menarik dua kereta
sorong yang sama tetapi magnitud daya bersih itu dikekalkan?
A porter at a train station pulls a cart. The net force acting on the cart causes it to accelerate. What happens to the acceleration of
the cart if he pulls two similar carts but the magnitude of the net force remains the same.

50
  Fizik  Tingkatan 4  Bab 2 Daya dan Gerakan 

Pemerhatian / Observation:
Pecutan kedua-dua kereta sorong adalah lebih kecil berbanding dengan pecutan kereta sorong tunggal.
The acceleration of the two carts is smaller compared to the acceleration of the single cart.

Inferens / Inference:
Pecutan sesuatu objek bergantung kepada jisimnya.
The acceleration of an object depends on its mass.

BAB
Hipotesis / Hypothesis:
Pecutan suatu objek berkurang apabila jisim objek itu bertambah.

.
The acceleration of an object decreases when the mass of the object increases. 2

hd
Tujuan / Aim:
Untuk menyiasat hubungan antara pecutan dengan jisim.

.B
To investigate the relationship between acceleration and mass.

Pemboleh ubah / Variables:

dn
(a) Pemboleh ubah dimanipulasi: Jisim troli
Manipulated variable: Mass of the trolley
(b) Pemboleh ubah bergerak balas: Pecutan troli

iS
Responding variable: Acceleration of the trolley
(c) Pemboleh ubah dimalarkan: Daya yang bertindak pada troli
Constant variable: Force applied on the trolley
ng
Bahan / Materials:
Bekalan kuasa 12 V a.u., jangka masa detik, pita detik, tali kenyal, tiga troli, pita selofan, landasan dan bongkah kayu
la
12 V a.c. power supply, ticker timer, ticker tape, elastic string, three trollies, cellophane tape, runway and wooden block
Pe

Radas / Apparatus:
Jangka masa
Pita detik detik Tali kenyal
Ticker tape Ticker timer Elastic string
n

Landasan terpampas
geseran
ita

Friction compensated runway


rb

Bongkah kayu
Wooden block
ne

Bekalan kuasa 12 V a.u


12V a.c. power supply
Pe

Prosedur / Procedure:
1. Hidupkan jangka masa detik. Gantung seutas tali kenyal pada troli dan tarik tali kenyal dari hujung troli. Satu
tali kenyal mewakili satu unit daya, F dan satu troli mewaliki satu unit jisim, m.
Switch on the ticker timer. Hook an elastic string onto the trolley and stretch it until the end of the trolley. The elastic string
represents a unit of force, F and the trolley will represent a unit of mass, m.

2. Tarik troli itu menuruni landasan terpampas geseran dengan tali kenyal itu sentiasa kekal pada panjang
yang sama. Potong pita detik yang didapati kepada keratan 10 detik dan bina sebuah carta pita detik untuk
menentukan pecutan.
Pull the trolley down the friction-compensated runway with the elastic string always maintained at the same length. Cut
the ticker tape obtained into 10-tick strips and construct a ticker-tape chart to determine the acceleration.

51
  Fizik  Tingkatan 4  Bab 2 Daya dan Gerakan

3. Ulangi eksperimen ini dengan menggunakan:


Repeat the experiment using:
(a) 2 troli bertindih untuk mewakili dua unit jisim,
2 stacked trolleys to represent two units of mass,
(b) 3 troli bertindih untuk mewakili tiga unit jisim.
3 stacked trolleys to represent three units of mass.

Penjadualan data / Tabulation of data:


BAB

Pecutan, a
Jisim, m 1 Acceleration, a
Mass, m
(kg–1)
m (m s–2)

.
2

hd
1 troli / trolley 1.00
2 troli / trolleys 0.50

.B
3 troli / trolleys 0.33

Analisa data / Analysis of data:

dn
a a

iS
0 m
ng 0
1

m
(a) Graf a melawan m 1
(b) Graf a melawan
la
Graph of a against m m
1
Graph of a against m
Pe

Kesimpulan / Conclusion:
Pecutan suatu objek berkurang apabila jisim objek itu bertambah. Hipotesis diterima.
The acceleration of an object decreases when the mass of the object increases. The hypothesis is accepted.
n
ita

Hubungan antara pecutan dengan daya untuk jisim tetap


Aktiviti 2.4 Relationship between acceleration and force with a fixed mass
rb

Situasi / Situation:
ne
Pe

Seorang porter di stesen kereta api menarik sebuah kereta sorong. Daya bersih yang bertindak pada kereta sorong
itu menyebabkannya memecut. Apakah yang berlaku kepada pecutan kereta sorong itu jika dua orang porter yang
sama menarik kereta sorong itu?
A porter at a train station pulls a cart. The net force acting on the cart causes it to accelerate. What happens to the acceleration of
the cart if two porters of the same mass pull the cart?

52
  Fizik  Tingkatan 4  Bab 2 Daya dan Gerakan 

Pemerhatian / Observation:
Pecutan kereta sorong yang ditarik oleh dua orang porter adalah lebih tinggi daripada seorang porter.
The acceleration of the cart pulled by two porters is higher than the single porter.

Inferens / Inference:
Pecutan sesuatu objek bergantung kepada daya yang bertindak padanya.
The acceleration of an object depends on the force acting on it.

BAB
Hipotesis / Hypothesis:
Pecutan suatu objek bertambah apabila daya bersih yang bertindak pada objek bertambah.

.
The acceleration of an object increases when the net force acting on the object increases. 2

hd
Tujuan / Aim:
Untuk menyiasat hubungan antara pecutan dengan daya untuk jisim tetap.

.B
To investigate the relationship between acceleration and force with a fixed mass.

Pemboleh ubah / Variables:

dn
(a) Pemboleh ubah dimanipulasi: Daya yang bertindak pada troli
Manipulated variable: Force acting on the trolley
(b) Pemboleh ubah bergerak balas: Pecutan troli

iS
Responding variable: Acceleration of the trolley
(c) Pemboleh ubah dimalarkan: Jisim troli
Constant variable: Mass of the trolley
ng
Bahan / Materials:
Pita detik, tiga tali kenyal dan pita selofan.
la
Ticker tape, three elastic strings and cellophane tape

Radas / Apparatus:
Pe

Bekalan kuasa 12 V a.u., jangka masa detik, troli, landasan dan bongkah kayu
12 V a.c. power supply, ticker timer, trolley, runway and wooden block

Susunan Radas / Apparatus set-up:


n

Jangka masa
ita

Pita detik detik Tali kenyal


Ticker tape Ticker timer Elastic string

Landasan terpampas
rb

geseran
Friction compensated runway
ne

Bongkah kayu
Wooden block
Pe

Bekalan kuasa 12 V a.u


12 V a.c. power supply

Prosedur / Procedure:
1. Hidupkan jangka masa detik. Gantung seutas tali kenyal pada troli dan tarik tali kenyal itu dari hujung troli.
Hal ini mewakili satu unit daya, F.
Switch on the ticker timer. Hook an elastic string onto the trolley and stretch it until the end of the trolley. This will represent
a unit of force, F.

53
  Fizik  Tingkatan 4  Bab 2 Daya dan Gerakan

2. Tarik troli itu menuruni landasan terpampas geseran dengan tali kenyal itu sentiasa kekal pada panjang
yang sama. Potong pita detik yang didapati kepada keratan 10 detik dan bina sebuah carta pita detik untuk
menentukan pecutan.
Pull the trolley down the friction-compensated runway with the elastic string always maintained at the same length. Cut
the ticker tape obtained into 10 tick strips and construct a ticker tape chart to determine the acceleration.

3. Ulangi eksperimen ini dengan menggunakan:


Repeat the experiment using
BAB

(a) 2 utas tali kenyal untuk mewakili dua unit daya,


2 pieces of elastic string to represent two units of force,
(b) 3 utas tali kenyal untuk mewakili tiga unit daya.

.
3 pieces of elastic string to represent three units of force.
2

hd
Penjadualan data / Tabulation of data:

Bilangan tali kenyal/Daya, Pecutan, a

.B
Number of elastic string/Force, Acceleration, a
F(N) (m s–2)
1

dn
2
3

iS
Analisa data / Analysis of data:
a
ng
Graf pecutan melawan daya
la
Graph of acceleration against force
Pe

Kesimpulan / Conclusion:
Pecutan suatu objek bertambah apabila daya bersih yang bertindak pada objek bertambah. Hipotesis diterima.
n

The acceleration of an object increases when the net force acting on the object increases. The hypothesis is accepted.
ita

Hubungan antara Daya, Jisim dan Pecutan


Relationship between Force, Mass and Acceleration
rb

1. Rajah 12 menunjukkan suatu daya, F bertindak pada satu objek berjisim, m. Objek itu bergerak dengan suatu
pecutan, a.
ne

Diagram 12 shows a force, F acting on an object of mass, m. The object moves with an acceleration, a.

a
Pe

m
F


Rajah 12 / Diagram 12

2. Hukum Gerakan Newton Kedua menyatakan bahawa kadar perubahan momentum adalah berkadar
terus dengan daya yang bertindak ke atas suatu objek pada arah yang sama dengan arah tindakan daya.
Newton's Second Law of Motion states that the rate of momentum change is directly proportional to the force acting
on an object in the same direction as the direction of the force.

54
  Fizik  Tingkatan 4  Bab 2 Daya dan Gerakan 

F ∝ ma
F ∝ m (v – u) Ungkapan Hukum Gerakan Newton Kedua
t Expression of Newton's Second Law of Motion
F ∝ mv – mu
t
F ∝ ma
F = kma, k ialah pemalar / k is constant.

BAB
Dengan menggantikan nilai-nilai ke dalam F = kma.
By replacing the values into F = kma
1 N = k × 1 kg × 1 m s–2

.
k=1 2

hd
Maka / Hence, F = ma
iaitu / that is

.B
F = daya bersih atau daya paduan = daya ke depan – daya ke belakang
net force or resultant force = forward force - backward force
m = jumlah jisim sistem tersebut

dn
the total mass of the system
a = pecutan sistem tersebut
acceleration of the system

iS
Menyelesaikan Masalah Melibatkan Rumus F = ma
Solve Problems Involving the Equation F = ma
ng
Contoh 15
la

1. Cari pecutan objek-objek itu. (a) Diberi / Given: F = 10 N, m = 5 kg, a = ?


Pe

Find the acceleration of the objects. F = ma


(a) (b) 10 = 5a
F = 10 N F = 45 N a = 2 m s–2
5 kg 3 kg
n

FR = 15 N (b) Diberi / Given:


F = 45 N, FR = 15 N, m = 3 kg, a = ?
ita

Permukaan tanpa geseran


Frictionless surface
F – FR = ma
45 – 15 = 3a
rb

a = 10 m s–2
ne

2. Cari nilai F. Diberi / Given:


Find the value of F.    FR = 5 N, m = 10 kg, a = 4 m s-2, F = ?
a = 4 m s–2
F – FR = ma
Pe

10 kg F
FR = 5 N F – 5 = (10)(4)
F = 45 N

3. Hitung nilai m. Diberi / Given:


Calculate the value of m. F = 60 N, FR = 10 N, a = 2 m s–2, m = ?
a = 2 m s–2
F – FR = ma
m
FR = 10 N F = 60 N 60 – 10 = (m)(2)
m = 25 kg

55
  Fizik  Tingkatan 4  Bab 2 Daya dan Gerakan

Tugasan 6
1. Suatu objek berjisim 10 kg memecut daripada rehat kepada Diberi / Given:
5 m s–1 dalam masa 2 s. Berapakah daya bersih yang bertindak m = 10 kg, u = 0, v = 5 m s–1, t = 2 s, F = ?
pada objek itu? v–u
F = ma = m 
t 

A 10 kg object accelerates from rest to 5 m s–1 in 2 s. What is the net force
BAB

acting on the object? 5–0


= 10 
2 
= 25 N

.
2 2. Suatu daya bersih 10 N bertindak pada satu objek pegun berjisim Diberi / Given:

hd
2 kg. Hitung jarak yang dilalui oleh objek itu dalam masa 5 s. F = 10 N, m = 2 kg, u = 0, t = 5 s, s = ?
A net force of 10 N acts on a 2 kg stationary object. Calculate the distance 10
travelled by the object in 5 s. F = ma, a = = 5 m s–2
2

.B
Guna / Use :
1
s = ut + at2
2
1

dn
= 0 + (5)(52) = 62.5 m
2

3. Suatu objek berjisim 5 kg bergerak dengan halaju seragam Halaju seragam bermaksud daya-daya dalam

iS
apabila ditarik dengan daya mengufuk sebanyak 5 N di atas satu keseimbangan,
permukaan satah. Berapakah pecutan objek itu apabila ditarik Constant velocity means forces in equilibrium,
dengan daya 25 N? FR = 5 N, m = 5 kg, F = 25 N, a = ?
An object with a mass of 5 kg moves at a constant velocity when
ng F – FR = ma
it is pulled by a horizontal force of 5 N on a level surface. What is
25 – 5 = 5a
the acceleration of the object if the object is pulled with a force of 25 N?
a = 4 m s–2
la
Pe

Impuls dan Daya Impuls


2.7 Impulse and Impulsive Force

besar jangka masa yang singkat


n

1. Daya impuls ialah suatu daya yang yang bertindak dalam semasa
perlanggaran atau letupan.
ita

An impulsive force is a large force that acts over a short period of time during a collision or an explosion.

2. Daripada rumus / From the formula, F = ma


rb

F = m  v – u 
t
Daya adalah / Force is
ne

(a) berkadar secara langsung dengan perubahan momentum


directly proportional
Pe

to the change in momentum

(b) berkadar songsang dengan masa


inversely proportional to time

3. Impuls ialah perubahan momentum . / Impulse is the change of momentum .

Impuls / Impulse, J = mv – mu F = Daya yang dikenakan / Force exerted


= Ft t = Masa impak / Time of impact

56
  Fizik  Tingkatan 4  Bab 2 Daya dan Gerakan 

4. Unit S.I. bagi impuls ialah newton saat (N s) .

S.I. unit for impulse is newton second (N s) .

Kesan Masa ke atas Daya Impuls


Effect of Time on the Impulsive Force

BAB
1. Sebiji gelas yang dijatuhkan di atas lantai simen pasti akan pecah. Oleh sebab permukaan lantai itu keras,
masa hentaman adalah sangat pendek . Dengan itu, daya impuls yang bertindak ke atas gelas adalah

.
sangat besar . 2

hd
A glass that is dropped on a cement floor will definitely break. As the surface of the floor is hard, the time of impact is
very short . Therefore, the impulsive force that acts on the glass is very large .

.B
2. Jika gelas yang sama dijatuhkan di atas satu alas getah yang lembut, kemungkinan gelas kaca itu tidak pecah.
Permukaan lembut alas getah memanjangkan masa hentaman seterusnya mengurangkan daya impuls

dn
yang bertindak pada gelas tersebut.
If an identical glass is dropped on a soft rubber mat, it might not break. The soft surface of the rubber mat lengthens/extends
reducing

iS
the impact time, thus the impulsive force acting on the glass.

3. Daripada rumus / From the formula, F = mv – mu


t ng
Jika perubahan momentum malar, magnitud daya impuls adalah berkadar songsang dengan masa
hentaman, maka
If the change of momentum is constant, the magnitude of the impulsive force is inversely proportional to the time of
la
impact, then

1
F ∝ t
Pe

Mengurangkan Daya Impuls dengan Memanjangkan Masa Hentaman


Reducing Impulsive Force by Extending Impact Time
n
ita

Situasi Penerangan
Situation Explanation
rb

Lompat tinggi (a) Apabila atlet jatuh di atas tilam, tilam yang tebal dan lembut itu
High jump
memanjangkan masa hentaman. Daya impuls yang bertindak ke atas
ne

atlet itu dikurangkan dan berkemungkinan kecil untuk dia


mengalami sebarang kecederaan. 3
Pe

When the athelete falls on a mattress, the thick and soft mattress extends the
impact time. The impulsive force acting on the athlete is reduced and he is
less likely to suffer any injuries.

57
  Fizik  Tingkatan 4  Bab 2 Daya dan Gerakan

Pendaratan ahli payung terjun membengkokkan memanjangkan masa


Parachutist landing (b) Ahli payung terjun kaki untuk
hentaman kakinya dengan tanah. 3

bends extend the impact time


The parachutist his legs to of his legs with
the ground.
BAB

.
2 Pembungkusan perkakas dengan (c) Polistirena digunakan semasa membungkus barangan mudah pecah. Semasa

hd
polistirena memanjangkan masa
Packing of appliance with polystyrene
perlanggaran, bahan polistirena yang lembut akan
hentaman . Hal ini akan mengurangkan daya impuls ke atas barangan

.B
Polistirena mencegah
Polystyrene tersebut, seterusnya daripada rosak. 3

Polystyrene is used in packing fragile goods. During impact, the soft polystyrene foam
extend the impact time reduce

dn
will  . This will the impulsive force
acting on the fragile goods, thus preventing them from breaking.

iS
Meningkatkan Daya Impuls dengan Mengurangkan Masa Hentaman
Increasing Impulsive Force by Reducing Impact Time ng
Pelantak cerucuk digunakan untuk memacu cerucuk ke dalam tanah
untuk menyediakan sokongan asas bangunan atau struktur lain. Tukul
la
Tukul cerucuk cerucuk diangkat pada ketinggian tertentu dan dilepaskan menghentam cerucuk. Halaju
Pile hammer
yang tinggi serta perlanggaran antara tukul cerucuk dengan cerucuk berlaku
Pe

dalam masa yang sangat singkat , menghasilkan satu daya impuls yang besar
Cerucuk
Pile yang memacu cerucuk ke dalam tanah.
A pile driver is used to drive piles
into soil to provide foundation support for buildings
n

or other structures. The pile hammer will be raised to a certain height and then released onto the piles. The high
ita

velocity of pile hammer and the collision between the pile hammer and the piles happen in a very short time , producing a

large impulsive force that drives the piles to the ground.


rb

Menyelesaikan Masalah Melibatkan Impuls dan Daya Impuls


ne

Solving Problems Involving Impulse and Impulsive Forces

1. Seorang pemain hoki menggunakan sebatang kayu hoki untuk memukul sebiji bola hoki, menyebabkan bola
Pe

itu bergerak dengan halaju 30 m s–1. Jika jisim bola hoki itu ialah 100 g dan masa hentaman bola hoki dengan
kayu hoki ialah 5 × 10–3 s, hitung daya impuls yang dikenakan oleh kayu hoki tersebut.
A hockey player uses a hockey stick to hit a hockey ball, causing the ball to travel with a velocity of 30 m s–1. If the mass of
the hockey ball is 100 g and the time of impact between the hockey stick and the ball is 5 × 10–3 s, calculate the impulsive
force exerted by the hockey ball.

F=m (v –t u)
= 0.1(
0.005 )
30 – 0
= 600 N

58
  Fizik  Tingkatan 4  Bab 2 Daya dan Gerakan 

2. Seorang budak lelaki berjisim 60 kg melompat melepasi sebuah pagar. Halaju budak lelaki itu sebelum mendarat
di atas tanah ialah 10 m s–1. Berapakah daya impuls pada kaki budak lelaki itu jika dia
A boy with a mass of 60 kg jumps over a fence. His velocity just before landing on the ground is 10 m s–1. What is the
impulsive force on the boy’s legs if he

(a) mengambil masa 0.5 s untuk berhenti, (b) membengkokkan kakinya dan berhenti dalam
takes 0.5 s to stop, masa 2.5 s?
bends his legs and stops in 2.5 s?
( )v–u
(v –t u)

BAB
F = m
t F = m

= 60( 0.5 ) = –1 200 N


0 – 10
= 60( 2.5 ) = –240 N
0 – 10

.
2

hd
Tugasan 7

.B
1. Sebuah troli dengan jisim 500 g berada dalam keadaan rehat di atas suatu permukaan licin. Troli itu kemudian dikenakan
impuls 5 N s. Berapakah halaju troli selepas hentaman itu?

dn
A trolley with a mass of 500 g is at rest on a smooth surface. The trolley is then given a horizontal impulse of 5 N s. What is the velocity of
the trolley after the impact?
Ft = m(v – u)

iS
5 = 0.5(v – 0)
v = 10 m s–1 ng
2. Satu impuls mengufuk 500 N s dikenakan pada sebuah troli pegun yang berjisim 2 kg. Berapakah halaju troli selepas
hentaman itu?
la
A horizontal impulse of 500 N s is exerted on a stationary trolley with a mass of 2 kg. What is the velocity of the trolley after the impact?
Ft = m(v – u)
Pe

500 = 2 (v – 0)
v = 250 m s–1
n

Berat
2.8
ita

Weight

1. Berat suatu objek ialah daya graviti yang bertindak ke atas objek itu.
rb

Weight of an object is the gravitational force exerted on the object.


ne

2. Berat suatu objek sama dengan magnitud daya graviti yang bertindak pada objek itu oleh Bumi. Maka,
unitnya ialah newton (N) .
Weight of an object equals the magnitude of the gravitational force acted by the Earth on the object. Therefore, its unit is
Pe

newton (N) .
• Daya graviti yang bertindak ke atas
3. Rajah 13 menunjukkan sebuah epal epal ialah beratnya, W.
mengalami jatuh bebas dari sebuah The gravitational force acting on the apple is
pokok ke Bumi. its weight, W.
Diagram 13 below shows an apple
experienced free fall from a tree to the • Pecutan epal itu ialah pecutan graviti, g
ground. The acceleration of the apple is the
acceleration of gravity, g.

Rajah 13 / Diagram 13

59
  Fizik  Tingkatan 4  Bab 2 Daya dan Gerakan

4. Untuk suatu objek yang jisimnya, m, beratnya, W ialah hasil darab jisim, m dengan pecutan graviti, g.
For an object with mass, m, its weight, W is the product of the mass, m and gravitational acceleration, g.

F = ma

W = mg
5. Unit SI untuk berat ialah Newton (N).
BAB

The SI unit for weight is newton (N).


6. Jadual di bawah menunjukkan perbezaan antara jisim dengan berat.
Table below shows the differences of mass and weight.

.
2

hd
Jisim / Mass Berat / Weight

Takrif: Kuantiti jirim dalam objek Takrif: Daya graviti pada objek

.B
Definition: Quantity of matter in the object Definition: Gravitational force on the objects

Kuantiti skalar Kuantiti vektor


Scalar Vector

dn
quantity quantity

Unit SI: kilogram (kg) Unit SI: Newton (N)

iS
SI unit: kilogram (kg) SI unit: Newton (N)

Nilainya adalah malar di mana-mana tempat. Nilainya berubah dengan kekuatan medan
constant graviti, g.
Its value is everywhere.
ng
Its value changes with gravitational field strength, g.
la
Contoh 18
Pe

1. Kekuatan medan graviti Bumi ialah 9.8 N kg–1 dan kekuatan medan graviti Bulan ialah 1.6 N kg–1. Jika
seorang lelaki mempunyai berat 120 N di Bulan, apakah beratnya di Bumi?
The gravitational field strength of the Earth is 9.8 N kg–1 and the gravitational field strength of the Moon is
1.6 N kg–1. If a man weighs 120 N on the Moon, what is his weight on the Earth?
n

Penyelesaian / Solution:
ita

Di Bulan / On the Moon, F = mgBulan Di Bumi / On Earth, F = mgBumi


120 = m × 1.6, F = 75 × 9.8 = 735 N
rb

m = 120 = 75 kg
1.6
ne

2. Seorang angkasawan menimbang beberapa biji batu dari sebuah planet X dengan neraca spring dan
mencatatkan bacaan neraca spring sebagai 80 N. Apabila kembali ke Bumi, dia menimbang semula
spesimen batu yang sama dengan menggunakan neraca spring yang sama. Dia mendapati bahawa
Pe

bacaan neraca spring sekarang ialah 400 N. Jika kekuatan tarikan graviti, g di Bumi ialah 10 N kg–1,
tentukan kekuatan tarikan graviti planet X.
An astronaut weighed some rocks from a planet X with spring balance and the spring balance reading is recorded
as 80 N. Upon returning to the Earth, he reconsider the same rock specimen by using the same spring balance. He
finds that the reading of the spring balance is now 400 N. If the force of the gravitational, g on the Earth is 10 N kg–1,
determine the gravity pull force of planet X.

Penyelesaian / Solution:
W = mg Di bumi / On the Earth, 400 = m(10), m = 40 kg
Di planet X / On the planet, W = mg, maka / hence 80 = 40(g), g = 2 N kg–1

60
  Fizik  Tingkatan 4  Bab 2 Daya dan Gerakan 

PRAKTIS SPM 2
Soalan Objektif

1. Rajah 1 menunjukkan jalan yang diambil oleh Ali 3. Rajah 3 menunjukkan sekeping kadbod ditarik

BAB
semasa cuti sekolah. Perjalanan Ali bermula dari Pulau dengan cepat, duit syiling 50 sen itu akan jatuh ke
2016 2015
Pinang ke Baling dan berakhir di Kuala Lumpur. dalam gelas. Inersia duit syiling itu mengekalkan
Diagram 1 shows the route taken by Ali during school holiday. keadaaan rehatnya.

.
His journey starts from Pulau Pinang to Baling and ended at Diagram 3 shows a cardboard is given a quick pull, the 50 cent 2

hd
Kuala Lumpur. coin will fall into the glass. The inertia of the coin maintains its
state at rest.
Pulau

.B
Pinang 300 km

Baling
500 km 750 km

dn

Kuala Rajah 3 / Diagram 3
Lumpur Situasi yang manakah mengurangkan inersia?

iS
2014
Rajah 1 / Diagram 1 Which situation decreases the inertia?
A Gunakan kadbod yang lebih licin.
Berapakah sesaran Ali? Use a smoother cardboard.
What is the displacement of Ali?
ng
B Gunakan kepingan kadbod yang besar.
A 300 km Use a bigger cardboard.
B 500 km C Menarik kadbod dengan daya yang lebih kecil.
la
C 750 km Pull the cardboard with smaller force.
D 1 550 km D Gunakan duit syiling lima sen.
Pe

Use a five cent coin.


2. Rajah 2 menunjukkan satu graf halaju-masa bagi
gerakan satu objek. 4. Sebuah troli berjisim 75 kg bergerak pada halaju 5 m s–1
2014
Diagram 2 shows a velocity-time graph for a motion of an berlanggar dengan sebuah troli berjisim 25 kg yang
2015
n

object. berada dalam keadaan rehat. Selepas perlanggaran


itu, kedua-dua troli bergerak bersama-sama. Apakah
ita

Halaju (m s–1)
Velocity (m s–1) halaju kedua-dua troli itu selepas perlanggaran?
A 75 kg trolley travelling at 5 m s–1 collide with a 25 kg trolley
60
which is at rest. After the collision, both the trolleys move
rb

50
together. What is their velocity after the collision?
40
A 2.50 m s–1 C 3.75 m s–1
ne

30 B 3.55 m s–1 D 5.00 m s–1


20
10 5. Rajah 4 menunjukkan dua penunggang basikal
Pe

0 Masa (s) dengan jisim yang sama


5 10 15 20 25 30 2014
Time (s) Diagram 4 shows two cyclists of same mass.
Rajah 2 / Diagram 2
Berapakah pecutan objek itu?
What is the acceleration of the object?
A 0.667 m s–2 Bukit
Hill
B 1.333 m s–2
Penunggang 1 Penunggang 2
C 2.000 m s–2 Cyclist 1 Cyclist 2
D 2.333 m s–2 Rajah 4 / Diagram 4

61
  Fizik  Tingkatan 4  Bab 2 Daya dan Gerakan

Mengapakah inersia bagi penunggang 1 adalah lebih Daya impuls dihasilkan apabila
besar daripada penunggang 2? Impulsive force is produced when
Why the inertia of the cyclist 2 is bigger than the cyclist 1? A penukul dihayun pada halaju yang tinggi.
A Halaju penunggang 2 lebih besar daripada the hammer is swung at high velocity.
penunggang 1. B penukul dihayun ke bawah.
The velocity of the cyclist 2 is bigger than the cyclist 1. the hammer is moving downwards.
B Jisim penunggang 1 lebih besar daripada C penukul menghentam paku.
penunggang 2. the hammer hits the nail.
BAB

The mass of the cyclist 2 is bigger than the cyclist 1. D paku bergerak ke dalam papan.
C Tinggi penunggang 2 lebih besar daripada the nail is driven into the wood.
penunggang 1.

.
2 8. Situasi yang manakah menunjukkan momentum

hd
The height of the cyclist 2 is more than the cyclist 1.
D Saiz penunggang 2 lebih besar daripada tidak terabadi?
2016
Which situation shows the momentum is not conserved?
penunggang 1.
A Sentakan senapang apabila sebiji peluru

.B
The size of the cyclist 2 is more than the cyclist 1.
ditembak.
6. Diagram 5 shows a high jumper when they hit the soft The rifle recoiled backward when a bullet is fired from a
mattress. rifle.

dn
2015
Rajah 5 menunjukkan pelompat tinggi mendarat pada tilam B Tukul cerucuk digunakan untuk memacu cerucuk
lembut. ke dalam tanah.
The pile hammer is used to drive the pile into the ground.

iS
C Sebuah roket dilancarkan dari stesen pelancar.
A rocket is launched from the launching station.
ng D Sebuah kereta terlanggar sepohon pokok yang
besar dan berhenti serta merta.
A car hit on a big tree and stops immediately.
la
9. Rajah 7 di bawah menunjukkan seorang pemain
badminton menerima sebiji bulu tangkis berjisim
2015
Pe

Rajah 5 / Diagram 5 50 g yang bergerak secara mengufuk pada 40.0 m s–1.


Apakah konsep fizik yang terlibat dalam situasi di Pemain itu mengembalikan bulu tangkis itu pada
atas? 45.0 m s–1 dalam arah yang bertentangan.
What is the physics concept involved in the above situation? Diagram 7 shows a badminton player receiving a shot
n

A Daya impuls with a shuttlecock of mass 50 g travelling horizontally at


Impulsive force 40.0 m s–1. The player returns the shot at 45.0 m s–1 in the
ita

B Tekanan opposite direction.


Pressure
40 m s–1 45 m s–1
C Keseimbangan daya
rb

Force in equilibrium
D Pemindahan daya
ne

Transmission of force Bulu tangkis


Shuttlecock
7. Rajah 6 menunjukkan sebuah penukul menghentam
sebatang paku pada halaju yang tinggi.
Pe

2015
Diagram 6 shows a hammer hits a nail with high velocity. Rajah 7 / Diagram 7
Penukul Apakah impuls yang bertindak ke atas bulu tangkis?
Hammer
What is the impulse acting on the shuttlecock?
A 2.55 kg m s–1
B 3.55 kg m s–1
C 3.75 kg m s–1
Paku
Nail D 4.25 kg m s–1

Rajah 6 / Diagram 6

62
  Fizik  Tingkatan 4  Bab 2 Daya dan Gerakan 

10. Rajah 8 menunjukkan pita detik bagi gerakan suatu A imbalance force
objek. ketidakseimbangan daya
2017
Diagram 8 shows a ticker tape for a motion of an object. B daya geseran yang kecil
small frictional force
C terdapat daya bersih
the existence of net force
Arah gerakan D the forces are in equilibrium
Direction of motion
daya-daya dalam keseimbangan

BAB
Rajah 8 / Diagram 8
12. Rajah 10 menunjukkan daya-daya yang bertindak
Pernyataan manakah menerangkan situasi tersebut? ke atas sebuah kereta. Jisim kereta tersebut adalah

.
2018
Which statement describes the situation? 1500 kg. 2

hd
A Objek itu memecut kemudian halaju seragam. Diagram 10 shows the forces acting on a car. The mass of the
The object accelerates then constant velocity. car is 1500 kg.
B Objek itu menyahpecut kemudian halaju seragam.

.B
The object decelerates then constant velocity. 1000 N
C Objek itu bergerak dengan halaju seragam 250 N
kemudian memecut.

dn
The object moves with constant velocity then accelerates.
D Objek itu bergerak dengan halaju seragam Rajah 10 / Diagram 10
kemudian menyahpecut.
Berapakah pecutan kereta tersebut?

iS
The object moves with constant velocity then decelerates.
What is the acceleration of the car?
11. Rajah 9 menunjukkan Ali yang bergerak dengan A 2 m s–2
halaju seragam di atas papan luncur. B 1 m s–2
2017
ng C 0.5 m s–2
Figure 9 shows Ali moving with constant velocity on a
skateboard. D 0.2 m s–2
la
13. Berat Halim di Bumi ialah 600 N. Apakah yang akan
berlaku kepada jisimnya di Bulan?
Pe

2018
Halim’s weight on the Earth is 600 N. What will happen to his
mass on the Moon?
A Bertambah
Increased
n

B Decreased
Berkurang
ita

C Unchanged
Tidak berubah
Rajah 9 / Diagram 9
rb

D Becomes zero
Pergerakan Ali disebabkan oleh Menjadi sifar
Ali's movement is caused by
ne
Pe

63
  Fizik  Tingkatan 4  Bab 2 Daya dan Gerakan

Soalan Struktur

Bahagian A

Situasi 1 Situasi 2
1. Rajah 1 menunjukkan dua biji telur yang sama masing-masing Situation 1 Situation 2
dijatuhkan ke atas simen dan span. Telur yang dijatuhkan di atas
permukaan simen pecah selepas hentaman tetapi tidak bagi telur di
atas span. Halaju telur sejurus sebelum mengenai permukaan ialah
BAB

4 m s–1.
Diagram 1 shows two similar eggs being dropped onto cement and sponge
respectively. The egg dropped on the cement surface cracked after an impact

.
2

hd
but not the one on the sponge. The velocity of the eggs just before reaching the
surfaces is 4 m s–1.
Simen Span
Cement Sponge

.B
(a) Namakan daya yang terlibat semasa hentaman dalam Situasi 1.
Name the force involved during the impact in Situation 1. Rajah 1 / Diagram 1

Daya impuls / Impulsive force

dn
[1 markah / mark]

(b) Beri satu sebab telur itu pecah dalam Situasi 1 tetapi tidak pecah dalam Situasi 2.

iS
Give one reason why the egg cracked in Situation 1 but not in Situation 2.
KBAT
Dalam Situasi 1, masa hentaman adalah singkat disebabkan oleh permukaan keras simen, maka daya impuls
ng
adalah besar dan telur akan pecah. Dalam Situasi 2, masa hentaman adalah lebih lama disebabkan oleh permukaan
lembut span, maka daya impuls lebih kecil dan telur tidak akan pecah.
la
In Situation 1, the impact time is short because of the hard surface of the cement, thus the impulsive force is big and the egg will
break. In Situation 2, the impact time is longer because of the soft surface of the sponge, thus the impulsive force is smaller and the
Pe

egg will not break.


[2 markah / marks]
(c) Jisim telur itu ialah 0.02 kg. / The mass of the egg is 0.02 kg.
n

(i) Hitung daya yang bertindak ke atas telur jika telur itu berhenti 0.05 s selepas hentaman dengan simen.
ita

Calculate the force acting on the egg if the egg stops 0.05 s after the impact with the cement.

0.02(4 – 0)
F = = 1.6 N
rb

0.05
[2 markah / marks]
ne

(ii) Hitung daya yang bertindak ke atas telur jika telur itu berhenti 2.0 s selepas hentaman dengan span.
Calculate the force acting on the egg if the egg stops 2.0 s after the impact with the sponge.

0.02(4 – 0)
Pe

F = 2 = 0.04 N
[2 markah / marks]

2. Rajah 2 menunjukkan satu keratan pita detik dengan 7 detik yang diperoleh daripada satu eksperimen.
Diagram 2 shows a ticker tape with 7 ticks obtained from an experiment.
2016
A B C D E F G

Arah gerakan
Direction of motion
Rajah 2 / Diagram 2

64
  Fizik  Tingkatan 4  Bab 2 Daya dan Gerakan 

(a) Satu detik adalah masa yang diambil dari A ke B .


One tick is the time taken from A to B . [1 markah / mark]
(b) Lengkapkan ayat berikut dengan menandakan (✓) pada jawapan yang betul dalam kotak yang disediakan.
Complete the following sentence by ticking (✓) at the correct answer in the box provided.
Pita detik dapat dianalisis untuk menentukan
The ticker tape can be analysed to determine

BAB
halaju suatu objek / velocity of an object ✓

.
pecutan suatu objek / acceleration of an object ✓ 2

hd
[2 markah / marks]
(c) (i) Apakah jenis gerakan yang ditunjukkan oleh pita detik dalam Rajah 2?

.B
What is the type of motion shown by the ticker tape in Diagram 2?
Pecutan seragam / Constant acceleration
[1 markah / mark]

dn
(ii) Beri satu sebab bagi jawapan anda dalam 2(c) (i).
Give one reason for your answer in 2(c)(i).

iS
Jumlah halaju bertambah kerana jarak detik bertambah.
Velocity increase because the distance for ticks increases.
ng [1 markah / mark]

3. Rajah 3.1 menunjukkan seorang pekerja di stesen kereta api menarik sebuah kereta sorong sejauh 10 m dalam masa
la
20 s. Rajah 3.2 menunjukkan dua orang pekerja menarik sebuah kereta sorong yang sama sejauh 10 m dalam masa 10 s.
2016
Diagram 3.1 shows a porter at the train station is pulling a cart to 10 m in 20 s. Diagram 3.2 shows two porters are pulling the same cart to
Pe

10 m in 10 s.
n
ita


rb

Rajah 3.1 / Diagram 3.1              Rajah 3.2 / Diagram 3.2

(a) Namakan daya yang bertindak di antara tayar kereta sorong dengan jalan.
ne

Name the force which acted between the cart tyre and the road.
Daya geseran / Frictional force
Pe

[1 markah / mark]
(b) Perhatikan Rajah 3.1 dan Rajah 3.2.
Observe Diagram 3.1 and Diagram 3.2.
(i) Bandingkan daya yang dikenakan oleh pekerja yang menyebabkan kereta sorong bergerak ke hadapan.
Compare the forces applied by the porters that cause the cart to move forward.
Daya yang dikenakan pada Rajah 3.2 lebih besar daripada daya yang dikenakan pada Rajah 3.1.
The force applied on Diagram 3.2 is greater than the force applied on Diagram 3.1.
[1 markah / mark]

65
  Fizik  Tingkatan 4  Bab 2 Daya dan Gerakan

(ii) Bandingkan masa yang diambil untuk bergerak pada jarak yang sama.
Compare the time taken to travel the same distance.
Masa yang diambil pada Rajah 3.2 adalah lebih pendek daripada Rajah 3.1.
The time taken in Diagram 3.2 is shorter than Diagram 3.1.
[1 markah / mark]
(iii) Berdasarkan jawapan anda di 3(b)(ii), bandingkan pecutan kereta sorong itu.
BAB

Based on the answer in 3(b)(ii), compare the acceleration of the cart.


Pecutan pada Rajah 3.2 adalah lebih besar daripada Rajah 3.1.

.
2 Acceleration in Diagram 3.2 is greater than Diagram 3.1.

hd
[1 markah / mark]
(iv) Hubung kait daya yang dikenakan oleh pekerja dengan pecutan kereta sorong itu.

.B
Relate the forces applied by the porters to the acceleration of cart.
Semakin besar daya yang dikenakan semakin besar pecutan.
The greater the force applied the greater is the acceleration.

dn
[1 markah / mark]
(c) Namakan hukum fizik yang terlibat di 3(b)(iv).

iS
Name the physics law involved in 3(b)(iv).
Hukum Gerakan Newton Kedua. / Newton’s Second Law of Motion.
[1 markah / mark]
ng
Bahagian B
la
4. Rajah 4.1 menunjukkan dua pelompat tinggi, Ali dan Ahmad yang mempunyai jisim yang sama melompat melepasi
galah yang mempunyai ketinggian yang sama. Rajah 4.2 menunjukkan keadaan dua pelompat tinggi apabila mereka
Pe

mendarat pada tilam keras dan tilam lembut masing-masing. Dapat diperhatikan bahawa Ali berhenti dengan lebih
cepat apabila dia mendarat pada tilam keras berbanding dengan Ahmad.
Diagram 4.1 shows two high jumpers, Ali and Ahmad, of equal mass jumping over the pole of the same height. Diagram 4.2 shows the
state of the high jumpers when they hit the hard mattress and soft mattress respectively. It is observed that Ali comes to a stop more quickly
n

when he lands on the hard mattress compared to Ahmad.


ita

Ali Ahmad
rb
ne

Tilam keras / Hard mattress Tilam lembut / Soft mattress


Pe

Rajah 4.1 / Diagram 4.1

Tilam keras / Hard mattress Tilam lembut / Soft mattress


Rajah 4.2 / Diagram 4.2

66
  Fizik  Tingkatan 4  Bab 2 Daya dan Gerakan 

(a) (i) Apakah yang dimaksudkan dengan daya impuls?


What is the meaning of impulsive force? [1 markah / mark]
(ii) Perhatikan Rajah 4.1 dan Rajah 4.2. Bandingkan bentuk permukaan pada tilam keras dan tilam lembut
sebelum dan apabila Ali dan Ahmad mendarat pada tilam mereka masing-masing. Hubung kaitkan
perubahan bentuk pada permukaan tilam untuk menyimpulkan satu konsep fizik.
Observe Diagram 4.1 and Diagram 4.2. Compare the surface of the hard mattress and the surface of the soft mattress before
and when both Ali and Ahmad land on their respective mattresses. Relate the changes in shape of surface of the mattresses to

BAB
deduce a relevant physics concept. [4 markah / marks]
(b) Berdasarkan konsep inertia
Based on the inertia concept

.
2

hd
(i) terangkan mengapa pemandu terhumban ke hadapan apabila kereta yang dipandunya berhenti secara
tiba-tiba,
explain why the driver stumbles forward when the car he drives suddenly stops,

.B
(ii) jelaskan dan terangkan langkah-langkah yang boleh diambil untuk mengelakkan keadaan di atas.
describe and explain the steps that can be taken to prevent the situation above.
[4 markah / marks]

dn
(c) Rajah 4.3 menunjukkan roket air yang diperbuat daripada botol air plastik bersaiz 1.5 liter dengan menambahkan
air dan tekanan udara untuk pelancaran. 5 6
Diagram 4.3 shows a water rocket made of 1.5 liter plastic water bottle by adding water and air pressure for launch.

iS
Kon
Cone
ng
Udara
termampat
Compressed air
la
Air
Water
Pe

Bilah
Fins

Muncung
Nozzle
n


ita

Rajah 4.3 / Diagram 4.3

Objektif utama dalam pelancaran roket ialah untuk mengekalkan roket tersebut di udara selama yang mungkin.
rb

Dengan menggunakan konsep fizik yang sesuai, cadangkan dan terangkan reka bentuk roket dari segi
The main objective of rocket launch is to keep the rocket in the air for as long as possible. Using the appropriate physics concept,
suggest and explain the design of the rocket in terms of
ne

(i) bentuk
shape
Pe

(ii) struktur
structure
(iii) pecutan
acceleration
(iv) daya apungan
buoyant force
(v) kestabilan gerakan
motion stability

[10 markah / marks]

67
  Fizik  Tingkatan 4  Bab 2 Daya dan Gerakan

Bahagian C

5. Rajah 5 menunjukkan sebuah kereta yang dipandu dengan halaju 20 m s–1.


Pemandu itu terlupa bahawa dia telah meletakkan sebuah kotak pada
bumbung kereta itu.
Diagram 5 shows a car being driven at 20 m s–1. The driver has forgotten that he placed a
box on the car roof.
BAB

(a) Apakah yang dimaksudkan dengan momentum?


What is meant by momentum? [1 markah / mark]
(b) Dengan menggunakan konsep momentum dan inersia, terangkan

.
2 Rajah 5 / Diagram 5

hd
momentum kereta itu dan arah pergerakan kotak itu apabila kereta
dihentikan secara tiba-tiba. Berikan satu sebab untuk pilihan arah gerakan anda.
Using the concepts of momentum and inertia, explain the car's momentum and the direction of the box when the car stops suddenly.

.B
Give one reason for your choice of direction. [4 markah / marks]
(c) Jadual 5 menunjukkan sistem brek kereta yang boleh dipasangkan. Anda dikehendaki mangkaji empat jenis ciri
sistem brek kereta dan tentukan jenis kereta yang mempunyai ciri-ciri sistem brek yang paling baik.

dn
Table 5 shows the car's brake system can be installed. You are required to analyse four types of brake system features and determine
the type of car that has the best braking system.

Nisbah luas keratan rentas

iS
Takat didih Ketebalan paip untuk omboh roda dan Jenis brek untuk
cecair brek brek omboh master roda hadapan
The boiling point of Thickness of brake The ratio of cross sectional Type of brake for
the brake fluid
ng
line area for wheel piston and front wheel
master piston
la
Sistem brek P Tinggi Nipis Rendah Dram
Brake system P High Thin Low Drum
Pe

Sistem brek Q Tinggi Tebal Tinggi Cakera


Brake system Q High Thick High Disc

Sistem brek R Rendah Tebal Rendah Cakera


n

Brake system R Low Thick Low Disc


ita

Sistem brek S Rendah Nipis Tinggi Dram


Brake system S Low Thin High Drum
rb

Jadual 5 / Table 5

Terangkan kesesuaian setiap ciri dalam Jadual 5. Berikan sebab-sebab untuk pilihan anda. 5 6
ne

Explain the suitability of each features in Table 5. Give the reasons for your choice.
[10 markah / marks]
Pe

(d) Sebuah motorsikal berjisim 400 kg memecut daripada keadaan rehat ke halaju 108 km j–1 dalam 14 s.
A motorcycle with a mass of 400 kg accelerates from rest to a speed of 108 km h–1 in the 14 s.
(i) Kira pecutan motor itu.
Calculate the acceleration of the motorcycle.
(ii) Kira impuls dan daya impuls yang bertindak pada motorsikal jika motorsikal itu melanggar sebuah tembok
dan berhenti dalam masa 0.1 s.
Calculate the impulse and impulsive force acting on the motorcycle when the motorcycle crashed into a wall and stopped
within 0.1 s.
[5 markah / marks]

68
Fizik  Tingkatan 4 Jawapan 

PRAKTIS SPM 2
JAWAPAN (iv) Daya apungan / Buoyant force
– Daya apungan boleh dibesarkan dengan
Soalan Struktur memasang payung terjun.
Bahagian B Buoyant force can be increased by fixing a parachute.
– Dengan payung terjun yang mempunyai
4. (a) (i) Daya impuls ialah kadar perubahan momentum.
permukaan yang besar akan meningkatkan daya
Impulsive force is the rate of change of momentum.
apungan menjadi lebih besar dengan bertindak

.
(ii) – Apabila Ali mendarat pada tilam yang keras, ke atas udara.

hd
masa impak adalah pendek. With the parachute, the bigger surface area will
When Ali hits the surface of the hard mattress, the time increase buoyant force by acting on the air.
of impact is shortened. (v) Kestabilan gerakan / Motion stability
– Hal ini menyebabkan daya impuls yang besar. – Kestabilan gerakan boleh ditingkatkan dengan

.B
This causes a large impulsive force.
memasang bilah di sekeliling botol air.
– Apabila Ahmad mendarat pada tilam yang The stability of the movement can be improved by
lembut, masa impak adalah panjang. installing the fins around the water bottle.
When Ahmad hits the surface of the soft mattress, the
– Dapat menstabilkan pergerakan dengan

dn
time of impact is lengthened.
mengimbangkan roket supaya dilancarkan
– Hal ini menyebabkan daya impuls yang kecil.
This causes a smaller impulsive force. dengan gerakan stabil.
Can stabilize movement by balancing rocket to launch
– Daya impuls adalah berkadar songsang dengan with stable motion

iS
masa impak.
The impulsive force is inversely proportional to the time
of impact. Bahagian C
(b) (i) Pada mulanya, pemandu bergerak bersama-sama 5. (a) Momentum ialah hasil darab jisim dengan halaju.
dengan kereta pada halaju seragam. Apabila
ng Momentum is the product of mass and velocity.
kereta berhenti secara tiba-tiba, inertia pemandu (b) Apabila brek kereta ditekan, momentum kereta itu
itu cenderung untuk terus bergerak. Hal ini berkurang. Hal ini adalah kerana halaju kereta telah
menyebabkan pemandu terhumban ke hadapan.
la
berkurang. Kotak akan bergerak ke hadapan kereta.
Initially, the driver is moving together with the car at Inersia kotak menyebabkannya terus bergerak ke hadapan.
constant velocity. When the car stops suddenly, the inertia When the car brake is pressed, the momentum of the car
Pe

of the driver tends to continue his forward motion. This decreases. This is because the speed of the car has decreased.
causes the driver to fall forward. The box will move forward of the car. Inertia of the box causes it
(ii) Pemandu harus memakai tali pinggang keledar to continue moving forward.
yang akan menariknya kembali daripada gerakan (c) – Takat didih cecair brek perlu tinggi supaya cecair brek
ke hadapan. tidak mudah meruap dan tidak mudah dimampatkan.
n

The driver should wear seat belt that will pull him back The boiling point of the brake fluid should be high so that the
from the forward motion. brake fluid is not volatile and not easily compressed.
ita

(c) (i) Bentuk / Shape – Ketebalan paip brek perlu tebal supaya paip brek
– Bentuk larus seperti peluru. dapat menahan tekanan yang tinggi.
Streamline shape like a bullet. The thickness of the brake line should be thick in order for
rb

– Bentuk aerodinamik mengurangkan rintangan the brake line to withstand high pressure.
udara. – Nisbah luas keratan rentas untuk omboh roda dan
Aerodynamic shape reduces air resistance. omboh master perlu tinggi supaya daya yang lebih
ne

(ii) Struktur / Structure besar dapat dikenakan pada omboh roda.


– Struktur roket dipenuhi dengan air dan udara Cross-sectional area ratio for wheel piston and master piston
pada tekanan tinggi. should be high so that a greater force can be applied to the
The rocket structure is filled with water and air at high piston wheel.
Pe

pressure. – Jenis brek untuk roda hadapan adalah cakera. Brek


– Tekanan tinggi dapat melancarkan roket ke atas cakera lebih berkesan dan cekap berbanding dengan
dengan daya yang lebih besar. brek dram.
High pressure can launch rocket upwards with greater The type of brake for front wheels is a disc. Disc brakes are
power. more effective and efficient than drum brakes.
(iii) Pecutan / Acceleration
– Brek sistem terbaik adalah Q kerana cecair brek
– Pecutan boleh ditingkatkan dengan
mempunyai takat didih yang tinggi, ketebalan paip
menggunakan jisim yang lebih kecil.
Acceleration can be increased by using smaller mass.
brek adalah tebal, nisbah luas keratan rentas bagi
– Jisim yang lebih kecil mempunyai inersia yang omboh roda dan omboh master adalah tinggi dan
kecil dan dapat dipecutkan dengan cepat. jenis brek untuk roda depan cakera.
The smaller mass has small inertia and can be The best brakes system is Q because the brake fluid has
accelerated rapidly. high boiling point, the thickness of the brake line is thick, the
cross-sectional area ratio for wheel piston and master piston
is high and the type of brake for front wheels is a disc.
  Fizik  Tingkatan 4 Jawapan

108 × 1000 m
(d) (i) 108 km j–1 = = 30 m s–1
3600 s
30 – 0
Pecutan / Acceleration = = 2.143 m s–1
14
(ii) Impuls = perubahan momentum
Impulse = change of momentum

F = mv – mu
t
400(0) – 400(30)
=
0.1
= 1.2 × 105 N

.
hd
.B
dn
iS
ng
la
Pe
n
ita
rb
ne
Pe
B
BA
3
Kegravitian
Gravitation

PETA Konsep

.
hd
KEGRAVITIAN
GRAVITATION

.B
Hukum Kegravitian

dn
Hukum-hukum Kepler Satelit buatan Manusia
Semesta Newton Kepler's Laws Man-made Satellite
Newton's Universal Law
of Gravitation

iS
Hukum Pertama Laju linear satelit,
First Law Linear speed of satellite,
Daya graviti, GM
v=
Gravitational force,
ng r
GMm
F= Hukum Kedua
r2
Second Law
la

Pecutan graviti,
Pe

Gravitational acceleration, Hukum Ketiga Satelit geopegun Satelit bukan


Third Law Geostationary geopegun
GM
g= 2 T 2 ∝ r 3 satellite Non-geostationary
r
satellite
n
ita

Daya memusat,
Centripetal force, Halaju lepas,
Escape velocity,
mv2
F= 2GM
r
rb

v=
r
ne

Jisim Bumi,
Mass of the Earth, Sistem Matahari – Planet Sistem Bumi – Satelit
4π2r 3 Sun – Planet System Earth – Satellite System
M=
Pe

GT 2

69
  Fizik  Tingkatan 4  Bab 3 Kegravitian

Hukum Kegravitian Semesta Newton


3.1 Newton's Universal Law of Gravitation

1. Isaac Newton menyatakan daya graviti yang melibatkan dua jasad mempunyai hubungan berikut: 1 2
Isaac Newton states that gravitational forces involving two bodies have the following relationship:

• Daya graviti berkadar terus dengan hasil darab jisim bagi dua jasad, iaitu F ∝ m1m2
The gravitational force is directly proportional to the product of the mass of two bodies, F ∝ m1m2
1
• Daya graviti berkadar songsang dengan kuasa dua jarak di antara pusat dua jasad tersebut, iaitu F ∝ 2 .
r
1

.
The gravitational force is inversely proportional to the square of the distance between the centres of the two bodies, F ∝ r 2 .

hd
Jisim, m1
Mass, m1 Jisim, m2
BAB

Mass, m2

.B
3 F F

dn
iS
Jarak r

ngDistance, r

2. Dalam bentuk persamaan, daya graviti,


In the form of equations, gravitational force,
la

F ∝ m1m2
Pe

m1m2
F∝
r2
1
F∝
r2
n


ita

3. Hukum Kegravitian Semesta Newton menyatakan bahawa daya tarikan graviti di antara dua jasad adalah
berkadar terus dengan hasil darab jisim-jisimnya dan berkadar songsang kepada kuasa
dua jarak pemisahan di antaranya.
rb

Newton’s Universal Law of Gravitation states that the gravitational attraction between two bodies is directly
inversely
ne

proportional to the product of the both masses and proportional to the square of the distance separating
them.
Pe

4. Rumus Hukum Kegravitian Semesta Newton,


The Newton’s Universal Law of Gravitation,
m1 m2
F = G 
r2
Dimana / Where,
F = daya graviti antara dua jasad /gravitational force between two bodies
m1 = jisim jasad pertama / mass of the first body
m2 = jisim jasad kedua / mass of the second body
r = jarak di antara pusat jasad pertama dan pusat jasad kedua / the distance between the centre of the first body and
the centre of the second body
G = pemalar kegravitian / universal gravitational constant (G = 6.67 × 10–11 N m2 kg–2)

70
Fizik  Tingkatan 4  Bab 3 Kegravitian 

Contoh 1
Kira daya antara dua objek yang mempunyai jisim 1000 kg dan 5000 kg dipisahkan dengan jarak 2 meter.
(G = 6.67 × 10–11 N m2 kg–2)
Calculate the force between two objects that have masses of 1000 kg and 5 000 kg separated by a distance of 1 m.
(G = 6.67 × 10–11 N m2 kg–2)

Penyelesaian / Solution:
m1 = 1000 kg
m2 = 5000 kg Senaraikan maklumat yang diberi dengan simbol.
List the information given using symbols.

.
r=2m

hd
G = 6.67 × 10–11 N m2 kg–2
m1m2 Kenal pasti dan tulis formula yang digunakan.
F=G

BAB
Identify and write the formula used

.B
r2

= 6.67 × 10
–11
× 1000 × 5000 Buat gantian numerikal ke dalam rumus dan lakukan penghitungan.
22 Substitue the numerals into the formula and do the calculations.
3

dn
= 8.34 × 10–5 N

iS
Menyelesaikan Masalah Melibatkan Hukum Kegravitian Semesta Newton
Solving Problems Involving Newton’s Universal Law of Gravitation
ng
1. Diberikan dua jasad berjisim m1 dan m2 mengalami daya graviti yang diberikan dengan formula berikut,
Given that two bodies of mass m1 and m2 experience the gravitational force given by the following formula,
la
m1 m2
F = G 
r2
Pe

(a) Jika kedua-dua jisim jasad digandakan, apa yang berlaku kepada daya graviti? 4
If the masses of both objects are doubled, what happens to the gravitational force?

Penyelesaian / Solution:
n

Daya graviti asal,


The original gravitational force,
ita

m1m2
F = G 
r2
rb

Jika kedua-dua jisim jasad digandakan,


If both the masses of the body doubled,
ne

2m 2m mm
F1 = G  1 2 2 = 4G  1 2 2
r r
Pe

Maka daya graviti baharu ialah empat kali ganda daya graviti asal.
Then the new gravitational force is four times the original gravitational force.

Semakin besar jisim, semakin besar daya graviti.


The larger the mass, the larger the gravitational force.

71
  Fizik  Tingkatan 4  Bab 3 Kegravitian

(b) Jika jisim kedua-dua jasad tidak berubah, tetapi jarak di antara dua jasad dikurangkan ke satu per empat
daripada jarak asal, apakah yang berlaku kepada daya graviti?
If the masses of the two objects do not change, the distance between the two bodies is reduced to one fourth of the
original distance, what happens to the gravitational force?

Penyelesaian / Solution:
Daya graviti asal,
The original gravitational force,
m1m2
F = G 
r2
Jika jarak di antara dua jasad diubah menjadi satu per empat,

.
hd
If the distance between two bodies is changed to one-fourth,
m1m2 mm
F1 = G  = 16G  1 2 2
1
( 4 r) r
BAB

.B
Maka daya graviti baharu ialah enam belas kali ganda daya graviti asal.
3 So the new gravitational force is sixteen times the original gravitational force.

dn
Daya graviti berkurang apabila jarak di antara dua objek bertambah .

iS
Gravitational force is reduced when the distance between two objects increases .
ng
Contoh 2
1. Kira daya graviti antara seorang budak lelaki berjisim 50 kg yang berada di atas permukaan Bumi.
la
Bumi mempunyai jisim 5.97 × 1024 kg dan jejari 6.37 × 106 m. (G = 6.67 × 10–11 N m2 kg–2)
Calculate gravitational force between a boy of 50 kg who is resting on the Earth's surface. The Earth has a mass
Pe

of 5.97 × 10 24 kg and a radius of 6.37 × 10 6 m. (G = 6.67 × 10–11 N m2 kg–2)

Penyelesaian / Solution:
m1 = 50 kg
m2 = 5.97 × 1024 kg
n

r = 6.37 × 106 m
ita

G = 6.67 × 10–11 N m2 kg–2


Gm1m2 6.67 × 10–11 × 50 × 5.97 × 1024
F = =
r2 (6.4 × 106)2
rb

= 486 N
2. Jarak di antara pusat Bumi dan Bulan ialah 3.8 × 108 m. Jisim Bumi ialah, me = 5.97 × 1024 kg dan jisim
ne

Bulan adalah, mm = 7.36 × 1022 kg. Berapakah daya graviti antara Bumi dan Bulan?
The distance between the Earth and the Moon centres is 3.8 × 10 8 m. The mass of the Earth is, me = 5.97 × 10 24 kg
and the mass of the Moon is, mm = 7.36 × 10 22 kg. What is the gravitational force between the Earth and the Moon?
Pe

Penyelesaian / Solution:
me = 5.97 × 1024 kg
mm = 7.36 × 1022 kg
r = 3.8 × 108 m
G = 6.67 × 10–11 N m2 kg–2
Gm1m2 6.67 × 10–11 × 5.97 × 1024 × 7.36 × 1022
F = =
r2 (3.8 × 108)2
= 2.03 × 10 N
20

72
Fizik  Tingkatan 4  Bab 3 Kegravitian 

Tugasan 1
1. Sebuah satelit mengorbit Bumi pada jarak 40 km. Satelit ini mempunyai jisim 750 kg dan jisim Bumi adalah
me = 5.97 × 1024 kg. Berapakah daya graviti antara planet dan satelit? (Jejari Bumi = 6.37 × 106 m)
A satellite is orbiting the Earth at a distance of 40 km. The satellite has a mass of 750 kg and the mass of the Earth is, me = 5.97 × 1024 kg.
What is the gravitational force between the planet and the satellite? (Earth Radius = 6.37 × 10 6 m)
ms = 750 kg
me = 5.97 × 1024 kg
h = 40 km

.
r = 6.37 × 106 m

hd
G = 6.67 × 10–11 N m2 kg–2
Gm1m2

BAB
F =

.B
(r + h)2
6.67 × 10–11 × 5.97 × 1024 × 750
=
(6.37 × 106 + 40 × 103)2 3

dn
= 7.27 × 103 N

iS
2. Jisim Matahari ialah 2.0 × 1030 kg dan jisim Bumi adalah, me = 5.97 × 1024 kg. Jarak di antara pusat Bumi dan pusat
Matahari adalah 1.50 × 1011 km. Berapakah daya graviti antara Matahari dan Bumi?
The mass of the Sun is 2.0 × 1030 kg and the mass of the Earth is, me = 5.97 × 1024 kg. The distance from the Earth is 1.50 × 1011 km. What is
the gravitational force between the Sun and the Earth?
ng
m
e = 5.97 × 1024 kg
ms = 2.0 × 1030 kg
la
r = 1.5 × 1011 m
G = 6.67 × 10–11 N m2 kg–2
Pe

Gm1m2
F =
r2
6.67 × 10–11 × 5.97 × 1024 × 2.0 × 1030
=
n

(1.5 × 1011)2
ita

= 3.54 × 1022 N
rb

Menghubung Kait Pecutan Graviti, g di Permukaan Bumi dengan Pemalar Kegravitian Semesta, G
Relating the Gravitational Acceleration, g on the Surface of the Earth to Universal Gravitational Constant, G
ne

1. Menurut Hukum Gerakan Kedua Newton, daya graviti boleh diungkapkan sebagai, F = mg. Daripada Hukum
mm
Kegravitian Semesta Newton, daya graviti, F diungkapkan sebagai F = G  1 2 2 .
r
Pe

According to Newton's Second Law of Motion, the gravitational force can be expressed as, F = mg. From the Newton’s
mm
Universal Law of Gravitation, gravitational force, F is expressed as F = G  12 2 .
r
2. Jika satu objek yang berada di permukaan Bumi, objek akan mengalami daya graviti, seperti dinyatakan dalam
Hukum Gerakan Kedua Newton .
If an object were on the surface of the Earth, the object would experience gravitational force, as stated in the
Newton's Second Law of Motion .

73
  Fizik  Tingkatan 4  Bab 3 Kegravitian

Objek
Object

Jisim Bumi = M
Mass of the Earth = M

.
hd
BAB

.B
3. Berdasarkan rajah di atas, jika, / Based on the above diagram, if,
M = jisim Bumi / the mass of the Earth
3

dn
m = jisim objek / object mass
r = jarak di antara pusat Bumi dan pusat objek
distance between the centre of the Earth and the centre of the object

iS
Kita boleh menghubungkan daya graviti yang menyebabkan objek jatuh dengan suatu pecutan graviti, g
Mm
menggunakan rumus F = mg dan daya graviti yang menarik objek ke arah pusat Bumi, F = G  2 , maka
r

ng
We can relate the gravitational force that causes objects to fall with a gravitational acceleration, g using the formula F = mg
Mm
and the gravitational force that pulls objects toward the center of the Earth, F = G  2 , hence
r
la
Mm
mg = G 
r2 Mansuhkan faktor sepunya, m
Pe

GM Cancel the common factor, m


g = 2
r

Contoh 3
n
ita

Hitungkan pecutan graviti sebuah objek berjisim 50 kg di atas permukaan Bumi. Diberikan jisim Bumi ialah
5.97 × 1024 kg dan jejari Bumi ialah 6.37 × 106 m. [G = 6.67 × 10–11 N m2 kg–2]
Calculate the gravitational acceleration of an object with mass of 50 kg above the surface of the Earth. Given that the mass
rb

of the Earth is 5.97 × 10 24 kg and the radius of the Earth is 6.37 × 10 6 m
[G = 6.67 × 10–11 N m2 kg–2]
ne

Penyelesaian / Solution:
M = 5.97 × 1024 kg
r = 6.37 × 106 m
Pe

G = 6.67 × 10–11 N m2 kg–2


6.67 × 10–11 × 5.97 × 1024
g = GM =
r 2
(6.37 × 106)2
= 9.81 m s–2

4. Jika satu satelit yang mengorbit mengelilingi Bumi, objek akan mengalami daya graviti, yang dinyatakan
dalam Hukum Gerakan Kedua Newton.
If a satellite is orbiting around the Earth, the object will experience a gravitational force, expressed in Newton's Second Law
of Motion.

74
Fizik  Tingkatan 4  Bab 3 Kegravitian 

5. Berdasarkan Rajah 1, jika / Based on Diagram 1, if, Satelit / Satellite


M = jisim Bumi / the mass of the Earth m
m = jisim objek / object mass
r = jarak di antara pusat Bumi dan pusat objek
the distance between the centre of the Earth and the centre of the object h
h = ketinggian objek dari permukaan Bumi / height of the object from the Earth
Kita boleh menghubungkan daya graviti yang menyebabkan objek jatuh
dengan suatu pecutan graviti, g menggunakan rumus F = mg dengan daya
GMm
graviti yang menarik objek ke arah pusat Bumi, F = , maka
r2 r

.
We can relate the gravitational force that causes objects to fall with a gravitational

hd
acceleration, g using the formula F = mg and the gravitational force that pulls the
GMm
objects toward the centre of the Earth, F = , hence Jisim Bumi = M
r2

BAB
Mass of the Earth = M

.B
GMm
mg =
(r + h)2 Mansuhkan faktor sepunya, m
GM Cancel the common factor, m
g = 3
(r + h)2

dn
Rajah 1 / Diagram 1

4

iS
Contoh
Sebuah satelit berjisim 300 kg mengorbit mengelilingi Bumi pada ketinggian 550 km. Diberikan jisim Bumi
ialah 5.97 × 1024 kg dan jejari Bumi ialah 6.37 × 106 m. Berapakah nilai pecutan graviti di kedudukan satelit
ng
itu? [G = 6.67 × 10–11 N m2 kg–2]
A 300 kg satellite is orbiting around the Earth at the height of 550 km. Given that the mass of the Earth is 5.97 × 10 24 kg
and the radius of the Earth is 6.37 × 10 6 m. What is the value of gravitational acceleration at the location of the satellite?
la
[G = 6.67 × 10 –11 N m2 kg–2]

Penyelesaian / Solution:
Pe

me = 5.97 × 1024 kg mg = G Mm
r2
r = 1.5 × 1011 m
g = GM 2 = 6.67 × 10 6× 5.97 × 102
–11 24

h = 550 × 103 m (r + h) (6.37 × 10 + 550000)


n

G = 6.67 × 10–11 N m2 kg–2 = 8.32 m s–2


ita

5. Rajah 2 menunjukkan lakaran graf variasi nilai pecutan graviti, g dengan jarak, r dari pusat Bumi.
rb

Diagram 2 show the skatch of graph of variation of the value of gravitational acceleration, g with the distance, r from the
centre of the Earth.
ne

g = GM
–—
Pe

R2

Nilai g adalah berkadar terus Nilai g adalah berkadar


dengan jarak dari pusat Bumi songsang pusat dengan
untuk kedudukan r < R. kuasa dua jarak dari Bumi
The value of g is directly untuk kedudukan r < R.
proportional to the distance from The value of g is inversely
the centre of the Earth for proportional to the square of the
distances r < R. distance from the centre of the
Earth for distances r < R.
0 r
R 2R 3R

Rajah 2 / Diagram 2

75
  Fizik  Tingkatan 4  Bab 3 Kegravitian

Kepentingan Mengetahui Nilai Pecutan Graviti


The Importance of Knowing the Value of the Gravitational Acceleration

1. Pecutan graviti adalah berbeza berdasarkan jisim planet atau bulan dan jarak dari pusat
jisimnya dan permukaannya. Atas sebab itu, graviti mempunyai tarikan yang lebih rendah pada jasad
berjisim kecil daripada Bumi. Formula untuk pecutan graviti ialah g = GM .
r2
The gravitational acceleration is different based on the mass of the planet or moon and the distance

from its center of mass and its surface. For that reason, gravity has a lesser pull on bodies of smaller mass than
GM

.
the Earth. The formula for gravitational acceleration is g = 2 .

hd
r
2. Contoh-contoh pecutan graviti bagi planet yang berlainan: i-THiNK Peta titi
Examples of gravitational accelerations on different planets:
BAB

.B
Planet Bumi Musytari Marikh
Planet Earth Jupiter Mars
as as as
Pecutan graviti 9.81 24.79 3.71
3

dn
Gravitational acceleration
(m s–2)

iS
Matahari
Sun Marikh Neptun
Zuhrah Neptune
Venus Mars
ng
Utarid
Mercury Bumi Uranus
Earth Zuhal
la
Musytari Uranus
Saturn
Jupiter
Pe

3. Badan manusia dicipta untuk hidup dalam persekitaran dengan pecutan graviti ialah 9.81 m s–2. Semasa
penerokaan angkasa yang jauh dari Bumi, badan angkasawan boleh terdedah kepada keadaan graviti rendah
atau graviti tinggi.
n

The human body is created to function in an environment with acceleration of gravity of 9.81 m s–2. During space exploration
ita

far from the Earth, astronauts can be exposed to low or high gravity conditions.

4. Apakah kesan graviti terhadap tumbesaran manusia? Kita boleh melihat kesan pecutan graviti yang rendah
seperti di Bulan(gbulan = 1.62 m s–2) kepada badan seperti di bawah.
rb

What is the effect of gravity on human growth? We can see the effect on low gravity acceleration such as on the Moon,
(gmoon = 1.62 m s–2)to the body as below.
ne

(a) Jantung berfungsi dengan baik di Bulan kerana graviti adalah kecil , maka lebih
senang untuk mengepam darah ke seluruh badan. Hal ini mungkin menyebabkan jantung
Pe

semakin lemah dan lebih kecil.


The heart works well on the Moon because the gravity is smaller so it is easier to
pump blood throughout the body. This may cause the heart to become weaker and smaller .

(b) Oleh kerana otot dan jantung kurang digunakan, jisim otot akan berkurang. Apabila
angkasawan kembali ke Bumi, otot mereka mungkin tidak cukup kuat untuk membolehkan
mereka bergerak dengan sendiri.
As the muscles and heart are used less, the muscles lose mass . When astronauts return to Earth, their
muscles may not be strong enough to allow them to move on their own.

76
Fizik  Tingkatan 4  Bab 3 Kegravitian 

(c) Oleh kerana kekurangan mampatan pada tulang belakang, cakera di antara tulang belakang menyerap
lebih banyak cecair berbanding dengan di Bumi. Hal ini menyebabkan para angkasawan semakin tinggi.
Pemanjangan tulang belakang ini juga boleh menyebabkan sakit belakang .
Due to the lack of compression on the spine, the discs in the spine absorb more fluid than they do on the Earth. This
causes astronauts becoming taller . This lengthening of the spine can also cause back pain .

(d) Darah dialirkan lebih ke bahagian atas


badan dan jauh dari anggota badan yang di bawah.
Semasa di angkasa, muka angkasawan akan mengalami bengkak kerana pengaliran bendalir dan
darah berkurangan di kaki dan mengalir lebih ke kepala dan bahagian atas badan.
upper

.
The blood is flowing more to the part of the body and away from the lower limbs. While in space,

hd
astronauts suffer from swollen faces as fluid and blood flow decreases to the legs and flow more to the head
and upper limbs.

BAB
.B
(e) Menurut penyelidikan NASA, tanpa graviti yang bekerja di tubuh anda, tulang anda kehilangan mineral,
dengan ketumpatan turun lebih dari 1% sebulan.
According to NASA research, without the gravity working on your body, your bones lose minerals, with a density of
3

dn
less than 1% a month.

Daya Memusat dalam Sistem Gerakan Satelit dan Planet

iS
Centripetal Force in the Motion of Satellites and Planet System

1. Rajah 1 menunjukkan kedudukan bagi sebuah satelit yang sedang mengorbit Bumi dengan laju seragam.
membulat
ng
Jasad yang sedang bergerak dalam gerakan sentiasa mengalami perubahan arah gerakannya
walaupun halajunya tetap .
la
Diagram 1 shows the position of a satellite orbiting the Earth at a uniform speed. The body that is moving in a circular

motion is constantly changing its direction of motion even when the velocity is constant .
Pe

Arah halaju
Direction of velocity

Satelit
n

Satellite

Bumi
ita

Earth
rb
ne

Arah halaju
Direction of
velocity
Pe

Arah halaju
Direction of
velocity

Rajah 1 / Diagram 1

77
  Fizik  Tingkatan 4  Bab 3 Kegravitian

2. Suatu daya memusat yang bertindak sepanjang masa ke arah pusat Bumi diperlukan untuk mengubah
arah gerakan jasad.
A centripetal
force acting all the time toward the centre of the Earth is needed to change the direction
of the motion of the body.

3. Dengan gabungan halaju seragam dan daya memusat yang bertindak ke arah satelit
tersebut akan menyebabkan satelit membuat gerakan membulat.
The combination of uniform velocity and the centripetal force acting on the satellite will cause
the satellite to make a circular motion.

.
4. Terdapat suatu daya yang bertindak ke atas satelit yang melakukan gerakan membulat dengan arah yang

hd
sentiasa menuju ke pusat Bumi itu. Daya ini dikenali sebagai daya memusat .
There is a force acting on the satellite which is moving in a circular motion in the direction of the centre of the Earth. This
BAB

.B
force is known as the centripetal force .

v
3

dn
T
v

iS
Penyumbat getah T
Rubber stopper T

v
ng
T = tegangan tali / tension of string
v = halaju linear / linear velocity
la
Pe

Jisim berslot
Slotted weight

Rajah 2 / Diagram 2
n

5. Rajah 2 menunjukkan tegangan benang, T yang bertindak sebagai daya memusat untuk gerakan
ita

penyumbat getah. Magnitud daya memusat bergantung pada jisim jasad, laju

gerakan membulat dan jejari bulatan.


rb

Diagram 2 shows the tension of the thread acting as a centripetal force for the movement of the rubber stopper. The
magnitude of the force depends on the mass of the body, the speed of movement circle and
ne

radius of the circle.

pecutan
Pe

6. Suatu jasad dalam gerakan membulat sentiasa dalam gerakan . Dalam gerakan pecutan,
sentiasa terdapat perubahan halaju sama ada dalam laju atau arah gerakan. Suatu
objek dalam gerakan membulat yang seragam bergerak dengan halaju seragam tetapi mengalami
pecutan akibat dari perubahan arah gerakan.
A body in circular motion is always in acceleration motion. In acceleration motion, there is always a change in
velocity either in speed or direction of motion. An object in a uniform circular motion
moves at a uniform velocity but experiences acceleration as a result of a change in direction .

78
Fizik  Tingkatan 4  Bab 3 Kegravitian 

7. Daya memusat yang bertindak dalam gerakan membulat ialah F = mv di mana


2

mv2 r
The centripetal force acting in a circular motion is F = where
r
F = Daya memusat / Centripetal force
m = jisim / mass
v = laju linear / linear speed
r = jejari bulatan / radius of circle
8. Sistem gerakan satelit dan planet adalah gerakan membulat yang sentiasa mengalami pecutan memusat, a.
Satellites and planetary motion systems are circular motions which are constantly experiencing a centripetal acceleration, a.

Maka, dengan rumus / So, with formula F = ma dan / and F = mv


2

r

.
ma = mv
2

hd

r
v2
a=

BAB
r

.B
Di mana / Where,
v = halaju linear / linear velocity
r = jejari orbit / radius of orbit
3

dn
Contoh 5

iS
Daripada Rajah 2, jika penyumbat getah berjisim 0.1 kg diputarkan dengan jejari bulatan, r = 40 cm,
berapakah daya memusat yang bertindak ke atas penyumbat getah apabila penyumbat getah itu sedang
bergerak dengan laju 10 m s–1? ng
From Diagram 2, if the 0.1 kg rubber stopper is rotated with a radius of circle, r = 40 cm, what is the centripetal force acting
on the rubber stopper when the rubber stopper is moving at a speed of 10 m s–1?

Penyelesaian / Solution:
la
m = 0.1 kg, r = 0.4 m, dan v = 10 m s–1
Daya memusat / Centripetal force, F = mv  , maka F = 0.1 × 10 = 25 N
2 2
Pe

r 0.4

Menentukan Jisim Bumi dan Matahari Menggunakan Rumus Hukum Kegravitian Semesta Newton dan
n

Daya Memusat
ita

Determining the Mass of the Earth and the Sun Using Newton's Universal Law of Gravitation and Centripetal Force

1. Oleh sebab daya graviti Matahari pada Bumi adalah daya memusat yang menyebabkan gerakan membulat
rb

Bumi mengelilingi Matahari, kita boleh menggunakan Hukum Kegravitian Semesta Newton untuk mencari
jisim Matahari.
centripetal force
ne

Since the gravitational force of the Sun on the Earth is the that causes the Earth's orbit around the
Sun, we can use Newton's Universal Law of Gravitation to find the mass of the Sun.

2. Jika jisim Matahari diwakili oleh M, jisim Bumi diwakili oleh m dan jejari orbit ialah r, maka hubungan antara
Pe

Hukum Kegravitian Semesta Newton dengan daya memusat ialah:


If the mass of the Sun is represented by M, the mass of the Earth is represented by m and the radius of the orbit is r, then
the relationship between the Newton's Universal Law of Gravitation with centripetal force is:

Fgraviti = Fmemusat
Matahari
GMm mv2 Sun r
=
r2 r Bumi
Persamaan di atas boleh disusun semula untuk mendapatkan jisim Matahari. Earth
The above equation can be rearranged to obtain the mass of the Sun. r = 1.5 x 10 km
8

Jisim Matahari / The mass of the Sun, M = rv


2

G

79
  Fizik  Tingkatan 4  Bab 3 Kegravitian

Di mana, nilai G ialah 6.67 × 10–11 N m2 kg–2 dan r ialah 1.5 × 108 km dan halaju Bumi, v = 2πr dengan T ialah
T
tempoh untuk Bumi mengorbit Matahari iaitu 365.25 hari.
2πr
Where, the value of G is 6.67 × 10 –11 N m2 kg–2 and r is 1.5 × 108 km and the velocity of the Earth, v= where T is the
T
period of the Earth orbiting the Sun that is 365.25 days.

Contoh 6
Tentukan jisim Matahari.
Determine the mass of the Sun.

Penyelesaian / Solution:

.
hd
G = 6.67 × 10–11 N m2 kg–2
r = 1.5 × 108 km = 1.5 × 1011 m
T = 365.25 hari × 24 jam × 60 minit × 60 saat = 31557600 saat
BAB

.B
365.25 days × 24 hours × 60 minutes × 60 seconds= 31557600 seconds

Maka / Hence, v = 2πr = 2(3.142)(1.5 × 10 ) = 2.99 × 104 m s–1


11

3 T 31557600

dn
M = rv = (1.5 × 10 )(2.99–11× 10 ) = 2.0 × 1030 kg
2 11 4 2

G 6.67 × 10

iS

3. Jisim Bumi juga boleh ditentukan dengan menggunakan Hukum Kegravitian Semesta Newton. Jika diberi
daya graviti Bumi sama dengan berat objek ke Bumi, maka

ng
The mass of the Earth also can be determined using Newton's Universal Law of Gravitation. If given the gravitational force
of the Earth equals the weight of the object to the Earth, hence

RE = 6.37 × 106 m
la
Objek di Bumi
Object on the Earth ME = jisim Bumi / mass of the Earth
m = jisim objek / mass of the object
Pe

G = 6.67 × 10–11 N m2 kg–2


g = 9.8 m s–2

r
GMEm
n

Fgraviti =
r2
ita

Berat objek / Weight of object, W = mg


Fgraviti = W
rb

GMEm
= mg
r2
r2
ne

Me = g
G

Tugasan 1
Pe

Tentukan jisim Bumi.


Determine the mass of the Earth.
gr2
Me =
G
9.8(6.37 × 106)2
Me = = 5.96 × 1024 kg
6.67 × 10–11

80
Fizik  Tingkatan 4  Bab 3 Kegravitian 

Hukum Kepler
3.2 Kepler's Laws

1. Hukum Kepler Pertama:


Hukum Kepler Pertama menyatakan bahawa orbit bagi setiap planet adalah elips dengan Matahari
berada di satu daripada fokusnya.
Kepler’s First Law:
Kepler’s First Law states that all planets move in elliptical orbits, with the Sun at one of its focus.
Paksi minor
Minor axis

.
Planet

hd
Planet

Matahari

BAB
Sun Paksi major
F1

.B
F2
Major axis

dn
Daripada model lintasan planet di atas, diperlihatkan orbit berbentuk elips yang mengelilingi
Matahari. Matahari berada pada salah satu titik fokusnya yang ditanda dengan F1 dan F2. Pada

iS
keadaan tersebut, planet memiliki dua jarak iaitu jarak terhadap F2 dan jarak terhadap F1.
From the model of the trajectory of the planet above is shown an
ng elliptical orbit around the Sun. The Sun is
at one of its focal points marked by F1 and F2. In that case, the planet has two distances
namely F2 and F1.
la
2. Hukum Kepler Kedua:
Hukum Kepler Kedua menyatakan bahawa satu garis yang menyambungkan planet dengan Matahari
Pe

mencakupi luas yang sama dalam selang masa yang sama apabila planet bergerak dalam orbit.
Kepler's Second Law:
Kepler's Second Law states that a line that connects a planet to the Sun sweeps out equal areas in equal times
when the planet moves in its orbit.
n

Planet
Planet
ita

Matahari
Sun
rb

Lambat Cepat
Slower Faster
ne

Luas Kawasan 1 dan 2 adalah sama.


Area of Region 1 and 2 is equal.
Kawasan 2 Kawasan 1
Region 2 Region 1
Pe

Planet akan bergerak lebih cepat ketika berada dekat dengan Matahari(kawasan 1), kemudian
akan bergerak lebih lambat ketika berjarak jauh dari Matahari(kawasan 2). Hukum Kepler Kedua
menyatakan bahawa luas kawasan 1 dan kawasan 2 adalah sama walau di mana pun planet
berada pada orbitnya dan diukur berdasarkan interval waktu yang sama.
The planets move faster when they are near the sun (area 1), and then move slower when they are
further away from the sun (area 2). Kepler's Law II states that the areas of the area 1 and area 2 are the same
regardless of whether the planet is in orbit and is measured by the same time interval.

81
  Fizik  Tingkatan 4  Bab 3 Kegravitian

3. Hukum Kepler Ketiga:


Hukum Kepler Ketiga menyatakan bahawa kuasa Planet 2
Planet 2
dua tempoh planet adalah berkadar terus
Tempoh, T2
dengan kuasa tiga jejari orbitnya. Period, T2
Kepler’s Third Law:
The square of the period of any planet is directly r2
proportional to the cube of the radius of its orbit.
T 2 ∝ r3
T12 T22
= = Pemalar / Constant r1
Planet 1

.
r13 r23
Matahari

hd
Planet 1
T1 = tempoh planet 1 Sun
Tempoh, T1
period of the planet 1 Period, T1
BAB

T2 = tempoh planet 2

.B
period of the planet 2
r1 = jarak planet 1 dari Matahari
the distance of planet 1 from the Sun
3

dn
r2 = jarak planet 2 dari Matahari
the distance of planet 2 from the Sun

iS
Merumuskan Hukum Kepler Ketiga / Express Kepler’s Third Law

1. Pertimbangkan sebuah planet dengan jisim m mengorbit dalam gerakan hampir membulat mengelilingi
ng
Matahari berjisim M. Kekuatan daya memusat bersih yang bertindak ke atas planet yang mengorbit ini
diberikan oleh hubungan berikut:
Consider a planet with mass m to orbit in a nearly circular motion around the Sun of mass M. The net centripetal force acting
la
upon this orbiting planet is given by the following relationship:
v2
Daya memusat / Centripetal force, FC = m 
Pe

r
Kekuatan daya memusat bersih ini adalah hasil daya graviti yang menarik planet ke arah Matahari dan boleh
diwakili sebagai
This net centripetal force is the result of the gravitational force that attracts the planet towards the Sun and can be represented
n

as
Mm
ita

Daya graviti / Gravitational force, FG = G 


r2
Oleh kerana FC = FG, ungkapan di atas untuk daya memusat dan daya graviti adalah sama. Oleh itu,
rb

Since FC = FG, the above expressions for centripetal force and gravitational force are equal. Thus,
v2 Mm
=G 2 m
ne

r r
GM
v =
2

r
Pe

Oleh kerana halaju sesuatu objek dalam orbit hampir bulat boleh dianggarkan sebagai
Since the velocity of an object in nearly circular orbit can be approximated as

v = 2πr 
r
• Bagi sistem planet dan Matahari, M
Gantikan  dalam , / Substitute  into , ialah jisim Matahari.
For planets and the Sun system, M is the mass

( )
2πr 2 = GM
T r
of the Sun.
• Bagi sistem satelit dan Bumi, M ialah
jisim Bumi.
( )
T2 = 4π r
2
3
For satellite and the Earth system, M is the
GM mass of the Earth.

82
Fizik  Tingkatan 4  Bab 3 Kegravitian 

3. Mengikut persamaan di atas, nilai sebelah kanan adalah sentiasa sama bagi setiap planet tanpa mengetahui
T2
jisim planet tersebut. Maka, boleh dirumuskan nisbah 3 adalah sama untuk semua planet jika daya yang
r
memegang planet di orbit mereka adalah daya graviti.
According to the equation above, the value on the right is always the same for every planet without knowing the mass
of the planet. It can be concluded that the ratio T 3 is the same for all planets if the force holding the planet in their orbit is
2

gravitational force. r 

T2
k=   k = pemalar / constant
r3
T2 = kr3

.
k = 4π
2

hd
GM

Menyelesaikan Masalah Menggunakan Rumus Hukum Kepler Ketiga

BAB
.B
Solve Problems Using Kepler's Third Law

T12 T22
= = Pemalar / Constant 3

dn
r13 r23
1. Bagi planet yang mengorbit Matahari / For a planet that orbits the Sun;

iS
r = jarak di antara pusat planet dengan pusat Matahari
the distance between the centre of the planet and the centre of the Sun.
2. Bagi satelit yang mengorbit Bumi / For satellites orbiting the Earth;
ng
r = R + h (jarak di antara pusat Bumi dengan pusat satelit / distance between centre of the Earth and satellite centre)
R = jejari Bumi / Earth radius = 6370 km
h = ketinggian satelit dari permukaan Bumi / satellite elevation from the Earth's surface
la

8
Pe

Contoh
T2
Tentukan nisbah untuk planet berikut yang mengorbit Matahari.
r3
n

Determine the ratio T 3 for the following planets orbiting the Sun.
2

r 
ita

Planet Tempoh masa (hari) Jejari (km) T 2 2 –3


(s m )
Planet Period (days) Radius (km) r3
rb

Merkuri (87.97 × 24 × 60 × 60)2


87.97 57.9 juta / million = 2.98 × 10–19
Mercury (57.9 × 106 × 103)3
ne

Zuhrah (224.9 × 24 × 60 × 60)2


224.70 108.2 juta / million (a) = 2.98 × 10–19
Venus (108.2 × 106 × 103)3
Pe

Musytari (4332.82 × 24 × 60 × 60)2


4,332.82 778.3 juta / million (b) = 2.98 × 10–19
Jupiter (778.3 × 106 × 103)3

Bumi (365.26 × 24 × 60 × 60)2


365.26 149.6 juta / million (c) = 2.98 × 10–19
Earth (149.6 × 106 × 103)3

83
  Fizik  Tingkatan 4  Bab 3 Kegravitian

Tugasan 2
Rajah 1 menunjukkan Satelit 1 mengorbit Bumi dengan tempoh Satelit 2
mengorbit T1 selama 24 jam atau 86 400 saat. Jika jarak Bumi ke Satellite 2
Satelit 1 ialah 3.59 × 107 m dan jejari Bumi ialah 6370 km,
Figure 1 shows the Satellite 1 orbiting the Earth with period of orbit T1 for
24 hours or 86 400 s. If the Earth's distance to Satellite 1 is 3.59 × 107 m and the r2
radius of the Earth is 6370 km,
T 2
(a) cari nilai nisbah untuk Satelit 1.
r3

.
Satelit 1
T 2 r1

hd
find the value of ratio for Satellite 1. Satellite 1
r 3 Bumi
Earth
(b) Jika jarak Bumi ke Satelit 2 ialah 3.8 × 107 m, cari tempoh
BAB

.B
mengorbit T2 Bumi untuk satelit 2.
If the Earth's distance to satellite 2 is 3.8 × 10 7 m, find the orbital period, T2
of the Earth for satellite 2.
3

dn
Penyelesaian / Solution:
(a) Jarak Satelit 1 ke pusat bumi, Rajah 1 / Diagram 1
Distance of Satellite 1 to the Earth's centre,

iS
r = R + h = 6370 km + 3.59 × 107 m = 3.59 × 107 m + 6.370 × 106 = 4.227 × 107 m
T2 864002
3 = = 9.88 × 10–14 s2 m–3
r (4.227 × 107)3
ng
T2
(b) Untuk kedua- dua satelit yang mengorbit Bumi, nilai 3 adalah sama (Hukum Kepler Ketiga).
r
T2
la
Maka, untuk Satelit 2, nilai 3 = 9.88 × 10–14 s2 m–3.
r
Jarak dari Satelit 2 ke pusat Bumi ialah
Pe

T 2
For both satellites orbiting the Earth, the value 3 is the same (Kepler's Third Law).
r 
T 2
Hence, for Satellite 2, the value 3 = 9.88 × 10 –14 s2 m–3.
r 
The distance from Satellite 2 to the centre of the Earth is
n

= 3.8 × 107 m + 6370 km = 3.80 × 107 m + 6.370 × 106 = 4.437 × 107 m


ita

T2
= 9.88 × 10–14
(4.437 × 107)3
T 2 = 9.88 × 10–14 × (4.437 × 107)3 = 8.63 × 109
rb

T = 9.29 × 104 s
ne

Satelit Buatan Manusia


3.3 Man-made Satellites
Pe

Orbit Satelit / Satellite Orbit

1. Satelit adalah objek buatan manusia yang mengorbit Bumi . Satelit digunakan untuk mengkaji
Bumi , komunikasi , dan juga untuk memerhatikan alam semesta . Satelit pertama
adalah Soviet Sputnik 1, yang dilancarkan pada tahun 1957. Sejak itu, banyak negara telah melancarkan
satelit, dengan lebih daripada 3,000 satelit yang sedang beroperasi di sekitar Bumi.
Satellites are man-made objects orbiting the Earth . Satellites are used to study the Earth , for
communication , and to observe the universe . The first satellite was the Soviet Sputnik 1, launched in 1957.
Since then, many countries have launched satellites, with more than 3,000 satellites operating around the Earth.

84
Fizik  Tingkatan 4  Bab 3 Kegravitian 

2. Satelit-satelit tersebut mengorbit di sekitar Bumi dalam orbit yang berukuran dari 240 km ke 36,200 km.
Satelit berada di orbit Bumi yang rendah, bergerak dengan halaju 27,400 km per jam supaya tidak dapat ditarik
semula ke atmosfera Bumi.
These satellites orbit around the Earth in orbit ranging from 240 km to 36,200 km. Satellites are in low Earth orbit, traveling
at very fast speed of 27,400 km per hour so they will not be pull back to the Earth's atmosphere.

3. Satelit sentiasa dalam gerakan membulat dengan halaju (v) , dengan daya tarikan graviti (F) antara
satelit dan Bumi. Satelit yang mengorbit lebih dekat dengan Bumi bergerak dengan kelajuan yang tinggi.
Satelit yang mengorbit lebih jauh dari Bumi bergerak dengan kelajuan yang lebih rendah.
The satellites are always in a circular motion with velocity (v) , with the gravitational force (F) between the

.
satellites and the Earth. The satellite orbiting closer to the Earth is moving at high speed. The satellite orbiting

hd
farther away from the Earth move at lower speeds.

4. Rajah di atas menunjukkan sebuah satelit mengorbit Bumi. Jika

BAB
.B
jisim Bumi diwakili dengan M, jisim satelit diwakili dengan m dan
jejari orbit satelit ialah r, maka halaju orbit diterbitkan menggunakan v
hubungan antara Hukum Kegravitian Semesta Newton dengan 3

dn
daya memusat .
The diagram above shows a satellite orbiting the Earth. If the mass of the F Satelit
Earth is represented by M, the mass of the satellite is represented by m, and Satellite

iS
the satellite orbital radius is r, hence orbital velocity is derived using the Bumi
Earth
relationship between the Newton's Universal Law of Gravitation with
centripetal force. .
ng
FMemusat = FGraviti
FCentripetal = FGravity
la
mv2 Mm
= G  2
Pe

r r
M
v2 = G 
r
v = GM
n

r
ita

Halaju orbit, v = GM ialah halaju yang diperlukan satelit yang bergerak untuk mengorbit bumi dalam orbit
r
membulat.
rb

The orbital velocity, v = GM is the velocity required by a moving satellite to orbit the Earth in a circular orbit.
r
ne

Satelit Geopegun dan Bukan Geopegun / Geostationary and Non-geostationary Satellites

sama
Pe

1. Satelit geopegun adalah satelit mengorbit Bumi dalam orbit geopegun pada kelajuan yang
dengan Bumi berputar pada garis latitud yang sama, secara khusus garis latitud ekuator. Orbit geopegun
mengorbit dalam jarak lebih kurang 35,786 km di atas Bumi. Sebuah satelit yang mengorbit dalam orbit
geopegun berada di tempat yang sama di langit, dan secara langsung di atas kawasan yang
sama pada setiap masa.
Geostationary satellites are satellites orbiting the Earth in geostationary orbit at the same speed as the Earth
rotating at the same latitude, specifically the equatorial latitude. Geostationary orbits orbit around 35,786 kilometres above
the Earth. A satellite orbiting in geostationary orbit in the same spot in the sky, and directly above the
same area at all times.

85
  Fizik  Tingkatan 4  Bab 3 Kegravitian

2. Satelit geopegun biasanya digunakan untuk pelbagai tujuan, seperti sistem komunikasi, Internet, TV kabel
penyiaran, Sistem Posisi Global , isyarat radio , dan ramalan cuaca .
Geopegun satellites are commonly used for a variety of purposes, such as communications systems, the Internet, broadcasting
cable TV, Global Position System , radio signals and weather forecasts .

3. Secara umumnya, kita boleh meletakkan satelit dalam tiga jenis orbit.
Generally we can place satellites in three orbits
(i) Orbit geopegun, kira-kira 35,786 km atas bumi
Geostationary Orbits, about 35.786 km above the Earth
(ii) Orbit Pertengahan Bumi (MEO), kira-kira 8000 km atas bumi

.
Middle-Earth Orbit MEO), about 8000 km above the earth

hd
(iii) Orbit Rendah Bumi, dari 160 hingga 2000 km di atas Bumi. Jadi satelit bukan geopegun adalah satelit
tidak
BAB

yang diletakkan dalam orbit geopegun.

.B
Low Earth Orbit, from 160 to 2000 km above the Earth. So a non-geostationary satellites are satellites that are
same placed in geostationary orbit.
3

dn
Halaju Lepas / Escape Velocity

iS
1. Halaju lepas, v adalah halaju minimum yang diperlukan suatu objek di permukaan Bumi untuk
mengatasi daya graviti dan terlepas ke angkasa. Sebagai contoh, sebuah roket yang masuk ke angkasa
ng
perlu mencapai halaju lepas untuk bebas dari Bumi.
The escape velocity, v is the minimum velocity required by an object on the Earth surface to overcome
gravitational force and escape to space. For example, a rocket that went into the space need to achieve escape velocity
la
to be free of the Earth.
Pe

2. Halaju lepas akan dicapai apabila tenega kinetik minimum yang Halaju lepas
dibekalkan kepada objek telah mengatasi tenaga keupayaan graviti. Escape velocity
Perlahan
Escape velocity is achieved when the minimum kinetic energy supplied Slow
to the object overcomes the gravitational potential energy.
n

Tenaga Kinetik Minimum


ita

Tenaga Keupayaan + =0
Potential Energy + Minimum Kinetic Energy =0 Cepat
Faster
rb

3. Objek berjisim m, boleh melepaskan diri daripada tarikan graviti


Bumi berjisim, M pada jarak dari pusat Bumi, r, hanya apabila tenaga
kinetiknya sama dengan tenaga keupayaan graviti. Tenaga kinetik
ne

Halaju orbit
objek tersebut dengan jisim, m yang bergerak dengan halaju, v ialah Orbital velocity
1
EK = mv2. Tenaga keupayaan graviti objek itu, diberikan dengan
2
Pe

rumus
An object with mass of m, can release from the gravitational pull of a large object with mass of M at the distance from the
centre of the Earth, r, only when its kinetic energy equals the gravitational potential energy. The kinetic energy of an object
1
with mass m moving with velocity v can be given by KE = mv2. The gravitational potential energy of this object can be
given by the formula 2

GMm
U=    G = pemalar kegravitian / gravitational constant (G = 6.67 × 10–11 N m2 kg–2)
r

86
Fizik  Tingkatan 4  Bab 3 Kegravitian 

1 2 GMm
4. Menyamakan kedua-dua persamaan, kita dapat: mv =
Equating both expressions, we found, 2 r

Maka, halaju lepas / Hence, escape velocity, v = 2GM


r
G = pemalar kegravitian / gravitational constant = 6.67 × 10–11
M = jisim planet atau jasad / mass of planet or body
r = jejari planet atau jasad / radius of planet or body

Contoh 10

.
hd
Jejari Bumi adalah 6.4 × 106 m, dan jisim Bumi adalah 5.97 × 1024 kg. Apakah halaju lepas dari permukaan
Bumi?

BAB
The radius of the Earth is 6.4 × 10 6 m, and the mass of the Earth is 5.97 × 10 24 kg. What is the escape velocity from the

.B
surface of the Earth?
(g = 9.8 m s–2, G = 6.67 × 10−11)
3

dn
Penyelesaian / Solution:
Untuk mencari halaju lepas daripada permukaan Bumi,
To find escape velocity from the surface of the Earth,

iS
v= 2GM = 2(6.67 × 10−11)(5.98 × 1024) = 11164 m s–1 = 11.2 km s–1
R 6.4 × 106 ng
Kaedah alternatif / Alternative method

v= 2GM dan / and g = GM


la
R R2
v = 2gR
Pe

= 2 × 9.8 × 6.4 × 106


= 11164 m s−1 = 11.2 km s−1

n
ita

Tugasan 3
rb

Jika jejari Bulan adalah 1.74 × 106 m, dan jisim Bulan ialah 7.35 × 1022 kg, apakah halaju lepas yang diperlukan untuk
meninggalkan Bulan?
If the radius of the Moon is 1.74 × 10 6 m, and the mass of the Moon is 7.35 × 10 22 kg, what is the escape velocity needed to reach in order to leave
ne

the Moon?

2GM = 2(6.67 × 10−11)(7.35 × 1022) = 2374 m s–1


Pe

The escape velocity / Halaju lepas, v =


R 1.74 × 106

87
  Fizik  Tingkatan 4  Bab 3 Kegravitian

PRAKTIS SPM 3
Soalan Objektif

1. Hukum Kegravitian Semesta Newton menyatakan 4. Jika jejari Bumi itu dikurangkan dan jisimnya tetap
daya graviti antara dua objek berkadar terus dengan sama, apa yang berlaku kepada pecutan disebabkan
hasil darab jisim kedua-dua jasad itu dan berkadar oleh graviti di permukaan Bumi?
songsang dengan If the radius of the Earth were to be reduced and its mass were

.
Newton’s Universal Law of Gravitation states that the to remain the same, what happens to the acceleration due to

hd
gravitational force between two objects is directly proportional gravity on the surface of the Earth?
to the product of masses of the two objects and inversely A Tetap sama C Sifar
BAB

proportional to Remain the same Zero

.B
A jarak di antara dua objek. B Menurun D Bertambah
distance between the two objects. Decreases Increases
3 B kuasa dua hasil darab jejari jasad.
5. Antara berikut, yang manakah boleh diaplikasikan

dn
square of the product of the body’s radii.
C kuasa dua jarak di antara pusat dua jasad. oleh Hukum Kegravitian Semesta?
square of the distance between the centres of the two Which of the following can be applied by the Universal Law of
Gravitation?

iS
objects.
D kuasa tiga jarak di antara dua jasad dari pusat. A Planet di sekitar Matahari.
cube of the distance between the centres of the two The planets around the Sun.
objects. B Bumi dan bulan.
ng The earth and the moon.
2. Sekiranya jarak di antara dua zarah adalah dua kali C Bumi dan epal.
ganda, apakah yang berlaku kepada daya graviti di The earth and the apple.
la
antara dua zarah itu? D Sebarang pasangan jasad.
If the distance between two particles is doubled, what Any pair of bodies.
Pe

happened to the gravitational force between them?


6. Apakah berat sesuatu objek dalam satelit yang
A Dikurangkan kepada satu perempat daripada
mengorbit di sekeliling Bumi?
nilai asal
What is the weight of an object in a satellite orbiting around
Is reduced to one-fourth of the original value
n

the Earth?
B Dikurangkan kepada separuh daripada nilai asal
A Berat sebenar
ita

Is reduced to half of the original value


Actual weight
C Menjadi dua kali ganda
B Kurang daripada berat sebenar
Becomes double
Less than the actual weight
rb

D Menjadi empat kali ganda


C Lebih besar daripada berat sebenar
Becomes four times
Greater than the actual weight
ne

D Sifar
3. Jika jisim Bumi menjadi empat kali, apakah yang akan
Zero
berlaku kepada nilai g?
If the mass of the Earth becomes four times, what will happen 7. Apakah yang menyebabkan pergerakan Bulan pada
Pe

to the value of g? orbitnya?


A Double What is the reason for the motion of the Moon on its orbit?
Dua kali ganda A Daya graviti yang dikenakan di Bumi oleh Bulan.
B Tiada kesan The gravitational force exerted on the Earth by the Moon.
No effect B Daya graviti yang dikenakan oleh Matahari.
C Empat kali The gravitational force exerted by the Sun.
Four times C Daya graviti yang diterapkan oleh planet-planet.
D Separuh The gravitational force exerted by the planets.
Half D Daya graviti yang dikenakan pada Bulan oleh Bumi.
The gravitational force exerted on the Moon by the Earth.

88
Fizik  Tingkatan 4  Bab 3 Kegravitian 

8. Suatu objek dengan berat 200 N berada di permukaan 12. Tempoh satelit geopegun ialah
Bumi. Cari beratnya jika ia adalah 1 × 106 m di atas The period of geostationary artificial satellite is
permukaan. Diberikan G = 6.67 × 10–11 N m2 kg–2, A 24 jam C 48 jam
MBumi = 6.0 × 1024 kg, RBumi = 6.4 × 106 m. 24 hours 48 hours
An object weighs 200 N on the surface of the Earth. Find its B 6 jam D 12 jam
weight if it was 1 × 10 6 m above the surface. 6 hours 12 hours
Given, G = 6.67 × 10 –11 N m2 kg–2, MEarth = 6.0 × 1024 kg,
REarth = 6.4 × 10 6 m. 13. Tempoh satelit di orbit bulatan berjejari R adalah T.
A 73.1 N Tempoh satelit lain dalam orbit bulat berjejari 4R
adalah
B 146.2 N
The period of a satellite in a circular orbit of radius R is T. The
C 218.2 N

.
period of another satellite in circular orbit of radius 4R is

hd
D 292.2 N A 8T C 2T
9. Daya graviti pada wanita 50 kg ialah 500 N. Wanita B T D T

BAB
8 4

.B
itu juga mengenakan daya tarikan graviti ke Bumi.
Berapa besar daya ini?
14. Jarak Zuhrah dan Zuhal dari Matahari masing-masing
The force of gravity on a 50 kg woman is 500 N. The woman
also exerts a gravitational force on the Earth. How large a force ialah hampir 1011 m dan 1012 m. Dengan anggapan 3

dn
is this? bahawa kedua-dua planet bergerak dalam orbit bulat,
A 0N nisbah tempoh kedua-dua planet ialah
The distance of Venus and Saturn from the Sun are nearly
B 245 N

iS
1011 m and 1012 m respectively. Assuming that they move in
C 500 N circular orbits, their periodic times would be in the ratio of
D 980 N ng A 0.1 0.1 C 10 10
10. Daya graviti antara dua jisim ialah 120 N apabila
B 100 D 10000
jaraknya ialah 20 m. Jika jarak di antara dua jisim
berubah dan daya graviti menjadi 480 N, berapakah 15. Hukum Kepler Ketiga menyatakan
la
jarak baharu di antara dua jisim tersebut? Kepler's Third Law states
The gravitational force between two masses was 120 N when A orbit planet di sekeliling Matahari adalah elips,
Pe

they were 20 m apart. If the distance between them was


dengan matahari pada satu fokus.
changed and the gravitational force became 480 N, how far
the orbit of a planet around the sun is an ellipse, with the
apart were they then?
sun at one focus.
A 2m B paksi semimajor adalah sama dengan jarak purata
n

B 8m planet dari Matahari.


C 10 m the semimajor axis is equal to the planet's average
ita

D 15 m distance from the Sun.


C kuasa dua tempoh orbit planet adalah berkadar
11. Satu garisan yang dilukis dari sebuah planet ke terus dengan kuasa tiga jejari orbitnya.
rb

Matahari sentiasa meliputi kawasan dengan luas yang the square of the orbital period of a planet is proportional
sama dalam selang masa yang sama, apabila planet to its orbital radius.
ne

itu bergerak lebih jauh dari Matahari, planet akan D tiada jawapan betul.
bergerak dengan none of these answers is correct.
A line drawn from a planet to the Sun always sweeps over
Pe

equal areas in equal intervals of time When the planet moves 16. Jika r mewakili jejari orbit sebuah satelit berjisim m
further from the Sun, the planet will move yang bergerak dalam orbit membulat di sekeliling
A cepat sebuah planet berjisim M, halaju satelit itu diberikan
faster oleh
B lambat If r represents the radius of the orbit of a satellite of mass m
slower moving round a planet of mass M, the velocity of the satellite is
C tetap pada setiap titik given by
constant at every point A v = (GM/r) C v = GMm
D tiada satu pun di atas r2
B v2 = GMm gM
none of the above D v2 =
r R

89
  Fizik  Tingkatan 4  Bab 3 Kegravitian

Soalan Struktur

Bahagian A

1. Rajah 1 menunjukkan dua jasad dengan jisim M dan m masing-masing. Kedua-dua jasad itu berada di angkasa lepas.
Diagram 1 shows two bodies with mass of M and m respectively. Both bodies are placed in space.

M m
720 N

.
hd

Rajah 1 / Diagram 1
BAB

.B
Kekuatan daya tarikan graviti antara dua jisim adalah 720 N.
The strength of gravitational force between the two bodies is 720 N.

3 (a) Berdasarkan Hukum Kegravitian Semesta Newton, lengkapkan ayat berikut.

dn
Based on Newton’s Universal Law of Gravitation, complete the following sentence.
Semakin besar jisim kedua-dua jasad, semakin besar daya tarikan graviti.

iS
Daya tarikan graviti berkurang apabila jarak di antara kedua-dua jasad bertambah.
The greater the mass of both bodies, the greater the force of gravitational attraction.
decreases
The force of gravitational attraction
ng
as the distance between the two bodies increases.
[2 markah / marks]
(b) Apakah nilai daya tarikan graviti yang akan berlaku jika satu daripada jisim menjadi dua kali ganda dan jarak di
la
antara dua jasad tidak berubah?
What will be the force of gravitational attraction if one of the mass is doubled and the distance between them unchanged?
Pe

F = G Mm
R2
720 = G Mm
n

R2
Jika satu daripada jisim menjadi dua kali ganda dan jarak di antara dua jasad tidak berubah,
ita

If one mass is doubled and the distance remains the same,


Fnew = G 2Mm = G 2Mm = 2G  Mm = 2(720) = 1440 N
R2 R2 R2
rb

[2 markah / marks]
(c) Apakah nilai daya tarikan graviti yang akan berlaku jika satu daripada jisim menjadi setengah kali ganda dan jarak
ne

di antara dua jasad bertambah tiga kali ganda?


What will be the force if one mass is halved and the distance between them is tripled?

F = G Mm
Pe

R2
720 = G Mm
R2
Jika satu daripada jisim menjadi separuh dan jarak menjadi tiga kali ganda,
If one of the mass is halved and the distance is tripled,
1 1
2 Mm 2 Mm
Fnew = G  = G  = 1  G  Mm = 1  (720) = 40 N
(3R)2 9R2 18 R2 18 [2 markah / marks]

90
Fizik  Tingkatan 4  Bab 3 Kegravitian 

(d) Tanpa menggunakan pengiraan, nyatakan apa yang berlaku kepada magnitud daya tarikan graviti jika jarak di
antara dua jasad dikurangkan menjadi separuh?
Without using calculation, state what happened to the magnitude of gravitational force attraction if the distance between two
bodies is halved?

Bertambah empat kali ganda / Become quadruple.


[1 markah / mark]

2. Rajah 2 di bawah menunjukkan kedudukan Matahari, Bumi dan Bulan.


Diagram 2 below shows the position of the Sun, Earth and Moon

.
hd
Bulan/ Moon
Jisim/ Mass = 7.4 × 1022 kg

BAB
.B
Matahari/ Sun
Jisim/ Mass = 2 × 1030 kg
Jarak ke Bumi/ Distance to the Earth
= 1.5 × 1011 m
3

dn
iS
Bumi/ Earth
Jisim/ Mass = 6 × 1024 kg
ng Jejari/ Radius = 6.4 × 106 m
la
Pe


Rajah 2 / Diagram 2

(a) Nyatakan Hukum Kegravitian Semesta Newton.


n

State the Newton’s Law of Universal Gravitation.


Hukum Kegravitian Semesta Newton menyatakan bahawa daya graviti antara dua jasad adalah berkadar terus
ita

dengan hasil darab jisim kedua-dua jasad dan berkadar songsang dengan kuasa dua jarak di antara pusat dua
rb

jasad tersebut.
Newton's law of universal gravitation states that the gravitational force between two objects is directly proportional to the product
ne

of their masses and inversely proportional to the square of the distance between their centres.
[2 markah / marks]
Pe

(b) Ungkapkan Hukum Kegravitian Semesta Newton dalam bentuk formula.


Express Newton’s Law of Universal Gravitation in formula form.

F = G Mm
R2
[1 markah / mark]

91
  Fizik  Tingkatan 4  Bab 3 Kegravitian

(c) Hitungkan daya graviti antara sebiji buah kelapa berjisim 1 kg dan Bumi.
Calculate gravitational force between a 1 kg coconut and the Earth.
(Diberikan / Given G = 6.67 × 10 –11 N m2 kg–2)
Jisim kelapa / Mass of the coconut = 1.0 kg
Jisim Bumi / Mass of the Earth = 5.97 × 1024 kg
Jarak di antara pusat kelapa dan pusat Bumi = 6.4 × 106 m
Distance between the centre of coconut and the centre of the Earth

F = G  Mm = 6.67 × 10 – 11 × 5.97 × 10 6×2 1 = 9.72 N


24

R2
(6.4 × 10 )

.
hd
[1 markah / mark]
(d) Hitungkan daya graviti antara Bumi dan Matahari.
BAB

Calculate gravitational force between the Earth and the Sun.

.B
(Diberikan / Given G = 6.67 × 10 –11 N m2 kg–2)
Jisim matahari / Mass of the Sun = 2 × 1030 kg
3 Jisim Bumi / Mass of the Earth = 5.97 × 1024 kg

dn
Jarak di antara pusat matahari dan pusat Bumi = 1.5 × 1011 m
Distance between the centre of the Sun and the centre of the Earth

F = G  Mm = 6.67 × 10 – 11 × 5.97 × 10 × 211 ×2 10 =3.54 × 1022 N

iS
24 30

R2
(1.5 × 10 )
ng [1 markah / mark]
(e) Daya graviti antara Bumi dan Bulan ialah 2.0 × 1020 N. Berapakah jarak di antara pusat Bumi dan pusat Bulan?
Gravitational force between the Earth and the Moon is 20 × 1020 N. What is the distance between the centre of the Earth and the
la
centre of the Moon?
Jisim bulan / Mass of the Moon = 7.4 × 1022 kg
Pe

Jisim Bumi / Mass of the Earth = 5.97 × 1024 kg


Jarak di antara pusat Bumi dan pusat Bulan / Distance between the centre of the Earth and the centre of the Moon = R m

F = G Mm
n

R2
2.0 × 1020 = 6.67 × 10 –11 × 5.97 × 10 ×2 7.4 × 10
24 22
ita


R
5.97 × 1024
× 7.4 × 1022 = 1.48 × 1017
R = 6.67 × 10  ×
2 –11
rb

2.0 × 10 20

R = 3.84 × 108 m
ne

[1 markah / mark]
Pe

3. Jadual berikut menunjukkan maklumat beberapa planet yang diperolehi pelajar.


The following table shows the information of a few planets obtained by the students.

Planet / Planet Jisim / Mass Jejari / Radius


(a) Bumi / Earth 6.0 × 1024 kg 6.4 × 106 m
(b) Marikh / Mars 6.4 × 1023 kg 3.4 × 106 m
(a) Uranus / Uranus 8.7 × 1025 kg 2.5 × 107 m

92
Fizik  Tingkatan 4  Bab 3 Kegravitian 

(a) Cari magnitud daya graviti yang dialami oleh suatu objek 50 kg apabila berada di atas permukaan setiap satu
daripada planet berikut.
Find the magnitude of the gravitational force a 50 kg object would experience when on the surface of each of the following planets.
(Diberikan / Given G = 6.67 × 10 –11 N m2 kg–2)
(i) Daya graviti Bumi
Gravitational force of the Earth

F = G  Mm
R2
F = 6.67 × 10 – 11 × 6 × 10 ×650
24
= 488.5 N
(6.4 × 10 )2

.
[1 markah / mark]

hd
(ii) Daya graviti Marikh
Gravitational force of the Mars

BAB
.B
F = G  Mm
R2
F = 6.67 × 10 – 11 × 6.4 × 10 ×6 250 = 184.6 N
23
3
(3.4 × 10 )

dn
[1 markah / mark]
(iii) Daya graviti Uranus

iS
Gravitational force of the Uranus

F = G  Mm
R2
ng
F = 6.67 × 10 – 11 × 8.7 × 10 ×7 250 = 464.23 N
25

(2.5 × 10 )
[1 markah / mark]
la
(b) Rajah 3 di bawah menunjukkan sebuah roket berada di tapak pelancaran.
Diagram 3 below shows a rocket at rocket launch site.
Pe

Roket
Rocket
n
ita

Bumi
rb

Earth
ne


Pe

Rajah 3 / Diagram 3

(i) Jika roket yang berada di tapak pelancaran mengalami daya graviti 5.0 × 105 N, berapakah jisim roket itu?
If the rocket at the launch site experienced gravitational force of 5.0 × 105 N, what is the mass of the rocket?

F = G  Mm
R2
5.0 × 105 = 6.67 × 10 – 11 × 6.4 × 10 ×6 2m
24

(6.4 × 10 )
5 × 105
× (6.4 × 10 )
6 2
m= = 47976 kg
6.67 × 10 –11 × 6.4 × 1024
[2 markah / marks]

93
  Fizik  Tingkatan 4  Bab 3 Kegravitian

(ii) Jika roket berada 100 km dari permukaan Bumi, apakah perubahan terhadap daya graviti yang bertindak ke
atas roket tersebut?
If the rocket at 100 km above the Earth surface, what is the change of gravitational force acted on the rocket?
(Bertambah / Increase, berkurang / decrease, sama / same)
[1 markah / mark]
(c) Mengikut Hukum Kegravitian Semesta Newton, daya graviti akan bertindak antara dua orang di permukaan
Bumi. Mengapakah dua orang di permukaan Bumi tidak akan merasai kesan daya graviti?
According to Newton’s Universal Law of Gravitation, the gravitational force is acted on two person on the surface of the Earth. Why
two people on the surface of the Earth will not feel the effects of gravity?
Daya graviti antara dua jasad berjisim kecil mempunyai magnitud yang sangat kecil.

.
hd
The gravitational force between two small mass bodies have small magnitude.
[2 markah / marks]
BAB

.B
4. Rajah 4 menunjukkan sebuah satelit berjisim m, pada ketinggian h dari permukaan Bumi berjisim, M. R merupakan
jejari Bumi dan h ialah jarak satelit itu dari permukaan Bumi.
3 Diagram 4 shows a satellite with mass m, at height h from the surface of the Earth with mass, M. R is the radius of the Earth and h is the

dn
distance between the satellite and the surface of the Earth.

iS
ng
Bumi
Earth
Satellite
la
h Satelit
R
Pe
n
ita


Rajah 4 / Diagram 4
rb

(a) Ungkapkan jejari orbit satelit dalam sebutan R dan h.


Express the radius of the satellite in terms of R and h.
ne

R+h
[1 markah / mark]
Pe

M untuk satelit pada ketinggian, h meter.


(b) Tunjukkan ungkapan pecutan graviti, g = G 
(R + h)2
M
Show the expression of gravitational acceleration, g = G  for the satellite at height, h meter.
(R + h)2
F = mg dan / and F = G Mm
R2
Mm
Maka / Then, mg = G  2
R
Oleh kerana jarak satelit dari pusat Bumi / Due to the satellite distance from the centre of the Earth = R + h
M
Maka / Then, mg = G 
(R + h)2
[2 markah / marks]

94
Fizik  Tingkatan 4  Bab 3 Kegravitian 

(c) Satelit tersebut mengorbit mengelilingi Bumi pada ketinggian 500 km. Berapakah nilai pecutan graviti di
kedudukan satelit tersebut?
(Diberikan G = 6.67 × 10 –11 N m2 kg–2, jisim Bumi M = 6 × 1024 kg, jejari Bumi R = 6.4 × 106 m)
The satellite orbiting around the Earth at height of 500 km. What is the value of gravitational acceleration athe satellite position?
(Given G = 6.67 × 10 –11 N m2 kg–2, mass of the Earth M = 6 × 1024 kg, radius of the Earth R = 6.4 × 106 m)
M 6 × 1024
g = G  2 = 6.67 × 10
 –11
× = 8.41 m s–2
(R + h) (6.4 × 106 + 500 000)2
[1 markah / mark]
(d) Hitungkan pecutan graviti di permukaan Bumi.

.
(Diberikan G = 6.67 × 10 –11 N m2 kg–2, jisim Bumi = 5.97 × 1024 kg, jejari Bumi = 6.37 × 106 m)

hd
Calculate the gravitational acceleration at the surface of the Earth.
(Given G = 6.67 × 10 –11 N m2 kg–2, mass of the Earth = 5.97 × 1024 kg, radius of the Earth = 6.37 × 106 m)

BAB
mg = G Mm , maka g = G  M2 = 6.67 × 10 –11 × 5.97 × 106 2 = 9.81 m s–2
24

.B

R2
R (6.37 × 10 )
[1 markah / mark]
3

dn
(e) Daripada jawapan (c) dan (d), apakah faktor yang menentukan nilai pecutan graviti? Terangkan hubungan antara
faktor tersebut dengan nilai pecutan graviti.
From the answer (c) and (d), what is the factor to determine the value of gravitational acceleration? Explain the relationship between

iS
the factor and the value of gravitational acceleration.
Ketinggian objek. Semakin besar ketinggian objek, semakin kecil nilai pecutan graviti.
ng
The height of the object. The higher the height of the object, the smaller the value of the gravitational acceleration.
[1 markah / mark]
la
Bahagian B
Pe

5. Rajah 5.1 menunjukkan dua buah satelit, satelit A dan satelit B berjisim m, di mana satelit A berada di permukaan Bumi
dan satelit B berada pada ketinggian h dari permukaan Bumi berjisim, M. R merupakan jejari Bumi dan h ialah jarak
satelit itu dari permukaan Bumi.
Diagram 5.1 shows two satellites, satellite A and satellite B with mass m, where satellite A is on the surface of the Earth and satellite B is at
n

the height h from the surface of the Earth with mass, M. R is the radius of the Earth and h is the distance of the satellite from the surface of
the Earth.
ita
rb
ne

Bumi
Earth
Pe

h
R Satelit A Satelit B
Satellite A Satellite B

Rajah 5.1 / Diagram 5.1

95
  Fizik  Tingkatan 4  Bab 3 Kegravitian

(a) Nyatakan Hukum Kegravitian Semesta Newton.


State the Newton’s Universal Law of Gravitation. [1 markah / mark]
(b) Dengan menggunakan Rajah 5.1,
By using Diagram 5.1,
(i) bandingkan jarak dari pusat Bumi, nilai pecutan graviti dan berat kedua-dua satelit,
compare the distance from the centre of the Earth, the gravitational acceleration value and the weight of both satellites,
[3 markah / marks]
(ii) nyatakan hubungan antara jarak dari pusat Bumi dan berat satelit,
state the relationship between the distance from the centre of the Earth and the weight of satellites, [1 markah / mark]

.
(iii) nyatakan kuantiti fizik yang malar dalam mendeduksikan antara jarak dari pusat bumi dan berat satelit

hd
seperti dalam jawapan 5(b)(ii).
state the physical quantity that is constant to deduce the relationship between the distance from the centre of the Earth and
BAB

the weight of the satellite as in answer 5(b) (ii). [1 markah / mark]

.B
(c) Sebuah satelit berjisim 200 kg mengorbit mengelilingi Bumi pada ketinggian 500 km. Diberikan jisim Bumi ialah
5.97 × 1024 kg, jejari Bumi ialah 6.37 × 106 m dan G = 6.67 × 10 –11 N m2 kg–2.
3

dn
Berapakah nilai pecutan graviti dan berat satelit apabila
A 200 kg satellite orbiting the Earth at an altitude of 500 km. Given that the mass of the Earth is 5.97 × 1024 kg, the radius of the Earth
is 6.37 × 10 6 m and G = 6.67 × 10 –111 N m2 kg–2.
What is the value of the gravitational acceleration and the weight of the satellite when

iS
(i) berada di permukaan Bumi?
on the surface of the Earth? ng
(ii) berada pada ketinggian 500 km dari permukaan Bumi?
at an altitude of 500 km from the surface of the Earth?
[4 markah / marks]
la
(d) NASA akan menggunakan roket Falcon untuk melancarkan satelit berkomunikasi canggih.
NASA will use its Falcon rocket to launch an advanced communication satellite.
Pe
n
ita
rb
ne
Pe


Rajah 5.2 / Diagram 5.2

Anda dikehendaki mencadangkan ciri-ciri roket yang sesuai yang boleh bergerak dengan pecutan yang lebih
besar dengan ciri keselamatan. Nyatakan dan terangkan cadangan anda berdasarkan ciri-ciri bahan binaan dan
bentuk roket.
You are required to suggest the suitable characteristics of the rocket that can travel with greater acceleration with safety features.
State and explain your suggestions based on the characteristics of material and shape of the rocket.
[10 markah / marks]

96
Fizik  Tingkatan 4  Bab 3 Kegravitian 

Bahagian C

6. Rajah 6.1 menunjukkan satelit MEASAT mengelilingi Bumi.


Diagram 6.1 shows the MEASAT satellite orbits around the Earth.

Tempoh, T(24 jam)


Period, T(24 hours)

Bumi
Earth MEASAT

.
hd
Jejari orbit, R

BAB
Orbital radius, R

.B

Rajah 6.1 / Diagram 6.1

(a) Apakah yang dimaksudkan dengan halaju lepas? 3

dn
What is meant by escape velocity? [1 markah / mark]
(b) Terangkan bagaimana satelit MEASAT boleh dibebaskan daripada tarikan graviti Bumi dengan menggunakan

iS
formula halaju lepas, v = 2GM .
R
Explain how the MEASAT satellite could be released from gravitational pull of the Earth by using formula of escape velocity,
ng
v= 2GM . [4 markah / marks]
R

(c) Satelit geopegun kekal di atas titik yang sama di Bumi kerana ia mengorbit. Ia kekal sebagai R jarak malar dari
la
pusat Bumi.
A geostationary satellite remains above the same point on the Earth as it orbits. It remains as constant distance R from the centre of
Pe

the Earth.

Tempoh, T = 24 jam
n

Period, T = 24 hours
ita

Bumi
Earth
Satelit geopegun
Geostationary satellite
rb

Jejari orbit, R
Orbital radius, R
ne


Rajah 6.2 / Diagram 6.2
Pe

(i) Tulis satu ungkapan, dalam sebutan R, untuk jarak ia bergerak dalam masa 24 jam.
Write down an expression, in terms of R, for the distance it travels in 24 hours.
(ii) Tuliskan, dalam sebutan R, ungkapan untuk kelajuannya dalam m s–1.
Write down, in terms of R, an expression for its speed in m s–1.
(iii) Dapatkan nilai R.
Find the value of R.
(G = 6.67 × l0 –11 N m2 kg–1; mass of Earth = 5.97 × 1024 kg)
[3 markah / marks]

97
  Fizik  Tingkatan 4  Bab 3 Kegravitian

(d) Untuk siaran secara langsung Piala Dunia bola sepak ke seluruh dunia, satu rangkaian satelit komunikasi diperlukan
untuk merangkumi seluruh Bumi. Untuk tujuan ini, beberapa satelit perlu ditempatkan di atas permukaan Bumi
pada tempat-tempat yang tertentu.
To broadcast live World Cup football to the world, a network of communications satellites needed to cover the entire Earth. For this
purpose, some satellites need to be placed above the surface of the Earth at certain places.
Jadual 6 menunjukkan ciri-ciri bagi sistem satelit yang berbeza.
Table 6 shows the characteristics of different satellite systems.

Satelit Jenis satelit Sudut liputan Tempoh orbit Bilangan minimum satelit
Satelitte Type of satellite Angle of coverage Period of orbit Minimum number of satellite

.
hd
Satelite geopegun
P 80º 24 jam/ hour 3
Geostationary satellite
BAB

.B
Satelite bukan geopegun
Q 120º 36 jam / hour 5
Non-geostationary satellite

3 Satelite geopegun

dn
R 120º 24 jam / hour 3
Geostationary satellite

Satelite bukan geopegun

iS
T 80º 36 jam / hour 5
Non-geostationary satellite

Jadual 6 / Table 6
ng
Kaji spesifikasi keempat-empat ciri bagi sistem tersebut. Terangkan kesesuaian setiap spesifikasi untuk semua
satelit dan tentukan sistem satelit yang paling sesuai digunakan untuk siaran secara langsung pertandingan Piala
Dunia. Beri sebab untuk pilihan anda. 5
la
Study the four specifications of the system above. Explain the suitability of each specification for all satellites and determine the most
suitable satellite system for live broadcast of World Cup matches. Give reasons for your choice. [10 markah / marks]
Pe
n
ita
rb
ne
Pe

98
PRAKTIS SPM 3
JAWAPAN (d)

Soalan Struktur Suggestions Kuantiti Vektor


Bahagian B Cadangan Vector Quantities

5. (a) Hukum Kegravitian Semesta Newton menyatakan Badan roket dibuat dari logam Boleh menahan tekanan
bahawa daya tarikan graviti di antara dua jasad adalah yang sangat kuat dan keras. dan daya yang tinggi tanpa

.
berkadar terus dengan hasil darab jisim-jisimnya dan The body of rocket is built with metal mengalami kerosakan.

hd
berkadar songsang dengan kuasa dua jarak pemisahan with high strength and hardness. Able to withstand high pressure and
di antara pusat jasad-jasad itu. force without causing damage.
The law of universal gravitation states that the gravitational
attraction between two bodies is directly proportional to the Rangka roket diperbuat daripada Lebih ringan, boleh

.B
product of the both masses and inversely proportional to the logam berketumpatan rendah. meningkatkan pecutan.
square of the distance between the centres of the bodies. Frame of rocket is made of low Lighter, can increase acceleration.
(b) (i) Jarak satelit A dari pusat Bumi adalah lebih kecil density metal.
dari satelit B. Nilai pecutan graviti satelit A adalah
Roket dibina dengan peringkat Setiap peringkat dengan bahan

dn
lebih besar dari satelit B. Berat satelit A adalah
yang boleh ditanggalkan. api, pam dan kebuk pembakaran
lebih besar dari Satelit B.
Rocket built with removable stage. berasingan. Ia akan
The distance of satellite A from the centre of the Earth
is smaller than satellite B. The gravitational acceleration ditanggalkan apabila bahan

iS
value of satellite A is greater than satellite B. The weight of bakar dibakar sepenuhnya. Bila
satellite A is greater than Satellite B. jisim berkurangan, pecutan akan
(ii) Lebih besar jarak dari pusat bumi, lebih kecil berat bertambah.
satelit. Each stage with separate fuel, pump
The greater the distance from the centre of the Earth, the
ng and combustion chamber. It will
be removed when the fuel is fully
smaller the weight of the satellite.
(iii) Jisim / Mass burned. As the mass decreases, the
acceleration increases.
(c) (i) berada di permukaan Bumi
la
on the surface of the Earth Badan roket diliputi dengan Geseran antara udara dan
M = 5.97 × 1024 kg penebat haba. badan akan meningkatkan haba.
Pe

R = 6.37 ×106 m The body of rocket covered with heat Penebat akan mengurangkan
G = 6.67 × 10–11 N m2 kg–2 insulator. kekonduksian haba.
Mm The friction between the air and the
mg = G  2 body increases the heat. Insulation
r
will reduce heat conductivity.
GM 5.97 × 1024
n

g = 2 = 6.67 × 10–11 ×
r (6.37 × 106)2 Bentuk aerodinamik Kurangkan geseran udara,
Aerodynamic shape bergerak lebih cepat.
ita

= 9.813 m s–2
Reduce air friction, move faster.
Berat / Weight = 200 × 9.813
= 1962 N Bahagian C
rb

(ii) berada pada ketinggian 500 km dari permukaan 4. (a) Halaju lepas adalah kelajuan minimum yang diperlukan
Bumi objek untuk untuk dibebaskan daripada pengaruh graviti
at an altitude of 500 km from the surface of the Earth jasad yang besar.
ne

M = 5.97 × 1024 kg Escape velocity is the minimum speed required for objects to be
released from the gravitational influence of a massive body.
R = 6.37 × 106 m
h = 550 × 103 m (b) – Satelit MEASAT boleh dibebaskan daripada tarikan
Pe

G = 6.67 × 10–11 N m2 kg–2 graviti Bumi apabila tenaga kinetiknya sama


Mm dengan tenaga keupayaan graviti.
mg = G  2 The MEASAT satellite can be released from gravitational
r
pull of the Earth when its kinetic energy is equal to the
M 5.97 × 1024 gravitational potential energy.
g = G  = 6.67 × 10–11
(r + h)2 (6.37 × 106 + 500 000)2
– Tenaga kinetik Satelit MEASAT dengan jisim m
= 8.44 m s–2 yang bergerak dengan halaju v boleh diberikan
1
Berat / Weight = 200 × 8.44 dengan EK = mv2.
2
= 1688 N The kinetic energy of the MEASAT satellite with mass m
1
moving with velocity v can be given by EK = mv2.
2
  Fizik  Tingkatan 4 Jawapan


Tenaga keupayaan graviti satelit MEASAT boleh (d) – Jenis satelit: Satelit geopegun
Mm Types of satellite: Geostationary satellite
diberikan dengan formula G  di mana
R Supaya berada di atas kawasan yang sama pada
G = pemalar kegravitian setiap masa.
(G = 6.67 × 10–11 N m2 kg–2) So that in the same spot above the same area at all times.
The gravitational potential of the MEASAT satellite can be – Sudut liputan: 120º
Mm
given by the formula G  where Angle of coverage: 120º
R
Supaya dapat meliputi kawasan yang lebih luas
G = gravity constant kegravitian (G = 6.67 × 10–11 Nm2 kg–2)
In order to cover a wider area
1 Mm – Tempoh orbit / Duration of orbit:
– Menyamakan kedua, kita dapat: mv2 = G 
2 R 24 jam / hour
1 2 Mm
Equating the two formulas, we obtain: mv = G  Supaya ia mengorbit pada kelajuan yang sama
2 R
seperti Bumi berputar
2GM

.
Maka, halaju lepas, v = So that it orbit at the same speed as the earth rotating.

hd
R – Bilangan minimum satelit: 3
2GM Minimum number of satellites: 3
So, escape velocity, v =
R Menjimatkan kos untuk siaran langsung meliputi
seluruh dunia.
(c) (i) Gerakan dalam bulatan, maka jarak ialah ukur lilit

.B
Save costs for covering live events around the world.
bulatan = 2πR Sistem satelit yang paling sesuai: R
Movement in a circle, so the distance is a circle's
circumference = 2πR
The most suitable satellite system: R
– Sebab sistem satelit R ialah satelite geopegun,
(ii) Masa satu putaran orbit

dn
sudut liputan ialah 120º, tempoh orbit ialah 24 jam
Time of one rotation orbit dan bilangan minimum satelit ialah 3.
= 24 × 60 × 60 = 86 400 s Because satellite system R is geostationary satellite, 120º
s 2πR angle of coverage, duration orbit is 24 hours, and the
v = =

iS
t 86 400 minimum number of satellites is 3.

(iii)
Fmemusat = Fgraviti
v 2 Mm
mS  = G  2 S
R
M
R
ng
v 2 = G 
R

2πR 2
( )
R = 6.67 × l0–11(5.97 × 1024)
la
86 400
86 4002
R3 = (6.67 × l0–11) (5.97 × 1024)
4π2
Pe

R = 4.2 × 107 m
n
ita
rb
ne
Pe
B
BA
4
Haba
Heat
Analisis Soalan SPM
Kertas 2011 2012 2013 2014 2015 2016 2017 2018
1 ✔ ✔ ✔ ✔ ✔ ✔ ✔ ✔

PETA Konsep 2
3


✔ ✔ ✔ ✔ ✔ ✔ ✔

.
hd
Keseimbangan terma berkait dengan Suhu diukur dengan Termometer
Thermal equilibrium relates to Temperature measured by Thermometer

.B
satu contoh
an example

Cecair-dalam-kaca

dn
Keabadian tenaga Liquid-in-glass
Conservation of energy

termasuk Cerek

iS
including
Kettle
Muatan haba tentu, Pemanasan dan diaplikasikan
Specific heat capacity, melibatkan dalam
Sistem penyejukan
Q involving
ng penyejukan applied in
c = mq Heating and cooling enjin kereta
Cooling system of a car
engine
la

Pelakuran Pepejal kepada cecair


Pe
HABA

Fusion Solid to liquid


HEAT

Haba pendam tentu,


Specific latent heat, dua jenis
Q two types
l= m
n

Pengewapan Cecair kepada gas


menyebabkan Vaporisation Liquid to gas
ita

causes

Perubahan fasa tanpa perubahan dalam Suhu


Change of phase without change in Temperature
rb
ne

Teori kinetik
Kinetic theory
Hukum Boyle boleh dinyatakan sebagai
menerangkan P1V1 = P2V2
Boyle’s law can be expressed as
Pe

sifat-sifat gas
explain properties
of gas

Hukum-hukum gas Hukum Charles boleh dinyatakan sebagai V1 V2


= T
Gas laws Charles’ law can be expressed as T1 2

Hukum Gay-Lussac boleh dinyatakan sebagai P1 P2


= T
Gay-Lussac law can be expressed as T1 2

99
  Fizik  Tingkatan 4  Bab 4 Haba

Memahami Keseimbangan Terma


4.1 Understanding Thermal Equilibrium

Keseimbangan Terma
Thermal Equilibrium

1. Apabila dua objek dengan suhu yang berbeza berada dalam sentuhan terma, tenaga haba dipindahkan antara
dua objek itu. Haba dipindahkan dalam kedua-dua arah . 1 2

When two objects of different temperatures come into thermal contact, heat energy is transferred between the two objects. The
heat is transferred in both directions .

.
hd
(a) Haba dipindahkan dalam kuantiti yang lebih besar.
Heat is transferred in a greater quantity.
Suhu A = Suhu B

.B
Temperature A = Temperature B

A B A B

dn
BAB

Panas Sejuk
Hot Cold

iS
(b) Haba dipindahkan dalam kuantiti yang lebih kecil. Tiada pengaliran bersih haba.
4 No net flow of heat.

Heat is transferred in a smaller quantity.
ng
2. Dua objek, A dan B dikatakan dalam keseimbangan terma jika kadar pengaliran bersih haba antara
la
objek-objek adalah sama . Tidak ada pengaliran bersih haba antara objek A dengan objek B.
The two objects, A and B are said to be in thermal equilibrium if the net rate of heat flow between the objects is equal .
Pe

There is no net flow of heat between objects A and B.

3. Jadual di bawah menerangkan keadaan sebelum dan selepas objek A dan objek B mencapai keseimbangan terma.
The table below explains the situation before and after object A and object B reach thermal equilibrium.
n

Sebelum keseimbangan terma Selepas mencapai keseimbangan terma


ita

Before thermal equilibrium After reaching thermal equilibrium

A dan B mempunyai suhu yang berbeza . A dan B mempunyai suhu yang sama .
rb

A and B have different temperatures. A and B have the same temperature.


ne

A hilang tenaga manakala B terima tenaga. Tidak ada pemindahan bersih tenaga.
A loses energy while B gains energy. transfer
There is no net of energy.
Pe

Tenaga dipindahkan pada kadar yang lebih tinggi dari Tenaga dipindahkan pada kadar yang sama
A ke B. antara A dengan B.
higher rate Energy is transferred at equal rate between A and B.
Energy is transferred at a from A to B.

4. Dua objek berada dalam keseimbangan terma apabila / Two objects are in thermal equilibrium when
(a) kedua-dua objek itu mempunyai suhu yang sama ,
same
both the objects have the temperature,
(b) tidak ada pemindahan bersih tenaga antara dua objek itu.
there is no net transfer of energy between the two objects.

100
Fizik  Tingkatan 4  Bab 4 Haba 

5. Contoh keseimbangan terma. / An example of thermal equilibrium.


Apabila makanan disimpan di dalam peti sejuk, haba daripada makanan dipindahkan
ke dalam udara di dalam peti sejuk. Pada keseimbangan terma, suhu makanan adalah
sama dengan suhu udara di dalam peti sejuk. Oleh itu, kesegaran makanan
dapat dikekalkan.
When food is kept in a refrigerator, the heat from the food is transferred into the air inside the

refrigerator. At thermal equilibrium, the temperature of the food is equal to the temperature

of the air inside the refrigerator. Thus, the freshness of food can be maintained.

.
hd
Termometer Cecair-dalam-Kaca
Liquid-in-Glass Thermometer

.B
1. Suhu ialah ukuran darjah kepanasan suatu objek.
Temperature is a measure of the degree of hotness of an object.

dn
2. Unit S.I. bagi suhu ialah kelvin (K) . Unit yang biasa digunakan bagi suhu ialah darjah Celsius (°C).

BAB
The S.I. unit of temperature is kelvin (K) . The commonly used unit of temperature is degree Celsius (°C).

iS
3. Suhu diukur dengan termometer . Termometer yang paling biasa digunakan ialah termometer
cecair-dalam-kaca. 4
thermometer
ng
Temperature is measured with a . The most commonly used thermometer is the liquid-in-glass thermometer.

Merkuri / Mercury
la
Pe

Bebuli / Bulb Tiub kapilari / Capillary tube

4. Termometer cecair-dalam-kaca berfungsi berdasarkan pengembangan dan pengecutan merkuri


dengan suhu.
n

The liquid-in-glass thermometer functions based on the expansion and contraction of mercury with temperature.
ita

5. Semua termometer baharu perlu ditentukur sebelum dapat digunakan.


All newly built thermometers need to be calibrated before it can be used.
rb

Termometer (a) Bebuli termometer yang belum ditentukur diletakkan di dalam


Thermometers
kiub ais . Aras merkuri yang sepadan kemudian ditandakan
ne

100°C
pada batang termometer sebagai 0°C .
Stim
0°C Steam The bulb of an uncalibrated thermometer is placed in ice cubes .
Pe

The corresponding level of mercury is then marked on the stem of the


Ais
Ice thermometer as 0°C .
(b) Kemudian, letakkan bebuli termometer itu di dalam stim .
Aras merkuri yang sepadan kemudian ditandakan pada batang
termometer sebagai 100°C .
Then, place the bulb of the thermometer in steam . The
l0 lθ
corresponding level of mercury is then marked on the stem of the
l100
thermometer as 100°C .

101
  Fizik  Tingkatan 4  Bab 4 Haba

(c) Panjang turus merkuri antara takat bawah yang ditetapkan ( 0°C ) dengan takat atas yang ditetapkan
( 100°C ) kemudian dibahagikan kepada 100 bahagian yang sama.
The length of mercury column between the lower fixed point ( 0°C ) and upper fixed point ( 100°C ) is then
divided into 100 equal divisions.
(d) Termometer itu sekarang sudah ditentukur dan boleh digunakan untuk mengukur suhu antara 0°C
dengan 100°C .
The thermometer is now calibrated and can be used to measure temperature between 0°C and 100°C .

6. Suhu, θ dapat ditentukan dengan rumus:

.
Temperature, θ can be determined using the formula:

hd
lθ – l0
θ= × 100 di mana / where
l100 – l0
lθ = panjang turus merkuri pada θ°C

.B
length of mercury column at θ°C
l0 = panjang turus merkuri pada takat bawah yang ditetapkan (0°C)
length of mercury column at lower fixed point (0°C)

dn
l100 = panjang turus merkuri pada takat atas yang ditetapkan (100°C)
length of mercury column at upper fixed point (100°C)
BAB

Contoh 1

iS
Panjang turus merkuri dalam sebuah termometer merkuri yang belum ditentukur ialah 3 cm apabila bebuli
4 direndamkan di dalam ais yang sedang melebur. Apabila bebuli termometer itu diletakkan di dalam stim,
ng
panjang turus merkuri meningkat sehingga 28 cm. Berapakah suhu jika panjang turus merkuri itu ialah 23 cm?
The length of mercury column in an uncalibrated mercury thermometer is 3 cm when its bulb is immersed in melting
ice. When the thermometer bulb is placed in steam, the length of mercury column increases to 28 cm. What would the
la
temperature be if the length of the mercury column is 23 cm?

Penyelesaian / Solution:
Pe

Diberi / Given: lθ = 23 cm, l0 = 3 cm, l100 = 28 cm, θ = ?


lθ – l0 23 – 3
θ= × 100 = × 100 = 80°C
l100 – l0 28 – 3
n


ita

7. Merkuri digunakan dalam termometer kerana


Mercury is used in the thermometer because
legap
rb

(a) dan boleh dilihat dengan mudah.


it is opaque and can be easily seen.
ne

(b) mengembang dan mengecut dengan cepat dan seragam .


it expands and contracts rapidly and uniformly .
melekat
Pe

(c) tidak pada dinding kaca tiub kapilari.


it does not stick to the glass wall of the capillary tube.
(d) konduktor haba yang baik .
it is a good conductor of heat.

8. Termometer merkuri mengandungi satu bebuli berdinding nipis yang diisi dengan merkuri. Bebuli
berdinding nipis itu akan menyebabkan termometer peka terhadap perubahan suhu. Bebuli itu
dihubungkan pada tiub kapilari.
Mercury thermometer consists of a thin-walled glass bulb which is filled with mercury. The thin-walled glass bulb will
cause the thermometer to be sensitive to the changes in temperature. It is connected to a capillary tube.

102
Fizik  Tingkatan 4  Bab 4 Haba 

9. Apabila suhu meningkat, merkuri mengembang di dalam tiub kapilari. Oleh itu, panjang turus
merkuri meningkat apabila suhu meningkat. Perubahan panjang turus merkuri akan memberi bacaan pada
termometer mengikut skala yang dilabelkan.
When temperature increases, the mercury expands in the capillary tube. Hence, the length of the mercury
column increases as the temperature increases. Changes in the length of mercury column will give the thermometer reading
based on the labelled scale.

10. Kepekaan suatu termometer dapat dipertingkatkan dengan cara-cara yang berikut:
The sensitivity of a thermometer can be increased in the following ways:

Cara / Way Sebab / Reason

.
hd
Bebuli yang lebih kecil Bebuli yang lebih kecil akan menyerap tenaga haba dalam masa yang lebih
A smaller bulb singkat dan isi padu akan berubah lebih cepat dengan perubahan suhu.

.B
A smaller bulb will absorb heat energy in a shorter time and its volume will
change faster with changes in temperature.

dn
Bebuli kaca dengan dinding nipis Haba dari persekitaran dapat dipindahkan ke merkuri dengan lebih cepat.
thinner wall transferred

BAB
A glass bulb with Heat from the surrounding can be to the mercury faster.

iS
Tiub kapilari yang lebih sempit Tiub kapilari yang lebih sempit akan menghasilkan turus merkuri yang
lebih panjang kerana peningkatan dalam merkuri adalah tetap.
Narrower capillary tube 4
ng
A narrower capillary tube will produce a longer mercury column because the
increase in the mercury is constant.
la

Memahami Muatan Haba Tentu


4.1
Pe

Understanding Specific Heat Capacity

Muatan Haba Tentu


Specific Heat Capacity
n

1. Muatan haba suatu objek ialah jumlah haba yang diperlukan untuk meningkatkan suhu objek itu
ita

sebanyak 1°C.
The heat capacity of an object is the amount of heat required to increase the temperature of the object by 1°C.
rb

2. Muatan haba suatu objek bergantung pada / The heat capacity of an object depends on the
(a) jisim objek (b) jenis bahan
ne

mass material
of the object type of

3. Muatan haba tentu suatu bahan ialah jumlah haba yang diperlukan untuk meningkatkan suhu
Pe

1 kg bahan itu sebanyak 1°C .

Specific heat capacity of a substance is the amount of heat required to increase the temperature of 1 kg of the
substance by 1°C .
Q
Muatan haba tentu / Specific heat capacity, c =
mq
di mana / where Q = jumlah haba yang dibekalkan / amount of heat supplied
m = jisim bahan / mass of the substance
θ = perubahan suhu / changes in temperature

103
  Fizik  Tingkatan 4  Bab 4 Haba

4. Unit S.I. bagi muatan haba tentu ialah J kg °C J kg–1 K –1 .


–1 –1
atau
The S.I. unit for specific heat capacity is J kg–1 °C–1 or J kg–1 K–1 .

Contoh 2
Suatu cecair berjisim 2 kg disejukkan daripada 40°C sehingga 10°C. Jika jumlah kehilangan haba daripada
cecair itu 120 000 J, berapakah muatan haba tentu cecair tersebut dalam J kg–1 °C–1?
A liquid of mass 2 kg is cooled from 40°C to 10°C. If the total heat loss from the liquid is 120 000 J, what is its specific heat
capacity in J kg–1 °C–1?

.
Penyelesaian / Solution:

hd
Diberi / Given: Q = 120 000 J, m = 2 kg, θ = 30°C, c = ?
Q
c =

.B

120 000
= 2 × 30 = 2 000 J kg–1 °C–1

dn
5. Jumlah haba yang diserap atau dibebaskan oleh sesuatu objek diberikan oleh Q = mcθ .
BAB

Q = mcθ

iS
The amount of heat absorbed or released by an object is given by .

4
Contoh 3 ng
Hitung jumlah haba yang diperlukan untuk memanaskan 5 kg besi dari 30°C sehingga 55°C. Muatan haba
tentu besi ialah 452 J kg–1 °C–1.
Calculate the amount of heat that is needed to heat 5 kg of iron from 30°C to 55°C. The specific heat capacity of iron is
la
452 J kg–1 °C–1.
Pe

Penyelesaian / Solution:
Diberi / Given: m = 5 kg, c = 452 J kg–1 °C–1, θ = 25°C, Q = ?
Q = mcθ
= 5 × 452 × 25 = 56 500 J
n
ita

6. Tenaga haba boleh ditukar daripada bentuk tenaga yang lain seperti tenaga elektrik, tenaga kinetik dan tenaga
keupayaan. Seperti yang dinyatakan dalam hukum keabadian tenaga, tenaga tidak boleh dimusnahkan
rb

atau dicipta, tetapi boleh ditukar dari satu bentuk tenaga kepada bentuk tenaga yang lain.
Heat energy can be converted from other forms of energy such as electrical energy, kinetic energy and potential energy. As
stated in the law of conservation of energy, energy cannot be destroyed nor created, but can be converted from one form of
ne

energy to another.

Tenaga elektrik Pemanas / Heater Tenaga haba


Pe

Pt = mc θ
Electrical energy Kuasa / Power = P Heat energy

Objek bergerak dihentikan


Tenaga kinetik oleh geseran Tenaga haba 1
mv2 = mc θ
Kinetic energy A moving object stopped by Heat energy 2
friction

Tenaga keupayaan Objek jatuh bebas Tenaga haba mgh = mc θ


Potential energy A falling object Heat energy

104
Fizik  Tingkatan 4  Bab 4 Haba 

Contoh 4
Sebuah cerek elektrik dengan kuasa 1.5 kW digunakan untuk mendidihkan 4 kg air. Diberikan bahawa
suhu awal air ialah x°C, cari x jika masa yang diperlukan untuk air mendidih ialah 10 minit.
[Muatan haba tentu air = 4 200 J kg–1 °C–1]
An electric heater with power 1.5 kW is used to boil 4 kg of water. Given that the initial temperature of the water is x°C,
find x if the time needed for the water to boil is 10 minutes.
[Specific heat capacity of water = 4 200 J kg-1 °C–1]

Penyelesaian / Solution:

.
Diberi / Given: P = 1 500 W, t = 600 s, m = 4 kg, c = 4 200 J kg-1 °C-1, θ = ?

hd
Pt = mcθ
1 500 × 600 = 4 × 4 200 × θ
θ = 53.57°C

.B

Air mendidih pada 100°C / Water boils at 100°C
∴ x = 100 – 53.57 = 46.4°C

dn

BAB
Aplikasi Muatan Haba Tentu

iS
Applications of Specific Heat Capacity

1. Cerek / Kettle 3 4
ng bukan logam
(a) Pemegang diperbuat daripada seperti kayu
Pemegang dan plastik yang merupakan konduktor haba yang lemah
Handle
la
dengan muatan haba tentu yang besar . Suhu pemegang
akan naik pada magnitud yang lebih kecil dan boleh
Pe

dipegang dengan mudah.


Badan non-metals
Body The handle is made of like wood and plastic which are
Tapak bad heat conductors with large specific heat capacity. The
n

Base
temperature of the handle will rise in smaller magnitude and it
ita

can be held easily.


(b) Badan cerek diperbuat daripada logam seperti aluminium yang berketumpatan rendah ; yang
rb

membuatkan badan cerek ringan . Aluminium juga merupakan konduktor haba yang baik
dan mempunyai muatan haba tentu yang kecil yang akan menyebabkan cerek menjadi panas
ne

dengan lebih cepat.


The body of the kettle is made of metals like aluminium which is low in density which makes the body of the
kettle light . Aluminium is also a good heat conductor and has small specific heat capacity
Pe

which will make the kettle become hot faster.


(c) Tapak cerek diperbuat daripada logam seperti kuprum yang merupakan konduktor haba yang
baik dan mempunyai muatan haba tentu yang kecil . Suhu tapak akan naik
dengan cepat untuk menjadi lebih panas untuk memanaskan air. Ketumpatan kuprum yang
tinggi membuatkan tapak lebih berat . Hal ini memastikan bahawa cerek adalah stabil .
The base is made of metals like copper which is a good heat conductor and has small specific heat
capacity. The temperature of the base will rise quickly to become hotter to heat up the water. The high
density of copper makes the base heavier . This ensures that the kettle is stable .

105
  Fizik  Tingkatan 4  Bab 4 Haba

2. Sistem penyejukan enjin kereta / Cooling system of a car engine 3


besar Kipas radiator
(a) Blok enjin menghasilkan jumlah haba yang Radiator fan
Air panas Silinder
Hot water Cylinders
daripada pembakaran petrol. Haba yang dihasilkan perlu
disingkirkan daripada enjin untuk mengelak pemanasan
lampau enjin.
Engine block produces large amounts of heat from the
combustion of petrol. The heat produced need to be removed
from the engine to prevent overheating of the engine.
(b) Suatu bahan penyejuk diperlukan untuk menyingkirkan

.
haba daripada enjin. Air yang mempunyai muatan haba tentu

hd
yang sangat tinggi (4 200 J kg–1 °C–1) merupakan Sirip penyejuk Pam air Turus berongga
cecair yang sesuai untuk bertindak sebagai agen penyejuk Cooling fin Water pump Hollow passages

.B
untuk menyerap sejumlah besar haba tanpa magnitud
kenaikan suhu yang besar .
high

dn
A suitable coolant is required to remove the heat from the engine. Water which has a very specific heat
capacity (4 200 J kg °C ) is the suitable liquid to act as a cooling agent to
–1 –1 absorb a great amount of heat
BAB

without big magnitude of temperature rise.

iS
(c) Air akan dipam ke dalam blok enjin oleh pam air. Blok enjin mengandungi turus berongga yang
membenarkan air menyerap haba yang dihasilkan daripada pembakaran campuran petrol-udara.
4 The water will be pumped into the engine block by the water pump. The engine block contains hollow passages that
absorb
ng
allow water to heat generated from the combustion of the petrol-air mixture.
(d) Air panas itu akan disejukkan oleh udara yang ditiup oleh kipas radiator dan mengalir melalui
la
sirip penyejuk radiator.
The hot water will be cooled by the air that is drawn in by the radiator fan and flows through the cooling fins
Pe

of the radiator.
(e) Air dipilih sebagai bahan penyejuk dalam enjin kerana / Water is chosen as a coolant in the engine because
(i) mempunyai takat didih yang tinggi / it has a high boiling point
n

(ii) mempunyai muatan haba tentu yang besar / it has a large specific heat capacity
ita

(iii) murah dan mudah didapati / it is cheap and easily available

(iv) tidak bertindak balas dengan komponen dalam enjin


rb

it does not react with the internal components of the engine


ne

3. Bayu laut / Sea breeze 3

(a) Pada waktu siang, sinar matahari akan memanaskan


kedua-dua darat dan laut. Darat mempunyai muatan Udara panas naik
Pe

haba yang lebih kecil berbanding dengan laut. Hot air rising

Maka, darat mengalami kenaikan suhu lebih cepat .


heat up Sejuk Panas
During the day, the rays of the sun will both Bayu laut
Cold
smaller Sea breeze Hot
the land and the sea. Land has a heat capacity
faster Laut Darat
compared to the sea. Thus, the land experiences a Land
Ocean
increase in temperature.
(b) Udara di atas darat dipanaskan lebih cepat dan menjadi kurang tumpat. Udara yang panas
mengembang dan naik ke atas.
The air above the land is heated up faster and becomes less dense. The warm air expands and rises
upwards.

106
Fizik  Tingkatan 4  Bab 4 Haba 

(c) Udara sejuk dari laut bergerak ke darat untuk mengisi ruang atau kawasan tekanan rendah
yang dicipta oleh udara panas di darat. Udara sejuk itu dikenal sebagai bayu laut .
Cool air from the sea moves towards the land to fill the space or the low pressure region created by
the warm air on the land. The cool air is called sea breeze .

4. Bayu darat / Land breeze 3

(a) Pada waktu malam, air laut yang mempunyai muatan Udara panas naik
haba yang lebih besar mengalami pengurangan suhu Hot air rising

lebih perlahan berbanding dengan darat.

.
During the night, the sea water which has larger heat Panas Sejuk

hd
Hot Bayu darat Cold
capacity experiences slower decrease in temperature compared Land breeze
to the land. Laut Darat
Ocean Land

.B
(b) Udara di atas laut dipanaskan lebih cepat dan ketumpatan
menjadi rendah . Udara yang panas mengembang dan naik ke atas.
The air above the sea is heated up faster and becomes low in density. The warm air expands and rises

dn
upwards.
darat laut

BAB
(c) Udara sejuk dari bergerak ke arah untuk mengisi ruang atau kawasan tekanan

iS
rendah di atas permukaan laut. Udara sejuk itu dikenal sebagai bayu darat .
Cool air from the land moves towards the sea to fill the space or the low pressure region on the
surface of the sea. The cool air is called land breeze . 4
ng
Tugasan 1
la

1. Dua biji cawan masing-masing mengandungi air kopi pada suhu 85°C dan susu pada suhu 35°C. Muatan haba tentu
Pe

susu adalah 1.5 kali muatan haba tentu air kopi. Jisim air kopi adalah dua kali jisim susu. Jika kedua-dua minuman itu
dicampurkan bersama, berapakah suhu akhir campuran?
Two cups contain coffee at 85°C and milk at 35°C respectively. The specific heat capacity of milk is 1.5 times the specific heat capacity of
coffee. The mass of coffee is twice the mass of milk. If the drinks are mixed together, what is the final temperature of the mixture?
n

Haba hilang daripada air kopi = Haba diterima oleh susu


ita

Heat lost by coffee Heat gained by milk


(mcθ)kopi = (mcθ)susu
mc(85 – x) = (0.5m)(1.5c)(x – 35)
rb

85 – x = 0.75x – 26.25
1.75x = 111.25
x = 63.57°C
ne

2. Suhu 8 kg plumbum meningkat sebanyak 30°C apabila dipanaskan. Hitung kenaikan suhu apabila 2 kg besi dipanaskan
dengan jumlah haba yang sama.
Pe

[Muatan haba tentu plumbum = 130 J kg–1 °C–1, Muatan haba tentu besi = 450 J kg–1 °C–1]
The temperature of 8 kg of lead rises by 30°C when heated. Calculate the temperature rise when 2 kg of iron is heated with the same
amount of heat.
[Specific heat capacity of lead = 130 J kg–1 °C–1, specific heat capacity of iron = 450 J kg–1 °C–1]
Haba diterima oleh plumbum = mcθ = (8)(130)(30) = 31 200 J
Heat gained by the lead
Haba diterima oleh besi = mcθ
Heat gained by the iron
31 200 = (2)(450)(θ)
θ = 34.67°C

107
  Fizik  Tingkatan 4  Bab 4 Haba

3. Satu pemanas elektrik 1 000 W digunakan untuk memanaskan sebuah bongkah kuprum berjisim 5 kg selama 50 saat.
Suhu bongkah kuprum itu naik sebanyak 26°C.
An electric heater of 1 000 W is used to heat a copper block of mass 5 kg for 50 seconds. The temperature of the copper block rises by 26°C.
(a) Hitung muatan haba tentu bongkah kuprum.
Calculate the specific heat capacity of the copper block.
Pt = mcθ
(1 000)(50) = (5)(c)(26)
c = 384.6 J kg–1 °C–1

(b) Apakah andaian yang dibuat dalam penghitungan ini?

.
What is the assumption made in the calculation?

hd
Semua tenaga elektrik ditukarkan kepada tenaga haba dan tiada haba yang hilang ke persekitaran.

.B
All electrical energy is converted to heat energy and no heat is lost to the surrounding.

4. Sebuah bongkah keluli berjisim 15 kg dijatuhkan secara menegak dari ketinggian 100 m. Berapakah peningkatan suhu

dn
bongkah keluli itu jika semua tenaga ditukarkan kepada tenaga haba?
[Muatan haba tentu keluli = 466 J kg–1 °C– 1, g = 10 m s–2]
BAB

A block of steel of mass 15 kg is dropped vertically from a height of 100 m. What is the increase in the temperature of the steel if all the

iS
energy is converted to heat energy?
[Specific heat capacity of steel = 466 J kg–1 °C– 1, g = 10 m s–2]

4 mgh = mcθ ng
(15)(10)(100) = (15)(466)( θ)
θ = 2.15°C
la
Pe

Memahami Haba Pendam Tentu


4.3 Understanding Specific Latent Heat

Haba Pendam
n

Latent Heat
ita

1. Empat fasa perubahan utama ialah / The four main changes of phase are:

Gas
rb

Gas
ne

Pendidihan / Boiling
Haba pendam diserap
Latent heat absorbed
Pe

Kondensasi / Condensation
Peleburan / Melting Haba pendam dibebaskan
Haba pendam diserap Latent heat released
Latent heat absorbed

Pemejalan / Solidification
Haba pendam dibebaskan
Latent heat released
Pepejal Cecair
Solid Liquid

108
Fizik  Tingkatan 4  Bab 4 Haba 

2. Graf menunjukkan lengkung pemanasan bagi bahan yang dalam keadaan pepejal semasa dipanaskan secara
seragam.
The graph shows the heating curve of a substance in a solid state when it is heated uniformly.

Suhu (°C)
Temperature (°C)

Cecair + Gas
Liquid + Gas Gas

.
hd
D E Gas
Takat didih
Boiling point
Cecair
Liquid

.B
Takat lebur B C
Melting point
Pepejal + Cecair
Solid + Liquid

dn
Pepejal

BAB
A Solid Masa (s)
Time (s)

iS
3. Jadual yang berikut menghuraikan setiap peringkat dalam lengkung pemanasan.
The following table describes each stage in the heating curve. 4
ng
(a) Haba yang diserap oleh pepejal meningkatkan
tenaga kinetik zarah-zarah dalam pepejal itu.
naik .
la
Zarah-zarah bergetar lebih cepat pada kedudukan tetap. Oleh itu, suhunya
AB
Heat absorbed by the solid increases the kinetic energy of the particles in the solid. The particles vibrate
Pe

faster at their fixed positions. Therefore, its temperature rises .

(b) Haba yang diserap


oleh pepejal yang sedang melebur tidak meningkatkan tenaga kinetik zarah-
zarah. Oleh itu, suhu kekal malar .
BC
n

Heat absorbed by the melting solid does not increase the kinetic energy of the particles. Therefore, the temperature
ita

remains constant .

(c) Haba yang diserap oleh cecair meningkatkan tenaga kinetik zarah-zarah dalam cecair itu.
rb

Oleh itu, suhunya naik .


CD
Heat absorbed by the liquid increases the kinetic energy of the particles in the liquid. Therefore, its
ne

temperature rises .

diserap
oleh cecair yang sedang mendidih tidak meningkatkan purata tenaga kinetik
Pe

(d) Haba yang


zarah-zarah. Oleh itu, suhu kekal malar .
DE
Heat absorbed by the boiling liquid does not increase the average kinetic energy of the particles. Therefore,
the temperature remains constant .

(e) Haba terus diserap oleh gas. Tenaga kinetik zarah-zarah dalam gas meningkat . Oleh itu,
suhunya naik .
EF
Heat is continuously absorbed by the gas. Kinetic energy of the particles in the gas increases . Therefore,
its temperature rises .

109
  Fizik  Tingkatan 4  Bab 4 Haba

4. Graf menunjukkan lengkung penyejukan bagi bahan yang dalam keadaan gas semasa disejukkan secara seragam.
The graph shows the cooling curve of a substance in a gas state when it is cooled uniformly.
Suhu (°C)
Temperature (°C)

Gas
Gas
A

Cecair + Gas
Liquid + Gas
Cecair
B C Liquid
Takat kondensasi

.
Condensation point

hd
Pepejal + Cecair
Solid + Liquid
Takat beku E

.B
Freezing point D
Pepejal
Solid

dn
F Masa (s)
Time (s)
BAB

5. Jadual yang berikut menghuraikan setiap peringkat dalam lengkung penyejukan.

iS
The following table describes each stage in the cooling curve.

dibebaskan oleh gas mengurangkan


4 (a) Haba yang tenaga kinetik zarah-zarah dalam gas. Oleh itu,
AB suhunya menurun  .
ng
Heat released by the gas decreases the kinetic energy of the particles in the gas. Therefore, its temperature
decreases  .
la
(b) Kondensasi gas ke cecair berlaku. Haba yang dibebaskan tidak mengurangkan tenaga kinetik zarah-
malar
Pe

BC zarah. Oleh itu, suhu kekal  .


Condensation of gas to liquid occurred. The heat released does not decrease the average kinetic energy of
the particles. Therefore, the temperature remains constant  .

(c) Haba yang dibebaskan oleh cecair mengurangkan tenaga kinetik zarah-zarah dalam cecair
n

menyebabkan zarah-zarah bergerak lebih perlahan dan mendekat di antara satu sama lain. Oleh itu,
CD
ita

suhunya menurun .
Heat released by the liquid decreases
the kinetic energy of the particles in the liquid, causing them to move
slower and closer to each other. Therefore, its temperature decreases  .
rb

(d) Haba yang dibebaskan oleh cecair yang sedang membeku tidak mengurangkan tenaga kinetik
malar
ne

DE zarah-zarah. Oleh itu, suhu kekal  .


Heat released by the freezing liquid does not decrease the kinetic energy of the particles. Therefore, its
temperature remains constant  .
Pe

(e) Haba terus dibebaskan ke persekitaran oleh pepejal. Purata tenaga kinetik zarah-zarah dalam pepejal
EF terus berkurang kerana zarah-zarah itu kekal pada kedudukan tetap. Oleh itu, suhunya menurun  .
Heat is continuously released to the surroundings by the solid. The average kinetic energy of the particles in the
solid continues to decrease as they remain in their fixed positions. Therefore, its temperature decreases  .

6. Tenaga haba yang diserap atau dibebaskan pada suhu malar semasa satu perubahan fasa dikenal
sebagai haba pendam .
The heat energy absorbed or released at constant temperature during a change of phase is known as
latent heat  .

110
Fizik  Tingkatan 4  Bab 4 Haba 

7. Terdapat dua jenis haba pendam iaitu haba pendam pelakuran dan haba pendam pengewapan .
There are two types of latent heat, namely latent heat of fusion and latent heat of vaporisation .

8. Haba pendam pelakuran / Latent heat of fusion


Haba dibebaskan
(a) Haba diserap apabila pepejal melebur Heat released
pada suhu malar.
absorbed solid Pembekuan
The heat when a melts at Freezing
constant temperature.
Haba pendam pelakuran

.
dibebas cecair Latent heat of fusion

hd
(b) Haba apabila membeku
pada suhu malar. Peleburan
Pepejal Melting Cecair
The heat released when a liquid freezes at Liquid
Solid

.B
constant temperature. Haba dibebaskan
Heat released
9. Haba pendam pengewapan / Latent heat of vaporisation

dn
(a) Haba diserap apabila cecair mendidih

BAB
pada suhu malar.

iS
absorbed liquid Haba dibebaskan
The heat when a boils at Heat released
constant temperature.
4
(b) Haba dibebaskan gas
apabila
ng
terkondensasi Kondensasi
Condensation
pada suhu malar.
released gas Haba pendam pengewapan
The heat when condenses at
la
Latent heat of vaporisation
constant temperature.
Pendidihan
Cecair Boiling Gas
Pe

(c) Haba yang dibekalkan digunakan untuk Gas


Liquid
memutuskan daya tarikan antara molekul Haba dibebaskan
dengan sepenuhnya tanpa perubahan suhu. Heat released
n

Heat supplied is used to break the forces of


attraction between molecules completely without changes in temperature.
ita

Haba Pendam Tentu


rb

Specific Latent Heat

1. Haba pendam tentu suatu bahan, l, ditakrifkan sebagai jumlah haba yang diperlukan untuk mengubah
ne

fasa bagi 1 kg bahan itu pada suhu malar .


Specific latent heat of a substance, l, is defined as the amount of heat required to change the phase of 1 kg of the
Pe

substance at constant temperature.

Q
l =
di mana / where
m
m = jisim bahan / mass of the substance
l = haba pendam tentu bahan / specific latent heat of the substance
Q = haba pendam yang diserap atau dibebaskan / latent heat absorbed or released

2. Unit S.I. bagi haba pendam tentu ialah joule per kilogram ( J kg
–1
).
The S.I. unit of specific latent heat is joule per kilogram ( J kg –1 ).

111
  Fizik  Tingkatan 4  Bab 4 Haba

3. Haba pendam tentu pelakuran, lf bagi suatu bahan ialah


kuantiti haba, Q yang diserap semasa peleburan atau kuantiti Peleburan
haba yang dibebaskan semasa pembekuan bagi 1 kg bahan itu 1 kg
Melting
1 kg
tanpa perubahan suhu. Pepejal Cecair
Solid Liquid
Specific latent heat of fusion, lf of a substance is the quantity of heat, Q
Pembekuan
that is absorbed during melting or the quantity of heat released during Freezing
Haba / Heat
freezing of 1 kg of the substance without change in temperature.

4. Haba pendam tentu pengewapan, lv bagi suatu bahan

.
ialah kuantiti haba yang diserap semasa pendidihan atau

hd
Pendidihan
kuantiti haba yang dibebaskan semasa kondensasi bagi 1 kg Boiling 1 kg
1 kg bahan itu tanpa perubahan suhu. Cecair Gas

.B
Liquid

Specific latent heat of vaporisation, lv of a substance is the quantity Kondensasi


of heat that is absorbed during boiling or the quantity of Condensation
Haba / Heat

dn
heat released during condensation of 1 kg of the substance without
change in temperature.
BAB

iS
4 Tugasan 2 ng
1. 200 g toluene dipanaskan dengan pemanas rendam berlabel 1 000 W pada takat didih 110°C. Cari masa yang diambil
untuk mengubah semua cecair toluene kepada gas. [Haba pendam tentu pengewapan toluene = 3.51 × 105 J kg–1]
la
200 g of toluene is heated with an immersion heater rated 1 000 W at boiling point 110°C. Find the time taken to change all the liquid
toluene to gas. [Specific latent heat of vaporisation of toluene = 3.51 × 105 J kg–1]
Pe

Pt = ml
1 000t = (0.2)(3.51 × 105)
1 000t = 7.02 × 104
t = 70.2 s
n

2. Hitung jumlah haba yang diperlukan untuk melebur 200 g emas padu pada 30°C kepada emas cecair pada 2 660°C.
ita

[Muatan haba tentu emas = 129 J kg–1 °C–1, haba pendam tentu pelakuran emas = 6.44 × 104 J kg–1]
Calculate the amount of heat needed to melt 200 g of solid gold at 30°C to liquid gold at 2 660°C.
[Specific heat capacity of gold = 129 J kg–1 °C–1, specific latent heat of fusion of gold = 6.44 × 104 J kg–1]
rb

Q = mcθ+ ml
= [(0.2)(129)(2 660 – 30)] + [(0.2)(6.44 × 104)]
ne

= 67 854 + (1.288 × 104)


= 80 734 J
Pe

3. Rajah ialah graf suhu-masa bagi satu bahan, W dengan jisim 250 g yang Suhu (°C)
dipanaskan dengan pemanas 1500 W. Tentukan Temperature (°C) F
The diagram is the temperature-time graph of a substance, W of mass 250 g that is
heated using a 1 500 W heater. Determine D
120 E
(a) muatan haba tentu W dalam keadaan pepejal
the specific heat capacity of W in solid state B C
30
Q = mcθ
(1 500 × 10) = (0.25)(c)(30 – 0) 0 Masa (s)
A 10 30 40 70 Time (s)
c = 2 000 J kg–1 °C–1

112
Fizik  Tingkatan 4  Bab 4 Haba 

(b) haba pendam tentu pelakuran W (d) haba pendam tentu pengewapan W
the specific latent heat of fusion of W the specific latent heat of vaporisation of W
Q = ml Q = ml
Q (1 500 × 30)
Q (1 500 × 20) l = =
l = = m 0.25
m 0.25
= 180 000 J kg–1
= 120 000 J kg–1
(c) muatan haba tentu W dalam keadaan cecair
the specific heat capacity of W in liquid state
Q = mcθ
(1 500 × 10) = (0.25)(c)(120 – 30)

.
c = 666.67 J kg–1 °C–1

hd
Memahami Hukum-hukum Gas
4.4

.B
Understanding the Gas Laws

1. Teori kinetik gas menganggap bahawa / Kinetic theory of gases assumes that

dn
(a) zarah-zarah gas sangat kecil dan terletak jauh di antara satu sama lain.
gas particles are infinitely small and located far from each other.

BAB
rawak

iS
(b) molekul-molekul gas sentiasa dalam gerakan yang berterusan dan sentiasa berlanggar
antara satu sama lain.
random
gas molecules are always in continuous motion and constantly collide with each other. 4
ng
(c) perlanggaran zarah gas antara satu sama lain dan dengan dinding bekas adalah kenyal sempurna.
the collisions of gas particles with each other and with the walls of the container are perfectly elastic .
la
2. Jadual menunjukkan sifat-sifat suatu gas dan penjelasan berdasarkan teori kinetik gas.
The table shows the properties of a gas and their explanation based on the kinetic theory of gases.
Pe

Sifat Penjelasan
Property Explanation

Isi padu • Molekul-molekul bergerak dengan bebas secara rawak dan mengisi seluruh ruang dalam
Volume
n

bekas.
The molecules move freely in random motion and fill up the whole space in the container.
ita

• Isi padu gas adalah sama dengan isi padu bekas.


The volume of the gas is equal to the volume of the container.
rb

Suhu • Molekul-molekul dalam gerakan rawak yang berterusan dan mempunyai tenaga kinetik
Temperature purata yang berkadar dengan suhu.
ne

The molecules are in continuous random motion and have average kinetic energy which is proportional
to the temperature.
• Semakin tinggi suhu, semakin laju gerakan molekul.
Pe

The higher the temperature, the faster the motion of the molecules.

Tekanan • Molekul-molekul dalam gerakan rawak yang berterusan serta melanggar dinding bekas dan
Pressure melantun balik.
The molecules are in continuous random motion and collide with the wall of the container and bounce back.
• Bagi setiap molekul, terdapat perubahan momentum  , dan suatu daya dikenakan pada dinding.
For every molecule, there is a change in momentum  , and a force is exerted on the wall.
• Daya per unit luas ialah tekanan gas itu.
The force per unit area is the pressure of the gas.

113
  Fizik  Tingkatan 4  Bab 4 Haba

Hukum Boyle
Boyle’s Law

1. Hukum Boyle menyatakan bahawa bagi suatu gas berjisim tetap, tekanan gas itu adalah berkadar songsang
dengan isi padu jika suhu gas itu kekal malar .
Boyle’s law states that for a fixed mass of gas, the pressure of the gas is inversely proportional to its volume when the
temperature of the gas is kept constant .

2. Hubungan antara tekanan dengan isi padu ditunjukkan dalam rajah di bawah.
The relationship between pressure and volume is shown in the diagram below.

.
hd
• Bilangan molekul per unit isi padu adalah lebih kecil ,
oleh itu frekuensi perlanggaran lebih rendah .

.B
The number of molecules per unit volume is smaller  , thus the
frequency of collisions is lower .

dn
• Isi padu lebih besar , tekanan lebih rendah .
BAB

Larger volume, lower pressure.

iS
Isi padu lebih besar Isi padu lebih kecil
Larger volume Smaller volume

4
3. Hukum Boyle dinyatakan secara matematik sebagai:
ng
Boyle’s law is expressed mathematically as:
1 1
V∝ atau / or P ∝ ,
la
P V
di mana / where
Pe

V = isi padu gas / volume of gas


P = tekanan gas / pressure of gas

Perhatikan / Note that:


n

PV = pemalar apabila suhu ditetapkan / constant when the temperature is kept constant
ita

P1V1= P2V2
rb

4. Hukum Boyle boleh ditunjukkan dalam bentuk graf.


Boyle’s Law can be shown graphically.
ne

(a) P (b) P (c) PV (d) PV


Pe

V 1
V P V
P berkadar songsang P berkadar secara langsung PV kekal malar apabila P PV kekal malar apabila V
dengan V 1 meningkat meningkat
dengan
P inversely proportional to V
V PV remains constant as P PV remains constant as V
1 increases increases
P directly proportional to
V

114
Fizik  Tingkatan 4  Bab 4 Haba 

Hubungan antara tekanan dengan isi padu gas


Eksperimen 4.1 Relationship between pressure and volume of a gas

Inferens / Inference:
Isi padu gas berjisim tetap dipengaruhi oleh tekanannya .
Volume of a fixed mass of gas is influenced by its pressure .

Hipotesis / Hypothesis:

.
Tekanan gas berjisim tetap meningkat apabila isi padu berkurang.

hd
The pressure of a fixed mass of gas increases when its volume decreases.

Tujuan / Aim:

.B
Menentukan hubungan antara tekanan dengan isi padu gas berjisim tetap pada suhu malar.
To determine the relationship between pressure and volume of a fixed mass of gas at constant temperature.

dn
Pemboleh ubah / Variables:

BAB
(a) Pemboleh ubah dimanipulasi: Isi padu udara

iS
Manipulated variable: Volume of air

(b) Pemboleh ubah bergerak balas: Tekanan


4
ng
Responding variable: Pressure

(c) Pemboleh ubah dimalarkan: Jisim dan suhu udara di dalam tiub
la

Constant variable: Mass and temperature of the air in the tube


Pe

Radas / Apparatus:
Tolok Bourdon , picagari dan tiub getah
n

Bourdon gauge, syringe, and rubber tube


ita

Susunan radas / Arrangement of apparatus:


rb

Tolok Bourdon
Bourdon gauge
ne

Tiub getah
Pe

Rubber tube
Picagari
Syringe
     
Prosedur / Procedure:
1. Isi padu picagari dilaraskan sehingga 140 cm3. Bacaan tekanan udara direkodkan.
The volume of the syringe is adjusted to 140 cm3. The reading of the air pressure is recorded.
2. Picagari itu kemudian ditolak dengan perlahan sehingga isi padu 120 cm3 dan tekanan udara direkodkan.
The syringe is then gently pushed to a volume of 120 cm3 and the air pressure is recorded.

3. Langkah 2 diulang bagi nilai isi padu yang berbeza, V = 100 cm3, 80 cm3 dan 60 cm3.
Step 2 is repeated for different values of volume, V = 100 cm3, 80 cm3 and 60 cm3.

115
  Fizik  Tingkatan 4  Bab 4 Haba

Penjadualan data / Tabulation of data:


Isi padu, V (cm3) 1 Tekanan, P (Pa)
Volume, V (cm3)
(cm–3) Pressure, P (Pa)
V
120
100
80
60
40

.
Analisis data / Analysis of data:

hd
P P

.B
1

dn
       V       V
Perbincangan / Discussion:
BAB

1. Picagari perlu ditolak dengan perlahan supaya suhu udara di dalam tiub getah dan picagari kekal

iS
malar .
gently
4 The syringe needs to be pushed so that the temperature of the air in the rubber tube and syringe remains
constant .
ng
2. Tiub getah yang digunakan mesti pendek dan berdiameter kecil supaya isi padu udara di dalam tiub
isi padu
la
itu tidak termasuk dalam penghitungan .
The rubber tube used must be short and has smaller diameter so that the volume of air in the tube is not included
Pe

in the calculation of volume .

Kesimpulan / Conclusion:
Isi padu udara di dalam picagari berkadar songsang dengan tekanan. Hipotesis diterima.
n

The volume of the air in the syringe is inversely proportional to pressure. The hypothesis is accepted.
ita

Tugasan 3
rb

1. Satu omboh ditolak dari A ke B. Cari panjang asal, y cm jika tekanan berubah y cm
A
daripada 10 Pa di A sehingga 18 Pa di B.
ne

A piston is pushed from A to B. Find the original length, y cm if the pressure changes from 10 Pa at
A to 18 Pa at B.
P1V1 = P2V2
Pe

B

10y = (18)(5) 5 cm
y = 9 cm
2. Satu gelembung udara naik ke permukaan dari dasar laut. Berapakah kedalaman laut
itu jika tekanan atmosfera setara dengan 10 m air? 1 cm3
An air bubble rises to the surface from the seabed. What is the depth of the sea if the atmospheric
pressure is equivalent to 10 m of water? hm
P1V1 = P2V2

(10)(1) = (10 + h)(0.05) 0.05 cm3
h = 190 m

116
Fizik  Tingkatan 4  Bab 4 Haba 

3. Jadual menunjukkan turus kaca yang sama mengandungi merkuri dalam kedudukan yang berbeza dengan gas yang
terperangkap di dalamnya. Cari tekanan dan panjang gas yang terperangkap itu.
The table shows the same glass tube containing mercury in different positions with trapped gas inside. Find the pressure and the length of
the gas trapped.
[Patm = 75 cm Hg]

P2
10 cm x cm
5 cm
Tiub kaca P1
Glass tube

.
5 cm 5 cm y cm

hd
P3

P1 = P2 = P3 =

.B
Tekanan gas P1 = Patm P2 + 5 cm Hg = Patm P3 = Patm + 5 cm Hg
Pressure of gas
= 75 cm Hg P2 = 75 – 5 = 75 + 5

dn
= 70 cm Hg = 80 cm Hg
l = 10 cm x= cm y= cm

BAB
Panjang gas

iS
terperangkap P1V1 = P2V2 P1V1 = P2V2
Length of gas (75)(10) = 70x (75)(10) = 80y
trapped
x = 10.71 cm y = 9.38 cm 4
ng
Hukum Charles
Charles’ Law
la
1. Hukum Charles menyatakan bahawa bagi suatu gas berjisim tetap, isi padu gas itu berkadar secara
suhu tekanan
Pe

langsung dengan mutlak apabila gas kekal malar.


Charles’ law states that for a fixed mass of gas, the volume of the gas is directly proportional to its absolute
temperature when the pressure of the gas is kept constant.
n

2. Rajah menunjukkan hubungan antara isi padu gas yang terperangkap dengan suhu, pada tekanan malar.
The diagram shows the relationship between the volume of gas trapped and the temperature, at a constant pressure.
ita

• Tenaga kinetik zarah-zarah adalah lebih besar ,


rb

oleh itu, frekuensi perlanggaran lebih tinggi .


The kinetic energy of the particles is greater , thus the
ne

frequency of collisions is higher .


Lower temperature
Higher temperature
Suhu lebih rendah
Suhu lebih •
tinggiIsi
padu yang lebih besar dihasilkan untuk
Pe

mengekalkan tekanan malar.


Larger volume produced to keep the pressure constant.
Suhu lebih rendah Suhu lebih tinggi
Lower temperature Higher temperature

3. Hukum Charles dinyatakan secara matematik sebagai:


Charles’ law is expressed mathematically as:
V ∝ T di mana / where V = pemalar / constant
T
maka / therefore,
V1 V2
=
T1 T2

117
  Fizik  Tingkatan 4  Bab 4 Haba

4. Hukum Charles dapat ditunjukkan dalam bentuk graf. Graf bagi gas yang mematuhi hukum itu adalah seperti
yang ditunjukkan di bawah.
Charles’s law can be shown graphically. The graphs of a gas obeying the law are as shown below.

V (cm3) V (cm3)

.
T (°C) 0 T (K)
– 273

hd
5. Suhu –273°C ialah suhu yang serendah mungkin dan dikenal sebagai suhu sifar mutlak .
absolute zero

.B
The temperature of –273°C is the lowest possible temperature and is known as the of temperature.
(a) Untuk menukarkan suhu darjah Celsius kepada kelvin
To change the temperature from degree Celsius to kelvin

dn
θ°C = ( θ + 273 )K
BAB

(b) Untuk menukarkan suhu kelvin kepada darjah Celsius

iS
To change the temperature from kelvin to degree Celsius
T K = ( T – 273 )°C
4
ng
Hubungan antara isi padu dengan suhu gas
Eksperimen 4.2 Relationship between volume and temperature of a gas
la

Inferens / Inference:
Pe

Isi padu suatu gas berjisim tetap dipengaruhi oleh suhu .


Volume of a fixed mass of gas is influenced by its temperature .
n

Hipotesis / Hypothesis:
ita

Apabila suhu suatu gas berjisim tetap meningkat, isi padu gas itu juga meningkat.
As the temperature of a fixed mass of gas increases, the volume of the gas increases.
rb

Tujuan / Aim:
isi padu suhu
ne

Menentukan hubungan antara dengan suatu gas berjisim tetap pada tekanan malar
To determine the relationship between the volume and the temperature of a fixed mass of gas at constant pressure
Pe

Pemboleh ubah / Variables:


(a) Pemboleh ubah dimanipulasi: Suhu gas
Manipulated variable: Temperature of gas

(b) Pemboleh ubah bergerak balas: Isi padu gas


Responding variable: Volume of gas

(c) Pemboleh ubah dimalarkan: Jisim dan tekanan gas di dalam tiub
Constant variable: Mass and pressure of gas in the tube

118
Fizik  Tingkatan 4  Bab 4 Haba 

Bahan / Materials:
Minyak parafin , air dan gelang getah
Paraffin oil, water, and rubber bands

Radas / Apparatus:
Tiub kapilari , termometer, penunu Bunsen , bikar, kaki retort dengan pengapit, tungku kaki tiga,
pengacau dan pembaris
Capillary tube, thermometer, Bunsen burner, beaker, retort stands with clamp, tripod stand, stirrer, and ruler

.
hd
Susunan radas / Arrangement of apparatus:

Termometer
110

Thermometer

.B
100

Tiub kapilari
90

Gelang getah
80

Capillary tube
Rubber band
70
60
50

Air
40

Pengacau

dn
Water
30
20

Minyak parafin Stirrer


10
0

Paraffin oil
–10

BAB
iS
4
    
ng
Prosedur / Procedure:
1. Air di dalam bikar dipanaskan secara perlahan dan dikacau berterusan.
la
The water in the beaker is heated slowly and stirred continuously.
Pe

2. Panjang udara yang terperangkap di dalam tiub kapilari, l dicatatkan apabila suhu, θ = 30°C.
The length of the trapped air in the capillary tube, l is recorded when the temperature, θ = 30°C.
3. Langkah 2 diulang bagi suhu yang berbeza, θ = 40°C, 50°C, 60°C, 70°C dan 80°C.
Step 2 is repeated for different temperatures, θ = 40°C, 50°C, 60°C, 70°C and 80°C.
n

Penjadualan data / Tabulation of data:


ita

Suhu / Temperature, θ (°C) 30 40 50 60 70 80


Panjang / Length, l (cm)
rb

Analisis data / Analysis of data:


ne

l (cm)
Pe

θ (°C)
0
     – 273 100

Perbincangan / Discussion:
1. Untuk mencapai suhu seragam di dalam bikar, air hendaklah dikacau secara berterusan .
To achieve uniform temperature in the beaker, the water needs to be stirred continuously .

119
  Fizik  Tingkatan 4  Bab 4 Haba

2. Apabila graf diekstrapolasi, garis akan mencapai –273°C pada paksi-x. Pada suhu ini, isi padu udara
adalah sifar kerana molekul udara tidak bergerak pada sifar mutlak (–273°C atau 0 K).
When the graph is extrapolated, the line will reach –273°C at the x-axis. At this temperature, the volume of air is zero
as the air molecules do not move at absolute zero (–273°C or 0 K).

Kesimpulan / Conclusion:
Isi padu suatu gas berjisim tetap adalah berkadar secara langsung dengan suhu mutlak gas itu jika tekanan
gas adalah malar. Hipotesis diterima.

.
The volume of a fixed mass of gas is directly proportional to the absolute temperature of the gas if the pressure of the gas is

hd
constant. The hypothesis is accepted.

.B
Tugasan
1. Sebiji belon diisikan udara pada suhu 27°C. Kemudian, belon itu dipanaskan pada suhu 77°C. Dengan tekanan kekal

dn
malar, cari isi padu akhir belon jika isi padu awal ialah 15 cm3.
A balloon is filled with air at a temperature of 27°C. Then, the balloon is heated to a temperature of 77°C. With the pressure kept constant,
BAB

find the final volume of the balloon if its initial volume is 15 cm3.

iS
V1 V
= 2
T1 T2 27°C 77°C
4 15 V2 V = 15 cm3 V=?
=
27 + 273 77 + 273
ng
V2 = 17.5 cm3
la
2. Suhu dan isi padu suatu gas berjisim tetap ditunjukkan dalam jadual.
Cari nilai θ dalam °C jika tekanan adalah kekal malar.
Pe

The temperature and the volume of a fixed mass of gas is shown in the table. Find the value of θ in °C if the pressure is kept constant.
Suhu / Temperature (°C) 127 θ
Isi padu / Volume (cm ) 3
25 40
n

V1 V
= 2
ita

T1 T2
25 40
=
127 + 273
rb

θ
θ = 640 K
= 640 – 273 = 367°C
ne

3. Suatu gas dengan isi padu 25 cm3 pada 37°C dipanaskan sehingga suhu menjadi θ°C pada tekanan malar. Apakah
perubahan dalam suhu jika isi padu selepas dipanaskan ialah 30 cm3?
Pe

A gas with the volume of 25 cm3 at 37°C is heated until its temperature becomes θ°C at constant pressure. What is the change in temperature
if the volume after being heated is 30 cm3?
V1 V
= 2
T1 T2
25 30
=
37 + 273 θ
θ = 372 K atau / or 99°C
∴ Peningkatan suhu / Increase in temperature, θ = 99 – 37 = 62°C

120
Fizik  Tingkatan 4  Bab 4 Haba 

Hukum Gay-Lussac
Gay-Lussac Law

1. Hukum Gay-Lussac menyatakan bahawa bagi suatu gas berjisim tetap, tekanan berkadar secara langsung
dengan suhu mutlak gas itu apabila isi padu gas adalah kekal malar.
Gay-Lussac law states that for a fixed mass of gas, the pressure is directly proportional to its absolute temperature

when the volume of the gas is kept constant.

2. Rajah menunjukkan bagaimana suhu mempengaruhi tekanan untuk isi padu yang sama.
The diagram shows how the temperature influences the pressure for the same volume.

.
hd
• Tenaga kinetik lebih besar .
P P
The kinetic energy is greater .

.B
• Molekul-molekul bergerak.
lebih laju .
P P P P

dn
The molecules move faster .

BAB
• Frekuensi perlanggaran

iS
lebih tinggi .
Panas Sejuk
The frequency of collisions is higher . Hot Cold

lebih tinggi
4
• Tekanan .
ng
The pressure is higher .
la

3. Hukum Gay-Lussac dinyatakan secara matematik sebagai:


Pe

Gay-Lussac law is expressed mathematically as:


P ∝ T
di mana / where
n

P = pemalar apabila isi padu kekal malar


ita

T constant when the volume is kept constant

P1 P
= 2
rb

T1 T2

ne

4. Hukum Gay-Lussac boleh ditunjukkan dalam bentuk graf. Graf bagi gas yang mematuhi hukum itu adalah
seperti yang ditunjukkan di bawah.
Gay-Lussac law can be shown graphically. The graphs of a gas obeying the law are as shown below.
Pe

P (Pa) P (Pa)

T (°C) 0 T (K)
– 273

121
  Fizik  Tingkatan 4  Bab 4 Haba

Hubungan antara tekanan dengan suhu gas


Eksperimen 4.3 Relationship between pressure and temperature of a gas

Inferens / Inference:
Tekanan suatu gas berjisim tetap dipengaruhi oleh suhu pada isi padu malar.
The pressure of a fixed mass of gas is influenced by its temperature at constant volume.

Hipotesis / Hypothesis:

.
suhu tekanan

hd
Apabila suatu gas berjisim tetap meningkat, gas itu juga meningkat.
As the temperature of a fixed mass of gas increases, the pressure of the gas increases.

.B
Tujuan / Aim:
Menentukan hubungan antara tekanan dengan suhu suatu gas berjisim tetap pada isi padu malar.

dn
To determine the relationship between the pressure and the temperature of a fixed mass of gas at constant volume.
BAB

Pemboleh ubah / Variables:

iS
(a) Pemboleh ubah dimanipulasi: Suhu udara
4 Manipulated variable: Temperature of air
ng
(b) Pemboleh ubah bergerak balas: Tekanan udara
Responding variable: Pressure of air
la

(c) Pemboleh ubah dimalarkan: Jisim dan isi padu udara


Pe

Constant variable: Mass and volume of air

Bahan / Materials:
n

Air / Water
ita

Radas / Apparatus:
Bikar, kelalang dasar bulat, termometer, tolok Bourdon , penunu Bunsen , tungku kaki tiga, kaki retort
dengan pengapit dan pengacau
rb

Beaker, round-bottomed flask, thermometer, Bourdon gauge, Bunsen burner, tripod stand, retort stand with clamp,
and stirrer
ne

Susunan radas / Arrangement of apparatus:


Pe

Tolok Bourdon
Bourdon gauge
110

Termometer
100
90

Thermometer
80
70
60
50

Udara
40
30
20

Air
10
0

Air
–10

Water

    

122
Fizik  Tingkatan 4  Bab 4 Haba 

Prosedur / Procedure:
1. Air di dalam bikar dipanaskan secara perlahan dan dikacau berterusan.
The water in the beaker is heated slowly and stirred continuously.
2. Tekanan udara, P pada tolok Bourdon dicatatkan apabila suhu, θ = 40°C.
The pressure of the air , P on the Bourdon gauge is recorded when the temperature, θ = 40°C.
3. Langkah 2 diulang bagi suhu yang berbeza, θ = 50°C, 60°C, 70°C dan 80°C.
Step 2 is repeated for different temperatures, θ = 50°C, 60°C, 70°C and 80°C.

Penjadualan data / Tabulation of data:


Suhu / Temperature, θ (°C) 40 50 60 70 80

.
hd
Panjang / Length, P (cm)

Analisis data / Analysis of data:

.B
P (Pa)

dn

BAB
iS
T (°C)
     – 273

Perbincangan / Discussion:
4
ng
1. Untuk mencapai suhu seragam di dalam bikar, air hendaklah dikacau secara berterusan .
continuously
la
To achieve uniform temperature in the beaker, the water needs to be stirred .

2. Apabila graf diekstrapolasikan, garis akan mencapai –273°C


pada paksi-x. Pada suhu ini, tekanan udara
Pe

adalah sifar kerana molekul-molekul udara tidak bergerak pada sifar mutlak ( –273°C or 0 K ).
When the graph is extrapolated, the line will reach –273°C
at the x-axis. At this temperature, the pressure of air is
zero as the air molecules do not move at absolute zero ( –273°C atau 0 K ).
n

Kesimpulan / Conclusion:
ita

Tekanan suatu gas berjisim tetap berkadar secara langsung dengan suhu mutlak gas itu jika isi padu gas
adalah kekal malar. Hipotesis diterima.
rb

The pressure of a fixed mass of gas is directly proportional to the absolute temperature of the gas if the volume of the gas is kept
constant. The hypothesis is accepted.
ne

Tugasan 5
Pe

1. Tekanan udara di dalam tayar motorsikal ialah 200 kPa pada suhu 37°C. Berapakah tekanan udara di dalam tayar itu
pada suhu 57 °C? [Anggap isi padu udara di dalam tayar itu adalah malar]
The air pressure in a motorcycle tyre is 200 kPa at a temperature of 37°C. What is the air pressure in the tyre at a temperature of 57°C?
[Assume that the volume of the air in the tyre is constant]
P1 P
= 2
T1 T2
200 P2
=
37 + 273 57 + 273
P2 = 212.90 kPa

123
  Fizik  Tingkatan 4  Bab 4 Haba

2. Sebiji tayar mempunyai tekanan udara 25 kPa apabila suhu 47°C. Berapakah tekanan tayar itu jika suhu ditingkatkan
sehingga 77 °C?
A tyre has air pressure of 25 kPa when the temperature is 47°C. What is the pressure of the tyre if the temperature is increased to 77°C?
P1 P
= 2
T1 T2
25 P2
=
47 + 273 77 + 273
P2 = 27.34 kPa

3. Suhu dan tekanan suatu gas berjisim tetap ditunjukkan dalam jadual. Cari nilai θ jika isi padu gas itu kekal malar.

.
The temperature and the pressure of a fixed mass of gas are shown in the table. Find the value of θ if the volume of the gas is kept

hd
constant.
Suhu / Temperature, θ (°C) 57 θ

.B
Tekanan / Pressure, P (Pa) 1 200 2 400

P1 P
= 2
T1 T2

dn
1 200 2 400
=
57 + 273 θ + 273
BAB

θ = 660 – 273 = 387°C

iS
4
ng
PRAKTIS SPM 4
la
Pe

Soalan Objektif

1. Suatu sudu logam dimasukkan di dalam suatu C Haba mengalir hanya daripada minuman panas
minuman panas, sudu itu akan menjadi panas dalam ke sudu.
n

2015
tempoh masa yang singkat. Pernyataan manakah Heat flows only from the hot drink to the spoon.
D
ita

yang betul mengenai pengaliran haba? Pengaliran haba bersih adalah sifar.
A metal spoon is placed in a hot drink, the spoon will become Net heat flow is zero.
hot in a short period of time. Which statement is correct about
2. 100 cm3 minyak zaitun memerlukan 8 minit untuk
rb

the heat flow?


meningkatkan suhu sebanyak 50°C dan 100 cm3 air
2016
memerlukan 10 minit untuk meningkatkan suhu
ne

sebanyak 50°C apabila dipanaskan dengan pemanas


yang sama. Pernyataan yang manakah menjelaskan
keadaan ini?
Pe

100 cm3 of olive oil needs 8 minutes to increase the temperature


by 50°C and 100 cm3 of water needs 10 minutes to increase
Rajah 1 / Diagram 1 the temperature by 50°C when heated with the same heater.
Which statement explains this situation?
A Pengaliran haba bersih adalah daripada A Ketumpatan minyak zaitun lebih rendah
minuman panas ke sudu. daripada ketumpatan air.
Net heat flow is from the hot drink to the spoon. The density of olive oil is lower than the density of water.
B Haba mengalir hanya daripada sudu ke B Jisim minyak zaitun lebih kecil daripada jisim air.
minuman panas. The mass of olive oil is smaller than the mass of water.
Heat flows only from the spoon to the hot drink.

124
Fizik  Tingkatan 4  Bab 4 Haba 

C Haba pendam tentu minyak zaitun lebih rendah B Kehilangan haba oleh badan pesakit adalah
daripada haba pendam tentu air. lebih rendah daripada penerimaan haba oleh
The specific latent heat of olive oil is lower than the termometer.
specific latent heat of water. The heat loss by patient’s body is less than heat gain by
D Muatan haba tentu minyak zaitun lebih rendah the thermometer.
daripada muatan haba tentu air. C Kehilangan haba oleh badan pesakit adalah
The specific heat capacity of olive oil is lower than the lebih besar daripada penerimaan haba oleh
specific heat capacity of water. termometer.
The heat loss by patient’s body is greater than heat gain
3. Rajah 2 menunjukkan satu picagari yang
by the thermometer.
disambungkan ke tolok Bourdon.
D Kehilangan haba oleh badan pesakit adalah sama

.
2016
Diagram 2 shows a syringe connected to a Bourdon

hd
gauge. dengan penerimaan haba oleh termometer.
The heat loss by patient’s body is equal to heat gain by
Omboh the thermometer.

.B
Picagari Piston
Syringe Udara 5. Rajah 4 menunjukkan lengkung penyejukan suatu
Air
Tolok Bourdon cecair.
2015
Bourdon gauge Diagram 4 shows a cooling curve of a liquid.

dn
Suhu, T (°C)
Rajah 2 / Diagram 2 Temperature, T (°C)

BAB
A

iS
Bacaan pada tolok Bourdon ialah 1.2 × 105 Pa apabila
isi padu udara dalam picagari ialah 20 cm3. Berapakah
B C
bacaan baru isi padu udara apabila bacaan tolok
4
Bourdon ialah 1.6 × 105 Pa?
ng 0
Masa, t (s)
The reading on Bourdon gauge is 1.2 × 10 Pa when the volume
5
Time, t (s)
of air in the syringe is 20 cm3. What is the new volume of air
Rajah 4 / Diagram 4
when the reading of Bourdon gauge is at 1.6 × 105 Pa?
la
A 10 cm3 C 25 cm3 Pernyataan manakah menerangkan apa yang berlaku
B 15 cm3 D 30 cm3 dalam bahagian BC?
Pe

Which statement explains what happens in section BC?


4. Rajah 3 menunjukkan termometer klinik diletakkan A Haba pendam pelakuran dibebaskan ke
di bawah lidah selama beberapa minit sebelum persekitaran.
2016
mengambil bacaan suhu badan pesakit. The latent heat of fusion is released to surrounding.
n

Diagram 3 shows a clinical thermometer will be placed under B Tiada haba yang dibebaskan oleh cecair.
tongues for a few minutes before taking the reading of patient
ita

No heat is released by the liquid.


body temperature. C Tenaga kinetik molekul berkurangan.
Kinetic energy of the molecules decreases.
rb

D Cecair mengalami perubahan fasa daripada gas


kepada cecair
The liquid experiences a phase change from gas to liquid.
ne

6. Rajah 5 menunjukkan satu belon pada suhu bilik 30°C.


Belon itu kemudian dimasukkan ke dalam peti sejuk
Pe

2014
Rajah 3 / Diagram 3 dengan suhu 5°C.
Diagram 5 shows a balloon at room temperature 30°C. The
Pernyataan yang manakah betul tentang suhu badan balloon is then put in a refrigerator with temperature 5° C.
dan bacaan termometer?
Which statement is correct about the body temperature and
the thermometer reading? 5°C
30°C
A Kehilangan haba oleh badan pesakit adalah
sama dengan kehilangan haba oleh termometer.
The heat loss by patient’s body is equal to heat loss by
the thermometer.
Rajah 5 / Diagram 5

125
  Fizik  Tingkatan 4  Bab 4 Haba

Antara yang berikut, graf yang manakah yang A caluminium > cbesi > cair
menunjukkan hubungan yang betul antara isi padu caluminium > ciron > cwater
dengan suhu suatu gas? B cbesi > caluminium > cair
Which of the following graph shows the correct relationship ciron> caluminium > cwater
between the volume and the temperature of gas?
C cair > cbesi > caluminium
A V (cm3) C V (cm3) cwater > ciron > caluminium
D cair > caluminium > cbesi
cwater > caluminium > ciron

9. Rajah 7 menunjukkan tekanan pada tayar motosikal

.
0 T (K) T (K) adalah 220 kPa pada suhu 60.3°C selepas satu

hd
0
2015
perjalanan yang jauh.
Diagram 7 shows the pressure of a motorcycle tyre is 220 kPa at
B D temperature 60.3°C after a long journey,

.B
V (cm )
3 V (cm3)

dn
BAB

0 T (K) 0 T (K)

iS
Rajah 7 / Diagram 7
7. Fenomena manakah yang menunjukkan aplikasi
Apabila suhu tayar itu menurun kepada 30°C,
4 haba pendam tentu pengewapan dalam kehidupan
berapakah tekanan yang baru?
2012
seharian?
ng
(Isi padu udara di dalam tayar tidak berubah)
Which phenomenon shows the application of latent heat of
When the temperature of the tyre decreased to 30°C, what is
vaporisation in everyday life?
the new pressure?
la
A Mengukus ayam (Volume of the air in the tyre does not change)
Steaming chicken
A 182 kPa
Pe

B Menyejukkan enjin kereta dengan air B 198 kPa


Cooling a car engine with water
C 200 kPa
C Menyejukkan minuman dengan ais
D 210 kPa
Cooling down a drink with ice
n

D Membekukan udang di dalam kiub-kiub ais 10. Rajah 8 menunjukkan perubahan keadaan jirim
Freezing prawns in ice cubes
ita

daripada 1 kg ais kepada stim.


2015
Diagram 8 shows the change of state of matter for 1 kg ice into
8. Rajah 6 menunjukkan 1 kg air, besi dan aluminium steam.
masing-masing dipanaskan dengan sumber
rb

2016 Ais Air Air


haba yang sama untuk tempoh masa yang sama. Ice Water Water
Pernyataan manakah yang betul? A B
ne

Diagram 6 shows 1 kg of water, iron and aluminium respectively 1 kg 1 kg 1 kg


C
being heated with the same source of heat in the same period. 0°C 0°C 30°C
Which statement is correct? Stim Air
Steam Water
Pe

Air
D
110

110

110

1 kg
100

100

100

Water Besi Aluminium


90

90

90
80

80

80

100°C 100°C
70

70

70

Iron Aluminium
60

60

60
50

50

50
40

40

40
30

30

30
20

20

20

Rajah 8 / Diagram 8
10

10

10
0

0
-10

-10

-10

Perubahan keadaan jirim manakah yang melibatkan


haba pendam tentu pengewapan?
Which change of state of matter involves the specific latent
heat of vaporisation?
40°C 260°C 170°C

Rajah 6 / Diagram 6

126
Fizik  Tingkatan 4  Bab 4 Haba 

Soalan Struktur

Bahagian A

1. Rajah 1 menunjukkan air mendidih sedang dipanaskan oleh pemanas rendam 1 500 W selama 120 saat. Haba pendam
tentu pengewapan adalah 2.26 × 106 J kg–1.
2015
Diagram 1 shows boiling water is being heated by 1 500 W immersion heater for 120 seconds. The specific latent heat of vaporisation is
2.26 × 10 6 J kg–1.
Bekalan kuasa
Power supply
Pemanas rendam

.
Immersion heater

hd
Air mendidih
Boiling water

Penimbang elektronik

.B
Electronic balance

dn
Rajah 1 / Diagram 1

(a) Apakah yang dimaksudkan dengan haba pendam tentu pengewapan?

BAB
What is the meaning of specific latent heat of vaporisation?

iS
Haba pendam tentu pengewapan untuk suatu bahan ditakrifkan sebagai jumlah haba yang diperlukan untuk
menukar 1 kg bahan daripada fasa cecair ke fasa gas tanpa perubahan dalam suhu.
4
ng
Specific latent heat of vaporisation of a substance is defined as the amount of heat required to change 1 kg of the substance from
liquid to gaseous phase without a change in temperature.
la
[1 markah / mark]
Pe

(b) Terangkan mengapa suhu tidak berubah apabila air bertukar ke stim.
Explain why the temperature does not change when the water change to steam.
Haba pendam pengewapan diserap untuk memutuskan sepenuhnya ikatan antara zarah-zarah. Haba pendam
n

pengewapan tidak meningkatkan tenaga kinetik purata zarah. Maka, suhu tetap malar.
ita

Latent heat of vaporisation is absorbed to totally break up the bonds between the particles. Latent heat of fusion though does not
increase the average kinetic energy of the particles. Therefore, the temperature remains constant.
rb

[2 markah / mark]
(c) Hitung
ne

Calculate
(i) tenaga yang diserap oleh air.
the energy absorbed by the water.
Pe

Q = Pt = 1 500 × 120 = 180 000 J

[2 markah / mark]
(ii) jisim air yang sudah bertukar ke stim.
the mass of water which has changed to steam.
Q = ml
180 000 = m(2.26 × 106)
m = 0.08 kg

[2 markah / mark]

127
  Fizik  Tingkatan 4  Bab 4 Haba

2. Rajah 2.1 menunjukkan bacaan Tolok Bourdon sebelum tangki gas dipanaskan. Rajah 2.2 menunjukkan bacaan Tolok
Bourdon apabila tangki gas dipanaskan ke suhu 47°C.
2014
Diagram 2.1 shows the reading of the Bourdon Gauge before the gas tank is heated up. Diagram 2.2 shows the reading of the Bourdon
Gauge when the gas tank is heated up to 47°C.

Tolok Tolok
Bourdon Bourdon
Bourdon Bourdon
Gauge Gauge

.
hd
Tangki gas Tangki gas
Gas tank Gas tank
27°C 47°C

.B
     

dn
Rajah 2.1 / Diagram 2.1                     Rajah 2.2 / Diagram 2.2

(a) Apakah kegunaan Tolok Bourdon? / What is the use of Bourdon Gauge?
BAB

iS
Mengukur tekanan gas / Measure gas pressure
[1 markah / mark]
4
ng
(b) Perhatikan Rajah 2.1 dan Rajah 2.2, / Observe Diagram 2.1 and Diagram 2.2,
(i) bandingkan tekanan di dalam tangki gas apabila suhu bertambah.
compare the pressure produced inside the gas tank when the temperature increases.
la
Apabila suhu rendah, tekanan rendah. / Apabila suhu tinggi, tekanan tinggi.
Pe

When the temperature is low, the pressure is low. / When the temperature is high, the pressure is higher.
[1 markah / mark]
(ii) Nyatakan hubungan antara tekanan gas di dalam tangki gas dengan suhu.
n

State the relationship between the pressure of gas inside the gas tank with the temperature.
ita

Suhu berkadar terus dengan tekanan. / Temperature is directly proportional to pressure.


[1 markah / mark]
rb

(ii) Nyatakan hukum fizik yang terlibat.


State the physics law involved.
ne

Hukum Gay-Lussac / Gay-Lussac law


[1 markah / mark]
Pe

(c) Berdasarkan jawapan dalam 3 (b), apakah yang berlaku kepada tenaga kinetik zarah gas apabila tangki gas
dipanaskan?
Based on the answer in 3(b), what happens to the kinetic energy of the gas particles when the gas tank is heated?
Tenaga kinetik zarah gas bertambah. / The kinetic energy of gas particles increases.
[1 markah / mark]

128
Fizik  Tingkatan 4  Bab 4 Haba 

3. Rajah 3 menunjukkan satu termometer merkuri tidak ditentukur diletakkan di dalam bikar yang mengandungi ketulan
ais pada 0°C. Panjang turus merkuri, l0 = 5 cm. Panjang turus merkuri apabila dimasukkan ke dalam suhu mendidih
2016
pada 100°C, adalah, l100 = 25 cm. Rajah 3 juga menunjukkan panjang turus merkuri, lθ = 12 cm apabila termometer itu
diletakkan di dalam satu cecair panas dengan suhu, θ°C.
Diagram 3 shows an uncalibrated mercury thermometer placed in a beaker containing ice cubes at 0°C. The length of the mercury column,
l0 = 5 cm. The length of the mercury column when put into boiling temperature at 100°C, is, l100 = 25 cm. Diagram 3 also shows the length
of mercury column, lθ =12 cm when the thermometer is placed in a hot liquid with temperature, θ°C.

Termometer
Thermometers
100°C

.
hd
Stim
0°C Steam

Ais

.B
Ice

dn

BAB

iS
l0
l100
Rajah 2.2 / Diagram 2.2
4
ng
(a) (i) Berdasarkan Rajah 3, tentukan perbezaan bagi panjang turus merkuri pada 0°C dan 100°C.
Based on Diagram 3, determine the difference in length of the mercury columns at 0°C and 100°C.
25 cm – 5 cm = 20 cm
la
[1 markah / mark]
Pe

(ii) Berdasarkan Rajah 3 dan jawapan dalam 3(b)(i), hitungkan suhu, θ bagi cecair panas tersebut.
Based on Diagram 3 and answer in 3(b)(i), calculate the temperature, θ of the hot liquid.

(12 – 5) 7
θ= × 100 = × 100 = 35°C
n

20 20
[2 markah / marks]
ita

(b) Jadual 3.1 menunjukkan beberapa ciri bagi termometer yang berbeza yang digunakan untuk menentukan suhu
badan seorang pesakit.
rb

Table 3.1 shows several characteristics of different thermometers which can be used to determine the body temperature of a patient.

Jadual 3.1 / Diagram 3.1


ne

Termometer Saiz bebuli Julat suhu (°C) Diameter tiub kapilari


Thermometer Size of bulb Range of temperature (°C) Diameter of capillary tube
Kecil Besar
Pe

P 0 – 100
Small Big
Kecil Kecil
Q 35 – 42
Small Small
Besar Kecil
S 35 – 42
Big Small
Besar Besar
T 0 – 100
Big Big

Jadual 3.1 / Table 3.1

129
  Fizik  Tingkatan 4  Bab 4 Haba

Berdasarkan Jadual 3.1,


Based on Table 3.1,
(i) pilih julat termometer yang sesuai untuk mengukur suhu badan pesakit.
choose the suitable range of thermometer to measure the body temperature of the patient.
35 – 42°C
[1 markah / mark]
(ii) Beri satu sebab bagi jawapan dalam 3(b)(i).
Give one reason for the answer in 3(b)(i).
Perubahan suhu badan manusia dalam julat 35 - 42°C sahaja.

.
hd
Changes in human’s body temperature in the range of 35 – 42°C only.
[1 markah / mark]

.B
(iii) Pilih saiz bebuli yang sesuai untuk termometer.
Choose the suitable size of bulb of the thermometer.
Saiz kecil / Small size

dn
[1 markah / mark]
BAB

(iv) Beri satu sebab bagi jawapan di 3(b)(iii).

iS
Give one reason for the answer in 3(b)(iii).
Bebuli yang lebih kecil akan menyerap dan membebaskan tenaga haba dalam masa yang lebih singkat dan
4
ng
bergerak balas dengan lebih cepat terhadap perubahan suhu.
A smaller bulb will absorb and release heat energy in a shorter time and responds faster to temperature change.
la
[1 markah / mark]
(v) Pilih diameter tiub kapilari termometer yang sesuai
Pe

Choose the suitable diameter of capillart tube of the thermometer.


Diameter tiub kapilari termometer perlu kecil.
Diameter of capillary tube of the thermometer needs to be small.
n

[1 markah / mark]
ita

(vi) Beri satu sebab bagi jawapan dalam 3(b)(v).


Give one reason for the answer in 3(b)(v).
rb

Diameter yang kecil akan meningkatkan kepekaan termometer.


The smaller diameter will increase the sensitivity of the thermometer.
ne

[1 markah / mark]
(vii) Berdasarkan jawapan di 3(b)(i), 3(b)(iii) and 3(b)(v). Pilih termometer yang paling sesuai yang dapat
Pe

digunakan bagi menentukan suhu badan pesakit.


Based on the answers in 3(b)(i), 3(b)(iii) and 3(b)(v). Choose the most suitable thermometer which can be used to determine
the body temperature of the patient.
Termometer Q.
Thermometer Q.
[1 markah / mark]

130
Fizik  Tingkatan 4  Bab 4 Haba 

Bahagian B

4 . (a) Rajah 4.1 menunjukkan dua objek dengan suhu yang berbeza dalam sentuhan terma. Kedua-dua objek
mempunyai suhu yang berbeza.
Diagram 4.1 shows two objects of different temperature in thermal contact. Both are having different temperatures.

Objek A Objek B
Object A Object B
120°C 50°C

.
hd
Rajah 4.1 / Diagram 4.1

(i) Apakah yang di maksudkan dengan suhu?
What is meant by temperature? [1 markah / mark]

.B

(ii) Berdasarkan Rajah 4.1 bandingkan suhu kedua-dua objek dan kesan haba yang dialami oleh kedua-dua
objek dan arah pengaliran haba. Hubung kaitkan arah pengaliran haba dengan perbezaan suhu antara
objek yang bersentuhan secara terma. Apakah yang berlaku pada suhu akhir apabila keseimbangan

dn
terma tercapai?
Based on the Diagram 4.1, compare the temperature of the objects, the effect of heat experienced by the objects and the

BAB
iS
direction of heat flow. Relate the direction of the heat flow to the difference in temperature between objects which are in
thermal contact. What will happen to the final temperature when the thermal equilibrium is obtained?
[5 markah / marks]
4
(b) Rajah 4.2 menunjukkan fenomena bayu darat yang berlaku pada waktu malam. Terangkan bagaimana fenomena
ng
ini berlaku.
Diagram 4.2 shows the phenomenon of land breeze which occur during the night. Explain this phenomenon.
la
[4 markah / marks]
Pe
n

Rajah 4.2 / Diagram 4.2


ita

(c) Rajah 4.3 menunjukkan sebuah lampu belajar yang digunakan oleh seorang
pelajar. Lampu ini tidak sesuai digunakan sebagai lampu untuk belajar. Terangkan
rb

pengubahsuaian pada lampu meja yang dapat digunakan oleh pelajar tersebut.
Diagram 4.3 shows a table lamp used by a student. This type of lamp is not suitable used as study
lamp. Explain the modifications that can be made to improvise the table lamp used by the student.
ne

Dalam penerangan anda berikan penekanan bagi aspek-aspek berikut: 4


Emphasize your explanation in the following aspects:
(i) memberi keselesaan kepada pelajar
Pe

Rajah 4.3 / Diagram 4.3


comfort the student
(ii) mengurangkan kehilangan haba pada lampu
reduce the energy loss from the lamp
(iii) reka bentuk lampu meja tersebut
the design of the table lamp
(iv) ciri-ciri keselamatan untuk penggunaan lampu
safety features of the lamp
(v) kecekapan tenaga lampu itu
the energy efficiency of the lamp
[10 markah / marks]

131
  Fizik  Tingkatan 4  Bab 4 Haba

Bahagian C

5. (a) Apakah yang dimaksudkan dengan keseimbangan terma?


What is meant by thermal equilibrium? [1 markah / mark]
(b) Seseorang dengan demam yang tinggi (lebih daripada 40°C) perlu menurunkan suhu badan. Salah satu cara
adalah dengan meletakkan tuala basah di dahinya. Jelaskan pemindahan tenaga haba yang berlaku.
A person with high fever (above 40°C) needs to bring down the body temperature. One way is by placing the towel on the forehead
of the person. Explain the heat transfer that takes place. [4 markah / marks]
(c) Anda ditugaskan untuk memilih sebuah termometer untuk mengukur suhu di dalam kotak pembeku dalam peti
sejuk. Suhu kotak pembeku haruslah di antara –10°C hingga 85°C.

.
You are assigned to choose a thermometer to measure the temperature in a freezer compartment in the refrigerator. The freezer

hd
compartment temperature must be between – 10°C to 85°C.
Jadual 5 menunjukkan ciri-ciri empat jenis termometer P, Q, R dan S.

.B
Table 5 shows the characteristics of four types of thermometers P, Q, R and S.

Jadual 5.1 / Diagram 5.1

Jenis Takat beku Takat didih Diameter

dn
Warna Dinding bebuli
Termometer cecair cecair (°C) cecair (°C) tiub kapilari
Thermometer
cecair kaca
Type of Freezing point Boiling point of Colour of Diameter of
BAB

Wall of glass bulb


liquid of liquid (°C) liquid (°C) liquid capillary tube

iS
Merkuri Legap Tebal Kecil
P –39 360
4 Mercury Opaque Thick Small

Alkohol
ng Lutsinar Nipis Kecil
Q –112 78
Alcohol Transparent Thin Small
la
Alkohol Lutsinar Tebal Besar
R –112 78
Alcohol Transparent Thick Big
Pe

Merkuri Legap Nipis Kecil


S –39 360
Mercury Opaque Thin Small
n

Berdasarkan kepada Jadual 5.1, 5


Based on Table 5.1,
ita

(i) Jelaskan ciri-ciri termometer yang sesuai supaya termometer tersebut boleh digunakan untuk mengukur
suhu kotak pembeku itu.
rb

Explain the suitable characteristics of the thermometer that can be used to measure temperatures of the freezer
compartment. [8 markah / marks]
ne

(ii)
Tentukan termometer yang manakah paling sesuai untuk digunakan dan berikan sebab-sebab bagi pilihan
anda.
Decide which thermometer is the most suitable to be used and give reasons for your choice. [2 markah / marks]
Pe

(d) Panjang turus merkuri sebuah termometer pada takat ais dan takat stim ialah 5 cm dan 25 cm masing-masing.
Apabila termometer itu direndam di dalam cecair X, panjang turus merkuri ialah 16 cm. Tentukan suhu cecair X
dalam Kelvin.
The lengths of the mercury column of a thermometer during ice point and steam point are 5 cm and 25 cm respectively. When the
thermometer is immersed in liquid X, the length of the mercury column is 16 cm. Determine the temperature of liquid X in kelvin.
[2 markah / marks]

132
PRAKTIS SPM 4
JAWAPAN – Haba berpindah dari kawasan yang panas ke
kawasan yang sejuk.
Soalan Struktur Heat flows from hot region to cold region.
Bahagian B – Kedua-dua objek akan mencapai suhu yang
sama pada keseimbangan terma.
4. (a) (i) Suhu adalah ukuran darjah kepanasan sesuatu
Both objects will reach the same temperature at
objek. thermal equilibrium.
Temperature is a measurement of degree of hotness of an

.
object. (b) – Pada waktu malam, haba hilang daripada darat dan

hd
air laut.
(ii) – Objek A lebih panas daripada objek B/
At night, heat is lost from land and sea water.
Objek A mempunyai suhu yang lebih tinggi
– Darat akan menjadi lebih sejuk daripada laut
daripada objek B.
kerana darat mempunyai muatan haba tentu yang
The object A is hotter than the object B/ Object A has

.B
higher temperature than the object B. lebih kecil berbanding air laut.
Land become cold first than sea because land has smaller
– Pengaliran haba berlaku dari kedua-dua arah.
heat capacity compare to the sea.
The heat flow occurs from both directions.
– Udara di atas laut yang lebih panas akan bergerak
– Kadar pengaliran haba daripada objek A ke

dn
naik ke atas.
objek B adalah lebih besar daripada kadar
Warmer air above the sea rises.
pengaliran haba daripada objek B ke objek A.
– Udara sejuk dari darat akan bertiup kearah laut
The rate of heat flow from the object A to the object B
is higher than the rate of heat flow from the object B sebagai bayu darat.

iS
to the object A. Cooler air from land moves towards the sea as land breeze.

(c) Cadangan Penjelasan


Suggestion
ng Explanation

Guna kaki yang boleh dilaraskan Ketinggian lampu boleh dilaraskan mengikut keselesaan pelajar
Use adjustable stand The height of the lamp can be adjusted according to the comfort of the student
la
Gunakan lampu pendarfluor Menggunakan kuasa yang kecil berbanding lampu filamen
Use the fluorescent lamp Use less power than filament lamp
Pe

Menggunakan penutup yang besar Kurang pantulan kepada mata dan kurang penyerapan tenaga haba kerana merupakan
dan berwarna putih pemantul haba yang baik
Bigger cover with white colour Less reflection on eyes and absorb less heat energy and good heat reflector

Sambung dengan wayar bumi Mengelakkan litar pintas dan kerosakan pada mentol
n

Connect with the earth wire Avoid short circuit and damage on the bulb
ita

Guna lampu penjimat tenaga Menghasilkan kecerahan yang sama dengan penggunaan kuasa yang rendah
Use an energy saver lamp Produce same brightness with less power consumption
rb

Bahagian C – Tuala yang basah kemudian dibilas menggunakan


5. (a) Dua objek dalam keadaan keseimbangan terma mempunyai air paip supaya haba dari tuala akan dipindahkan
ne

suhu yang sama. Tiada pemindahan bersih haba antara ke air sehingga keseimbangan terma tercapai.
dua objek yang berada dalam keseimbangan terma. The wet towel is then rinsed under the tap water so that
Two objects in thermal equilibrium have the same temperature. the heat from towel will be transferred to the tap water until
There is no net heat transfer between two objects in the thermal thermal equilibrum is reached.
Pe

equilibrium. – Langkah-langkah ini diulang sehingga jumlah haba


(b) – Tuala dibilas dengan air paip yang suhunya lebih yang cukup dipindahkan keluar dari badan untuk
rendah daripada suhu badan. menurunkan suhu badan.
The towel is rinsed with tap water which is lower than the The steps are repeated until the right amount of the heat
body temperature. transferred out of the body to bring the fever down.
– Apabila tuala basah diletakkan di dahi, haba dari
dahi akan dipindahkan ke tuala basah sehingga
keseimbangan terma tercapai.
When the towel is placed on the forehead, the heat from
the body will be transferred to the wet towel until thermal
equilibrium is reached.
  Fizik  Tingkatan 4 Jawapan

(c) (i) Cadangan Penjelasan


Suggestion Explanation

Menggunakan merkuri Merkuri mempunyai titik beku -39°C dan takat didih pada 360°C jadi merkuri
Use mercury dapat mengukur suhu antara - 10°C hingga 85°C
Mercury has freezing point -39°C and boiling point of 36 o C so it able to measure
temperature between –10°C to 85°C

Cecair dalam termometer harus legap Supaya bacaan termometer boleh dibaca dengan mudah
Liquid in the thermometer should be opaque So that the reading of thermometer can be read easily

Dinding bebuli yang nipis Sensitif kepada perubahan suhu yang kecil
Use thin wall of bulb Sensitive to small change of the temperature

.
Termometer harus mempunyai diameter Termometer akan menjadi lebih sensitif

hd
tiub kapilari yang kecil The thermometer will be more sensitive
The thermometer should have small diameter of
capillary tube

.B
(ii) Termometer paling sesuai adalah termometer S. Termometer S menggunakan merkuri, cecair legap, dinding bebuli yang nipis
dan diameter tiub kapilari yang kecil.
The most suitable thermometer is thermometer S. Thermometer S uses mercury, opaque liquid, thin wall of glass bulb and small diameter of
capillary tube.

dn
(d) Suhu cecair X /Temperature of liquid X
16 – 5
= × 100 = 55°C
25 – 5

iS
Suhu cecair X dalam Kelvin /Temperature of liquid X in Kelvin
= 55 + 273 K
= 328 K ng
la
Pe
n
ita
rb
ne
Pe
B
BA
5
Gelombang
Waves
Analisis Soalan SPM
Kertas 2011 2012 2013 2014 2015 2016 2017 2018
1 ✔ ✔ ✔ ✔ ✔ ✔ ✔ ✔

PETA Konsep 2
3


✔ ✔ ✔ ✔

.
hd
GELOMBANG
WAVES

.B
dn
Jenis-jenis Pelembapan Fenomena Gelombang
gelombang dan resonans gelombang elektromagnet
Types of waves Damping and Phenomena of Electromagnetic

iS
resonance wave wave

Gelombang Gelombang
ng Pantulan Spektrum
progresif pegun gelombang elektromagnet
Progressive wave Stationary wave Gelombang Gelombang Reflection of Electromagnetic
la
mekanik elektromagnet wave spectrum
Mechanical Electromagnetic
wave wave
Pe

Gelombang Gelombang Pembiasan


melintang membujur Gelombang Gelombang gelombang
Refraction of
Transverse wave Longitudinal air cahaya
wave
Water wave Light wave
n

wave
ita

Pembelauan
Gelombang air Gelombang Gelombang Gelombang gelombang
Water wave bunyi bunyi Radio Diffraction of
rb

Sound wave Sound wave Radio wave wave


ne

Gelombang
cahaya Interferens
Light wave gelombang
Interference of
Pe

wave
Gelombang
radio
Radio wave

133
  Fizik  Tingkatan 4  Bab 5 Gelombang

Asas Gelombang
5.1 Fundamental of Waves

Gelombang / Waves

1. Gelombang dihasilkan oleh sistem bergetar . Getaran yang merambat melalui satu lintasan yang tetap
secara berulang-alik menghasilkan gelombang. 1 2

The waves are generated by vibrating system . The vibrations propagates through a fixed path to and fro produce waves.
2. Contoh jenis gelombang yang biasa ialah gelombang air, gelombang bunyi, gelombang elektromagnet dan

.
gelombang cahaya.

hd
Examples of common types of waves are water waves, sound waves, electromagnetic waves and light waves.

3. Sistem bergetar merupakan punca gelombang dan boleh memindahkan tenaga dari satu tempat
ke tempat yang lain.

.B
The vibrating system is the source of the wave and transfers energy from one place to another.

4. Apabila gelombang merambat melalui satu medium, zarah-zarah dalam medium itu akan bergetar

dn
pada kedudukan keseimbangannya.Walaupun begitu, zarah-zarah medium itu tidak bergerak bersama-
sama dengan gelombang itu.
vibrate

iS
When a wave travels through a medium, the particles in the medium will at its equilibrium position.
However, the medium particles do not move along with the wave.
Arah gelombang
ng
Wave direction
BAB

la
Arah getaran
Direction of vibration Lembangan / Trough
Gelombang melintang dalam spring slinki
5 Transverse waves in a slinky spring
Pe

Rajah 1 / Diagram 1
5. Rajah 1 menunjukkan satu spring slinki yang diayun oleh seorang pelajar. Gelombang yang terhasil merambat
ke depan tetapi zarah-zarah medium tidak bergerak bersama-sama dengan gelombang yang dihasilkan
n

tetapi hanya bergetar pada tempat yang sama .


ita

Diagram 1 shows a slinky spring vibrated by a student. The waves produced propagate forward but the medium particles
do not move along with the waves produced but only vibrate in the same place.
rb

Jenis-jenis Gelombang / Types of Waves


ne

1. Terdapat dua jenis gelombang iaitu gelombang progresif dan gelombang pegun .
There are two types of waves that are progressive waves and stationary waves.
Pe

2. Gelombang progresif memindahkantenaga dari satu tempat ke tempat lain. Gelombang bunyi yang datang
dari sumber ke telinga kita adalah satu contoh gelombang progresif.
Progressive waves transfer energy from one place to another. The sound wave coming from the source to our
ear is an example of a progressive wave.

3. Gelombang pegun tidak memindahkan tenaga dari satu tempat ke tempat lain. Gelombang pegun terbentuk
apabila dua gelombang progresif dengan frekuensi yang sama dan amplitud sama, merambat dalam arah yang
bertentangan, bertembung antara satu sama lain.
Stationary waves do not transfer energy from one place to another. Stationary waves are formed when two progressive
waves with the same frequency and similar amplitude, travelling in opposite directions, interfere with each other.

134
Fizik  Tingkatan 4  Bab 5 Gelombang 

4. Contoh gelombang pegun ialah gelombang yang terhasil di dalam alat-alat muzik seperti apabila tali gitar dipetik.
Example of stationary waves is the resulting waves in musical instruments such as when the guitar strings are plucked.

5. Perbezaan antara gelombang pegun dan progresif adalah gelombang progresif memindahkan tenaga
dari satu tempat ke tempat lain manakala gelombang pegun tidak memindahkan tenaga dari satu
tempat ke tempat lain.
The difference between stationary and progressive waves is that progressive waves transfer energy from one place
to another whereas stationary waves do not transfer energy from one place to another.

6. Gelombang juga boleh dibahagikan kepada gelombang elektromagnet dan gelombang mekanik .
electromagnetic mechanical

.
Waves can also be divided into waves and waves.

hd
7. Gelombang elektromagnet adalah gelombang yang tidak memerlukan medium untuk merambat sedangkan
gelombang mekanikal memerlukan medium untuk perambatannya.

.B
Electromagnetic waves are waves that do not require medium to propagate while mechanical waves require
medium to propagate.

8. Contoh gelombang elektromagnet ialah gelombang cahaya dan gelombang radio . Contoh

dn
gelombang mekanik pula ialah gelombang air dan bunyi .
Examples of the electromagnetic wave is light waves and radio waves. Examples of the mechanical

iS
waves are water waves and sound waves.

Perbezaan antara Gelombang Melintang dengan Gelombang Membujur


The Differences between Transverse Waves and Longitudinal Waves
ng

BAB
1. Terdapat dua jenis gelombang yang utama, iaitu gelombang melintang dan gelombang membujur .
la
There are two main types of waves, namely transverse waves and longitudinal waves.
5
Pe

2. Gelombang melintang ialah gelombang di mana zarah-zarah mediumnya bergetar pada arah yang
berserenjang dengan arah perambatan gelombang.
A transverse wave is a wave in which the particles of the medium vibrate in the direction perpendicular
n

to the direction of wave propagation.

Contoh: Gelombang cahaya air


ita

dan gelombang .
Example: Light waves and water waves.
puncak lembangan
rb

3. Gelombang melintang terdiri daripada satu siri dan .


Transverse waves consist of a series of crests and troughs .
ne

Arah perambatan
Puncak Direction of propagation Arah getaran
Crest Direction of vibration
Pe

Arah
perambatan
Direction of
Zarah dan arah getarannya propagation
Lembangan Particles and direction of vibration
Trough

4. Gelombang membujur adalah gelombang di mana getaran zarah selari dengan arah gerakan gelombang.
Longitudinal waves are waves in which the vibration of the particles is parallel to the direction of motion of the waves.

Contoh: Gelombang bunyi .


Example: Sound waves.

135
  Fizik  Tingkatan 4  Bab 5 Gelombang

5. Gelombang membujur terdiri daripada siri pemampatan dan rengangan .


Longitudinal waves consist of a series of compressions and rarefactions .

Mampatan Regangan
Compression Rarefaction Arah getaran zarah Arah perambatan gelombang
Direction of vibration of particle Direction of propagation of the wave

Ciri-ciri Gelombang / Characteristics of Waves

.
hd
1. Suatu sistem ayunan merujuk kepada suatu sistem yang bergerak secara ulang-alik dalam suatu lintasan yang
tetap.

.B
An oscillating system refers to a system that moves in a fixed track with to and fro movement.
Contoh: Ayunan bandul ringkas dan spring berbeban.
Spring
Example: A simple pendulum and weighted spring. Spring
Benang

dn
String
2. Satu ayunan lengkap berlaku apabila bandul atau spring itu
telah bergerak melalui lintasan P ke Q dan balik dari P

iS
Q ke P . Jisim
Ladung Mass O
A complete oscillation occurs when the pendulum bob and spring has
ng Bob
moved from position P to Q and back from Q to P . P Q
O Q
BAB

3. Amplitud, A, satu sistem bergetar ialah sesaran maksimum dari kedudukan keseimbangannya.
la
Amplitude, A, is the maximum displacement from the equilibrium position.
ketinggian kedalaman
5
Pe

4. Amplitud gelombang melintang adalah sama dengan puncak atau palung.


The amplitude of transverse waves is equal to the height of a crest or the depth of a trough.

5. Tempoh, T satu sistem bergetar ialah masa yang diambil oleh sistem itu untuk membuat satu
n

ayunan lengkap .
ita

Period, T of a vibrating system is the time taken by the system to make one complete oscillation.

6. Frekuensi, f ialah bilangan ayunan lengkap yang dilakukan oleh satu sistem bergetar dalam masa
rb

satu saat.
Frequency, f is the number of complete oscillations made by a vibrating system in one second.
ne

7. Panjang gelombang, λ ialah jarak di Puncak


Crest
antara dua titik berturutan pada satu
Panjang gelombang, λ
Pe

gelombang yang bergetar pada fasa yang Wavelength, λ


sama. (e.g. jarak di antara dua puncak
berturutan atau dua palung berturutan). Amplitud, A
Wavelength, λ is the distance between two Amplitude, A Kedudukan
consecutive points on a wave that vibrates in keseimbangan
the same phase. (e.g., distance between two Equilibrium
consecutive crests or two consecutive troughs). position
A

Lembangan
Trough

136
Fizik  Tingkatan 4  Bab 5 Gelombang 

panjang gelombang λ wavelength λ


8. Laju gelombang = , v = / Wave speed = ,v=
T
tempoh T period
1
Maka / Therefore v =  λ
T
v = f λ

Contoh 1
Dalam satu eksperimen, Mat memerhati bahawa bandul ringkas membuat 20 ayunan lengkap dalam 36.0 saat.
In an experiment, Mat observes that a simple pendulum completes 20 oscillations in 36.0 seconds. Berapakah
(a) tempoh ayunan bandul itu? / the period of oscillation of the pendulum?

.
hd
(b) frekuensi ayunan bandul itu? / the frequency of oscillation of the pendulum?
Penyelesaian / Solution
Masa yang diambil 36

.B
(a) Tempoh, T = = = 1.8 s
Bilangan ayunan lengkap 20
Masa yang diambil 36
Tempoh, T = = = 1.8 s
Bilangan ayunan lengkap 20

dn
1 1
(b) Frekuensi / Frequency, f = = = 0.56 Hz
T 1.8

iS

Contoh 2 ng
Satu zarah dalam gelombang bunyi berayun 80 ayunan lengkap dalam 16 s.

BAB
A particle in a sound wave makes 80 complete oscillations in 16 s.
la
Kira / Calculate
(a) tempoh getaran / the period of the vibration
5
Pe

(b) frekuensi gelombang / the frequency of the wave


Penyelesaian / Solution
(a) 80 ayunan dalam 16 s / 80 oscillations in 16 s.
n

16
1 ayunan / oscillation in = = 0.2 s
80
ita

Tempoh / Period, T = 0.2 s


1
(b) Frekuensi / Frequency = = 5 Hz
rb

0.2

ne

Melakar dan Mentafsir Graf Gelombang


Sketching and Interpreting Waves Graphs
Pe

1. Gerakan gelombang boleh diwakili dua jenis graf.


Wave motion can be represented by two types of graph.

A Graf sesaran melawan masa / Displacement against time graph


Graf sesaran melawan masa menunjukkan bagaimana sesaran zarah tertentu berubah dengan
masa . Graf sesaran melawan masa memberikan amplitud dan tempoh gelombang.
A displacement against time graph shows how the displacement of a particular particle changes with time .
A displacement against time graph gives the amplitude and period of the wave.

137
  Fizik  Tingkatan 4  Bab 5 Gelombang

Contoh 3
Rajah menunjukkan graf sesaran melawan masa suatu sistem yang bergetar. Tentukan
Diagram shows the displacement againsts time graph of a vibrating system. Determine the
(a) amplitud / amplitude,
Sesaran, x/cm
(b) tempoh / period, Displacement, x/cm
(b) frekuensi / frequency. 10

Penyelesaian / Solution 0
6 Masa, t/s
(a) Amplitud / Amplitude, A = 10 cm Time, t/s
–10

.
3

hd
(b) Tempoh / Period, T = 6 s
4
T=8s
1
(c) Frekuensi / Frequency, f = = 0.125 Hz

.B
8

dn
B Graf sesaran melawan jarak / Displacement against distance graph
Graf sesaran maelawan jarak menunjukkan bagaimana sesaran zarah tertentu berubah dengan

iS
jarak . Graf sesaran melawan jarak memberikan amplitud dan panjang gelombang .
A displacement against distance graph shows how the displacement
ng of a particular particle changes with distance .
A displacement against distance graph gives the amplitude and wavelength .
BAB

Contoh 4
la
Rajah 1.6 menunjukkan suatu graf sesaran melawan jarak. Sesaran /cm
5
Pe

Diagaram 1.6 shows a displacement against distance graph. Displacement /cm


Tentukan / Determine
(a) amplitud gelombang / the amplitude of the wave, 20
(b) panjang gelombang / the wavelength.
0 Jarak /cm
n

Penyelesaian / Solution 75 Distance /cm


ita

–20
(a) Amplitud / Amplitude, A = 30 cm
5
(b) Panjang gelombang / The wavelength, λ = 75 s
4
rb

λ = 60 cm
75
atau / or × 8 = 60 cm
ne

10

Pe

Menentukan Panjang Gelombang, λ, Frekuensi, f dan Laju Gelombang, v


Determining wavelength, λ, frequency, f, and wave speed, v

1. Hubungan antara halaju v, frekuensi f dan panjang gelombang λ boleh dinyatakan seperti berikut
The relationship between velocity v, frequency f and wavelength λ can be expressed as follows

v = f λ

2 Graf gelombang: (i) sesaran melawan masa (ii) sesaran melawan jarak boleh menunjukkan amplitud, tempoh
dan panjang gelombang dan seterusnya halaju gelombang.
Wave graph: (i) displacement versus time (ii) against a displacement can show amplitude, period and wavelengths and
finally the velocity of the wave.

138
Fizik  Tingkatan 4  Bab 5 Gelombang 

Sesaran /cm Sesaran /cm


Displacement /cm Displacement /cm

A A
Masa /s Jarak /cm
Time /s Distance /cm

(i) (ii)

.
Tugasan

hd
1. Gelombang air dihasilkan oleh sumber bergetar 40 Hz. Jika panjang gelombang air itu ialah 6.0 cm, berapakah tempoh

.B
dan halaju perambatan gelombang air itu?
The water waves are produced by 40 Hz vibrating source. If the wavelength of water was 6.0 cm, what is the period and velocity of the
propagation of water waves?
1 1

dn
(a) Tempoh / Period, T = = = 0.025 s
f 40
(b) Halaju / Velocity, v = f λ = 40 (0.06) = 2.4 m s–1

iS
2. Rajah-rajah di bawah menunjukkan graf gelombang untuk graf sesaran melawan jarak dan graf sesaran melawan
masa, untuk satu punca gelombang. Daripada dua graf tersebut, tentukan
The diagrams below shows the wave graph for displacement against distance graph and displacement against time graph for one wave
source. From the two graphs, determine the
ng

BAB
Sesaran /cm Sesaran /cm
Displacement /cm Displacement /cm
la

10
5
Pe

5
0 Jarak /cm 0 Masa /s
–5 9 Distance /cm 40 Time /s
–10
n


ita

(a) amplitud gelombang / amplitude of the wave,


(b) tempoh gelombang / period of the wave,
(c) frekuensi gelombang / frequency of the wave,
rb

(d) panjang gelombang / wavelength,


(e) halaju gelombang / velocity of the wave.
ne

(a) Amplitud / Amplitude, A = 5 × 3 = 15 cm


5
(b) Tempoh / Period,
Pe

T = 40 s
4
40
T = 32 s  atau / or  × 8 = 32 s
10
1
(c) Frekuensi / Frequency, f = = 0.03125 Hz
32
3
(d) Panjang gelombang / Wavelength = λ = 9 s
4
9
λ = 12 cm  atau / or  × 8 = 12 cm
6
(e) Halaju gelombang / Velocity of the wave, v = f λ = 0.03125 (12) = 0.375 cm s–1

139
  Fizik  Tingkatan 4  Bab 5 Gelombang

Pelembapan dan Resonans


5.2 Damping and Resonance

1. Dalam suatu sistem berayun seperti ayunan bandul ringkas, amplitud ayunan bandul akan berkurangan

secara beransur-ansur dan menjadi sifar apabila ayunan berhenti.


In an oscillating system such as the oscillation of a simple pendulum, the amplitude of oscillation of the simple pendulum
will gradually decrease and become zero when the oscillation stops.

2. Pengurangan dalam amplitud sistem berayun dipanggil pelembapan .


The decrease in the amplitude of an oscillating system is called damping .

.
hd
Amplitud, A/cm Sesaran
Amplitude, A/cm Displacement

.B
Amplitud berkurang secara beransur-ansur
Amplitude gradually decrease

0 Jarak

dn
Distance

Masa, t/s
Time, t/s

iS
3. Sistem berayun mengalami pelembapan apabila tenaganya bertukar sebagai tenaga haba .
The oscillating system experiences damping when it changes as
ng heat energy.

(a) Pelembapan luar adalah kehilangan tenaga untuk mengatasi daya geseran atau
BAB

rintangan udara.
la
External damping is the loss of energy to overcome friction or air resistance.
dalaman pemampatan
5
Pe

(b) Pelembapan adalah kehilangan tenaga disebabkan oleh regangan dan


molekul dalam sistem.
Internal damping is a loss of energy due to the extension and compression of molecules in the system.
frekuensi asli
n

4. Frekuensi sistem yang berayun secara bebas tanpa tindakan daya luaran dipanggil .
The frequency of a system which oscillates freely without the external force acting, is called the natural frequency .
ita

5. Resonans berlaku apabila sistem dipaksa untuk berayun pada frekuensi yang sama dengan frekuensi
asli oleh daya luar. Sistem ayunan akan berayun dengan amplitud maksimum .
rb

Resonance occurs when a system is made to oscillate at a frequency equivalent to its natural frequency by an external
ne

force. The resonating system oscillates at maximum amplitude.

Pelembapan dan Resonans bagi Satu Sistem Ayunan dan Getaran


Pe

Damping and Resonance for an Oscillating and Vibrating System

1. Pelembapan dalam sistem berayun menyebabkan amplitud Sesaran


Displacement
dan tenaga sistem berkurangan tetapi tempoh, T dan
frekuensi, f tidak berubah seperti ditunjukkan dalam rajah.
Damping in an oscillating system causes amplitude and energy system 0 Masa
period, T frequency, f Time
reduced but the and does not
change as shown in the diagram. T T T

140
Fizik  Tingkatan 4  Bab 5 Gelombang 

2. Untuk membolehkan sistem berayun diteruskan secara berterusan, daya luar perlu ditindakkan pada sistem.
Daya luar menyebabkan sistem berayun dalam keadaan yang dikenali sebagai ayunan paksa .
To enable the oscillating system to oscillate continuously, external force must be applied to the system. The external force
causes the system to oscillate in a state called forced oscillation .

3. Jika sistem berayun berayun secara bebas tanpa daya luar bertindak, maka ia berayun dengan frekuensi asli .
If oscillating system oscillates independently without external force acting, it is oscillating in natural frequency .

4. Ciri-ciri resonans dapat dinyatakan dengan Tali penyokong


sistem bandul Barton seperti yang ditunjukkan Supporting string

.
dalam Rajah.

hd
The characteristics of resonance can be demonstrated Benang halus
with Barton’s pendulum system as shown in the Fine thread
Diagram. Bandul

.B
Pendulum
5. Apabila bandul Y berayun, semua bandul lain
Beban berat
terpaksa berayun . P Y Heavy weight
Q
When pendulum Y oscillates, all the other pendulums R

dn
are forced to oscillate . S

6. Antara bandul-bandul berkenaan, hanya bandul

iS
P mempunyai frekuensi asli yang sama dengan frekuensi bandul Y kerana kedua-dua bandul itu mempunyai
panjang tali yang sama . Maka, P berayun dalam resonans dan amplitudnya adalah
ng maksimum .
Among the pendulums, only pendulum P has the same natural frequency as the frequency of pendulum Y because both
same maximum .

BAB
pendulum has the length string. So, P will oscillate in the resonance and the amplitude is
la
Kesan Resonans dalam Kehidupan Seharian
The Effects of Resonance in Daily Lives
5
Pe

1. Keruntuhan jambatan Tacoma Narrows adalah contoh kerosakan akibat resonans. Tindakan angin menyebabkan
jambatan itu bergetar dengan amplitud yang besar akibat daripada resonans . 3
The Tacoma Narrows bridge collapse is an example of the destructive potential of resonance. The action of the wind caused
n

the bridge to vibrate with a large amplitude as a result of resonance .


ita

2. Satu lagi contoh resonans adalah buaian. Apabila kita mula menolak orang di
atas buaian, gerakan pada mulanya perlahan dan ia berayun dengan amplitud
rb

yang kecil . 3
Another example of resonance is a swing. When we start to push the person on the swing,
ne

the motion initially is slow and it swings with a small amplitude.

Apabila ayunan mencapai frekuensi asli , tolakan yang kecil akan


Pe

mengekalkan amplitud ayunan maksimum disebabkan oleh resonans.


But once when the swing reaches its natural frequency of oscillation, a gentle push to
the swing helps it maintain the maximum amplitude of swing due to resonance.

3. Seorang penyanyi opera boleh memecahkan sebiji gelas wain jika menyanyi dengan suara soprano pada
frekuensi tinggi yang sama dengan frekuensi asli gelas wain. Dinding gelas akan bergetar dengan amplitud
yang besar apabila resonans berlaku dan akan pecah. 3

An opera singer can break a wine glass if she sings with soprano voice at a high frequency that equal to the natural frequency
of the glass. The wall of the glass vibrates with large amplitudes when resonance occurs thus breaking the glass.

141
  Fizik  Tingkatan 4  Bab 5 Gelombang

Pantulan Gelombang
5.3 Reflection of Waves

1. Pantulan gelombang berlaku apabila gelombang tuju terkena pemantul atau halangan. Gelombang
mengalami perubahan dalam arah perambatan gelombang.
Reflection of a wave occurs when incident wave strikes an obstacle or barrier. The wave undergoes a change in
direction of wave propagation.

2. Pantulan gelombang air boleh diperhatikan dalam sebuah tangki riak.

.
Reflection of water waves can be observed in a ripple tank.

hd
Getaran ke atas
dan ke bawah
oleh motor

.B
Vibrated up and Lampu
down by motor Lamp
Bekas berdasar kaca
Glass-bottomed tray
Pencelup

dn
Dipper

iS
ng
BAB

Corak gelombang
Skrin putih
la
atas skrin
White screen
Wave pattern
on screen
5
Pe

3. Rajah menunjukkan gelombang tuju sebelum ia terkena halangan atau Muka gelombang
Wavefront
pemantul. Gelombang pantulan adalah gelombang yang telah mengalami
perubahan dalam arah perambatan gelombang selepas pantulan.
n

The diagram shows the the incident waves before it strikes the obstacle or barrier.
The reflected wave is the wave which has undergone a change in direction of wave
ita

propagation after reflection.

4. Pantulan gelombang mematuhi hukum pantulan, iaitu sudut tuju adalah


rb

sama Pemantul / Reflector


dengan sudut pantulan. Panjang gelombang bagi gelombang
tuju adalah sama dengan panjang gelombang pantulan.
ne

Reflection of a wave obeys the law of reflection, which is the angle of incidence is equal to the angle of reflection.
The wavelength of the incident wave is equal to the wavelength of the reflected wave.
Pe

5. Frekuensi bagi gelombang tuju, gelombang pantulan, dan penggetar adalah sama. Gelombang yang
dipantulkan mempunyai magnitud halaju yang sama tetapi arah yang berlainan dengan
gelombang tuju.
The frequency for incident wave, reflected wave, and vibrator is the same. The reflected wave has the same velocity
magnitude but the direction is different with the incident wave.

142
Fizik  Tingkatan 4  Bab 5 Gelombang 

6. Rajah di bawah menunjukkan gelombang yang dipantulkan oleh sebuah pemantul.


Diagram below shows wave reflected by a reflector.

i r

.
Pemantul / Reflector

hd
Istilah Penerangan
Term Explanation

.B
Sudut tuju, i Sudut di antara arah perambatan gelombang tuju dan normal.
Angle of incidence, i incident wave

dn
The angle between the direction of propagation of and the normal.

Sudut pantulan, r Sudut di antara arah perambatan gelombang pantulan dan normal.

iS
Angle of reflection, r reflected wave
The angle between the direction of propagation of and the normal.
ng muka gelombang
Panjang gelombang, λ Jarak di antara dua berturutan.
Wavelength, λ

BAB
Distance between two consecutive wavefronts .
la

Bilangan ayunan lengkap yang dilakukan dalam satu saat.


Frekuensi, f 5
Pe

Frequency, f
The number of complete oscillations made in a second.

Jarak perambatan gelombang dalam satu saat, v = f λ.


Laju, v
n

Speed, v Distance travelled by a wave in one second, v = f λ.


ita

Arah gelombang yang berserenjang dengan muka gelombang.


Arah perambatan gelombang
rb

Direction of propagation of waves perpendicular


Wave direction to the wavefront.
ne

7. Fenomena pantulan gelombang mematuhi Hukum Pantulan di mana:


The phenomenon of reflection of waves obeys the Laws of Reflection where:
Pe

(a) sudut tuju, i sama dengan sudut pantulan, r,


the angle of incidence, i is equal to the angle of reflection, r,

(b) gelombang tuju, gelombang pantulan dan garis normal adalah pada satah yang sama.
the incident wave, the reflected wave and the normal lie in the same plane .

143
  Fizik  Tingkatan 4  Bab 5 Gelombang

8. Jadual di bawah menunjukkan perbandingan gelombang asal dan gelombang yang dipantulkan.
The table below shows the comparison of the original wave and the reflected wave.

Gelombang asal Gelombang yang dipantulkan


Original wave Reflected wave

Frekuensi, f Tidak berubah = f


Frequency, f No changes = f

Laju, v Tidak berubah = v


Speed, v No changes = v

.
Panjang gelombang, λ Tidak berubah = λ

hd
Wavelength, λ No changes = λ

Sudut tuju, i sama dengan sudut pantulan, r.


Arah

.B
Direction equal
The angle of incidence, i is to the angle of reflection, r.

Melukis Gambar Rajah Pantulan Gelombang Air

dn
Drawing Diagrams of Reflection of Water Waves

1. Jadual di bawah menunjukkan gelombang air satah yang ditujukan ke suatu pemantul satah. Lakarkan

iS
gelombang air yang dipantul balik dalam ruang yang disediakan.
The table below shows plane water wave directed to a plane reflector. Sketch the reflected water waves in the space provided.

Gelombang air satah


ng Gelombang air yang dipantulkan
Plane water wave Reflected water wave
BAB

la
Pemantul / Reflector

Pemantul / Reflector
5
Pe
n
ita Pemantul / Reflector

Pemantul / Reflector
rb
ne
Pe

Pemantul / Reflector Pemantul / Reflector

144
Fizik  Tingkatan 4  Bab 5 Gelombang 

Aplikasi Pantulan Gelombang dalam Kehidupan Harian


Application of Reflection of Waves in Daily Lives

1. Di tempat berhampiran pantai dengan rumah dan


bangunan, gelombang laut yang membawa tenaga
Pantai
kinetik yang banyak menyebabkan kerosakan yang terlindung
besar dan berterusan terhadap harta benda. Oleh itu, dari hakisan
dinding konkrit atau dinding penghalang dibina untuk The beach is
proctected
memantulkan gelombang laut dan melindungi from erosion

harta benda daripada musnah. 3 Laut


Sea

.
In places near seaside with houses and buildings, sea waves

hd
that carry large amounts of kinetic energy often cause huge
and continuous damages to the properties. Therefore, concrete Gelombang pantul Gelombang tuju Dinding penghalang
reflect Reflected wave Incident wave Concrete walls
walls or barrier walls are built to sea waves

.B
and protect the properties from destroy.

2. Endoskop terdiri daripada sekumpulan fiber optik. Cahaya diletakkan pada satu hujung dan ia mengalami
pantulan

dn
banyak dalam penuh. Ia digunakan untuk memeriksa organ-organ dalaman pesakit. 3
Endoscope is made up of a narrow bunch of optical fibres.
Light is put in at one end and it undergoes multiple total internal reflection . It is used to examine the internal organs
of patients.

iS
ng

BAB
la

5
Pe

3. Cermin sisi dan cermin pandang belakang digunakan untuk melihat kereta di belakang dan di dua sisi kereta
ketika memotong, mengundur, mengubah arah dan memakir kereta. Cermin ini memantulkan gelombang
ita

cahaya dari kenderaan atau benda lain ke mata pemandu. 3


Side mirror and rear view mirror are used to view cars behind and two sides of the car when overtaking, reversing, turning
rb

and parking the car. These mirrors reflect light waves from other vehicles or objects into the driver’s eyes.

Cermin pandang belakang


ne

Rear view mirror


Pe

Cermin sisi
Side mirror

Cermin sisi
Side mirror

145
  Fizik  Tingkatan 4  Bab 5 Gelombang

Menyelesaikan masalah melibatkan pantulan gelombang


Solving Problems Involving Reflection of Waves

Contoh 5
Gelombang ultrasonik dihantar dari kapal ke dasar laut. Gelombang yang dipantulkan diterima 1.8 saat selepas
penghantaran.
Ultrasonic waves are transmitted from a ship to the bottom of the sea. The reflected waves are received 1.8 s after transmission.
Pemancar dan penerima
Transmitter and receiver

.
hd
.B
Kedalaman, D
Depth, D

dn
Apakah kedalaman laut?
(Laju gelombang ultrasonik di dalam air laut = 1 500 m s–1)

iS
What is the depth of the sea?
(Speed of ultrasonic waves in sea water = 1 500 m s–1)

Penyelesaian / Solution
s=v×t
ng
2D = 1 500 × 1.8
BAB

2D = 2700
la
D = 1 350 m

5
Pe

Tugasan 2
n

1. Seorang pelajar berdiri pada jarak 60 m dari dinding. Dia memberikan tepukan
yang kuat dan gema akan kedengaran selepas 0.4 s. Hitung kelajuan gelombang
ita

bunyi di udara.
A student is standing at a distance of 60 m from a wall. He gives a loud clap and the echo is
heard after 0.4 s. Calculate the speed of sound wave in air.
rb

s
v=
t
ne

v = × 60)
(2

0.4
v = 300 m s–1
Pe

2. Ahmad menjerit kuat di hadapan dinding konkrit. Selepas 0.8 s, dia mendengar
gema suaranya. Jika laju bunyi di udara ialah 300 m s–1, cari jarak di antara Ahmad
dan dinding itu.
Ahmad shouts loudly in front of a concrete wall. After 0.8 s, he heard the echo of his voice. If the
speed of sound in air is 300 m s–1, find the distance between Ahmad and the wall.
s=v×t
2X = 300 × 0.8
x
2X = 240
X = 120 m

146
Fizik  Tingkatan 4  Bab 5 Gelombang 

3. Seorang pelajar berdiri di antara dua dinding tinggi pada jarak 25 m dan
75 m seperti dalam rajah di bawah. Dia memberikan jeritan yang kuat.
Kirakan selang masa antara dua gema yang dia dengar jika kelajuan bunyi
di udara adalah 300 m s–1.
A student is standing between two high walls at a distance of 25 m and 75 m as shown
in diagram below. He gives a loud scream. Calculate the time interval between the two
echoes he hears if the speed of sound in air is 300 m s–1.
s
t1 =
v
t1 = (2 × 25)

.
300

hd
t1 = 0.1667 s
s
t2 =

.B
v
t2 = (2 × 50)
300

dn
t2 = 0.3333 s

Selang masa / Time interval = 0.333 – 0.1667 = 0.1663 s

iS
4. Suatu denyut ultrasonik dipancarkan oleh alat sonar 240 m dari dasar laut. Pemancar dan penerima
Laju ultrasonik ialah 1 600 m s–1. Transmitter and receiver
An ultrasonic pulse transmitted by a sonar device 240 m above the seabed. The speed
of ultrasound in water is 1 600 m s–1.
ng

BAB
Berapakah tempoh masa yang diambil denyut itu untuk dipantulkan
la
semula ke alat?
What is the time taken for the pulse to be reflected back to the device? 240 m
5
Pe

s
t1 =
v
t1 = (2 × 240)
1 600
n

t1 = 0.3 s
ita

Pembiasan Gelombang
5.4
rb

Refraction of Waves

Pembiasan Gelombang
ne

Refraction of Wave

1. Pembiasan gelombang berlaku apabila arah perambatan gelombang berubah disebabkan oleh perubahan
Pe

laju gelombang tersebut semasa merambat dari suatu medium ke medium yang berbeza.
Wave refraction occurs when the direction of wave propagation changes due to changes in the wave velocity while
propagating across different medium.

2. Pembiasan gelombang air berlaku apabila gelombang air merambat dari satu kawasan ke kawasan yang
mempunyai kedalaman yang berbeza .
Refraction of water wave occurs when water wave propagate through areas of different depths.

147
  Fizik  Tingkatan 4  Bab 5 Gelombang

Halaju di kawasan dalam > Halaju di kawasan cetek λDalam > λCetek
Velocity in deep region > Velocity in shallow region λDeep > λShallow
d s

Kawasan cetek / Shallow region


Kawasan dalam Plat kaca
Deep region Glass plate

3. (a) Apabila gelombang air merambat dari kawasan dalam ke kawasan cetek, gelombang dibiaskan

.
hd
ke arah normal.
When a water wave propagates from deep region into shallow region, the wave is refracted towards the
normal.

.B
(b) Apabila gelombang air merambat dari kawasan cetek ke kawasan dalam, gelombang dibiaskan
menjauhi normal.

dn
When the water wave propagates from shallow region into deep region, the wave is refracted away from
the normal.

iS
Kawasan cetek
Shallow region
i
ng
Kawasan Kawasan
dalam cetek
BAB

Deep r Shallow
Kawasan dalam
la
area area
Deep region

5
Pe

(a) (b)
4. Jadual menunjukkan perbandingan gelombang asal dan gelombang yang dibiaskan.
The table shows the comparison of the original wave and the refracted wave.
n
ita

Dari kawasan air dalam Dari kawasan air cetek


Ciri gelombang ke kawasan air cetek ke kawasan air dalam
Characteristic of wave From deep water region to From shallow water region to
rb

shallow water region deep water region

Frekuensi, f Tidak berubah Tidak berubah


ne

Frequency, f No changes No changes

berkurang bertambah .
Pe

Laju gelombang, v v . v
Wave speed, v decreasing increasing
v . v .

λ berkurang bertambah .
Panjang gelombang, λ . λ
Wavelength, λ decreasing increasing
λ . λ .

Dibias mendekati normal. Dibias menjauhi normal.


Arah perambatan
Direction of propagation towards away
Refracted normal. Refracted from normal.

148
Fizik  Tingkatan 4  Bab 5 Gelombang 

Melukis Gambar Rajah untuk Menunjukkan Pembiasan Gelombang bagi Dua Kedalaman yang Berbeza
Drawing Diagrams to Show the Refraction of Waves for Two Different Depths

1. Jadual di bawah menunjukkan gelombang air satah yang merambat ke kedalaman berbeza. Lakarkan
gelombang air yang dibiaskan dalam ruang yang disediakan.
The table below shows plane water wave propagates into different depth. Sketch the refracted water waves in the space
provided.

Gelombang air satah / Plane water wave Gelombang air yang dibiaskan / Refracted water wave

.
hd
.B
Kawasan dalam Kawasan cetek Kawasan dalam Kawasan dalam Kawasan cetek Kawasan dalam
Deep region Shallow region Deep region Deep region Shallow region Deep region

dn
Garis Normal / Normal line

iS
ng

BAB
la
Kawasan dalam Kawasan cetek
Kawasan dalam Kawasan cetek
5
Pe

Deep region Shallow region


Deep region Shallow region
n
ita
rb
ne

Kawasan cetek Kawasan dalam Kawasan cetek Kawasan dalam


Shallow region Deep region Shallow region Deep region
Pe

Kawasan dalam Kawasan cetek Kawasan dalam Kawasan dalam Kawasan cetek Kawasan dalam
Deep region Shallow region Deep region Deep region Shallow region Deep region

149
  Fizik  Tingkatan 4  Bab 5 Gelombang

Kawasan dalam Kawasan cetek Kawasan dalam Kawasan dalam Kawasan cetek Kawasan dalam
Deep region Shallow region Deep region Deep region Shallow region Deep region

.
hd
Menjelaskan Fenomena Semula Jadi Akibat Pembiasan Gelombang dalam Kehidupan Harian
Explaining Natural Phenomena of Refraction of Waves in Daily Life

.B
1. Kesan pembiasan menyebabkan muka gelombang di laut mengikuti Teluk
bentuk pantai apabila gelombang air mendekati pantai. Bay

The effect of refraction


causes the wavefronts in the sea follow the shape of the

dn
coastline as the water wave approaches the coastline. Tanjung
Cape
• Di tengah-tengah lautan, muka gelombang hampir lurus

iS
antara satu sama lain. Hal ini kerana kedalaman laut adalah lebih kurang
Teluk
sama dan gelombang air bergerak pada kelajuan seragam . Bay
straight
In the centre of the ocean, the wavefronts are almost
ng to each other.
This is because the depth of the sea is almost the same and the water waves travel at uniform speed.
BAB

cetek
la
• Apabila gelombang air mendekati pantai, air menjadi lebih . Kelajuan gelombang
berkurang dan pembiasan berlaku. Gelombang dibiaskan dan mengikut bentuk pantai.
5
Pe

When the water waves approach the beach, the water becomes shallower . The wave speed desreases and
refraction occurs. The waves are refracted and follow the shape of the coastline.

Udara lebih sejuk-laju rendah


Udara lebih panas-laju tinggi
n

Air is cooler-speed is lower


Air is hotter-speed is higher
ita

Gelombang bunyi terbias


rb

menjauhi rumah pangsa Gelombang bunyi terbias


Sound waves are refracted mendekati rumah pangsa
away from the flats Sound waves are refracted
ne

towards the flats


Udara lebih
panas-laju tinggi
Air is hotter-speed Udara lebih sejuk-laju rendah
Pe

is higher Air is cooler-speed is lower



(a) (b)
2. (a) Pada waktu siang, bunyi dari kenderaan tidak jelas kedengaran di rumah pangsa. Pada waktu siang, udara
berhampiran jalan raya lebih panas dan kurang tumpat. Hal ini menyebabkan gelombang bunyi
dibias menjauhi rumah pangsa. Penduduk rumah pangsa mendengar bunyi yang lebih perlahan .
During the day, sound from the vehicle is not very loud in the flats. During the day, the air near the road is hotter and
less dense. This causes the sound waves refracted away from the flats. Residents of flats heard
a softer sound.

150
Fizik  Tingkatan 4  Bab 5 Gelombang 

(b) Pada waktu malam, bunyi dari kenderaan sangat jelas kedengaran di rumah pangsa. Pada waktu malam,
udara berhampiran tanah lebih sejuk dan lebih tumpat. Gelombang bunyi dibias mendekati
rumah pangsa. Penduduk rumah pangsa mendengar bunyi yang lebih kuat .
At night, the sound of the vehicle is very clear at night. The air near the ground is cooler and more dense.
Sound waves are being refracted towards the flats. Residents of the flats will hear louder sounds.

Menyelesaikan Masalah Melibatkan Pembiasan Gelombang


Solving Problems Involving Diffraction of Waves

.
hd
Contoh 6
Rajah menunjukkan sebuah tangki riak yang mempunyai Penggetar satah

.B
Plane vibrator
kedalaman dasar yang berbeza. Jika sebatang penggetar
satah bergetar di atas permukaan air, lukiskan corak
gelombang terbentuk.

dn
The diagram shows a ripple tank which has a different base
depth. If a plane vibrator vibrates on the water surface, draw a
wave pattern that is formed. Tangki riak
Ripple tank

iS
Penyelesaian / Solution
ng

BAB
la

5
Pe

Contoh 7
n
ita

Dalam rajah, PQRS ialah kawasan air cetek. Gelombang satah dengan kelajuan 3 cm 1 cm Q
P
4 cm s–1 merambat daripada kawasan dalam ke kawasan cetek.
In the diagram, PQRS is a region of shallow water. Plane waves with a velocity of 4 cm
rb

s–1 propagate from a deep area to a shallow area.

Berapakah frekuensi dan halaju kawasan PQRS?


ne

What is the frequency and velocity in the region PQRS?

Penyelesaian / Solution R S
Pe

Jika / If, v = f λ


Di kawasan yang dalam / At the deeper area, 4 = f (3)
f = 1.33 Hz
Frekuensi kawasan dalam = frekuensi kawasan cetek = 1.33 Hz
Frequncy of deep area = frequency of shallow area
Halaju kawasan cetek / Velocity of shallow area, v = f λ = 1.33 (1) = 1.33 cm s–1

151
  Fizik  Tingkatan 4  Bab 5 Gelombang

Tugasan 3
1. Suatu penggetar yang bergetar dengan frekuensi x Hz menghasilkan gelombang air satah yang merambat dengan
halaju v cm s–1. Apabila gelombang air itu memasuki kawasan dalam, panjang gelombangnya menjadi 3.2 cm dan
halaju menjadi 2v cm s–1. Tentukan panjang gelombang di kawasan cetek.
A vibrator vibrated with frequency x Hz and produced plane water wave that propagated at velocity of v cm s–1. When the water wave
moved to deep region, its wavelength and velocity are 3.2 cm and 2v cm s–1 respectively. Determine the wavelength at shallow region.
Frekuensi kawasan dalam = frekuensi kawasan cetek
Frequncy of deep area = frequency of shallow area

.
v
v = f λ, f =

hd
λ
Maka, Then fdalam = fcetek

( ) ( )

.B
v v
  =  
λ dalam λ cetek
( )
v
  = ( )
2v
λ dalam 3.2 cetek

dn
λ = 1.6 cm

2. Rajah di bawah menunjukkan gelombang satah air dalam sebuah tangki riak yang bergerak menuju ke arah sekeping

iS
perspeks yang diletakkan di dalam air itu. Pada rajah itu, lukiskan muka gelombang di atas kepingan perspeks dan
muka gelombang selepas melalui kepingan perspeks.
The diagram below shows plane waves in a ripple tank water moving towards a piece of perspex placed in the water. On the diagram, draw
ng
wavefronts on top of perspex wave and the wavefronts after passing through perspex.
BAB

(a) (b)
la

5
Pe
n

3. Gelombang bergerak dari kawasan cetek ke kawasan yang dalam dengan frekuensi 10 Hz. Nisbah panjang gelombang
ita

di kawasan cetek dan kawasan dalam ialah 4:5. Kelajuan gelombang di kawasan cetek adalah 8 cm s–1.
A wave travels from a shallow region to a deep region with a frequency of 10 Hz. The ratio of the wavelength in the shallow region and the
rb

deep region is 4:5. The wave speed at the shallow region is 8 cm s–1.
Kirakan / Calculate
(i) panjang gelombang ombak di kawasan cetek. / the wavelength of the waves in the shallow region.
ne

(ii) kelajuan gelombang di kawasan dalam. / the wave speed in the deep region.
(i) Frekuensi kawasan dalam = frekuensi kawasan cetek / Frequency of deep area = frequency of shallow area
Pe

v = f λ, 8 = (10)λ
λcetek = 0.8 cm

(ii) Kelajuan gelombang di kawasan dalam / the wave speed in the deep area,
v = f λ
5
λcetek =  (0.8) = 0.1 cm
4
v = 10(0.1) = 1.0 cm s–1

152
Fizik  Tingkatan 4  Bab 5 Gelombang 

Pembelauan Gelombang
5.5 Diffraction of Waves

Pembelauan Gelombang
Diffraction of Waves

1. Pembelauan gelombang adalah fenomena yang berlaku apabila gelombang merambat melalui celah

atau tepi suatu halangan .


Diffraction of waves is a phenomenon occurs when the waves pass propagate through slit or by the edge of an
obstacle

.
.

hd
2. Kesan pembelauan lebih jelas hanya jika / The effect of diffraction is more obvious only if
(a) saiz celah atau halangan adalah kecil ,

.B
the size of the slit or obstacle is small ,
λ λ

dn
a a

iS

(b) panjang gelombang yang lebih panjang


ng .
longer

BAB
the wavelength of the waves.
la
λ λ

5
Pe

a a

3 Jadual menunjukkan perbandingan gelombang asal dan gelombang yang dibelaukan.


ita

The table shows the comparison of the original wave and the diffracted wave.

Gelombang asal Gelombang yang dibelaukan


rb

Original wave Diffracted wave

Frekuensi, f Tidak berubah = f


ne

Frequency, f No changes = f

Laju, v Tidak berubah = v


Speed, v No changes = v
Pe

Panjang gelombang, λ Tidak berubah = λ


Wavelength, λ No changes = λ

Arah Berubah dari segi corak dan arah .


Direction directions
Changed in terms of patterns and .

Amplitud Berkurang kerana tenaga gelombang disebar meliputi kawasan yang Iebih luas.
Amplitude Decreases because the wave energy is distributed over a wider area.

153
  Fizik  Tingkatan 4  Bab 5 Gelombang

Faktor-faktor yang Mempengaruhi Pembelauan Gelombang


The Factors Affecting Diffraction of Waves

1. (a) Apabila gelombang satah melalui suatu celah yang besar, gelombang akan mengalami
pembelauan dan arah gelombang berubah.
When plane waves pass through a big slit, the waves experienced diffraction and changes
in direction of propagation.

.
(b) Jika celah itu lebih kecil, lebih besar sedikit dari saiz satu panjang gelombang,

hd
gelombang itu akan mangalami pembelauan yang lebih jelas.
If the slit is smaller, slightly bigger than one wavelength width, the waves experienced more

.B
obvious diffraction .

dn
(c) Jika celah itu sangat kecil , iaitu lebih kurang sama dengan lebar satu
panjang gelombang, gelombang itu akan mengalami pembelauan yang sangat jelas
dan menjadi bulatan.

iS
If the slit is very small , about one wavelength width, the waves experienced very
obvious diffraction that they become circular. ng
2 (a) Apabila gelombang satah merambat melalui tepi suatu penghalang, gelombang dibengkokkan di
sekeliling.
BAB

bent
la
When plane waves move pass the edge of an obstacle, they around the edge.

5
Pe

Penghalang
n

Obstacle
ita


rb

(b) Gelombang akan lebih membengkok jika panjang gelombang bertambah .


There is more bending if the wavelength is increased .
ne
Pe

Penghalang
Obstacle


(c) Jika saiz penghalang adalah besar berbanding dengan panjang gelombang, gelombang merambat
mengelilingi penghalang dan membengkok dengan sudut yang lebih kecil .
If the size of the obstacle is bigger compared with the wavelength, the wave passes around the obstacle and bends at a
smaller angle.

154
Fizik  Tingkatan 4  Bab 5 Gelombang 

(d) Jika saiz penghalang adalah kecil berbanding dengan panjang gelombang, gelombang merambat
mengelilingi penghalang dan membengkok dengan sudut yang lebih besar .
If the size of the obstacle is small compared with the wavelength, the wave passes around the obstacle and bends at a
bigger angle.

λ λ λ λ

.
hd

.B
3. (a) Cahaya mengalami pembelauan apabila melalui celah sempit yang hampir sama dengan panjang
gelombang cahaya. Walau bagaimanapun, kesannya tidak jelas jika saiz celah meningkat. Hal ini kerana
pendek

dn
panjang gelombang cahaya adalah sangat .
The light experiences diffraction when passing through narrow slit of approximately wavelength of light. However,
the effect is not clear if the size of the gap increases. This is because the wavelength of light is very short .

iS
ng

BAB
Pemerhati
la
Observer

Cahaya monokromatik Celah sempit


5
Pe

Monochromatic light Narrow slit

(b) Apabila cahaya monokromatik melalui suatu celah sempit, satu jalur cerah yang lebih lebar daripada
celah itu terbentuk pada skrin. Terdapat penyebaran cahaya, iaitu pembelauan telah berlaku.
n

When the monochromatic light passes through a narrow slit, a bright band of light that is wider than the slit is formed
diffraction
ita

on the screen. There is spreading of light, thus of light waves occur.

Saiz celah
Size of the slit
rb
ne
Pe

Jalur cerah Saiz celah


Bright band Size of the slit

(c) Apabila cahaya monkromatik lalu menerusi suatu celah lebar, satu jalur
cerah merah pada skrin mempunyai lebar yang sama seperti celah itu.
Cahaya tidak disebar, iaitu pembelauan cahaya tidak berlaku.
When the monochromatic light passes through a wide slit, the bright band on
the screen had the same width as the slit. There is no spreading of light, thus
Jalur cerah
diffraction of light waves does not occur. Bright band

155
  Fizik  Tingkatan 4  Bab 5 Gelombang

Gambar Rajah Corak Pembelauan Gelombang Air dan Kesan Pembelauan Cahaya
The Patterns of Diffraction of Water Waves and the Effect of Diffraction of Light Waves

Jadual di bawah menunjukkan gelombang satah yang ditujukan ke suatu bukaan atau halangan. Lakarkan
gelombang yang dibelaukan dalam ruang yang disediakan.
The table below shows plane wave directed to a gap or obstacle. Sketch the diffracted waves in the space provided.

Panjang gelombang tetap / Fixed wavelength Jawapan / Answer:


Celah besar / Large slit

.
hd
.B
dn
Panjang gelombang tetap / Fixed wavelength Jawapan / Answer:
Celah kecil / Small slit

iS
ng
BAB

la

5
Pe

Panjang gelombang tetap / Fixed wavelength Jawapan / Answer:


Penghalang besar / Large obstacle
n
ita
rb
ne

Panjang gelombang tetap / Fixed wavelength Jawapan / Answer:


Penghalang kecil / Small obstacle
Pe

156
Fizik  Tingkatan 4  Bab 5 Gelombang 

Bukaan tetap-panjang gelombang kecil Jawapan / Answer:


Fixed size of slit-short wavelength

.
hd
Bukaan tetap- panjang gelombang besar Jawapan / Answer:
Fixed size of slit-longer wavelength

.B
dn
iS
Aplikasi Pembelauan Gelombang dalam Kehidupan Harian ng
Application of Diffraction of Waves in Daily Life

BAB
1. Gelombang radio dengan panjang gelombang yang panjang dapat dibelaukan dengan mudah apabila
la
melalui kawasan berbukit. Rumah-rumah di belakang bukit dapat menerima siaran dengan kuat akibat
pembelauan . 3
5
Pe

Radio waves with longer wavelength can be diffracted easily when the waves propagate through the hilly terrain.
The houses behind the hills can receive stronger signals because of diffraction effect.
n
ita
rb
ne
Pe


Gelombang radio dengan panjang gelombang yang lebih pendek dibelaukan pada sudut yang lebih
kecil apabila melalui kawasan berbukit. Rumah-rumah di belakang bukit tidak dapat menerima
siaran dengan kuat akibat pembelauan pada sudut yang kecil.
Radio waves with shorter wavelength will be diffracted at smaller angle when the waves propagate through
the hilly terrain. The houses behind the hills won’t be able to receive strong signal because of smaller angle of
diffraction.

157
  Fizik  Tingkatan 4  Bab 5 Gelombang

.
hd
2 Dinding laut di Pelabuhan Jeti / Sea-walls in a Harbour Jetty
Dinding konkrit yang kuat dibina berhampiran dengan pelabuhan melindungi kapal daripada rosak yang
disebabkan oleh gelombang kuat. 3

.B
Strong concrete wall is built close to the harbour to protect the ships from ruined caused by strong waves.

dn
Dinding konkrit
Concrete wall

iS
Laut
Sea
ng
Teluk
BAB

Bay
la

Jeti
5
Pe

Jetty
Ship
Kapal
n

Waves Tanjung
Cape
ita

Ombak

Dalam rajah di atas, gelombang yang berlanggar dengan dinding konkrit akan dipantulkan dan dibelaukan
rb

apabila melalui celah. Tenaga gelombang itu akan tersebar meliputi kawasan yang lebih luas di
belakang dinding konkrit itu. Oleh itu, air di kawasan jeti lebih tenang dan sesuai untuk meletak bot.
ne

In the diagram above, the waves colliding with concrete walls will be reflected and diffracted when passing through
slits. The wave energy will be spread to cover the wider area behind the concrete walls. Thus, the water in the
Pe

jetty area is calmer and is ideal for parked boats.

158
Fizik  Tingkatan 4  Bab 5 Gelombang 

Interferens Gelombang
5.6 Interference of Waves

Menghuraikan Prinsip Superposisi Gelombang


Principle of Superposition of Waves

1. Interferens ialah fenomena gelombang yang berlaku apabila dua atau lebih gelombang koheren saling
bertindih(superposisi) antara satu dengan yang lain.
Interference is a wave phenomenon that occurs when two or more coherent waves collide (superposition) between
one and the other.

.
hd
Sumber-sumber gelombang yang koheren ialah gelombang yang dihasilkan pada perbezaan fasa

yang tetap dan frekuensi yang sama.

.B
Coherent sources of waves are waves produces at a constant phase difference and same frequency .

2. Terdapat dua jenis interferens iaitu interferens membina dan interferens memusnah .

dn
There are two types of interference that are constructive interference and destructive interference.

(a) Interferens membina berlaku apabila puncak bertembung puncak lain atau

iS
lembangan bertembung lembangan .
Constructive interference occurs when a crest meets another crest or a trough meets
trough
another .
ng
(b) Interferens memusnah berlaku apabila puncak bertembung dengan lembangan .

BAB
crest trough
la
Destructive interference occurs when a meets a .

3. Jadual di bawah menunjukkan perbezaan antara inteferens membina dan interferens memusnah.
5
Pe

The table below shows the difference between constructive inteferens and destructive interferences.

Jenis interferens Sebelum superposisi Semasa superposisi Selepas superposisi


Type of interference Before superposition During superposition After superposition
n

(a) Interferens
Sesaran paduan = 2A
ita

membina apabila
dua puncak Resultant displacement = 2A
bersuperposisi X Y Y X
rb

Constructive
2A
interference when
two crests are in
ne

superposition

(b) Interferens
membina apabila Sesaran paduan = –2A
Pe

dua lembangan Resultant displacement = –2A


bersuperposisi X Y Y X
Constructive 2A
interference when
two troughs are in
superposition

(c) Interferens Sesaran paduan = 0


memusnah Y Resultant displacement = 0 Y
Destructive X X
interference

159
  Fizik  Tingkatan 4  Bab 5 Gelombang

Kesan superposisi menghasilkan interferens gelombang


Principle of Superposition of Waves

1. Penghasilan gelombang air yang koheren boleh dihasilkan dengan meletakkan dua pencelup yang dipasang
pada satu motor.
Coherent water waves can be produced by placing two dippers that are mounted on one motor.

Bekalan elektrik Bar kayu


Electrical supply Wooden bar
Motor
Motor Pencelup
Dipper

.
hd
S2

.B
S1

dn
2. Corak interferens dapat diperhati seperti dalam rajah.
The interference pattern can observed as in diagram.
A
N

iS
Garis antinod ialah garis yang menyambungkan titik-
titik interferens membina . (Garisan A)
X A
The antinod line is a line that connects the dots of
S1
constructing interferences. (Line A)
ng N
A = Garisan antinod
Y A = Antinode line
A
BAB

Garis nod ialah garis yang menyambungkan titik-titik N = Garisan nod


N
la
S2 N = Nod line
Z
interferns memusnah . (Garisan N) A
destructive
5
Pe

The node line is the line that connects dots of


interferences. (Line N) N
A
Pada titik Y / At point Y:
membina
n

Pada titik Y, puncak S1 bertemu dengan puncak S2, oleh itu interferens berlaku. Amplitud
2A
ita

yang dihasilkan adalah .


At point Y, the crest of S1 meets with the crest of S2, thus constructive interference occur. The resultant amplitude is
2A .
rb

Pada titik X / At point X:


ne

Pada titik X, lembangan S1 bertemu dengan lembangsan S2, oleh itu interferens membina berlaku.
Amplitud yang dihasilkan adalah –2A .
Pe

At point X, the trough of S1 meets with the trough of S2, thus constructive interference occur. The resultant amplitude
is –2A .

Pada titik Z / At point Z:


Pada titik Z, puncak S1 bertemu dengan lembangan S2, interferens memusnah berlaku. Amplitud yang
dihasilkan adalah 0 , dengan itu menghasilkan air yang tenang.
At point Z, the crest of S1 meets with the trough of S2, destructive interference occurs. The resultant amplitude is
0 , thus producing calm water.

160
Fizik  Tingkatan 4  Bab 5 Gelombang 

Melukis Corak Interferens Gelombang


Drawing Patterns of Interference of Waves

Lukiskan corak interferens gelombang air untuk dua sumber


koheren S1 dan S2.
Draw a water wave interference pattern for two coherent S1 and
S2 sources.
S2
S2

.
hd
S1 S1

.B
dn
Lukiskan corak interferens gelombang cahaya yang terbentuk
pada skrin.

iS
Draw a light wave interference pattern that is formed on the screen.

ng

BAB
Sumber cahaya
la
monokromatik
Monochromatic
light source Pinggir gelap Pinggir cerah
5
Pe

Dark fringe Bright fringe


Dwicelah
Double slit

Tuliskan huruf K dan L untuk bunyi kuat (K) dan lemah


n

(L) yang dihasilkan oleh interferens gelombang bunyi dua


ita

sumber koheren S1 dan S2.


Write the letters K and L for strong (K) and weak (L) sound
generated by sound waves interferences of two coherent S1 and K
rb

S2 sources. L

K
ne

S1
L

K
Pe

S2

161
  Fizik  Tingkatan 4  Bab 5 Gelombang

Menghubung kait λ, a, x dan D berdasarkan Corak Intereferens Gelombang


Relating λ, a, x and D Based on Pattern of Wave Interference

Jika corak interferens diperhatikan seperti di bawah, panjang gelombang diberikan oleh formula:
If the interference pattern is observed as below, the wavelength of the wave is given by the following formula:

λ = ax
D
Garis antinod Garis nod
Antinodal line Nodal line
x

.
a = Jarak pemisahan di antara dua sumber koheren

hd
Distance between the two coherent sources

λ = Panjang gelombang
Wavelength

.B
D
λ
x = Jarak pemisahan di antara dua garis nod atau antinod yang
bersebelahan

dn
Distance between two successive nodal or antinodal lines.

D = Jarak dari dua punca gelombang koheren ke x diukur


S1 a S2 Distance from the two coherent sources of waves to where x is measured

iS
Muka gelombang Muka gelombang
lembangan puncak
Through wavefront Crest wavefront ng
Formula x boleh digunakan untuk meramalkan perubahan corak interferens. Apabila D atau a dimanipulasi, x
berubah.
BAB

The formula x can be used to predict changes in an interference pattern. When D or a is manipulated, x changes.
la

Jenis
5
Pe

Gelombang air Gelombang cahaya Gelombang bunyi


gelombang Water wave Light wave Sound wave
Type of waves

Celah tunggal
Single slit D
n

Pinggir K
Dwicelah gelap
Garis nod Double slit L
ita

Nodal line Dark


S1 x fringe
K
a x a a
ax a
λ= Pinggir x L
rb

D S2 Garis nod Cahaya cerah


Nodal line putih Bright
White
K
fringe
light
ne

Pembesar suara L
D D Loudspeaker
Pe

x K
L
Kesan x S1 Garis nod Garis nod K
Nodal line Nodal line x L
apabila a S1
K
dibesarkan a a x x a L
a
Effect on x K
S2 Garis nod Garis nod
when a is S2 L
Nodal line Nodal line
increased K
L
D K
D

162
Fizik  Tingkatan 4  Bab 5 Gelombang 

Garis nod
Nodal line Garis nod x
Nodal line K
Kesan x
S1 L
apabila a S1 a
dikecilkan a x a K
a x a
Effect on x S2 x L
S2
when a is K
decreased L

.
D

hd
Jarak pemisahan di antara dua sumber, a berkadar songsang dengan jarak pemisahan di antara dua garis nod atau antinod, x.
The distance between the two sources is inversely proportional to the distance between two successive nodal or antinodal line.

.B
Menyelesaikan masalah yang melibatkan interferens gelombang
Solving Problems Involving Interference of Waves

dn
Contoh 8

iS
Dalam interferens dua punca gelombang koheren, jarak pemisahan antara dua pencelup sfera ialah 4 cm dan
jarak antara dua garis nod yang bersebelahan ialah 5 cm apabila diukur pada jarak 20 cm dari punca gelombang
ng
koheren. Hitungkan panjang gelombang air bagi punca gelombang itu.
In the interference of two coherent wave sources, the separation distance between two spherical dippers is 4 cm and

BAB
the distance between two adjacent nodes is 5 cm when measured at a distance of 20 cm from the coherent wave
source. Calculate the wavelength of the water for the cause of the wave.
la
Penyelesaian / Solution
5
Pe

Diberi / Given a = 4 cm, x = 5 cm dan / and D = 20 cm


ax
Gunakan / Use: λ =
D
n

λ = 4(5) = 1.0 cm
20
ita

Contoh 9
rb

Cahaya monokromatik dilalukan menerusi dua celah yang terpisah sejauh 4.0 × 10– 4 m. Pinggir-pinggir
ne

interferens terbentuk pada skrin yang terletak 1 m. Jika jarak di antara dua pinggir terang yang bersebelahan
ialah 2.0 × 10– 3 m, berapakah panjang gelombang cahaya monokromatik itu?
The monochromatic light is passed through two separate slits as far as 4.0 × 10 – 4 m. The interferences are formed on the screen
Pe

at 1 m. If the distance between two adjacent edges is 2.0 × 10 –3 m, what is the wavelength of the
monochromatic light?

Penyelesaian / Solution
Diberi / Given a = 4.0 × 10–4 m, x = 2 × 10–3 m dan / and D = 1 m
ax
Gunakan / Use: λ =
D
4.0 × 10– 4 × 2.0 × 10–3 = 8.0 × 10–7
λ= m
1

163
  Fizik  Tingkatan 4  Bab 5 Gelombang

Contoh 10
Dalam satu eksperimen untuk mengkaji interferens gelombang bunyi, dua pembesar suara disambung kepada
sebuah penjana isyarat audio yang mengeluarkan bunyi dengan frekuensi 1200 Hz. Seorang pelajar terdengar
bunyi kuat dan bunyi lemah berselang-seli di sepanjang satu garis lurus yang 20 m dari pembesar suara.
In an experiment to study the interference of sound waves, two speakers are connected to an audio signal generator that
produces sound with a frequency of 1200 Hz. A student heard a weak sound and weak sound alternating along a straight line
20 m from the speakers.
(a) Halaju bunyi dalam udara diberikan sebagai 300 m s–1. Tentukan panjang gelombang yang dikeluarkan oleh
kedua-dua pembesar suara itu.
The velocity of sound in air is given as 300 m s–1. Determine the wavelength issued by the two speakers.

.
hd
(b) Jika jarak di antara dua pembesar itu ialah 3.0 m, tentukan jarak terdekat di antara kawasan bunyi lemah
dengan kawasan bunyi lemah yang berturutan.
If the distance between the two speakers is 3.0 m, determine the shortest distance between the weak sound with a weak

.B
sound sequence.

Penyelesaian / Solution
(a) v = f λ

dn
300 = 1200 (λ)
λ = 0.25 m

iS
(b) Diberi / Given a = 3.0 m, λ =0.25 m dan / and D = 20 m
ax , maka / hence x = λD
Gunakan / Use: λ=
D a
ng
0.25 × 20 = 1.667 m
x=
BAB

3.0
la

5
Pe

Aplikasi Interferens Gelombang dalam Kehidupan Harian


Applications of Interference of Waves in Daily Life
n

1. Pembatalan Bunyi / Sound Cancelation 3


Pembatalan bunyi boleh digunakan ditempat seperti kabin kapal terbang atau di kawasan yang mengeluarkan
ita

bunyi bising dari sumber yang tetap.


Sound cancellation can be used in places such as aircraft cabins or in areas producing
noise from a fixed source. Punca bunyi
rb

Noise source
Mikrofon digunakan untuk mengesan bunyi di dalam kabin dan maklumat
dimasukkan ke dalam komputer kecil. Komputer ini akan menganalisis
ne

bunyi dan menghantar isyarat ke pembesar suara. Pembesar besar


Bunyi paduan
memancarkan gelombang bunyi dengan amplitud yang sama tetapi dengan Resulting noise
terbalik
Pe

fasa kepada bunyi asal. Gelombang bergabung untuk Anti bunyi


Anti noise
membentuk gelombang baru, dalam proses yang dipanggil interferens
memusnah dan membatalkan satu sama lain.
Microphone is used to detect sound in the cabin and information is inserted into the computer. The computer will analyze
the sound and send the signal to the speakers. Big speakers emit sound waves with the same amplitude but with the
reverse phase to the original sound. The waves combine to form a new wave, in a process called destructive
interference and canceling each other

164
Fizik  Tingkatan 4  Bab 5 Gelombang 

Gelombang Elektromagnet
5.7 Electromagnetic Wave

Ciri-ciri gelombang elektromagnet


Characteristics of Electromagnetic Waves

1. Gelombang elektromagnet ialah gelombang merambat dengan ayunan medan elektrik dan medan magnet
yang berserenjang kepada satu sama lain dan dengan arah perambatan.
Electromagnetic waves are propagating waves with electric field and magnetic field oscillate perpendicular to each other
and to the direction of propagation.

.
hd
Medan magnet, B
Magnetic field, B

.B
Medan elektrik, E
Electric field, E

dn
iS
Arah perambatan
Propagation direction
ng
2. Gelombang elektromagnet adalah gelombang melintang dan boleh bergerak melalui vakum
tanpa memerlukan medium untuk merambat.

BAB
transverse vaccum
la
The electromagnetic waves are waves and can travel through without the need of a
medium for propagation.
5
Pe

3. Gelombang elektromagnet bergerak melalui vakum pada kelajuan 3.0 × 10 m s


8 –1
tetapi bergerak pada
kelajuan yang lebih rendah dalam medium berbanding dengan di dalam vakum
Electromagnetic waves travel through a vacuum at a speed of 3.0 × 10 8 m s–1 but travel at a lower speed in
n

the medium than in vacuum.


ita

4. Semua gelombang elektromagnet boleh menunjukkan empat fenomena gelombang utama iaitu
pantulan , pembiasan , pembelauan dan interferens .
reflection refraction
rb

All electromagnetic waves can show the four main wave phenomena namely , ,
diffraction and interference .
ne

Spektrum Elektromagnet
Electromagnetic Spectrum
Pe

1. Terdapat tujuh jenis gelombang dalam spektrum elektromagnet iaitu gelombang radio, gelombang mikro ,
inframerah, cahaya nampak, ultra ungu , sinar-X dan sinar gama .
There are seven types of waves in the electromagnetic spectrum of radio waves, microwaves, infrared, visible light, ultra
violet, X-rays and gamma rays.

2. Spektrum elektromagnet terdiri daripada pelbagai frekuensi dan panjang gelombang di mana gelombang
elektromagnet disebarkan. Spektrum elektromagnet ditunjukkan di dalam rajah.
The electromagnetic spectrum consists of a different range of frequencies and wavelengths over which electromagnetic
waves are propagated. The electromagnetic spectrum is shown in the diagram.

165
  Fizik  Tingkatan 4  Bab 5 Gelombang

Panjang Gelombang radio Gelombang mikro Sinaran inframerah Cahaya nampak Sinaran ultraungu Sinar-X Sinar gama
Radio waves Microwaves Infrared rays Visible light Ultraviolet rays X-rays Gamma rays
gelombang
Wavelength
104 103 102 101 1 10–1 10–2 10–3 10–4 10–5 10–6 10–7 10–8 10– 9 10–10 10–11 10–12 10–13 10–14

Hampir
menyerupai
saiz...
About

.
the size
of...

hd
Bangunan Manusia Besbol Jarum peniti Protozoa Molekul Atom Nukleus atom
Frekuensi
Buildings Humans Baseball Pin head Protozoa Molecules Atoms Atomic nuclei
Frequency
f / Hz 105 106 107 108 109 1010 1011 1012 1013 1014 1015 1016 1017 1018 1019 1020 1021 1022 1023

.B
Aplikasi Setiap Komponen Spektrum Elektromagnet dalam Kehidupan
Applications of Each Component in the Electromagnetic Spectrum in Daily Life

dn
Jenis gelombang Sumber Aplikasi
Type of wave Source Application

iS
• Telekomunikasi , telefon bimbit.
Arial pemancar radio dan
Gelombang radio
televisyen
ng Telecommunications , mobile phones.
Radio waves
Arial radio and television transmitters • Penyiaran siaran radio dan televisyen.
BAB

Radio and television broadcast transmission.


la
• Komunikasi antarabangsa melalui penggunaan satelit
5
Pe

Ketuhar gelombang mikro dan International communications through satellite use


Gelombang mikro pemancar gelombang mikro • Pengesanan radar bagi pesawat
Microwave Microwave oven and microwave Radar detection of aircraft
transmitter • Masakan
n

Cooking
ita

Objek panas seperti Matahari, kawalan jauh


• Fisioterapi, penglihatan malam dan alat
Sinaran inframerah mentol elektrik dan api
Infrared rays Hot object like the Sun, electric
untuk TV
rb

Physiotherapy, night vision and remote controls for TV


bulbs and fire

• Membolehkan benda hidup untuk melihat


ne

Allows living things to see


• Fotografi
Matahari, mentol elektrik dan Photography
Pe

Cahaya nampak lampu pendarfluor


Visible light Sun, electric bulb and fluorescent • Fotosintesis dalam tumbuhan hijau
lamp
Photosynthesis in green plants
• Cahaya laser yang digunakan dalam sasaran pada senapang,
The laser light used in the target on the gun,

166
Fizik  Tingkatan 4  Bab 5 Gelombang 

• Pengenalpastian duit kertas palsu

Identification of counterfeit money


Objek sangat panas, Matahari,
Sinaran ultraungu
Ultraviolet rays
lampu merkuri • Penulenan air minuman
Very hot object, the Sun, lamps Purification of drinking water
• Alat perangkap serangga
Insects trap

• Pengesanan keretakan pada tulang


Detection of cracks in bone

.
hd
Sinar-X Tiub sinar-X • Radiografi (gambar sinar-X )
X-ray X-ray tube
Radiography ( X-ray
photograph)
• Mengesan sama ada hasil lukisan lama adalah tulen atau tidak

.B
Detect whether old painting is genuine or not

• Membunuh sel kanser dan pensterilan

dn
alat pembedahan
Sinaran gama Bahan radioisotop
Gamma rays Radioisotopes sunstance Kills cancer cells and surgical instrument sterilization

iS
• Pensterilan makanan supaya tahan lama
Sterilization of food so last longer
ng
SPM 5

BAB
PRAKTIS
la

5
Pe

Soalan Objektif

1. Rajah 1 menunjukkan seorang budak lelaki dapat 2. Rajah 2 menunjukkan suatu pola pinggir dalam
mendengar muzik dari radio walaupun berdiri di eksperimen dwicelah Young yang dihasilkan oleh
n

2018 2018
belakang dinding. sumber monokromatik dengan jarak 3.0 × 10–2 m
Diagram 1 shows a boy able to hear the music from the radio antara satu sama lain. Skrin diletakkan pada jarak
ita

even standing behind the wall. 5.0 m dari celah.


Diagram 2 shows a fringe pattern in a Young’s double slit
experiment produced by monchromatic source that are
rb

3.0 × 10–2 m apart. The screen is placed at a distance of 5.0 m


Dinding
Wall from the slit.
ne

4.0 × 10–4 m
Pe


Rajah 1 / Diagram 1
Fenomena yang menyebabkan budak itu dapat Pola pinggir
mendengar muzik dikenali sebagai Fringe pattern
The phenomenon that causes the boy able to hear music is Rajah 2 / Diagram 2
known as
A Pantulan / Reflection Berapakah panjang gelombang?
What is the wavelength?
B Pembiasan / Refraction
A 1.2 × 10–6 m C 1.2 × 10–7 m
C Pembelauan / Diffraction
B 4.8 × 10–6 m D 4.8 × 10–7 m
D Interferens / Interference

167
  Fizik  Tingkatan 4  Bab 5 Gelombang

3. Rajah 3 menunjukkan satu siri mampatan dan 5. Cahaya merah dengan panjang gelombang 7400 nm
regangan molekul udara bagi gelombang bunyi. merambat melalui vakum dengan laju 3.0 × 108m s–1.
2018 2017
Diagram 3 shows a series of compression and rarefaction of air Berapakah frekuensi cahaya biru itu?
molecules for sound waves. A blue light of wavelength 7400 nm propagates through a
Mampatan vaccum with a speed of 3.0 × 108 m s–1. What is the frequency
Compression of the blue light?
A 4.05 × 1016 Hz C 2.22 × 1012 Hz
B 4.05 × 1013 Hz D 2.22 × 1010 Hz

6. Kuantiti manakah yang tidak akan berubah apabila


Regangan gelombang air dibiaskan?

.
Rarefaction 2017
Which quantity will not change when water wave is refracted?

hd
Rajah 3 / Diagram 3 A Velocity / Halaju
Pernyataan manakah yang salah untuk ciri gelombang B Frequency / Frekuensi
bunyi? C Wavelength / Panjang gelombang

.B
Which is the wrong statement for the characteristic of sound D Direction of propagation / Arah perambatan
wave?
A Gelombang bunyi tidak boleh merambat melalui 7. Rajah 5 menunjukkan suatu tangki riak dengan plat

dn
vakum. perspeks dan penggetar lurus.
2017
Sound wave cannot propagate through a vacuum. Diagram 5 shows a ripple tank with a perspex plate and a
B Gelombang bunyi memindahkan tenaga semasa plane vibrator.

iS
perambatannya. Penggetar
Sound wave transfers energy during its propagation. Vibrator

C Zarah-zarah udara bergetar dalam arah yang


selari dengan arah perambatan gelombang bunyi
ng
Air particles vibrate in a direction parallel to the direction
BAB

propagation of sound wave.


la
D Panjang gelombang bunyi adalah sama dengan Plat perspek

jarak di antara mampatan dan regangan. Perspex plate
Rajah 5 / Diagram 5
5
Pe

Wavelength of the sound is the same as the distance


between a compression and rarefaction. Apabila penggetar itu dihidupkan, corak gelombang
manakah yang dilihat pada skrin?
4. Rajah 4 menunjukkan imej pengimbas keselamatan When the vibrator is turned on, which wave pattern is seen on
lapangan terbang. the screen?
n

2018
Diagram 4 shows the image of airport security scanner. A C
ita

B D
rb
ne

8. Rajah 6 menunjukkan kedudukan sebuah stesen


0.5 litre
radio dan suatu penerima.
2017
Diagram 6 shows the position of a radio station and a receiver.
Pe


Rajah 4 / Diagram 4
Gelombang elektromagnet manakah yang digunakan
dalam pengimbas keselamatan lapangan terbang?
Which electromagnetic wave is used in airport security scanner?
A Sinar-X C Sinaran gama
X-rays Gamma rays
B Inframerah D Sinaran ultraungu
Infrared Ultraviolet rays Rajah 6 / Diagram 6

168
Fizik  Tingkatan 4  Bab 5 Gelombang 

Gelombang radio yang manakah terbaik supaya 11. Gelombang elektromagnet manakah mempunyai
isyarat dari stesen radio dapat diterima oleh penerima? frekuensi paling tinggi dan panjang gelombang
2017
Which of the best radio waves so that the signal from the radio paling kecil?
station will be able to be received by the receiver? Which electromagnetic wave has the highest frequency and
A Panjang gelombang radio yang panjang the shortest wavelength?
Long wavelength radio waves A Gelombang radio
B Panjang gelombang radio yang pendek Radio wave
Short wavelength radio waves B Gelombang mikro
C Gelombang radio frekuensi tinggi Microwave
High-frequency radio waves C Sinaran gama
D Gelombang radio tenaga rendah Gamma ray

.
Low energy radio waves D Sinaran ultraungu

hd
Ultraviolet ray
9. Rajah 7 menunjukkan corak interferens terbentuk
dalam eksperimen dwicelah Young. 12. Rajah 9 menunjukkan gelombang air yang dihasilkan

.B
2017
(Diberi a = 4 × 10–3 m, D = 2.5 m, λ = 5.0 × 10–7 m) oleh suatu penggetar satah.
2016
Diagram 7 shows an interference pattern formed by the Diagram 9 shows water waves produced by a plane vibrator.
Young’s double slit experiment.
A C
(Given, a = 4 × 10–3 m, D = 2.5 m, λ = 5.0 × 10–7 m.)

dn

B D
Dwicelah Young Skrin
Young’s double slit Screen F

iS
G H
Celah tunggal
Single slit E
K
a L L
ng I
J

Sumber cahaya

BAB
hijau Rajah 9 / Diagram 9
la
Green light source D Titik-titik sambungan manakah adalah muka
Corak interferens gelombang?
5
Pe

Interference pattern
Which joining points is the wavefront?
Rajah 7 / Diagram 7 A ACD
B BEI
Berapakah nilai L?
What is the value of L? C CFK
n

A 3.125 × 10– 4 m C 3.125 × 10–3 m D IFD


ita

B 2.5 × 10–3 m D 2.5 × 10– 4 m 13. Rajah 10 menunjukkan bandul Barton. Bandul X
10. Rajah 8 menunjukkan kawasan regangan, R dan ditolak supaya berayun.
2016
rb

Diagram 10 shows a Barton’s pendulum. Pendulum X is pushed


mampatan, C bagi gelombang bunyi.
2017 to oscillate.
Diagram 8 shows a rarefaction, R and compression, C region of
a sound wave.
ne

Pembesar suara
Loudspeaker A B C D E F
Pe

A
Mampatan Regangan B
Compression Rarefaction
C X
Rajah 8 / Diagram 8 D

Panjang gelombang diwakili oleh Rajah 10 / Diagram 10
The wavelength is represented by
Pernyataan yang manakah benar tentang bandul
A AB C DE
yang lain?
B BF D DF Which of the statement is true about the other pendulum?

169
  Fizik  Tingkatan 4  Bab 5 Gelombang

A Frekuensi asli bandul D adalah sama dengan 15 Rajah 12 menunjukkan gelombang air merambat
frekuensi asli bandul X. dari kawasan dalam ke kawasan cetek apabila suis
2016
The natural frequency of pendulum D is equal to the penggetar dihidupkan.
natural frequency of pendulum X. Diagram 12 shows a water wave propagates from the deep
B Bandul C didapati berayun dengan amplitud area to the shallow area when the switch of vibrator is turned
maksimum. on.
Pendulum C is found to oscillate with maximum amplitude
C Pendulum X transfers force to all pendulum. Kawasan cetak
Penggetar Shallow area
Bandul X memindahkan dàya ke semua bandul. Vibrator
D Pendulum X transfers energy only to pendulum C.
Bandul X memindahkan tenaga hanya ke bandul C.

.
hd
14. Rajah 11 menunjukkan suatu susunan radas untuk Kawasan dalam Plat perspek
menyiasat pantulan gelombang bunyi. Deep area Perspex plate
2016
Diagram 11 shows an apparatus set-up to investigate the Rajah 12 / Diagram 12

.B
reflection of sound wave.
Ciri-ciri gelombang yang manakah dalam kawasan
X Hujung terbuka cetek bagi situasi di atas adalah yang betul?
Open end Which characteristics of wave in the shallow area for above

dn
Hujung terbuka Tiub kadbod situation is correct?
Open end Cardboard tube
i r
Kertas putih N Panjang
Frekuensi Halaju

iS
White paper
gelombang
Tiub kadbod Frequency Velocity
Cardboard tube
50

45

40
55

35
25

20
0
MIN

15
5

10
5

25
10

15

20
Wavelength
30

Hujung tertutup A Bertambah Berkurang Sama


Closed end
ng Increase Decrease Same
Jam randik Y
Stopwatch B Sama Berkurang Berkurang
BAB

Same Decrease Decrease


la
Rajah 11 / Diagram 11
Antara berikut, yang manakah betul untuk gelombang C Bertambah Sama Bertambah
5
Pe

tuju dan gelombang pantulan? Increase Same Increase


Which of the following is correct for the incident wave and the D Bertambah Bertambah Sama
reflected wave? Increase Increase Same

Sudut tuju (i) dan


n

16. Rajah 13 menunjukkan dinding konkrit yang kuat


Gelombang Gelombang sudut pantulan(r) dibina berhampiran dengan pelabuhan melindungi
ita

tuju pantulan Angle of incident 2016


kapal dari rosak yang disebabkan oleh gelombang
Incident wave Reflected wave and angle of
kuat.
reflection
Diagram 13 shows a strong concrete wall is built close to the
rb

harbour to protect the ship from ruined caused by strong


A Frekuensi kecil Frekuensi besar
i>r waves.
Small frequency Big frequency
ne

B Frekuensi tidak Frekuensi tidak Tanjung


Cape
berubah berubah
Pe

i=r
Frequency is Frequency is
unchanged unchanged
Taluk
C Frekuensi besar Frekuensi kecil
i<r
Bay

Big frequency Small frequency


Jeti
D Frekuensi kecil Frekuensi besar
i=r Jetty
Small frequency Big frequency
Rajah 13 / Diagram 13

170
Fizik  Tingkatan 4  Bab 5 Gelombang 

Apakah fenomena yang berlaku apabila gelombang A reflection of wave


air melalui dinding konkrit? pantulan gelotnbang
What is the phenomenon that occurs when the water wave B refraction of wave
passes through the concrete wall? pembiasan gelombang
A Pantulan C Pembiasan C diffraction of wave
Reflection Refraction pembelauan gelombang
B Pembelauan D Interferens D interference of waves
Diffraction Interference interferens gelombang

17. Rajah 14 menunjukkan gelombang di laut mengikuti 18. Rajah 15 menunjukkan suatu spektrum gelombang
bentuk pantai apabila gelombang air mendekati elektromagnet.

.
2016 2016
pantai. Diagram 15 shows an electromagnetic spectrum of wave.

hd
Diagram 14shows the waves in the sea follow the shape of the
coastline as the water wave approaches the coastline. Sinaran Ultra Cahaya
Sinaran Gelombang Gelombang
gama ungu nampak Inframerah
X Infrared mikro radio
Gamma Ultra Visible

.B
Laut / Sea X-Ray Microwave Radio wave
ray violet light

Rajah 15 / Diagram 15
Gelombang elektromagnet yang manakah digunakan

dn
untuk merawat kanser?
Which of the electromagnetic wave is used to treat cancer?
A Gelombang mikro

iS
Tanjung Teluk / Bay Tanjung Microwave
Headland Headland
Pantai / Beach B Inframerah
Infrared
Rajah 14 / Diagram 14
ng C Ultraungu
Corak gelombang air yang terhasil itu adalah Ultraviolet

BAB
disebabkan oleh D Sinaran gama
la
The wave pattern produced is due to Gamma ray

5
Pe

Soalan Struktur
n

Bahagian A
ita

1. Rajah 1 menunjukkan satu imej gelombang satah air apabila ia merambat dari kawasan air cetek ke kawasan air dalam.
Diagram 1 shows an image of a plane water waves as it propagates from shallow water region to deep water region.
2016
rb

Kawasan air cetak


Shallow water region
ne

X
Pe

Kawasan air dalam


Deep water region


Rajah 1 / Diagram 1

171
  Fizik  Tingkatan 4  Bab 5 Gelombang

(a) Namakan fenomenon gelombang yang ditunjukkan dalam Rajah 1.


Name the wave phenomenon shown in Diagram 1.
Pembiasan / Refraction
[1 markah / mark ]
(b) Apakah yang berlaku kepada laju gelombang di kawasan air dalam?
What happens to the speed of wave in the deep water region?
Berkurang / Decreases
[1 markah / mark ]

.
(c) Pada Rajah 1:

hd
On Diagram 1:
(i) Lukis satu garisan untuk menunjukkan garisan normal pada X.

.B
Draw a line to show the normal line at X.
[1 markah / mark]
(ii) Lukis satu garisan bagi menunjukkan arah perambatan gelombang air di kawasan air cetek dan di kawasan

dn
air dalam sebelum dan selepas melalui X.
Draw a line to show the direction of propagation of water wave in shallow water region and in deep water region before and
after passing through X.

iS
c(ii)
ng X
BAB

c(i)
la

5
Pe


[2 markah / marks]
n

2. Rajah 2.1(a) dan Rajah 2.2(a) menunjukkan titisan air jatuh ke permukaan air di dalam besen. Imej pegun untuk titisan
ita

air yang menitis dan kedua-dua pili diambil pada masa yang sama.
2017
Rajah 2.1(b) dan Rajah 2.2(b) menunjukkan muka gelombang membulat yang terhasil apabila titisan air mencecah
permukaan air dalam besen.
rb

Diagram 2.1(a) and Diagram 2.2(a) show the water droplets drop onto the water surface in a basin. The static images of the water droplets
dripping from both taps were taken at the same time.
ne

Diagram 2.1(b) and Diagram 2.2(b) show the circular wavefronts produced when the water droplets hit the water surface in the basin.
Pe

Besen / Basin      Besen / Basin


(a) (b) (a) (b)
Rajah 2.1 / Diagram 2.1 Rajah 2.2 / Diagram 2.2

172
Fizik  Tingkatan 4  Bab 5 Gelombang 

(a) Apakah yang dimaksudkan dengan muka gelombang?


What is meant by wavefront?
Muka gelombang adalah satu garisan yang menghubungkan semua titik yang mempunyai sama fasa.

A wave front is a line joining all the points which have the same phase.
[1 markah / mark]

(b) Perhatikan Rajah 2.1 dan 2.2.


Observe Diagram 2.1 and 2.2.
(i) Bandingkan bilangan titisan air yang menitis pada masa yang sama.

.
Compare the number of water droplets dripping at the same time.

hd
Bilangan titisan air dalam Rajah 2.1 adalah sama dengan bilangan titisan air dalam Rajah 2.2.

.B
The number of water droplets in Diagram 2.1 is the same as the number of water droplets in Diagram 2.2.
[1 markah / mark]

(ii) Bandingkan kedalaman air di dalam besen.

dn
Compare the depth of water in the basin.
Kedalaman air dalam Rajah 2.1 adalah lebih dalam dari kedalaman air dalam Rajah 2.2

iS
The depth of water in Diagram 2.1 is deeper than the water depth in Diagram. 2.2
ng [1 markah / mark]

(iii) Bandingkan panjang gelombang bagi gelombang membulat yang tersebar keluar di dalam besen.
Compare the wavelength of the circular wave which is spreading outwards in the basin.

BAB
la
Panjang gelombang bagi gelombang membulat dalam Rajah 2.1 adalah lebih besar daripada panjang

gelombang bagi gelombang membulat dalam Rajah 2.2. 5


Pe

The wavelength of the circular wave in Diagram 2.1 is bigger than the wavelength of the circular wave in Diagram 2.2.
[1 markah / mark]
n

(iv) Hubung kait kedalaman air di dalam besen dengan panjang gelombang bagi gelombang membulat.
ita

Relate the depth of the water in the basin to the wavelength of the circular wave.
Semakin dalam kedalaman air, semakin besar panjang gelombang bagi gelombang membulat.
rb

The deeper the depth of the water, the greater the wavelength of the circular wave.
[1 markah / mark]
ne

(c) Bilangan titsan air ke dalam besen pada Rajah 2.1(a) ditingkatkan.
The number of drops of water into the basin in Diagram 2.1 (a) is increased.
Pe

(i) Apakah yang berlaku kepada panjang gelombang bagi gelombang membulat tersebut?
What happens to the wave length of the circular wave?
Panjang gelombang bagi gelombang membulat dalam Rajah 2.1 adalah lebih kecil.

The wavelength of the circular waves in Diagram 2.1 is smaller.


[1 markah / mark]

173
  Fizik  Tingkatan 4  Bab 5 Gelombang

(ii) Beri satu sebab bagi jawapan di 2(c)(i).


Give one reason for the answer in 2(c)(i).
Halaju gelombang tidak berubah, apabila titsan air bertambah, frekuensi bertambah maka panjang

gelombang berkurang (v = f λ) / The velocity of the wave does not change, when water drops increase, the frequency
increases then the wavelength decreases (v = f λ)
[1 markah / mark]

3. Rajah 3.1 menunjukkan corak gelombang apabila gelombang air bergerak dari kawasan A ke kawasan B di dalam
tangki riak.

.
2015

hd
Diagram 3.1 shows the wave pattern when water wave moves from region A to region B in a ripple tank.

.B
dn
iS

Kawasan A / Region A  Kawasan B / Region B
Rajah 3.1 / Diagram 3.1
ng
Rajah 3.2 menunjukkan keratan rentas tangki riak itu.
BAB

Diagram 3.2 shows the cross-section of the ripple tank.


la

Kawasan A
5
Pe

Region A Kawasan B
Region B

Rajah 3.2 / Diagram 3.2


ita

Panjang gelombang bagi gelombang air itu berubah apabila ia bergerak dari kawasan A ke kawasan B.
The wavelength of the water wave changes as it moves from region A to region B.
(a) Apakah yang dimaksudkan dengan panjang gelombang?
rb

What is the meaning of wavelength?


Panjang gelombang ialah jarak di antara dua titik berturutan yang sama fasa dalam suatu gelombang.
ne

The wavelength is the distance between successive points of the same phase in a wave.
[1 markah / mark]
Pe

(b) Berdasarkan Rajah 3.1 dan Rajah 3.2,


Based on Diagram 3.1 and Diagram 3.2,
(i) bandingkan panjang gelombang antara kawasan A dengan kawasan B.
compare the wavelength between region A and region B.
Panjang gelombang di kawasan A lebih pendek berbanding panjang gelombang kawasan B.

The wavelength in region A is shorter than the wavelength in the region B.


[1 markah / mark]

174
Fizik  Tingkatan 4  Bab 5 Gelombang 

(ii) bandingkan kedalaman air antara kawasan A dengan kawasan B.


compare the depth of water between region A and region B.
Kedalaman dalam kawasan A lebih cetek daripada kawasan B.

Depth in region A is more shallow than region B.


[1 markah / mark]
(c) Berdasarkan jawapan anda dalam 3(b),
Based on answers in 3(b),
(i) nyatakan hubungan antara kedalaman air dengan panjang gelombang.

.
state the relationship between the depth of water and the wavelength.

hd
Lebih dalam kedalaman air, lebih besar panjang gelombang.

The deeper the water depth, the greater the wavelength.

.B
[1 markah / mark]
(ii) lengkapkan rajah di bawah dengan muka gelombang dalam kawasan B.
Complete the diagram below with the wavefront on region B.

dn
iS
ng

BAB
la

5
Pe

[2 markah / marks]
(d) Rajah menunjukkan bentuk pantai. Di manakah chalet sesuai dibina? Mengapa?
n

The diagram shows the shape of the beach. Where is the chalet suitable to be built? Why?
ita

Laut / Sea
rb
ne

Tanjung Teluk / Bay Tanjung


Headland Headland
Pe

Pantai / Beach

Di teluk . Gelombang air mengalami pembiasan yang terbias ke arah tanjung. Hal ini menyebabkan air kawasan

teluk lebih tenang dan sesuai untuk aktiviti rekreasi dan berenang. / In the bay. The water wave undergoes refraction and

refracts towards the cape. This causes the water of the bay area to be calmer and suitable for recreational and swimming activities.
[2 markah / marks]

175
  Fizik  Tingkatan 4  Bab 5 Gelombang

Bahagian B

4. Rajah 4.1 menunjukkan sebuah pembesar suara menghasilkan bunyi yang dapat didengar oleh telinga manusia.
Diagram 4.1 shows a speaker producing a sound that can be heard by a human ear.
2012

Diafragma
Diaphragm

.
Rajah 4.1 / Diagram 4.1

hd
(a) (i) Apakah jenis gelombang bagi gelombang bunyi?
What type of wave is the sound wave? [1 markah / mark]

.B
(ii) Terangkan bagaimana bunyi dihasilkan oleh pembesar suara itu dan dapat didengar oleh telinga manusia.
Explain how the sound is produced by the speaker and can be heard by the human’s ear. [4 markah / marks]
(b) Rajah 4.2 dan Rajah 4.3 menunjukkan gelombang bunyi dihasilkan oleh kereta yang jauh pada waktu malam dan

dn
waktu siang yang dapat didengar oleh penghuni pangsapuri.
Diagram 4.2 and Diagram 4.3 shows sound wave produced by a distant car at night and daytime that can be heard by the resident
of apartment.

iS
Udara sejuk
ng
Cool air
BAB

la

Udara panas
5
Pe

Hot air

Rajah 4.2 / Diagram 4.2
n
ita

Udara panas
Hot air
rb
ne

Udara sejuk
Cool air

Rajah 4.3 / Diagram 4.3
Pe

(i) Berdasarkan Rajah 4.2 dan Rajah 4.3, bandingkan kekuatan bunyi pada waktu malam berbanding pada
waktu siang.
Based on Diagram 4.2 and Diagram 4.3, compare the loudness of sound at night compared to day time. [1 markah / mark]
(ii) Berdasarkan Rajah 4.2 dan Rajah 4.3, bandingkan panjang gelombang bunyi antara kawasan sejuk dengan
kawasan panas.
Based on Diagram 4.2 and Diagram 4.3, compare the wavelength of sound waves between the cold region and the warm
region. [1 markah / mark]

176
Fizik  Tingkatan 4  Bab 5 Gelombang 

(iii) Berdasarkan Rajah 4.2 dan Rajah 4.3 bandingkan sudut antara kawasan udara sejuk dengan kawasan udara
panas.
Based on Diagram 4.2 and Diagram 4.3, compare the angles between the cold air region and the warm air region.
[1 markah / mark]
(iv) Berdasarkan sudut antara kawasan sejuk dan kawasan panas, nyatakan arah perambatan gelombang bunyi
dari kawasan panas ke kawasan sejuk.
Based on the angles between the cold region and the warm region, state the direction of propagation of sound wave from the
warm region to the cold region. [1 markah / mark]
(v) Nyatakan konsep fizik yang berkaitan.
State the relevant physics concept. [1 markah / mark]

.
hd
(c) Rajah 4.4 menunjukkan satu sistem komunikasi yang terlibat dalam penghantaran siaran radio melalui kawasan
berbukit.
Diagram 4.4 shows a communication system involved in the transmission of radio broadcasts through hilly terrain.

.B
dn
iS
ng

BAB

la
Rajah 4.4 / Diagram 4.4

Gelombang dipancarkan pemancar dan diterima oleh rumah-rumah di belakang bukit tersebut. Walau
5
Pe

bagaimanapun, isyarat yang diterima tidak jelas.


Wave is transmitted from the transmitter and received by the houses behind the hills. However, the signal received is not clear.
Anda dikehendaki memberi beberapa cadangan untuk mereka bentuk satu sistem komunikasi yang dapat
menambah baik kualiti penghantaran isyarat. Menggunakan pengetahuan tentang gelombang, terangkan
n

cadangan anda berdasarkan aspek-aspek berikut: 4


ita

You are required to give some suggestions to design a communication system which can improve the quality of signal transmission.
Using the knowledge on waves, explain your suggestions based on the following aspects:
(i) Jenis gelombang yang dipancarkan / Type of wave transmitted
rb

(ii) Frekuensi gelombang / Frequency of the wave


(iii) Panjang gelombang yang dipancarkan / Wavelength of the wave transmitted
ne

(iv) Kaedah yang terlibat yang membolehkan gelombang itu merambat dalam jarak yang lebih jauh
The method involved which enable the wave to propagate in a longer distance
Pe

(v) Lokasi pemancar / Location of the transmitter


[10 markah / marks]

Bahagian C

5. Dalam imbasan ultrabunyi, gelombang ultrasonik digunakan untuk menghasilkan imej fetus untuk mengetahui
keadaan fetus dalam kandungan ibu.
2014
In ultrasound scan, ultrasonic waves are used to produce images of foetuses to determine the condition of the foetus in the womb.
(a) Rajah 1.1 menunjukkan gambar imbasan fetus yang dihasilkan oleh gelombang ultrasonik.
Diagram 1.1 shows the foetus scan image produced by ultrasonic waves.

177
  Fizik  Tingkatan 4  Bab 5 Gelombang

.
Rajah 1.1 / Diagram 1.1

hd
(i) Apakah yang dimaksudkan dengan gelombang ultrasonik?
What is the meaning of ultrasonic wave? [1 markah / mark]

.B
(ii) Terangkan bagaimana gelombang ultrasonik digunakan untuk menghasilkan imej fetus.
Explain how the ultrasonic waves are used to produce image of foetus. [4 markah / marks]
(b) Satu alur sonar dengan kelajuan 1200 m s–1 dihantar ke dasar laut dari sebuah bot nelayan.

dn
A sonar beam at a speed of 1200 m s–1 was sent into the seabed from a fishing boat.
Hitung / Calculate

iS
(i) Kedalaman dasar laut jika gema diterima balik selepas 50 milisaat.
Depth of the seabed if the echoes are received back at 50 milliseconds. [3 markah / marks]
(ii) Panjang gelombang untuk gelombang sonar apabila frekuensi ialah 25 kHz.
ng
The wavelength of the sonar waves when its frequency is 25 kHz. [3 markah / marks]
BAB

(c) Tiga bilik darjah di sekolah anda digabungkan untuk menjadi sebuah dewan kuliah. Jadual 1 menunjukkan empat
la
jenis sistem bunyi yang akan digunakan dalam dewan kuliah tersebut.
Three classrooms in your school are combined to become a lecture hall. Table 1 shows four types of sound system to be used in the
5
Pe

hall.

Jarak di antara Bahan yang


kedua-dua digunakan untuk
Sistem bunyi Lantai dewan Kedudukan mikrofon
pembesar suara menutup dinding
n

Sound system Hall flooring The position of the microphone


The distance between The material used to
ita

two loudspeakers cover the wall


Simen Kecil Tirai tebal Di belakang pembesar suara
P
Cement Small Thick curtain Behind the speaker
rb

Permaidani Besar Tirai tebal Di belakang pembesar suara


Q
Carpet Large Thick curtain Behind the speaker
ne

Simen Besar Hardwood Di hadapan pembesar suara


R
Cement Large Kayu keras In front of the speaker
Pe

Permaidani Kecil Hardwood Di hadapan pembesar suara


T
Carpet Small Kayu keras In front of the speaker
Jadual 1 / Table 1

Anda dikehendaki menyiasat reka bentuk sistem bunyi untuk menghasilkan kualiti bunyi yang lebih baik.
Terangkan kesesuaian setiap ciri pada sistem bunyi tersebut. Tentukan sistem bunyi yang paling sesuai. Berikan
sebab-sebab untuk pilihan anda. 5
You are asked to investigate the design of the sound system to produce better quality of sound. Explain the suitability of each
characteristic of the sound system. Determine the most suitable sound system. Give reasons for your choice [10 markah / marks]

178
PRAKTIS SPM 5
JAWAPAN Bahagian C
5. (a) (i) Ultrabunyi adalah gelombang bunyi dengan
Soalan Struktur
frekuensi lebih tinggi daripada 20 kHz.
Bahagian B Ultrasound is sound waves with frequencies higher than
20 kHz.
4. (a) (i) Gelombang membujur / Longitudinal Wave.
(ii) – Getaran diafragma pembesar suara yang (ii) Ultrabunyi bergerak melalui tisu lembut dan cecair
di dalam badan.

.
berterusan menghasilkan satu siri lapisan udara
Ultrasound dipantulkan kembali sebagai gema

hd
mampatan dan regangan yang bergerak ke
hadapan. apabila ia mengena permukaan yang lebih tumpat.
The vibration of diaphragm speaker produces a series Data gema akan dihantar ke komputer untuk
of rarefaction and compression of air that move forward. diproses. Komputer akan menghantar imej fetus ke

.B
– Gelombang bunyi yang terhasil akan merambat skrin komputer.
ke hadapan mengikut frekuensi diafragma. Ultrasound travels freely through fluid and soft tissues in
The sound waves produced will propagate forward the body. Ultrasound is reflected back as echoes when
according to the diaphragm frequency. it hits a denser surface. The echo data will be sent to
the computer for processing. The computer will send the

dn
– Getaran molekul udara ke hadapan dan ke
image of the foetus to the computer screen.
belakang masuk ke dalam telinga manusia.
The vibrations of the air molecules forward and vt
(b) (i) Kedalaman / Depth, d =
backward will enter into human ears. 2

iS
– Molekul udara yang bergetar mengikut frekuensi 1200 (50 × 10–3)
=
yang dihasilkan oleh pembesar suara akan 2
memukul gegendang telinga dan bunyi tersebut = 30 m
boleh didengari oleh manusia.
ng (ii) v = fλ
The air molecules vibrating by the frequency generated v 1200
λ = = = 0.048 m
by the loudspeaker will hit the eardrum and the sound f 25 000
can be heard by humans. (c)
(b) (i) Kekuatan bunyi pada waktu malam lebih kuat dari
Cadangan / Suggestion Penjelasan / Explaination
la
waktu siang.
The loudness of sound at night is higher than daylight.
(ii) Panjang gelombang bunyi kawasan sejuk lebih Lantai Dewan – Permaidani Boleh menyerap bunyi dan kurang
Pe

pendek daripada kawasan panas. Hall flooring – Carpet gangguan.


The wavelength in the cold region is shorter than the hot Can absorb sound and less disruption.
region.
(iii) Sudut kawasan udara sejuk lebih kecil berbanding Jarak di antara kedua-dua Apabila jarak di antara kedua-dua
sudut kawasan udara panas. pembesar suara – besar pembesar suara besar, jarak di
n

The angle of the cold air region is smaller than the angle Distance between two loud antara garis antinod (x) menjadi lebih
of hot air region. speakers – Large kecil.
ita

(iv) Arah perambatan gelombang bunyi akan mendekati When the distance between the two
normal. speakers is large, the distance between
The direction of sound wave propagation will be towards the antinode lines (x) becomes smaller.
the normal.
rb

(v) Pembiasan Bahan yang digunakan Boleh menyerap bunyi dan tidak
Refraction
untuk menutupi dinding – memantulkan bunyi.
(c) (i) Gelombang radio – Boleh merambat pada jarak jauh Can absorb sound and prevent reflection
Tirai tebal
ne

Radio wave – Can propagate in long distance of sound.


Material used to cover the wall
(ii) Frekuensi gelombang tinggi – Mempunyai tenaga – Thick curtain
yang lebih tinggi
High wave frequency – Has higher energy
Pe

Mikrofon di belakang Untuk mengelakkan hingar akibat


(iii) Gelombang dengan panjang gelombang yang pembesar suara bunyi dari pembesar suara masuk
panjang – supaya pembelauan berlaku pada sudut Microphone behind the speaker semula ke mikrofon.
yang lebih besar. To avoid feedback due to the sound from
Waves with long wavelengths – so that the diffraction the speaker pick up by the microphone.
occurs at a larger angle.
(iv) Menggunakan stesen geganti- untuk menguatkan Sistem bunyi yang paling sesuai ialah Q kerana lantai dewan
isyarat dan menghantar semula ke jarak yang jauh menggunakan permaidani, jarak di antara kedua-dua pembesar
Using relay stations – to amplify the signal and retransmit
to a further distance suara adalah besar, bahan yang digunakan untuk menutup
(v) Lokasi pemancar perlu di kawasan yang tinggi – dapat dinding ialah tirai tebal dan mikrofon di belakang pembesar suara.
The most suitable sound system is Q because hall flooring uses carpet,
memancarkan gelombang radio tanpa halangan distance between two loud speakers is large, material used to cover the wall
The location of the transmitter needs to be in high ground
is thick curtain and microphone behind the speake.
– can transmit radio waves without any obstacles
B
BA
6
Cahaya dan Optik
Lights and Optics
Analisis Soalan SPM
Kertas 2011 2012 2013 2014 2015 2016 2017 2018
1 ✔ ✔ ✔ ✔ ✔ ✔ ✔ ✔

PETA Konsep 2
3


✔ ✔ ✔ ✔ ✔ ✔ ✔

.
hd
Dalam nyata dan
Indeks biasan Hukum snell dalam ketara
Pembiasan cahaya Refractive index Real depth and

.B
Snell law
Refraction of light c apparent depth
n= v n1 sin θ1 = n2 sin θ1
H
n= h

dn
Apikasi
Pantulan dalam Sudut genting Fenomena semula jadi
Application
penuh Critical angle Natural phenomena

iS
• Periskop / Periscope
Total internal 1 • Pelangi / Rainbow
sin c = n • Gentian optik
reflection • Logamnya / Mirage
ng Optical fibre

Cermin cekung
Concave mirror
Pembesar linear
Linear magnification
la
Kanta
CAHAYA DAN OPTIK

Lenses h1 v
LIGHTS AND OPTICS

n= v =
0 u
Cermin cembung
Pe

Convex mirror

Formula kanta
n

Formula kanta nipis Lens formula


Thin lens formula 1 1 1
= +
ita

f u v
rb

• Kanta pembesar / Magnifying lens


Peralatan optik
• Mikroskop majmuk / Microscope
Optical instruments
• Teleskop / Telescope
ne

Apikasi / Applications
Pe

• Cermin keselamatan / Security mirror


Cermin cembung
Convex mirror • Cermin selekoh tersembunyi
Blind corners mirror
• Cermin sisi kereta / Side mirror
Cermin sfera
Spherical mirror
Apikasi / Applications
• Cermin cukur / Shaving mirror
Cermin cekung
Concave mirror • Cermin solek / Make-up mirror
• Cermin pergigian
Dental mirror

179
  Fizik  Tingkatan 4  Bab 6 Cahaya dan Optik

Pembiasan Cahaya
6.1 Refraction of Light

Normal
1. Pembiasan cahaya ialah pembengkokan sinar cahaya di sempadan apabila 1 Normal

sinar itu bergerak dari satu medium ke medium yang lain yang berlainan ketumpatan. Udara i
Air
Refraction of light is the bending of a light ray at the boundary as it travels
from one medium to another that is different density. 1 2 Kaca r 2
Glass

.
Sinar muncul 3

hd
Emergent ray

Sinar cahaya Penerangan

.B
Light ray Explanation

(a) Sinar tuju merambat dari satu medium (udara) dan memasuki sempadan di antara dua medium
1
(sempadan udara-kaca)

dn
Incident ray travels from a medium (air) and strikes the boundary between the two media (air-glass
boundary)

iS
(b) Sinar tuju kemudian dibiaskan apabila merambat ke dalam medium yang lain dan terus
2
merambat di dalam medium ini sehingga memasuki satu lagi sempadan udara-kaca
ng
The incident ray is then refracted when it travels into another medium and continues to travel within
this medium until it meets another air-glass boundary
la
(c) Sinar biasan mengalami pembiasan sekali lagi di sempadan dan sinar cahaya yang muncul
3
dari kaca ke udara dikenal sebagai sinar muncul.
Pe

The refracted ray undergoes refraction again at the boundary and the light ray emerging from
glass to air is known as the emergent ray.
BAB

2. Semasa pembiasan cahaya:


n

During refraction of light:


6
ita

(a) Sudut di antara sinar tuju dengan normal ialah sudut tuju, i .

The angle between the incident ray and the normal is the angle of incidence, i .
rb

(b) Sudut di antara sinar biasan dengan normal ialah sudut biasan, r .
ne

The angle between the refracted ray and the normal is the angle of refraction, r .

3. Sinar muncul dalam rajah merambat dalam arah yang sama dengan sinar tuju. Sinar muncul itu
Pe

selari dengan sinar tuju tetapi tidak sejajar .

The emergent ray in the diagram travels in the same direction as the incident ray. The emergent ray is parallel

to the incident ray but not in line .

4. Berikut menghuraikan cara sinar cahaya dibiaskan apabila merambat ke medium yang berbeza ketumpatan
(lebih tumpat atau kurang tumpat).
The following describes ways in which a light ray is refracted when it travels to a medium of different density (denser or less
dense).

180
  Fizik  Tingkatan 4  Bab 6 Cahaya dan Optik 

(a) Cahaya merambat dari Normal ke arah


Normal Sinar cahaya dibiaskan
medium optik kurang
normal. Kelajuan cahaya berkurang .
tumpat ke medium optik i Udara
lebih tumpat. Air
The light ray is refracted towards the
Light travels from an optically Kaca decreases
r Glass normal. The speed of light .
less dense medium to an
optically denser medium.

(b) Cahaya merambat dari Normal Sinar cahaya dibiaskan menjauhi


Normal

.
medium optik lebih bertambah .

hd
tumpat ke medium optik normal. Kelajuan cahaya
r Udara
kurang tumpat. Air
The light ray is refracted away from
Light travels from an optically Kaca increases .

.B
i the normal. The speed of light
denser medium to an Glass
optically less dense medium.

dn
(c) Cahaya merambat dari Sinar cahaya tidak berubah arah.
Udara
dua medium yang berbeza Air berubah .
Kelajuan cahaya

iS
ketumpatan optik secara
Kaca change
tegak lurus. Glass The light ray does not its
Light travels through two changes
direction. The speed of light .
media of different optical
ng
densities in straight line.
la
Indeks Biasan
Refractive Index
Pe

1. Hukum pembiasan menyatakan bahawa apabila cahaya merambat dari satu medium ke medium lain yang

BAB
mempunyai ketumpatan optik yang berbeza,
The law of refraction states that when light travels from one medium to another medium of different optical density,
n

(a) sinar tuju, sinar biasan dan normal berada pada satah yang sama,
6
ita

the incident ray, the refracted ray and the normal lie in the same plane ,
sin i
rb

(b) nisbah sin r adalah malar, di mana i ialah sudut tuju dan r ialah sudut biasan.
sin i
ne

the ratio sin r is constant, where i is the angle of incidence and r is the angle of refraction.

2. Hukum pembiasan juga dikenal sebagai Hukum Snell .


Pe

The law of refraction is also known as Snell’s Law .

3. Bagi sinar cahaya yang memasuki suatu medium dari vakum atau udara, indeks biasan, n untuk medium itu
boleh dinyatakan sebagai:
For light rays entering a medium from vacuum or air, the refractive index, n for the medium can be expressed as:

sin i (vakum atau udara) sin i (vacuum or air)


Indeks biasan, n = / Refractive index, n =
sin r (medium) sin r (medium)

181
  Fizik  Tingkatan 4  Bab 6 Cahaya dan Optik

4. Indeks biasan suatu medium ialah nisbah kelajuan cahaya di dalam vakum atau udara dengan kelajuan cahaya
di dalam medium itu.
The refractive index of a medium is the ratio of the speed of light in vacuum or air to the speed of light in the medium.

Kelajuan cahaya di dalam vakum atau udara


Indeks biasan =
Kelajuan cahaya di dalam medium

Speed of light in vacuum or air


Refractive index =
Speed of light in medium

.
n =

hd
v

.B
Hubungan antara sudut tuju dengan sudut biasan
Eksperimen 6.1 Relationship between angle of incidence and angle of refraction

dn
Pernyataan masalah / Problem statement:
Apakah hubungan antara sudut tuju dengan sudut biasan ?

iS
What is the relationship between the angle of incidence and the angle of refraction ?

Tujuan / Aim:
ng
Untuk mengetahui hubungan antara sudut tuju dengan sudut biasan .
angle of incidence angle of refraction
la
To find the relationship between the and .

Hipotesis / Hypothesis:
Pe

Apabila sudut tuju, i meningkat , sudut biasan, r juga meningkat .


BAB

As the angle of incidence, i increases , the angle of refraction, r also increases .


n

Pemboleh ubah / Variables:


6
ita

(a) Pemboleh ubah dimanipulasi: Sudut tuju, i

Manipulated variable: Angle of incidence, i


rb

(b) Pemboleh ubah bergerak balas: Sudut biasan, r


Angle of refraction, r
ne

Responding variable:

(c) Pemboleh ubah dimalarkan: Jenis kaca


Pe

Constant variable: Type of glass

Bahan / Material:
Kertas putih / White paper

Radas / Apparatus:
Bongkah kaca segi empat tepat , kotak sinar , protraktor dan pensel
Rectangular glass block , ray box , protractor, and pencil

182
  Fizik  Tingkatan 4  Bab 6 Cahaya dan Optik 

Susunan radas / Arrangement of apparatus:


Kotak sinar
Ray box

i
O

r Bongkah kaca
Glass block

.
    

hd
Prosedur / Procedure:
1. Radas disediakan seperti ditunjukkan di atas.
The apparatus is set up as shown above.

.B
2. Garis luar bongkah kaca disurih pada sehelai kertas putih dan garis normal dilukis melalui titik O.
External lines of the glass block is traced on a white paper and the normal line is drawn through the point O.
3. Lima garis dilukis pada sudut tuju yang berbeza, i = 20°, 30°, 40°, 50° dan 60° dengan menggunakan protraktor.

dn
Five lines are drawn with different angles of incidence, i = 20°, 30°, 40°, 50° and 60° by using a protractor.
4. Satu sinar cahaya dari kotak sinar ditujukan pada garis, i = 20°. Sinar muncul, PQ dilukis.
A ray of light from the ray box is directed at the line, i = 20°. The emergent ray, PQ is drawn.

iS
5. Sinar biasan dilukis di sepanjang OP, dan sudut biasan, r diukur dan direkodkan.
The refracted ray is drawn along OP, and the angle of refraction, r is measured and recorded.
6. Langkah 4 dan 5 diulang untuk sudut tuju, i = 30°, 40°, 50° dan 60°.
ng
Steps 4 and 5 are repeated for angles of incidence, i = 30°, 40°, 50° and 60°.
sin i
7. Nilai sin i, sin r dan dihitung dan dijadualkan.
la
sin r
sin i
The values of sin i, sin r and are calculated and tabulated.
sin r
Pe

8. Graf r melawan i dan graf sin i melawan sin r diplotkan.


Graphs of r against i and sin i against sin r are plotted.

BAB
Penjadualan data / Tabulation of data:
n

Sudut tuju, i Sudut biasan, r sin i


sin i sin r
6
ita

Angle of incidence, i Angle of refraction, r sin r


20°
rb

30°
40°
ne

50°
60°
Pe

Analisis data / Analysis of data:


Graf r melawan i Graf sin r melawan sin i
Graph of r against i Graph of sin r against sin i
r sin r

    i      sin i

183
  Fizik  Tingkatan 4  Bab 6 Cahaya dan Optik

Kesimpulan / Conclusion:
Apabila sudut tuju, i meningkat , sudut biasan, r juga meningkat . Hipotesis diterima.
As the angle of incidence, i increases , the angle of refraction, r also increases . The hypothesis is accepted.

Dalam Nyata dan Dalam Ketara


Real Depth and Apparent Depth

1. Sekeping syiling di dalam segelas air kelihatan lebih dekat kepada


pemerhati kerana

.
A coin in a glass of water appears to be nearer to the observer because

hd
(a) sinar cahaya daripada syiling nyata merambat melalui air ke
Dalam
permukaan,
ketara, d
light rays from the real coin travel through the water to the surface, Dalam

.B
Apparent
depth, d 50 nyata, D
(b) sinar cahaya itu kemudian dibiaskan menjauhi normal dan syiling
20 13

Real
maya (imej) dilihat oleh pemerhati kelihatan datang dari imej itu, depth, D
yang berada di atas syiling nyata.

dn
20 50 13

the light rays are then refracted away from the normal and the virtual coin
(image) seen by the observer appears to come from the image, which is above the real coin.

iS
2. Indeks biasan, n bagi suatu medium juga boleh dinyatakan secara matematik sebagai
The refractive index, n, of a medium can also be mathematically expressed as

Indeks biasan, n = Dalam nyata, D


ng
/ Refractive index, n = Real depth, D
Dalam ketara, d Apparent depth, d
la
Hubungan antara dalam nyata dengan dalam ketara
Eksperimen 6.2 Relationship between real depth and apparent depth
Pe

Pernyataan masalah / Problem statement :


BAB

Apakah hubungan antara dalam nyata dengan dalam ketara ?


n

real depth apparent depth


6
ita

What is the relationship between and ?

Tujuan / Aim :
rb

Mengkaji hubungan antara dalam nyata dengan dalam ketara .


To investigate the relationship between real depth and apparent depth .
ne

Hipotesis / Hypothesis:
bertambah bertambah
Pe

Apabila dalam nyata suatu objek di dalam air , dalam ketara objek itu juga .
As the real depth of an object in water increases , its apparent depth also increases .

Pemboleh ubah / Variables:


(a) Pemboleh ubah dimanipulasi: Dalam nyata, D
Manipulated variable: Actual depth, D

(b) Pemboleh ubah bergerak balas: Dalam ketara, d

Responding variable: Apparent depth, d

184
  Fizik  Tingkatan 4  Bab 6 Cahaya dan Optik 

(c) Pemboleh ubah dimalarkan: Indeks biasan medium (air), n

Constant variable: Refractive index of medium (water), n

Bahan / Material:
Air / Water
Radas / Apparatus:
Bikar tinggi, pin, pembaris meter dan kaki retort
Tall beaker, pins, metre rule, and retort stand

Susunan radas / Arrangement of apparatus:

.
hd
Pemerhati
Observer

Pembaris meter

.B
Bikar Metre rule
Beaker
Air
d
Water
D

dn
Imej
ketara P1
Apparent
image P0
   

iS
Prosedur / Procedure:
1. Radas disediakan seperti yang ditunjukkan dalam rajah.
The apparatus is set up as shown in the diagram.
ng
2. Sebatang pin, PO diletakkan di dasar bikar tinggi.
A pin, PO is placed at the base of the tall beaker.
la
3. Pin yang satu lagi, PI diapit pada pengapit alih kaki retort.
Pe

Another pin, PI is mounted on a movable clamp of a retort stand.


4. Bikar diisi dengan air pada kedalaman 4.0 cm.

BAB
The beaker is filled with water to a depth of 4.0 cm.
5. PO diperhatikan dari atas permukaan air itu.
n

PO is observed from above the surface of the water.


6
ita

6. PI diselaraskan sehingga kedudukan pin itu sama aras dengan imej PO di dalam bikar.
PI is adjusted until its position is at the same level as the image of PO in the beaker.
7. Kedudukan PI diukur dan direkodkan sebagai dalam ketara, d.
rb

The position of PI is measured and recorded as the apparent depth, d.


8. Eksperimen diulang dengan mengubah dalam nyata, D = 5.0 cm, 6.0 cm, 7.0 cm, 8.0 cm dan 9.0 cm.
ne

The experiment is repeated by changing the real depth, D = 5.0 cm, 6.0 cm, 7.0 cm, 8.0 cm and 9.0 cm.

Penjadualan data / Tabulation of data:


Pe

Dalam nyata, D (cm) Dalam ketara, d (cm)


Real depth, D (cm) Apparent depth, d (cm)
4.0
5.0
6.0
7.0
8.0
9.0

185
  Fizik  Tingkatan 4  Bab 6 Cahaya dan Optik

Analisis data / Analysis of data:


Dalam ketara, d (cm)
Apparent depth, d (cm)

0 Dalam nyata, D (cm)


     Real depth, D (cm)

Kesimpulan / Conclusion:

.
hd
Dalam ketara bertambah apabila dalam nyata bertambah . Nisbah dalam nyata kepada dalam ketara ialah
indeks biasan medium. Hipotesis diterima.

.B
Apparent depth increases when real depth increases . The ratio of real depth to apparent depth is the
refractive index of the medium. The hypothesis is accepted.

dn
Tugasan 1

iS
1. Rajah menunjukkan satu sinar cahaya merambat dari udara ke kaca. Cari sudut tuju, i dan
sudut biasan, r. Kemudian, hitung indeks biasan, n kaca itu.
The diagram shows a ray of light travelling from air to glass. Find the angle of incidence, i and the angle of
ng
refraction, r. Then, calculate the refractive index, n of the glass. 50°
60°

la
40° i = 90 – 50 = 40°, r = 90 – 60 = 30°
sin i sin 40°
n= = = 1.29
Pe

sin r sin 30°


30°
BAB

2. Satu sinar cahaya merambat dari udara ke kaca seperti yang ditunjukkan dalam
rajah. Tentukan sudut θ jika indeks biasan kaca ialah 1.42.
6
ita

A light ray travels from air to glass as shown in the diagram. Determine the angle θ if the refractive Udara
index of the glass is 1.42. Air 47°
sin i sin 43°
rb

Kaca θ
i = 90 – 47 = 43°, r = 90 – θ, n = = = 1.42 Glass
sin r sin 90° – θ
90 – θ = 28.7°
ne

θ = 61.3°
Pe

3. Kelajuan cahaya di dalam sebutir berlian ialah 1.2 × 108 m s–1. Jika kelajuan cahaya di dalam vakum ialah 3.0 × 108 m s–1,
hitung indeks biasan berlian itu.
The speed of light in a diamond is 1.2 × 108 m s–1. If the speed of light in vacuum is 3.0 × 108 m s–1, calculate the refractive index of the
diamond.
Diberi / Given: v = 1.2 × 108 m s–1, c = 3.0 × 108 m s–1, n = ?
c 3.0 × 108
n= v = = 2.5
1.2 × 108

186
  Fizik  Tingkatan 4  Bab 6 Cahaya dan Optik 

4. Rajah menunjukkan satu sinar cahaya melalui cecair X.


The diagram shows a light ray passing through liquid X.
Indeks biasan cecair X ialah 1.15. Jika kelajuan cahaya di dalam vakum ialah 55°
3.0 × 108 m s–1, cari
The refractive index of liquid X is 1.15. If the speed of light in vacuum is 3.0 × 108 m s–1, find Cecair X
Liquid X
(a) kelajuan cahaya di dalam cecair X, (b) sudut biasan, r.
the speed of light in liquid X, the angle of refraction, r.
sin i sin 55°
Diberi / Given , i = 55°, n = 1.15,
(a) (b) 1.15 = =
sin r sin r
c = 3.0 × 108 m s–1, v = ?
sin 55°

.
c sin r = = 0.7123

hd
n = v 1.15
3.0 × 108 r = 45.4°
1.15 =
v

.B
v = 2.61 × 108 m s–1

5. Sekeping syiling diletakkan di dasar sebuah bikar yang berisi air dengan kedalaman 6.0 cm. Berapakah dalam ketara
syiling itu apabila diperhatikan dari atas permukaan air? [Indeks biasan air = 1.2]

dn
A coin is placed at the bottom of a beaker filled with water to a depth of 6.0 cm. What is the apparent depth of the coin when viewed from
above the water surface? [Refractive index of water = 1.2]

iS
Diberi / Given: D ­= 6.0 cm, n = 1.2, d =?
D
n =
d
6.0
ng
1.2 =
d
d = 5.0 cm
la
6. Satu objek diletakkan di bawah satu bongkah kaca. Mata pemerhati
Pe

berada pada jarak 15 cm di atas bongkah kaca itu. Jika indeks biasan
bongkah kaca ialah 1.25, berapakah jarak di antara mata dengan imej 15 cm
yang terbentuk di dalam bongkah kaca itu?

BAB
An object is placed under a glass block. The eye of an observer is at a distance of
Bongkah kaca
n

15 cm above the glass block. If the refractive index of the glass block is 1.25, what is Imej 3 cm
Glass block Image
the distance between the eye and the image formed in the glass block?
6
ita

D Objek
n = Object
d
3.0
rb

1.25 =
d
d = 2.4 cm
ne

∴ Jarak di antara mata dengan imej / Distance between the eye and the image
= 15 + 2.4 = 17.4 cm
Pe

7. Sekeping syiling diletakkan di bawah satu bongkah kaca berketinggian 4.0 cm. Jarak di antara imej syiling dengan
syiling nyata ialah 0.5 cm. Berapakah indeks biasan bongkah kaca itu?
A coin is placed at the bottom of a glass block of height 4.0 cm. The distance between the image of the coin and the real coin is 0.5 cm. What
is the refractive index of the glass block?
D
n =
d
4.0 cm
4.0
=
4.0 – 0.5 0.5 cm
= 1.14

187
  Fizik  Tingkatan 4  Bab 6 Cahaya dan Optik

8. Seketul batu kelihatan lebih dekat dengan permukaan sebuah kolam. Kedalaman air kolam itu ialah 1.8 m.
[Indeks biasan air = 1.35]
A stone appears closer to the surface of a pond. The depth of water in the pond is 1.8 m.
[Refractive index of water = 1.35]
(a) Lukis sinar cahaya dari batu ke mata pemerhati. Pemerhati
Draw the light ray from the stone to the eye of the observer. Observer

        
(b) Berapakah dalam ketara batu itu?
What is the apparent depth of the stone?

.
D Batu
n =

hd
d Stone
1.8
1.35 =
d

.B
d = 1.33 cm

Pantulan Dalam Penuh

dn
6.2 Total Internal Reflection

1. Kedua-dua gambar foto menunjukkan fenomena

iS
pantulan dalam penuh di dalam air.
The two photographs show the phenomena of total internal
reflection in water.
ng
2. Pantulan dalam penuh ialah pantulan lengkap
suatu sinar tuju di sempadan antara dua medium,
la
apabila sinar cahaya itu di dalam medium dengan
indeks biasan yang lebih besar .
Pe

Total internal reflection is the complete reflection of a light ray at the boundary between two media, when the light ray is in
the medium with greater refractive index.
BAB

3. Berikut menghuraikan pantulan dalam penuh dan sudut genting.


n

The following describes the total internal reflection and the critical angle.

6
ita

(a) Sinar tuju merambat • i < r


melalui medium lebih • Sinar cahaya dibiaskan menjauhi normal
tumpat ke medium kurang
rb

The light ray refracts away from normal


tumpat
Incident ray travels through a i • Sinar pantulan yang lemah dapat dilihat
ne

denser medium to a less dense The weak reflected ray can be seen
medium
Pe

(b) Sudut tuju dibesarkan • i < r


sehingga cahaya dibiaskan • r = 90°
pada sudut 90° dari
• i = c (sudut genting / critical angle)
normal
Angle of incidence is • Sinar cahaya dibiaskan menjauhi normal
increased until the light is away
i=c The light ray refracts from normal
refracted at an angle of 90°
from normal • Sinar pantulan yang lemah dapat dilihat
The weak reflected ray can be seen

188
  Fizik  Tingkatan 4  Bab 6 Cahaya dan Optik 

(c) Sudut tuju bertambah • i > c


sehingga lebih besar dibiaskan
• Tiada cahaya
daripada sudut genting
Angle of incidence is No refracted light
increased until bigger than
• Cahaya dipantulkan di sempadan kaca dan
critical angle
i>c udara
Light is reflected at the boundary of glass and air.

• Pantulan dalam penuh berlaku

.
Total internal reflection happens

hd
4. Dua syarat yang mesti dipenuhi untuk pantulan dalam penuh berlaku ialah:

.B
The two conditions that must be fulfilled for total internal reflection to take place are:

(a) sinar cahaya mesti merambat dari medium optik lebih­ tumpat ke medium optik kurang tumpat ,

light ray must travel from an optically denser medium into an optically less dense medium,

dn
(b) sudut tuju di dalam medium optik lebih tumpat mesti lebih besar daripada sudut genting, c.
the angle of incidence, i in the optically denser medium must be greater than the critical angle, c.

iS
Sudut Genting dan Indeks Biasan
Critical Angle and Refractive Index
ng
1. Sudut genting
, c ditakrifkan sebagai sudut tuju, i apabila sinar cahaya dibiaskan Udara / air, n = 1
pada 90° dari normal.
la
The critical angle , c is defined as the angle of incidence, i when a light ray is refracted at
90° from normal.
Pe

c
2. Daripada Hukum Snell, / From Snell’s Law,
n = sin i

BAB
n1, sin θ1, = n2 sin θ2
sin r
n

3. Apabila sudut tuju, i adalah sudut genting, c, sudut biasan, r ialah 90°. Maka,
6
ita

When the angle of incidence, i is the critical angle, c, the angle of refraction is 90°. Hence,
sin 90°
n=
rb

sin c

1 , 1 1
sin c = atau/ or c = sin–1 
ne

n=
sin c n n

Pe

Fenomena Semula Jadi Melibatkan Pantulan Dalam Penuh


Natural Phenomena Involving Total Internal Reflection

1. Logamaya / Mirage Sinar cahaya dari langit


Light ray from sky
(a) Pada hari panas terik, lopak air kelihatan di atas tanah di
Udara sejuk / Cool air
hadapan kita. Lopak air itu hilang apabila kita menghampiri.
On a hot and sunny day, a puddle of water seems to appear on the
ground ahead. The puddle of water disappears as we approach it. Udara panas / Hot air

Logamaya / Mirage

189
  Fizik  Tingkatan 4  Bab 6 Cahaya dan Optik

(b) Fenomena ini dikenal sebagai logamaya yang disebabkan oleh pembiasan dan pantulan dalam penuh  .
This phenomenon is known as mirage which is caused by refraction and total internal reflection .

(c) Hal ini berlaku apabila cahaya melalui lapisan udara dengan suhu yang berbeza-beza. Matahari
memanaskan tanah. Lapisan udara yang dekat dengan tanah lebih panas dan kurang tumpat.
Lapisan yang lebih atas adalah lebih sejuk dan lebih tumpat .
This happens when light passes through layers of air with different temperatures. The Sun heats up the ground. The
layers of air near the ground are hotter and less dense. The upper layers are cooler and denser .
ketumpatan

.
(d) Indeks biasan udara bergantung pada masing-masing. Lapisan bawah atau yang lebih

hd
panas mempunyai indeks biasan yang lebih rendah daripada lapisan atas.
The refractive index of air depends on its density . The lower or hotter layers have a lower refractive index

.B
than the layers above them.

(e) Apabila sinar cahaya daripada langit merambat ke arah permukaan bumi, sinar cahaya itu dibiaskan
menjauhi normal akibat pengurangan ketumpatan udara dan pengurangan indeks biasan

dn
udara di atmosfera.
When light rays from the sky travel towards the earth’s surface, the light rays are refracted away from the

iS
normal due to decreasing air density and decreasing refractive index of air in the atmosphere.

(f) Sudut tuju bertambahapabila sinar cahaya memasuki lapisan udara yang seterusnya. Sinar cahaya
ng
itu dipantulkan apabila sudut tuju melebihi sudut genting udara dalam lapisan tersebut. Maka,
pantulan dalam penuh terjadi.
la
The angle of incidence increases as the light rays enter the next layer of air. The light rays are reflected when the
angle of incidence exceeds the critical angle of air in that layer. Hence, total internal reflection occurs.
Pe

(g) Sinar pantulan memasuki mata pemerhati dan satu imej maya terbentuk di atas tanah seolah-olah imej
BAB

itu berasal dari tanah.


The reflected rays enter the observer’s eyes and a virtual image is formed on the ground as if the image originates from
n

the ground.
6
ita

2. Pelangi / Rainbow Cahaya putih Cahaya merah


White light Red light
(a) Pembentukan pelangi berlaku apabila cahaya matahari menyinari
titisan air di udara. Kejadian ini melibatkan beberapa fenomena optik,
rb

Cahaya ungu
iaitu pembiasan , penyerakan dan pantulan dalam penuh  . Violet light
Merah
ne

The formation of rainbow happens when sunlight shines on water droplets


Red
in the air. It involves a few optical phenomena, namely refraction ,
total internal reflection Ungu
dispersion and .
Pe

Violet

(b) Apabila suatu cahaya putih memasuki permukaan hadapan setitis air, cahaya putih itu mengalami
penyerakan dan dipisahkan kepada warna-warna yang berbeza, yang disebabkan oleh sudut biasan
yang berbeza bagi warna yang berbeza.
When a white light enters the front surface of a water droplet, the white light undergoes dispersion and is separated into
different colours due to different angles of refraction for different colours of light.

(c) Cahaya merah dibiaskan paling sedikit manakala cahaya ungu dibiaskan paling banyak ke
arah normal.
Red light is refracted the least and violet light is refracted the most towards the normal.

190
  Fizik  Tingkatan 4  Bab 6 Cahaya dan Optik 

(d) Kesemua warna cahaya yang berbeza kemudian mengalami pantulan dalam penuh di permukaan
belakang titisan air.
All of the different colours of light undergo total internal reflection at the back surface of the water droplet.
(e) Sinar cahaya yang dipantulkan kembali ke permukaan hadapan dan mengalami pembiasan sebelum
merambat ke arah mata pemerhati dengan spektrum cahaya.
The reflected light rays return to the front surface and undergo refraction before travelling towards the observer’s
eyes with spectrum of colour.

Aplikasi Pantulan Dalam Penuh

.
Applications of Total Internal Reflection

hd
1. Periskop / Periscopes 3

(a) Sebuah periskop dapat dibina menggunakan dua prisma Prisma bersudut tegak

.B
Right-angled prisms
bersudut tegak.
A periscope can be constructed using two right-angled (45°) glass
prisms.

dn
(b) Sinar cahaya daripada suatu objek memasuki prisma atas secara
tegak lurus dan mengenai permukaan hipotenus (mengufuk
pantulan dalam penuh Prisma bersudut tegak

iS
45°) prisma itu dan mengalami
Right-angled prisms
(sudut genting kaca, iaitu 42°).
Light rays from an object enter the top prism perpendicularly and strike the hypotenuse face (45° to the horizontal) of
ng
the prism and undergo total internal reflection (critical angle of glass, which is 42°).

(c) Sinar cahaya yang dipantulkan itu kemudian merambat ke prisma bawah di mana sekali lagi
pantulan dalam penuh
la
berlaku sebelum sampai ke mata pemerhati.
The reflected light rays then travel to the bottom prism where another round of total internal reflection takes place
Pe

before reaching the observer’s eye.

(d) Imej yang dibentuk ialah tegak , maya dan mempunyai saiz yang sama dengan objek.

BAB
The image formed is upright , virtual and has the same size as the object.
n

2. Gentian optik / Optical fibre 3


6
ita

(a) Suatu gentian optik terdiri daripada satu teras dalam dengan indeks
biasan yang lebih tinggi dan salutan luar dengan indeks biasan Salutan
rb

lebih rendah . Cladding Teras dalam


yang Inner core
higher
An optical fibre consists of an inner core with refractive index and
ne

an outer cladding with lower refractive index.


Pantulan dalam penuh
(b) Di dalam gentian optik, suatu cahaya merambat dari medium optik
Pe

di sempadan
lebih tumpat (teras dalam) ke medium optik kurang tumpat Total internal reflection
at the boundaries
(salutan luar).
Inside the optical fibre, a light travels from the optically denser medium (inner core) to the optically less dense
medium (outer cladding).
(c) Apabila cahaya mengenai sempadan di antara teras dengan salutan pada sudut tuju yang lebih besar

daripada sudut genting, cahaya itu mengalami pantulan dalam penuh .


As light strikes the boundary between the core and the cladding at angle of incidence greater than the critical
angle, the light undergoes total internal reflection .

191
  Fizik  Tingkatan 4  Bab 6 Cahaya dan Optik

(d) Cahaya merambat di sepanjang gentian dan mengalami pantulan dalam penuh berulang dalam
suatu lintasan bengkang-bengkok.
The light propagates along the fibre and undergoes repeated total internal reflection in a zig-zag path.

(e) Gentian optik berguna dalam banyak aplikasi praktikal yang penting. Hal ini termasuklah alat diagnostik
gentian optik dalam perubatan dan kabel gentian optik dalam telekomunikasi .
Optical fibres are useful in many important practical applications. This includes fibre optic diagnostic tools in
medicine and fibre optic cables in telecommunications .

(i) Endoskop yang dibina daripada suatu kabel gentian optik digunakan oleh doktor untuk melihat di

.
hd
dalam organ manusia seperti perut dan duodenum.
An endoscope which consists of a fibre optic cable is used by doctors to see the inside of the human organs such
as the stomach and the duodenum.

.B
(ii) Dalam telekomunikasi, kabel kuprum kini digantikan dengan kabel gentian optik yang mampu
membawa data yang lebih banyak pada kadar pemindahan yang lebih cepat .
more

dn
In telecommunications, copper cables are now replaced by fibre optic cables which are able to carry
data at a faster transmission rate .

iS
Tugasan 2 ng
1. Satu sinar cahaya ditujukan kepada dua bongkah prisma. Lukiskan sinar cahaya di dalam prisma dan sinar muncul jika
sudut genting bagi kedua-dua prisma ialah 42°.
A light ray is directed to two prism blocks. Draw the light ray in the prisms and the emergent ray if the critical angle of both the prisms is 42°.
la
(a) (b)
70°
Pe

45°
70°
70°
45°
BAB

45°
n

6
ita

2. Sudut genting suatu kaca ialah 42°. Jika satu sinar cahaya merambat dari kaca itu ke udara,
rb

The critical angle of a glass is 42°. If a light ray travels from the glass to air,
(a) nyatakan fenomena yang akan berlaku jika sudut tuju ialah
ne

state the phenomenon that will happen if the angle of incidence is

(i) 40°: Pembiasan / Refraction


Pe

(ii) 42°: Pembiasan / Refraction

(iii) 44°: Pantulan (pantulan dalam penuh) / Reflection (total internal reflection)

(b) tentukan indeks biasan kaca itu.


determine the refractive index of the glass.
1 1
n = sin c = sin 42° = 1.49

192
  Fizik  Tingkatan 4  Bab 6 Cahaya dan Optik 

3. Sebutir berlian mempunyai indeks biasan 2.42. Hitung sudut genting, c bagi berlian itu.
A diamond has a refractive index of 2.42. Calculate the critical angle, c, of the diamond.
1
n = sin c
1 1
c = sin–1 n  = sin–1 2.42  = 24.4°

4. Lintasan satu sinar cahaya dari A ke O di dalam sebutir berlian seperti ditunjukkan di O
bawah. Sudut genting bagi berlian itu ialah 24°. Jika θ = 74°, lukis lintasan cahaya selepas θ θ
melepasi O.
The path of a light ray from A to O in a diamond is as shown below. The critical angle of the diamond is A

.
24°. If θ = 74°, draw the light path after passing through O.

hd
Pembentukan Imej oleh Kanta
6.3

.B
Image Formation by Lenses

1. Terdapat dua jenis kanta iaitu kanta penumpu dan kanta pencapah .

dn
There are two types of lenses: converging and diverging lenses.

2. Kanta penumpu dikenal sebagai kanta cembung . Kanta ini lebih tebal di pusat berbanding di sisi.

iS
A converging lens is called a convex lens . It is thicker at the centre than at the edges.

kanta cekung . Kanta ini lebih nipis di pusat berbanding di sisi.


3. Kanta pencapah dikenal sebagai
ng
A diverging lens is called a concave lens . It is thinner at the centre than at the edges.
la
Titik Fokus dan Panjang Fokus Suatu Kanta
Focal Point and Focal Length of a Lens
Pe

1. Perbezaan di antara kanta cembung dengan kanta cekung ditunjukkan di bawah.

BAB
The differences between convex lens and concave lens are shown below.
n

Kanta cembung Kanta cekung


Convex lens Concave lens
6
ita
rb

Paksi utama
Principal axis Paksi utama
ne

Principal axis
O F
F O
Pe

f
f

Sinar cahaya yang selari dan hampir dengan paksi utama Sinar cahaya yang selari dan hampir dengan paksi utama
terbias ke dalam dan menumpu pada suatu titik, terbias ke luar dan kelihatan mencapah dari F
F selepas merambat melalui kanta selepas merambat melalui kanta
Light rays parallel and close to the principal axis are Light rays parallel and close to the principal axis are
refracted inwards converge refracted outwards diverge
and to F on the principal and appear to from F
axis after passing through the lens after passing through the lens

193
  Fizik  Tingkatan 4  Bab 6 Cahaya dan Optik

2. Istilah umum yang digunakan adalah seperti yang berikut:


The common terms used are as follows:

Istilah Penerangan
Term Explanation

Pusat optik, O (a) Titik tengah kanta


Optical centre, O
The centre point of a lens

Paksi utama Garis yang melalui pusat optik, O


Principal axis The line that passes through the optical centre, O

.
Titik fokus, F (b) • Kanta cembung / Convex lens

hd
Focus point, F menumpu selepas merambat melalui kanta
– Satu titik di mana sinar cahaya selari
A point where the parallel light rays converge to after passing through the lens

.B
• Kanta cekung / Concave lens
– Satu titik di mana sinar cahaya selari kelihatan mencapah selepas merambat melalui
kanta

dn
A point where the parallel light rays appear to diverge from after passing through the
lens

iS
Panjang fokus, f (c) Jarak di antara pusat optik, O dengan fokus utama, F
Focal length, f
optical centre, O principal focus, F
The distance between the ng and the

Jarak objek, u Jarak di antara objek dengan pusat optik, O suatu kanta
Object distance, u The distance between the object and the optical centre, O of a lens

Jarak imej, v Jarak di antara imej dengan pusat optik, O suatu kanta
la
Image distance, v The distance between the image and the optical centre, O of a lens
Pe

Kuasa Kanta
Power of Lenses
BAB

1. Kuasa kanta ditakrifkan sebagai kebolehan untuk membengkokkan


n

suatu sinar cahaya yang memasuki


6 kanta itu. Kuasa kanta diukur dalam unit diopter (D) .
ita

The power of a lens is defined as the ability to bend a light ray entering it. The power of a lens is measured in
dioptre (D) .
rb

2. Kuasa suatu kanta adalah berkadar songsang dengan panjang fokus, f dalam meter. Kuasa boleh ditunjukkan
ne

secara matematik sebagai:


Power of a lens is inversely proportional to the focal length, f in metres. It can be mathematically shown as:
Pe

1
Kuasa kanta =
panjang fokus
1
Power of a lens =
focal length
1
atau / or P=
f
Semakin tinggi kuasa suatu kanta, semakin pendek panjang fokus kanta itu dan sebaliknya.

The higher the power of a lens, the shorter its focal length and vice versa.

194
  Fizik  Tingkatan 4  Bab 6 Cahaya dan Optik 

3. Bagi kanta cembung, kuasa kanta yang lebih tinggi akan menumpukan sinar cahaya pada sudut yang
lebih besar . Kuasa bagi kanta cembung diambil sebagai positif .
For a convex lens, the higher power lens will converge light ray at a bigger angle. The power of a convex lens

is taken to be positive .

O F O F

.
hd
f f

.B
Kanta cembung kuasa tinggi Kanta cembung kuasa rendah
High power convex lens Low power convex lens

4. Semakin tebal pusat suatu kanta cembung, semakin pendek panjang fokus kanta itu.

dn
The thicker the centre of a convex lens, the shorter the focal length of the lens.

5. Bagi kanta cekung, kuasa kanta yang lebih tinggi akan mencapah sinar cahaya pada sudut yang

iS
lebih besar . Kuasa bagi kanta cekung diambil sebagai negatif .
For a concave lens, the higher power lens will diverge light ray at a bigger angle. The power of a concave lens
ng
is taken to be negative .
la
Pe

O O

BAB
n

f f
6
ita

Kanta cekung kuasa tinggi Kanta cekung kuasa rendah


High power concave lens Low power concave lens
rb

6. Semakin nipis pusat suatu kanta cekung, semakin pendek panjang fokus kanta itu.
The thinner the centre of a concave lens, the shorter the focal length of the lens.
ne

Mengganggar Panjang Fokus bagi Suatu Kanta Cembung Menggunakan Objek Jauh
Estimate Focal Length for a Convex Lens Using Distant Object
Pe

Panjang fokus kanta cembung boleh dianggarkan


seperti berikut. Kanta cembung
The focal length of a convex lens can be determined as Convexs lens
Skrin
follows. Screen
1. Kanta cembung dipasang pada pemegang
kanta dan diletakkan di atas meja seperti
yang ditunjukkan dalam rajah. Pemegang kanta
The convex lens is mounted on a holder and Lens holder
placed on a table as shown in the figure.

195
  Fizik  Tingkatan 4  Bab 6 Cahaya dan Optik

2. Kanta diselaraskan untuk menghala ke objek jauh.


The lens is adjusted to face a distant object.

3. Kedudukan skrin diselaraskan sehingga imej terbalik tajam terbentuk pada skrin.
The position of the screen is adjusted until a sharp inverted image is formed on the screen.

4. Jarak skrin dari pusat optik kanta diukur. Jarak ini ialah panjang fokus yang dianggarkan, f.
The distance of the screen from the optical centre of the lens is measured. This distance is the estimated focal length, f.

Imej yang Dibentuk oleh Kanta


Image Formed by Lenses

.
hd
1. Kanta cembung
Convex lens
(a) Tiga peraturan untuk melukis gambar rajah sinar ditunjukkan di bawah. Lengkapkan rajah-rajah di

.B
bawah.
The three rules to draw a ray diagram are shown below. Complete the diagrams below.

dn
Peraturan Sinar selari dengan paksi utama terbias untuk
Rule melalui F
A ray parallel to the principal axis is refracted to

iS
① pass through F

F O ngF
la

Peraturan Sinar yang melalui F terbias selari dengan


Pe

Rule paksi utama


A ray passing through F is refracted parallel to the
② principal axis
BAB

F O
n

6
ita
rb

Peraturan Sinar yang melalui O tidak dibiaskan dan


Rule merambat pada garis lurus
ne

A ray passing through O is not refracted and travels


③ in a straight line
F O F
Pe

196
  Fizik  Tingkatan 4  Bab 6 Cahaya dan Optik 

(b) Ciri-ciri imej yang dibentuk oleh kanta ialah:


Characteristics of images formed by the lens are:

Kedudukan
objek Gambar rajah sinar Ciri-ciri imej Aplikasi
Position of Ray diagram Characteristics of image Application
object

Di antara F • v > f Kanta pembesar


dengan O Magnifying lens
• Maya / Virtual
Between F

.
and O • Tegak / Upright

hd
• Dibesarkan / Magnified
u<f O
F F
• Berada di sebelah yang

.B
sama dengan objek
same
On the side as

dn
the object

Pada F • v = ∞ (Infiniti / Infinity) Eyepiece of

iS
At F Maya / Virtual telescope
• Kanta mata
u=f • Tegak / Upright teleskop

Dibesarkan / Magnified
O
ng •
F F
• Berada di sebelah yang
sama dengan objek
la
On the same side as
the object
Pe

Di antara F • v > 2f Kanta objek

BAB
dengan 2F Nyata / Real mikroskop
Between F • Objective lens of
n

and 2F • Songsang / Inverted microscope

6
ita

• Dibesarkan / Magnified
f < u < 2f 2F F O F 2F
• Berada di sebelah
bertentangan dengan
rb


objek
Opposite
ne

side of the object

Pada 2F • v = 2f Mesin fotostat


Photocopying
Pe

At 2F Nyata / Real
• machine
u = 2f
• Songsang / Inverted
• Sama saiz / Same size
2F F O F 2F • Berada di sebelah
bertentangan dengan
objek (pada 2F)
Opposite
side of the
object (at 2F)

197
  Fizik  Tingkatan 4  Bab 6 Cahaya dan Optik

Jauh dari 2F • f < v < 2f Kanta kamera


Further than Lens of camera
• Nyata / Real
2F
• Songsang / Inverted
u > 2f • Dikecilkan / Diminished
• Berada di sebelah
2F F O F 2F
bertentangan dengan
objek (di antara F dengan
2F)

.
Opposite

hd
side of the
object (between F and 2F)

Di infiniti • v < u Kanta objek

.B
At infinity teleskop
• Nyata / Real
Objective lens of
u=∞ • Songsang / Inverted telescope

dn
• Dikecilkan / Diminished
F O F
• Berada di sebelah
bertentangan dengan

iS
objek (pada F)
Opposite side of the object
(at F)
ng
2. Kanta cekung
la
Concave lens
(a) Tiga peraturan untuk melukis gambar rajah sinar ditunjukkan di bawah. Lengkapkan rajah berikut.
Pe

The three rules to draw a ray diagram are shown below. Complete the diagrams below.

Peraturan Sinar selari dengan paksi utama


BAB

Rule terbias dan kelihatan datang dari F


A ray parallel to the principal axis is
n

① refracted and appears to come from F


6
ita

F O
rb

Peraturan Sinar yang melalui F terbias selari


Rule dengan paksi utama
ne

A ray passing through F is refracted


② parallel to the principal axis
O F
Pe

Peraturan Sinar yang melalui O tidak dibiaskan


Rule dan merambat pada garis lurus
③ A ray passing through O is not refracted
and travels in a straight line
F O F

198
  Fizik  Tingkatan 4  Bab 6 Cahaya dan Optik 

(b) Imej yang dibentuk oleh kanta ini tidak bergantung pada kedudukan objek. Jadual di bawah
menunjukkan ciri-ciri imej dan aplikasi kanta. Lengkapkan rajah berikut.
The image formed by this lens is not dependent on the position of the object. The table below shows the characteristics
of the image and the applications of the lens. Complete the following diagram.

Gambar rajah sinar Ciri-ciri imej


Ray diagram Characteristics of image

• Maya / Virtual

• Tegak / Upright

.
• Dikecilkan / Diminished

hd
F Image O sama
Imej • Berada di sebelah yang di antara objek dengan
kanta
same

.B
On the side between the object and the lens

Aplikasi
Applications

dn
• Cermin mata untuk membetulkan penglihatan rabun jauh atau miopia
Spectacles to correct short-sightedness or myopia

iS
• Lubang intip pada pintu untuk melihat objek di luar pintu
Peepholes in doors to see object outside the door
• Bersama dengan kanta cembung untuk mengatasi kesan aberasi, iaitu sejenis kecacatan dalam kanta
Together with a convex lens to overcome aberration, which is a type of defect in lenses
ng
Pembesaran Linear
la
Linear Magnification
Pe

1. Saiz imej bergantung pada saiz objek. Untuk mengira pembesaran linear, kita boleh membandingkan saiz imej
dengan saiz objek.

BAB
Size of image is dependent on the size of object. To calculate the linear magnification, we can compare the size of image to the
n

size of object.

6
ita

2. Pembesaran linear untuk kanta boleh diwakilkan secara matematik sebagai:


Linear magnification for lenses can be represented mathematically as:
Saiz imej
= Tinggi imej
rb

Pembesaran linear =
Saiz objek Tinggi objek
ne

Size of image Height of image


Linear magnification = =
Size of object Height of object
hi
m=
Pe

ho

Nota / Note:
Pembesaran linear tidak mempunyai unit kerana merupakan suatu nisbah.
Linear magnification does not have unit since it is a ratio.

199
  Fizik  Tingkatan 4  Bab 6 Cahaya dan Optik

3. Nilai m akan menunjukkan saiz imej: / The value of m will indicate the size of the image:
saiz imej = saiz objek
m=1
size of image size of object

saiz imej lebih besar daripada saiz objek


m>1
size of image is bigger than size of object

saiz imej lebih kecil daripada saiz objek


m<1
smaller

.
size of image is than size of object

hd
4. Pembesaran linear juga boleh dihitung dengan menggunakan jarak imej dan jarak objek:
Linear magnification can also be calculated using image distance and object distance:

.B
Jarak imej Image distance
Pembesaran linear = Linear magnification =
Jarak objek Object distance
v

dn
m=
u

iS
Formula Kanta Nipis
6.4 Thin Lens Formula
ng
Hubungan antara u, v dan f
Eksperimen 6.3 Relationship between u, v and f
la

Tujuan / Aim:
Pe

Mendapatkan hubungan antara jarak objek , u, jarak imej , v dan jarak fokus , f untuk suatu kanta
BAB

To find the relationship between the object distance , u, image distance , v and the focal length , f of a lens
n

Radas / Apparatus:
6
ita

Kanta cembung (f = 20 cm), mentol, bateri, pemegang kanta, pembaris meter dan skrin putih
Convex lens (f = 20 cm), light bulb, battery, lens holder, metre rule, and white screen

Susunan radas / Arrangement of apparatus:


rb

Kanta
Lens Skrin
ne

Mentol Screen
Light bulb

Bateri
Pe

Battery

      
Prosedur / Procedure:
1. Radas disusun seperti yang ditunjukkan dalam rajah di atas.
The apparatus is set up as shown in the above diagram.
2. Mentol diletakkan pada jarak 25 cm (u = 25 cm) dari pusat optik kanta itu.
The light bulb is placed at a distance of 25 cm (u = 25 cm) from the optical centre of the lens.

200
  Fizik  Tingkatan 4  Bab 6 Cahaya dan Optik 

3. Skrin digerakkan ke hadapan dan ke belakang sehingga satu imej yang tajam terbentuk padanya.
The screen is moved forward and backward until a sharp image is formed on it.

4. Jarak di antara pusat optik kanta dengan skrin (jarak imej, v) diukur dan dicatatkan.
The distance from the optical centre of the lens to the screen (image distance, v) is measured and recorded.

5. Langkah 2 hingga 4 diulang untuk jarak objek, u = 30 cm, 35 cm, 40 cm, 45 cm dan 50 cm. Semua nilai jarak
imej, v yang sepadan diukur dan dicatatkan.
Steps 2 to 4 are repeated for object distance, u = 30 cm, 35 cm, 40 cm, 45 cm and 50 cm. All the corresponding values of the
image distance, v are measured and recorded.

Penjadualan data / Tabulation of data:

.
hd
Jarak objek, u Jarak imej, v 1 1 1 1
Object distance, u Image distance, v u v u +v
(cm) (cm)

.B
25 100.0 0.040 0.010 0.050

30 58.8 0.033 0.017 0.050

dn
35 47.6 0.029 0.021 0.050

iS
40 40.0 0.025 0.025 0.050

45 35.7 0.022 ng 0.028 0.050

50 33.3 0.020 0.030 0.050

Analisis data / Analysis of data:


la

v (cm) 1
v (cm )
–1
Pe

BAB
n

u (cm) 1
u (cm )
–1

         6
ita

Perbincangan / Discussion:
1 1
1. Nilai u + v adalah tetap. Nilai ini sama dengan nilai 1 .
rb

f
1 1 1
The value of   + is constant. The value is equal to .
ne

u v f
2. Apabila jarak objek, u bertambah, jarak imej, v berkurang.
As the object distance, u increases, the image distance, v decreases.
Pe

Kesimpulan / Conclusion:
Hubungan antara jarak objek, u, jarak imej, v, dan jarak fokus, f untuk suatu kanta ialah
The relationship between the object distance, u, the image distance, v and the focal length, f of a lens is
1 1 1
= +
f u v

201
  Fizik  Tingkatan 4  Bab 6 Cahaya dan Optik

Tugasan 3
1. Panjang fokus suatu kanta cembung ialah 20 cm. Berapakah kuasa kanta itu?
The focus length of a convex lens is 20 cm. What is the power of the lens?
1
Kuasa kanta / Power of a lens, P =
f
1
= = +5 D
0.2

2. Panjang fokus suatu kanta cekung ialah 10 cm. Berapakah kuasa kanta itu?

.
hd
The focus length of a concave lens is 10 cm. What is the power of the lens?

1
Kuasa kanta / Power of a lens, P =
f

.B
1
= = –10 D
–0.1

3. Rajah menunjukkan kedudukan imej suatu objek yang dibentuk oleh

dn
30 cm
satu kanta cembung. Cari tinggi imej, h.
The diagram shows the position of image of an object formed by a convex lens. 9 cm

iS
Find the height of the image, h.
Objek Imej
h v Object
h cm
Image
= m=
9 u ng
h 10 40 cm
=
9 30
h = 3 cm
la

4. Suatu objek dengan ketinggian 8 cm diletakkan 20 cm dari satu kanta cekung. Suatu imej terbentuk di sebelah yang
Pe

sama dengan kanta. Jika ketinggian imej ialah 2 cm dan terbentuk pada jarak imej v cm dari kanta cekung itu, cari
An object with a height of 8 cm is placed 20 cm from a concave lens. An image is formed on the same side as the object. If the image is 2 cm
BAB

in height and is formed at a distance of v cm from the concave lens, find


(a) pembesaran linear, m (b) jarak imej, v
n

the linear magnification, m the image distance, v


6
ita

hi 2 v h
m= = = 0.25 m = = i
ho 8 u ho
v
= 0.25
rb

20
v = 5 cm
ne

5. Pembentukan imej suatu objek ditunjukkan dalam rajah. Jika jarak imej itu ialah
4 cm dan panjang fokus ialah 10 cm, hitung jarak objek.
The formation of the image of an object is shown in the diagram. If the image distance is 4 cm and
Pe

the focal length is 10 cm, calculate the object distance.

1 1 1 F Imej O
+ = Objek
u v f Object Image
1 1 1
+ =
u –4 –10
1 1 1 3
= + =
u –10 4 20
u = 6.67 cm

202
  Fizik  Tingkatan 4  Bab 6 Cahaya dan Optik 

6. Suatu objek dengan ketinggian 5 cm diletakkan 12 cm dari satu kanta cembung yang mempunyai panjang fokus 4 cm.
Tentukan
An object of height 5 cm is placed 12 cm from a convex lens of focal length 4 cm. Determine
(a) kedudukan imej yang terbentuk, (b) ciri-ciri imej yang terbentuk.
the position of the image formed, the characteristics of the image formed.
1 1 1 v h
u + v = f = h i
u o
1 1 1 6 hi
12 + v = 4 = 5
12
1 1 1 2 hi = 2.5 cm
= – =

.
v 4 12 12
∴ Imej adalah songsang, nyata dan dikecilkan

hd

v = 6 cm The image is inverted, real and diminished in size
7. Berdasarkan graf, berapakah panjang fokus bagi kanta yang digunakan?

.B
Based on the graph, what is the focal length of the lens used?
1 1 1 1
u + v = f v (cm )
–1

dn
1
Pada pintasan-x / At the x-intercept, v = 0 0.05
1 1
=
u f

iS
1
0.05 = 1
f u (cm )
–1

0.05
f = 20 cm
ng
8. Suatu objek dengan ketinggian 5 cm diletakkan pada jarak u cm dari satu kanta cembung yang mempunyai panjang
fokus 10 cm. Tentukan
An object with a height of 5 cm is placed at a distance of u cm from a convex lens of focal length 10 cm. Determine
la
(a) jarak objek jika imej yang terbentuk ialah 12 cm di (b) pembesaran linear,
Pe

belakang kanta, the linear magnification,


the object distance if the image formed is 12 cm behind the lens, v 12 1
m = = =
u 60 5

BAB
1 1 1
+ =
u v f (c) ketinggian imej itu.
n

1 1 1 the height of the image.


+ =
u 12 10 6
ita

h
1 1 1 2 m = h i
= – =
u 10 12 120 o
1 h
u = 60 cm
rb

= i
5 5
hi = 1 cm
ne

Peralatan Optik
6.5 Optical Instruments
Pe

1. Kanta pembesar (mikroskop ringkas) / Magnifying lens (simple microscope)


(a) Sebuah kanta pembesar terdiri daripada satu kanta cembung .
A magnifying lens consists of a single convex lens .

(b) Objek perlu diletakkan di antara fokus utama , F dengan


Imej F Objek O F
pusat optik , O.
Image Object
The object needs to be placed between principal focus , F and
optical centre , O.

203
  Fizik  Tingkatan 4  Bab 6 Cahaya dan Optik

(c) Sinar cahaya daripada objek tidak bertemu selepas melalui kanta tetapi akan memintas di hadapan objek.
The light ray from the object does not meet after passing through the lens but will intersect in front of the object.

(d) Imej yang terbentuk di hadapan kanta adalah tegak , maya dan dibesarkan .
The image formed in front of the lens is upright , virtual and magnified .

(e) Kuasa pembesar meningkat jika kuasa kanta lebih tinggi .


The magnifying power increases if the power of the lens is higher .

2. Mikroskop majmuk / Compound microscope


(a) Sebuah mikroskop majmuk digunakan untuk melihat objek yang sangat kecil seperti mikroorganisma  .

.
hd
A compound microscope is used to view very small objects like microorganisms .

(b) Mikroskop majmuk dibina dengan dua kanta cembung berkuasa tinggi sebagai kanta objek
kanta mata

.B
dan .
The microscope is built with two high power convex lenses as objective lens and eyepiece .

(c) Kanta objek dengan kuasa yang lebih tinggi berada lebih dekat dengan objek .

dn
Objective lens with a higher power is nearer to the object .

(d) Kanta mata dengan kuasa yang lebih rendah Kanta mata

iS
berada lebih dekat dengan mata . Kanta objek
Eyepiece

lower Objective lens L > f + f


Eyepiece with a power is nearer to the o e
eye .
ng ho Fe

(e) Jarak di antara dua kanta itu, L lebih besar sedikit Fo O Fe h1


Fo
daripada jumlah jarak panjang fokus bagi kanta objek h2
la
dan kanta mata (L . fo + fe).
The distance of the two lenses, L is slightly bigger
Pe

than the total distance of the focal length of objective lens and
eyepiece (L . fo + fe).
BAB

(f) Suatu objek diletakkan di antara Fo dan 2Fo untuk menghasilkan imej pertama yang
n

nyata , songsang dan dibesarkan .


6
ita

The object is placed between Fo and 2Fo to produce the first image which is real ,
inverted and magnified .
rb

(g) Imej pertama, I1 akan jatuh di antara pusat optik , O dengan titik fokus kanta mata, Fe. Imej
pertama sekarang menjadi objek bagi kanta mata.
ne

The first image, I1 will fall between the optical centre , O and the focal point of the eyepiece, Fe. The first
image now becomes the object for the eyepiece.
Pe

(h) Kanta mata menggunakan prinsip kerja kanta pembesar. Imej akhir, I2 yang dihasilkan adalah maya  ,
songsang dan dibesarkan dengan objek asal.
The eyepiece uses the work principle of a magnifying glass. The final image, I2 produced is virtual ,
inverted and magnified to the original object.
(i) Pembesaran bagi mikroskop majmuk adalah bergantung pada pembesaran bagi kanta objek dan kanta
mata.
Magnification of a compound microscope is dependent on the magnification of objective lens and eyepiece.

204
  Fizik  Tingkatan 4  Bab 6 Cahaya dan Optik 

Jika / If
h0 = tinggi objek / height of object
h1 = tinggi imej pertama / height of first image
h2 = tinggi imej kedua / height of second image
Pembesaran linear bagi kanta objek / Linear magnification of objective lens,
h
mo = 1 ,
h0
Pembesaran linear bagi kanta mata / Linear magnification of eyepiece,
h
me = 2 ,

.
h1

hd
∴ Pembesaran linear bagi mikroskop majmuk, mT
Linear magnification of compound microscope, mT
h1 h h

.B
= mo × me = × 2 = 2
h0 h1 h0
3. Teleskop / Telescope
planet bintang  .

dn
(a) Sebuah teleskop astronomi digunakan untuk melihat objek yang jauh seperti dan
A telescope is used to view faraway objects like planets and stars .

iS
(b) Teleskop ini dibina dengan dua kanta cembung yang berbeza kuasa.
The telescope is built with two convex lenses of different power.

(c) Kanta objek dengan kuasa yang rendah lebih dekat dengan objek. Kanta mata
Objective lens with a low
ng
power is nearer to the object.
Eyepiece
Kanta objek
Objective lens L = f + f
(d) Kanta mata dengan kuasa yang tinggi lebih o e
la
dekat dengan mata. FoFe Fe
high O I1
Eyepiece with a power is nearer to the eye.
Pe

(e) Jarak di antara dua kanta itu, L adalah sama

BAB
dengan jumlah panjang fokus bagi kanta objek
dan kanta mata (L = fo + fe).
n

The distance of the two lenses, L is equal the total distance of focal length of objective lens and eyepiece (L = fo + fe).
6
ita

(f) Sinar cahaya selari daripada objek yang jauh menumpu pada fokus utama, Fo kanta objek untuk
membentuk imej pertama, I1 yang nyata  , songsang dan dikecilkan .
rb

Parallel light rays from the faraway object converge at the principal focus, Fo of the objective lens to form
the first image, I1 which is real  , inverted and diminished in size .
ne

(g) Imej pertama, I1 sekarang menjadi objek bagi kanta mata. Imej pertama berada pada fokus utama kanta
mata.
The first image, I1 now becomes the object for the eyepiece. The first image is at the principal focus of the eyepiece.
Pe

(h) Imej akhir, I2 terbentuk di infiniti dan adalah maya  , songsang dan dibesarkan .
The final image, I2 is formed at infinity and is virtual , inverted and magnified .
(i) Pembesaran linear bagi sebuah teleskop dihitung seperti yang berikut:
Linear magnification of a telescope is calculated as follows:

m= Kuasa kanta mata = Pe atau m = Panjang fokus kanta objek = fo


Kuasa kanta objek Po Panjang fokus kanta mata fe
Power of eyepiece P Focal length of objective lens f
m= = e = or m = = o
Power of objective lens Po Focal length of eyepiece fe

205
  Fizik  Tingkatan 4  Bab 6 Cahaya dan Optik

Perbandingan antara mikroskop majmuk dengan teleskop astronomi


Comparison between a compound microscope and an astronomical telescope

Alat optik Mikroskop majmuk Teleskop


Optical device Compound microscope Telescope

Dua kanta cembung: satu berkuasa tinggi


Jenis kanta Dua kanta cembung berkuasa tinggi dan satu berkuasa rendah
Type of lens Two high powered convex lenses high powered
Two convex lenses: one and
one low powered

.
hd
Kuasa kanta objek Kanta kuasa lebih tinggi Kanta kuasa lebih rendah
Power of objective
Higher power lens Lower power lens
lens

.B
Kuasa kanta mata Kanta kuasa lebih rendah Kanta kuasa lebih tinggi
Power of eyepiece Lower power lens Higher power lens

dn
Panjang fokus
fe . fo fo . fe
Focal length

Jarak di antara

iS
Lebih besar sedikit daripada fo + fe Sama dengan fo + fe
kanta
Slightly bigger than fo + fe Equal to fo + fe
Distance of lenses ng
Nyata , songsang dan
Imej pertama Nyata , songsang dan dikecilkan
dibesarkan
First image Real , inverted and diminished in size
la
Real , inverted and magnified

Maya songsang dan dibesarkan (merujuk pada objek asal)


Pe

Imej akhir ,
Final image Virtual , inverted and magnified (referring to original object)
BAB

Pembesaran linear P f
m = mo × me m = Pe atau / or m = e
n

Linear magnification o fo

6
ita

Aplikasi Kanta Bersaiz Kecil dalam Teknologi Peralatan Optik


Application of Small Lenses in Optical Instrument Technology
rb

1. Projektor slaid / Slide projector Mentol projektor


ne

Projection lamp
(a) Projektor slaid ialah satu alat optik yang
menggunakan kanta cembung untuk Kondenser Kanta projektor
Condenser Projector lens
membentuk imej slaid atau filem yang nyata,
Pe

songsang dan dibesarkan pada satu skrin.


A slide projector is an optical device using
convex lenses
to form a real, inverted and
magnified
image of the slide or film on a screen. Imej nyata, songsang
Objek / Slaid
Cermin cekung Object / Slide dan dibesarkan
(b) Mentol projektor diletakkan pada Concave mirror Real, inverted
pusat lengkungan cermin cekung. and magnified image

The projection lamp is placed at the centre of curvature of the concave mirror.

206
  Fizik  Tingkatan 4  Bab 6 Cahaya dan Optik 

(c) Cermin cekung digunakan untuk memantulkan semua cahaya daripada mentol balik ke kondenser.
The concave mirror is used to reflect all the light from the lamp back to the condenser.

(d) Kondenser mengandungi dua kanta plano cembung. Semua cahaya akan merebak pada slaid selepas
merambat melalui kanta-kanta itu.
The condenser consists of two plano-convex lenses. All the light will be spread out onto the slide after passing through
the lenses.
(e) Slaid diletakkan di antara F dengan 2F kanta projektor. Hal ini akan menghasilkan imej
yang dibesarkan .
F 2F magnified

.
The slide is placed between and of the projection lens. This will produce a image.

hd
(f) Slaid perlu diletakkan terbalik supaya dapat membentuk imej yang tegak pada skrin. Imej yang
terbentuk adalah nyata , tegak dan dibesarkan .

.B
The slide needs to be put upside down in order to form an upright image on the screen. The image formed is
real upright magnified
, and .

dn
(g) Kanta projektor boleh anjak dapat diselaraskan sehingga satu imej yang tajam terbentuk pada skrin.
The movable projector lens can be adjusted until a sharp image is formed on the screen.

iS
2. Kamera kanta tunggal / Single lens camera
(a) Kamera terdiri daripada sebuah kanta cembung Gelang fokus
yang digunakan untuk menangkap imej pada Kanta Focusing ring
filem.
ng Lens
Filem
convex lens Film
A camera consists of a which is used
Objek
la
to capture the image on film. Imej yang nyata,
Object songsang dan
(b) Oleh sebab filem bersaiz kecil, imej yang dibentuk Diafragma dikecilkan
Pe

oleh kanta cembung mestilah nyata , Diaphragm Pengatup Real, inverted and
diminished in size
Shutter
songsang dan dikecilkan . image

BAB
Since the film is small in size, the image formed by the Jarak > 2f
Distance > 2f
n

convex lens must be real , inverted and


diminished in size .
6
ita

(c) Objek mesti diletakkan pada jarak lebih daripada dua kali panjang fokus ( > 2fo ) kanta
kamera untuk membentuk imej tersebut.
rb

The object must be placed at a distance of more than twice the focal length ( > 2fo ) of the camera
lens to form the image.
ne

(d) Gelang fokus digunakan untuk memfokuskan imej pada filem dengan menyelaraskan jarak di
antara kanta dengan filem.
Pe

Focusing ring is used to focus the image on the film by adjusting the distance between lens and film.

(e) Diafragma digunakan untuk mengawal saiz bukaan, yang menentukan jumlah cahaya
yang memasuki kamera.
The diaphragm is used to control the size of aperture, which determines the amount of light that enters
the camera.

(f) Kelajuan pengatup mengawal panjang masa pembukaan , maka bahagian ini digunakan untuk
mengawal masa pendedahan semasa mengambil gambar.
The shutter speed controls the length of time of the opening , thus is used to control the exposure time
when taking photographs.

207
  Fizik  Tingkatan 4  Bab 6 Cahaya dan Optik

Pembentukan Imej oleh Cermin Sfera


6.6 Image Formation by Spherical Mirror

1. Cermin melengkung ialah cermin dengan permukaan pantulan melengkung yang berbentuk seperti sebahagian
daripada sfera kaca.
A curved mirror is a mirror with a curved reflective surface which is shaped like part of a glass sphere.
2. Pantulan cahaya pada cermin melengkung seperti ditunjukkan dalam rajah. Lengkapkan rajah berikut.
Reflections of light on curved mirrors are as shown in the diagram. Complete the following diagram.

.
hd
Paksi utama
P F C Principal axis C F P

.B
f f

dn
Cermin cembung Cermin cekung
Convex mirror Concave mirror

3. Istilah umum bagi cermin melengkung diterangkan dalam jadual di bawah.

iS
Common terms of curved mirrors are explained in the table below.

Istilah / Term ng Penerangan / Explanation


Pusat lengkungan, C (a) Pusat sfera cermin melengkung
Centre of curvature, C Centre of the sphere
of the curved mirror
la
Kutub cermin, P (b) Titik tengah di permukaan cermin melengkung
Pole of mirror, P central point
The on the surface of the curved mirror
Pe

Paksi utama (c) Garis yang melalui pusat lengkungan, C dan kutub cermin, P
Principal axis
BAB

The line passing through the centre of curvature, C to the pole of mirror, P
n

Jejari lengkungan, r (d) Jarak di antara kutub cermin, P dengan pusat lengkungan, C
Radius of curvature, r
pole of mirror, P centre of curvature, C
6
ita

The distance between the and the

Fokus utama, F (e) • Cermin cembung / Convex mirror


Principal focus, F – Titik pada paksi utama di mana semua sinaran pantulan kelihatan mencapah
rb

dari belakang cermin (fokus maya)


The point on the principal axis where all the reflected rays appear to diverge from
ne

behind the mirror (virtual focus)


• Cermin cekung / Concave mirror
– Titik pada paksi utama di mana semua sinaran pantulan menumpu di hadapan
Pe

cermin
The point on the principal axis where all the reflected rays converge in front of the mirror

Panjang fokus, f Jarak di antara kutub cermin, P dengan fokus utama, F


Focal length, f The distance between the pole of mirror, P and the principal focus, F

Bukaan cermin (f) Bahagian permukaan cermin yang memantulkan cahaya


Aperture
The portion of the surface of the mirror which reflects light

Jarak objek, u (g) Jarak di antara objek dengan kutub cermin, P


Object distance, u
The distance between the object and the pole of mirror, P

208
  Fizik  Tingkatan 4  Bab 6 Cahaya dan Optik 

4. Terdapat tiga sinar asas yang perlu dilukis dalam melukis gambar rajah sinar bagi cermin melengkung.
There are three basic rays that need to be drawn in drawing ray diagrams for curved mirrors.

Sinar pantulan
Sinar Sinar tuju Reflection ray
Ray Incident ray Cermin cembung Cermin cekung
Convex mirror Concave mirror

1 Satu sinar cahaya Dipantulkan seolah-olah datang dari F Dipantulkan melalui F


selari dengan Is reflected as if it comes from F Is reflected through F
paksi utama

.
1 1
A ray parallel to

hd
the principal axis

.B
P F C C F P

dn
Sinar pantulan
Sinar Sinar tuju Reflection ray

iS
Ray Incident ray Cermin cembung Cermin cekung
Convex mirror
ng Concave mirror

2 Satu sinar cahaya Dipantulkan selari dengan paksi utama Dipantulkan selari dengan paksi utama
melalui F Is reflected parallel to the principal axis Is reflected parallel to the principal axis
A ray through F
la
2
2
Pe

C F P

BAB
P F C
n

6
ita

3 Satu sinar cahaya Dipantulkan mengikut lintasan asal Dipantulkan mengikut lintasan asal
rb

melalui C Is reflected along the original path Is reflected along the original path
A ray through C
3
ne

3
Pe

C F P
P F C

209
  Fizik  Tingkatan 4  Bab 6 Cahaya dan Optik

5. Ciri-ciri imej yang dibentuk oleh cermin cembung adalah sama untuk kedudukan jarak objek yang berbeza.
Lengkapkan rajah berikut.
The characteristics of the image formed by a convex mirror are the same for different positions of the object. Complete the
following diagrams.

Imej
Imej Image
Objek Image Objek
Object Object

F P F C F P F C

.
hd
     

.B
• Imej yang dibentuk oleh cermin cembung adalah sentiasa maya , tegak dan dikecilkan
serta di belakang cermin.

dn
The image formed by a convex mirror is always virtual , upright and diminished , and behind the
mirror.

iS
• v = negatif / negative

6. Ciri-ciri imej yang dibentuk oleh cermin cekung berdasarkan kedudukan yang berbeza bagi suatu objek
ditunjukkan dalam jadual yang berikut. Lengkapkan semua rajah berikut.
ng
The characteristics of image formed by a concave mirror based on different positions of an object are shown in the following
table. Complete all the following diagrams.
la

Kedudukan objek Gambar rajah sinar Kedudukan imej Ciri-ciri imej


Pe

Position of object Ray diagram Position of image Characteristics of image

Objek, O di antara Imej di belakang • Maya / Virtual


BAB

F dengan P cermin
Object, O is Image is behind • Tegak / Upright
n

O
between F and P the mirror
• Dibesarkan / Magnified
6
ita

u<f C F P v = negatif • Digunakan sebagai


negative cermin solek dan cermin
pemeriksaan pergigian
rb

Used as make-up mirror


and dental inspection mirror
ne

Objek, O pada F Imej di infiniti • Maya / Virtual


Object, O is at F Image is at infinity
• Tegak / Upright
Pe

u=f O v=∞ • Dibesarkan / Magnified


• Digunakan sebagai
lampu sorot di mana
C F P pemantul menghasilkan
sinar cahaya selari
Used as spotlight where the
reflector produces parallel
light rays

210
  Fizik  Tingkatan 4  Bab 6 Cahaya dan Optik 

Objek, O di antara Imej melebihi • Nyata / Real


F dengan C kedudukan C
Object, O is O Image is beyond • Songsang / Inverted
between F and C C
• Dibesarkan / Magnified
f < u < 2f C F P v > 2f
v>r

.
Objek, O pada C Imej pada C • Nyata / Real

hd
Object, O is at C Image is at C
O • Songsang / Inverted
u = 2f v = 2f • Sama saiz dengan objek

.B
v=r
Same size as the object
C F P
• Digunakan sebagai
pemantul dalam projektor

dn
Used as a reflector in projector

iS
Objek, O Imej di antara C • Nyata / Real
melebihi C dengan F
Object, O is beyond Image is between • Songsang / Inverted
O
C C and F
• Dikecilkan / Diminished
ng
C F P f < v < 2f
1r < v < r
la
2
Pe

Objek di infiniti Image is at F • Nyata / Real

BAB
Object is at infinity Imej pada F
• Songsang / Inverted
n

u=∞
v=f • Dikecilkan / Diminished
6
ita

F 1
v= r
C P 2
rb
ne

Aplikasi Cermin Cekung dan Cermin Cembung dalam Kehidupan Harian


Applications of Concave and Convex Mirrors in Daily Life
Pe

1. Cermin cembung / Convex mirrors 3

(a) Sebuah cermin keselamatan yang dibina daripada cermin cembung


membantu meluaskan medan penglihatan bagi tujuan
keselamatan.
A security mirror made from a convex mirror helps to
widen field of vision for security purposes.

211
  Fizik  Tingkatan 4  Bab 6 Cahaya dan Optik

(b) Sebuah cermin cembung digunakan untuk membantu pemandu melihat


lalu lintas di selekoh tersembunyi .

A convex mirror is used to help drivers to view traffic around blind corners .

(c) Cermin cembung sisi kereta membolehkan pemandu melihat


kenderaan di kedua-dua belah kereta. Sebuah cermin cembung

.
hd
pandang-belakang digunakan untuk membantu pemandu melihat
lalulintas di belakang .
Convex wing mirrors enable the driver to see vehicles on both sides

.B
of the car. A convex rear-view mirror is used to help drivers to view
the traffic at the back .

dn
2. Cermin cekung / Concave mirrors 3

(a) Cermin parabola digunakan dalam lampu suluh dan lampu kereta

iS
sebagai pemantul . Satu lampu kecil diletakkan pada titik fokus
cermin untuk menghasilkan sinaran selari.
reflectors

ng
Parabolic mirrors are used in torches and car headlamps as .
A small lamp is placed at the focus point of the mirror to produce parallel
rays.
la

(b) Cermin cekung dengan jarak fokus yang panjang boleh digunakan sebagai
Pe

cermin cukur atau cermin solek kerana cermin ini membentuk imej yang besar dan
tegak.
BAB

Concave mirrors with long focal lengths can be used as shaving mirrors or make-up
n

mirrors as they form magnified and upright images.

6
ita

(c) Cermin pemeriksaan pergigian digunakan oleh doktor gigi untuk memeriksa
gigi pesakit. Cermin cekung membentuk imej gigi yang dibesarkan .
rb

Dental inspection mirrors are used by dentists to examine the teeth of a patient. The
concave mirror forms a magnified image of the teeth.
ne

(d)
Sebuah teleskop pantulan astronomi menggunakan
Pe

cermin parabola yang besar untuk mengumpul cahaya


malap daripada bintang-bintang yang jauh. Sebuah cermin satah
digunakan untuk memantulkan imej pada kanta mata .
An astronomical reflecting telescope uses a large parabolic mirror to
gather dim light from distant stars. A plane mirror is used to reflect the image
to the eyepiece .

212
  Fizik  Tingkatan 4  Bab 6 Cahaya dan Optik 

PRAKTIS SPM 6
Soalan Objektif

1. Rajah 1 menunjukkan sinar cahaya bergerak melalui A Pantulan


suatu kanta cembung. Reflection
2016
Diagram 1 shows light rays passing through a convex lens. B Pembiasan
Refraction

.
C Pembelauan

hd
Diffraction
A B C D D Pantulan dalam penuh
Total internal reflection

.B
4. Rajah manakah yang menunjukkan laluan cahaya
Rajah 1 / Diagram 1
yang betul apabila cahaya merambat melalui dua
2016

dn
Antara titik A, B, C atau D, yang manakah adalah titik lapisan udara yang berbeza suhu?
fokus? Which diagram shows the correct light path when light
Which point A, B, C or D, is the focal point? propagates through two layers of air with different

iS
temperature?
2. Rajah 2 menunjukkan sinar cahaya yang bergerak dari A C
udara ke dalam kaca dan sebaliknya. Udara panas Udara panas
2015 Hot air Hot air
Diagram 2 shows light rays travelling from air into glass and
vice versa.
ng
Udara sejuk
Cold air
Udara sejuk
Cold air
la
C
D B D
B
Pe

Udara panas
Udara panas Hot air
A Hot air

BAB
Udara sejuk
Cold air Udara sejuk
Cold air
n

Rajah 2 / Diagram 2
6
ita

Yang manakah sudut adalah genting kaca? 5. Rajah 4 menunjukkan sinar cahaya ditujukan ke
Which angles is the critical angle of the glass? dalam satu bongkah kaca lut sinar.
2015
Diagram 4 shows a ray of light entering a transparent glass
rb

3. Rajah 3 menunjukkan satu gentian optik dan cahaya 2013


block.
merambat di dalam gentian optik.
2015
Diagram 3 shows an optical fibre and light propagates inside
ne

38º
optical fibre.
48º
Bongkah
kaca lut sinar
Lapisan luar Teras dalam
Pe

Transparent
Outer core Inner core glass block

Rajah 4 / Diagram 4
Berapakah indeks biasan bongkah kaca itu?
Rajah 3 / Diagram 3 What is the refractive index of the glass block?
Fenomenon cahaya manakah menerangkan A 0.83
perambatan cahaya dalam gentian optik? B 1.18
Which light phenomenon explain the light propagation inside C 1.21
optical fibre? D 1.58

213
  Fizik  Tingkatan 4  Bab 6 Cahaya dan Optik

6. Rajah 5 menunjukkan seorang budak memancing 9. Rajah 7 menunjukkan seorang budak lelaki dengan
ikan di kolam. Ikan kelihatan lebih dekat apabila imejnya dalam cermin satah.
2016 2016
dilihat dari atas permukaan air. Diagram 7 shows a boy with his image in a plane mirror.
Diagram 6 shows a boy fishing in the pond. The fish appears to
be closer when viewed from the surface of the water.

Rajah 7 / Diagram 7
Pasangan manakah yang betul untuk menghasilkan

.
satu imej budak yang nyata, besar dan songsang?

hd
Which pair is correct to produce real, magnified and an
inverted image of the boy?
Rajah 5 / Diagram 5 Kedudukan budak

.B
Alasan manakah yang betul untuk menerangkan Jenis cermin The position of boy
situasi tersebut? Type of mirror f = panjang fokus cermin
f = focal length of mirror
Which reason is correct to explain this situation?

dn
A Indeks biasan udara < indeks biasan air A Cembung u>f
Convex
The refractive index of air < the refractive index of water
B Laju cahaya dalam udara < laju cahaya dalam air B Cembung u<f

iS
Convex
The speed of light in air < the speed of light in water
C Ketumpatan udara > ketumpatan air C Cekung f < u < 2f
Concave
The density of air > the density of water
D Keamatan cahaya dalam udara < keamatan
ng D Cekung u < 2f
Concave
cahaya dalam air
The intensity of light in air < the intensity of light in water
la
10. Rajah 8 menunjukkan satu objek di hadapan suatu
cermin satah.
7. Satu objek diletakkan 10 cm di hadapan sebuah kanta 2012
Diagram 1 shows an object in front of a plane mirror.
Pe

cembung dengan panjang fokus 10 cm. Berapakah


2015 A
jarak imej? B
BAB

An object is placed 10 cm in front of a convex lens with a focal 4m 5m


length of 10 cm. What is the image distance? C
n

A 5 cm C 20 cm 5m

6 B 10 cm D ∞ cm
ita

4m D
5m
5m 5m
8. Rajah 6 menunjukkan duit syiling dalam segelas air 6m
rb

yang menjadi lebih dekat kepada pemerhati. Rajah 8 / Diagram 8


2014
Diagram 6 shows a coin in a glass of water that appears to be
nearer to the observer. Pada kedudukan manakah, A, B, C atau D, imej
ne

terbentuk?
At which position, A, B, C or D, is the image formed?
Pe

11. Rajah 9 menunjukkan satu sinar cahaya merambat


daripada kaca ke dalam air.
2013
5 cm Diagram 4 shows a light ray propagating from glass into water.
2 cm
Normal
Normal
Rajah 6 / Diagram 6
Kaca
Berapakah indeks biasan air? Glass
What is the refractive index of the water?
A 1.40 C 1.67 Air
Water
B 1.50 D 2.50
Rajah 9 / Diagram 9

214
  Fizik  Tingkatan 4  Bab 6 Cahaya dan Optik 

Apakah yang berlaku pada sinar cahaya di dalam air 13. Rajah 11 menunjukkan kedudukan suatu imej yang
jika indeks biasan kaca lebih besar daripada indeks terbentuk oleh kanta cembung.
biasan air? Diagram 11 shows the position of an image formed by a
What happens to the light ray in the water if the refractive convex lens.
index of glass is bigger than that of water?
A Dibiaskan ke arah normal
Refracts towards normal
B
B Dibiaskan menjauhi normal
Refracts away from normal
C Mengalami pantulan Image Objek
Imej Object C
Experiences reflection

.
D Bergerak lurus tanpa pembiasan

hd
Travel straight without reflection A D

12. Rajah 10 menunjukkan sebuah binaan kamera dan Rajah 11 / Diagram 11

.B
hubungan antara jarak objek, u dan jarak imej, v Pada kedudukan manakah antara A, B, C dan D,
2014
sebuah kanta cembung. pemerhati tidak dapat melihat imej tersebut?
Diagram 10 shows a built-up of a camera and the relationship Which position of A, B, C or B, the observer cannot see the

dn
between object distance, u and image distance, v of a convex image?
lens.
u v 14. Sebutir berlian berkilauan apabila dihentam oleh

iS
Filem
Film
sinar tuju.
Fenomenon ini disebabkan oleh
A diamond glitters when struck by incident rays.
Objek This phenomenon is caused by
Object
ng
A pantulan cahaya
Rajah 10 / Diagram 10
reflection of light
B pembiasan cahaya
la
Antara yang berikut, perubahan yang manakah perlu refraction of light
dilakukan untuk imej menjadi lebih kecil? C interferens cahaya
Pe

Which of the following change needed to be done for image to interference of light
become smaller?
D pantulan dalam penuh cahaya

BAB
A Tukarkan kanta cembung dengan kanta cekung total internal reflection of light
Change the convex lens to conCAVE lens
n

B Besarkan saiz objek


Increase the size of object
6
ita

C Tambahkan jarak objek, u


Increase the object distance, u
rb

D Kurangkan jarak objek, u


Decrease the object distance, u
ne
Pe

215
  Fizik  Tingkatan 4  Bab 6 Cahaya dan Optik

Soalan Struktur

Bahagian A

1. Rajah 1.1 menunjukkan suatu objek, O diletakkan di hadapan sebuah cermin cekung.
Diagram 1.1 shows an object, O placed in front of a concave mirror.
F ialah fokus utama cermin itu dan C ialah pusat lengkungan. O
F is the principal focus of the mirror and C is the centre of curvature.
(a) Apakah maksud fokus utama?
What is the meaning of principal focus?
C F P

.
Titik di mana semua sinar cahaya selari menumpu selepas pantulan

hd
daripada cermin. / A point where all parallel light rays converge after
reflection from the mirror.

.B
Rajah 1.1 / Diagram 1.1
[1 markah / mark]
(b) Pada Rajah 1.1, lukis gambar rajah sinar untuk menunjukkan bagaimana imej objek itu terbentuk.

dn
On Diagram 1.1, draw a ray diagram to show how the image of the object is formed.
[3 markah / marks]
(c) Rajah 1.2 menunjukkan keratan rentas pemantul cekung yang digunakan

iS
dalam lampu kereta.
Diagram 1.2 shows the cross section of a concave reflector used in a car headlamp.
A
F ialah fokus utama cermin itu dan C ialah pusat lengkungan. Sinar R dari
ng R
mentol filamen ditujukan ke titik A pada pemantul cekung.
F is the principal focus of the mirror and C is the centre of curvature. Ray R from the
F C
filament bulb is incident at point A on the concave reflector.
la
(i) Pada Rajah 1.2, lengkapkan lintasan cahaya sinar R.
On Diagram 1.2, complete the path of ray R.
Pe

[2 markah / marks]
(ii) Nyatakan fenomena cahaya yang terlibat di 1(c)(i). Rajah 1.2 / Diagram 1.2
BAB

State the light phenomenon involved in 1(c)(i).


n

Pantulan / Reflection
6
ita

[1 markah / mark]
(d) Lampu kereta dalam Rajah 1.2 tidak mengeluarkan sinaran selari yang terang.
The headlamp in Diagram 1.2 does not produce bright parallel rays.
rb

Cadangkan satu pengubahsuaian yang boleh dilakukan pada kedudukan mentol untuk menghasilkan sinaran
selari yang terang.
ne

Suggest one modification on the position of the bulb to produce bright parallel rays.
Letakkan mentol itu pada fokus utama, F.
Pe

Place the bulb on the principal focus, F.


[1 markah / mark]
(e) Cadangkan satu kaedah lain untuk lampu kereta yang ditunjukkan dalam Rajah 1.2 menghasilkan sinaran yang
lebih terang.
Suggest one other method for the headlamp shown in Diagram 1.2 to produce brighter rays.
Guna mentol dengan kadar kuasa yang lebih tinggi.
Use bulb with higher power rating.
[1 markah / mark]

216
  Fizik  Tingkatan 4  Bab 6 Cahaya dan Optik 

2. Rajah 2.1 menunjukkan struktur prisma yang akan digunakan untuk membina periskop yang ditunjukkan dalam Rajah
2016
2.2. Sudut genting prisma ialah 42°.
Diagram 2.1 shows the structure of a prism to be used for building a periscope shown in Diagram 2.2. The critical angle of the prism is 42°.

40°

.
hd
Rajah 2.1 / Diagram 2.1 Rajah 2.2 / Diagram 2.2

(a) Apakah yang dimaksudkan dengan sudut genting?

.B
What is the meaning of critical angle?
Sudut genting ditakrifkan sebagai sudut tuju, i dalam medium yang lebih tumpat apabila sudut biasan, r dalam

medium kurang tumpat adalah 90°.

dn
Critical angle is defined as the angle of incidence, i in the denser medium when the angle of refraction, r in the less dense medium

iS
is 90°.
[2 markah / marks]
(b) (i) Pada Rajah 2.3, lengkapkan sinar cahaya selepas memasuki prisma.

ng
On diagram 2.3, complete the ray of light after entering the prism.
la
40°
Pe

BAB
Rajah 2.3 / Diagram 2.3
[2 markah / marks]
n

(ii) Berdasarkan jawapan pada 2(b)(i), apakah yang berlaku kepada sinar cahaya selepas memasuki prisma itu?
6
ita

Beri satu sebab bagi jawapan anda.


Based on answer in 2(b)(i), what happens to the ray of light after entering the prism? Give one reason for your answer.
Apabila sinar cahaya keluar dari prisma, sinar cahaya terbias menjauhi normal kerana sudut tuju kecil dari
rb

sudut genting.
ne

When light rays coming out of prism, the light ray is refracted away from the normal because of the angle of incidence

smaller than the critical angle.


Pe

[2 markah / marks]
(c) Berdasarkan pemerhatian dalam 2(b)(ii), pelajar itu mendapati beberapa pengubahsuaian perlu dibuat supaya
prisma boleh digunakan dalam periskop. Pengubahsuaian adalah berdasarkan aspek berikut:
Based on the observation in 2(b)(ii), the student found that a few modifications should be made so that the prism can be used in the
periscope. The modifications are based on the following aspect:
(i) Sudut prisma yang digunakan
The angle of prism used
Sudut prisma digunakan hendaklah 45°. / The angle of the prism used should be 45°.

217
  Fizik  Tingkatan 4  Bab 6 Cahaya dan Optik

(ii) Sebab
Reason
Supaya sudut tuju akan lebih besar daripada sudut genting dan sinar cahaya akan mengalami pantulan

dalam penuh. / So that angle of incidence will be bigger than the critical angle and the light ray will experience total

internal reflection.
[2 markah / marks]
(d) Berdasarkan prisma pada Rajah 2.1, hitung indeks biasan prisma plastik yang digunakan.
Based on the prism in Diagram 2.1, calculate the refractive index of the plastic prism used.

.
hd
1 1
n= = = 1.49
sin c sin 42°

[2 markah / marks]

.B
3. Rajah 3.1 dan Rajah 3.2 menunjukkan kedudukan imej yang dilihat oleh seorang pemerhati apabila dua objek yang
serupa diletakkan di bawah dua bikar yang serupa dengan ketinggian air yang berbeza.

dn
2013
Diagram 3.1 and Diagram 3.2 show the positions of the image seen by an observer when two identical objects are placed in two similar
beakers with different heights of water.

iS
Pemerhati Pemerhati
Observer Observer

Air
ng
Water
Dalam nyata Dalam nyata Air
Dalam ketara Water
la
Real depth Apparent depth Real depth
Dalam ketara
Apparent depth

Pe

Rajah 3.1 / Diagram 3.1 Rajah 3.2 / Diagram 3.2

(a) Nyatakan hubungan antara indeks biasan air dengan dalam nyata dan dalam ketara.
BAB

State the relationship between refractive index of water and real depth and apparent depth.
n

Indeks biasan = dalam nyata


6 dalam ketara
ita

real depth
Refractive index =
apparent depth
rb

[1 markah / mark]
(b) Perhatikan Rajah 3.1 dan Rajah 3.2,
ne

Observe Diagram 3.1 and Diagram 3.2,


(i) bandingkan dalam nyata dengan dalam ketara objek-objek yang dilihat oleh pemerhati,
compare the real depth and the apparent depth of the objects seen by the observer,
Pe

Apabila dalam nyata berkurang, dalam ketara berkurang / As the real depth decreases, the apparent depth decreases
[1 markah / mark]
(ii) namakan fenomena cahaya yang terlibat. / name the light phenomenon involved.
Pembiasan / Refraction
[1 markah / mark]

218
  Fizik  Tingkatan 4  Bab 6 Cahaya dan Optik 

(c) Rajah 3.3 menunjukkan satu gambar rajah sinar yang tidak lengkap.
Diagram 3.3 shows an incomplete ray diagram.
Normal
Normal

Imej
Image

.
Objek

hd
Object
Rajah 3.3 / Diagram 3.3

Lengkapkan gambar rajah sinar itu untuk menunjukkan bagaimana imej terbentuk.

.B
Complete the ray diagram to show how the image is formed. [3 markah / marks]

4. Rajah 4.1 dan Rajah 4.2 menunjukkan gambar rajah sinar bagi satu objek pada dua jarak yang berbeza di hadapan satu

dn
kanta cembung.
2015
Diagram 4.1 and Diagram 4.2 show the ray diagrams of an object at two different distances in front of a convex lens.

iS
Objek Objek
Object F Object F
2F F O 2F 2F F O 2F Imej
Imej
ng Image
Image
    
la
Rajah 4.1 / Diagram 4.1                     Rajah 4.2 / Diagram 4.2

(a) Nyatakan dua ciri imej yang sama untuk imej yang terbentuk dalam Rajah 4.1 dan Rajah 4.2?
Pe

Sate two common characteristics of the image formed in Diagram 4.1 and Diagram 4.2?
Songsang dan nyata / Inverted and real

BAB
[2 markah / mark]
n

(b) Perhatikan Rajah 4.1 dan Rajah 4.2.


Observed Diagram 4.1 and Diagram 4.2.
6
ita

(i) Bandingkan jarak objek.


Compare the object distance.
rb

Jarak objek dalam Rajah 4.1 lebih panjang daripada Rajah 4.2.
Object distance in Diagram 4.1 is longer than Diagram 4.2.
ne

[1 markah / mark]
(ii) Bandingkan jarak imej.
Pe

Compare the image distance.


Jarak imej dalam Rajah 4.1 lebih pendek daripada Rajah 4.2.
Image distance in Diagram 4.1 is shorter than Diagram 4.2.
[1 markah / mark]
(iii) Bandingkan saiz imej.
Compare the size of the image.
Saiz imej dalam Rajah 4.1 lebih kecil daripada Rajah 4.2.
Size of the image In Diagram 4.1 is smaller than Diagram 4.2.
[1 markah / mark]

219
  Fizik  Tingkatan 4  Bab 6 Cahaya dan Optik

(c) Berdasarkan jawapan dalam 4(b),


Based on the answer in 4(b),
(i) nyatakan hubungan antara jarak objek dan jarak imej.
state the relationship between object distance and the image distance.
Lebih besar jarak objek, lebih kecil jarak imej./ The bigger the object distance, the smaller the image distance.
[1 markah / mark]
(ii) nyatakan hubungan antara jarak objek dan saiz imej.
state the relationship between object distance and the size of the image.
Lebih besar jarak objek, lebih kecil saiz imej./ The bigger the object distance, the smaller the size of image.

.
[1 markah / mark]

hd
(d) Rajah 5.3 menunjukkan satu kanta pembesar dengan cahaya matahari menembusinya. Sekeping kertas
diletakkan di bawah kanta pembesar.

.B
Diagram 5.3 shows a magnifying glass with sunlight passing through it. A piece of paper is put under the magnifying glass.

Matahari
Sinar cahaya Sun
Sun light

dn
iS
Kanta pembesar
Magnifying glass

Rajah 4.3 / Diagram 4.3


ng
(i) Di manakah kertas tersebut sepatutnya diletakkan untuk memperoleh haba yang paling kuat?
Where the paper should be placed to obtain the strongest heat from the sun?
Pada titik fokus kanta cembung. / At the focus point of convex lens.
la
[1 markah / mark]
Pe

(ii) Beri satu sebab untuk jawapan anda dalam 4(d)(i).


Give one reason for your answer in 4(d)(i).
BAB

Semua cahaya selari dari cahaya matahari akan difokuskan pada satu tiitk iaitu titik fokus kanta cembung.
n

All the parallel light from the sun will be focus on a single point that is the focal point of the convex lens.
6
ita

[1 markah / mark]

Bahagian B
rb

5. Rajah 5.1 dan Rajah 5.2 menunjukkan lintasan cahaya memasuki satu blok kaca semibulatan. Kedua-dua blok kaca
menyebabkan lintasan cahaya dibengkokkan.
ne

Diagram 5.1 and Diagram 5.2 shows a ray of light ray enter a semicircular glass block. Both the block causes the light ray to bend.
Pe

A A B B

Rajah 5.1 / Diagram 5.1 Rajah 5.2 / Diagram 5.2



(a) (i) Namakan fenomena bagi Rajah 5.2.
Name the phenomena in Diagram 5.2.
[1 markah / mark]

220
  Fizik  Tingkatan 4  Bab 6 Cahaya dan Optik 

(ii) Berdasarkan Rajah 5.1 dan Rajah 5.2, bandingkan sudut sinar cahaya yang merambat melalui blok kaca
semibulatan, ketumpatan dan juga indeks biasan bagi kedua–dua blok kaca semibulatan. Hubung kaitkan
antara ketumpatan blok kaca semibulatan dengan indeks biasan.
With reference to diagram 5.1 and 5.2 compare the angles of the light rays pass through semicircular glass block, density
and refractive index in both semicircular glass blocks. Relate the density of semicircular glass block with refractive index.
[4 markah / marks]
(b) Terangkan mengapa ketika memandu pada hari yang panas, anda akan melihat satu lopak air atas jalan tetapi
apabila mendekainya, lopak air tersebut tidak wujud.
Explain why while driving a car on a hot day, you may see a pool of water on the road but as you approach that particular spot, you
find that the pool does not really exist. [5 markah / marks]

.
(c) Gentian optik menggunakan prinsip pantulan dalam penuh dalam menghantar isyarat. Gentian optik digunakan

hd
sebagai endoskop yang merupakan suatu alat perubatan yang boleh digunakan untuk melihat bahagian dalam
manusia. Anda dikehendaki mereka satu gentian optik yang sesuai digunakan sebagai endoskop. Nyatakan dan
terangkan rekaan anda berdasarkan aspek-aspek berikut: 4

.B
Optical fibre uses a total internal reflection principle in transmit signals. Optical fibre is used as an endoscope, medical device that
can be used to look inside of a human. You are required to design an appropriate optical fibre to be used as an endoscope. State and
explain your design based on the following aspects:

dn
• Indek biasan bagi lapisan dalam dan lapisan luar.
Refractive index for inner layer and outer layer.
• Kebolehlenturan bahan.

iS
Flexibility of the material.
• Bahan yang diperlukan untuk membina gentian optik.
The types of materials used in the construction of the optical fibres.
ng
• Diameter gentian optik dan sebab ia dicadangkan.
The diameter of the optical fibres and why the diameter must be as suggested.
[8 markah / marks]
la
(d) Dalam bidang telekomunikasi, gentian optik kini secara beransur-ansur telah menggantikan penggunaan kabel
kuprum. Nyatakan dua kelebihan kabel gentian optik berbanding kabel kuprum.
Pe

In telecommunication, gentian optic now slowly replaced the copper cable. State two advantages of fibre optic cables compared
to copper cables. [2 markah / marks]

BAB
n

Bahagian C
6
ita

6. (a) Hyperopia juga dikenali sebagai rabun dekat. Hyperopia adalah ketidakupayaan mata untuk memfokuskan objek
yang dekat. Imej yang terbentuk oleh kanta mata jatuh di belakang retina. Rajah 6.2 menunjukkan kanta yang
rb

digunakan untuk membetulkan penglihatan seseorang dengan hyperopia.


Farsightedness also known as hyperopia. Hyperopia is an inability of the eye to focus near objects. Image formed by the eye lens
falls behind the retina. Diagram 6.2 shows a lens is used to correct the sight of a person with hyperopia.
ne

Kanta mata
Kanta mata
Eye lens Eye lens Retina Retina
Retina Retina
Pe

Otot silia Otot silia


Ciliary muscle
Ciliary muscle

Rajah 6.1 / Diagram 6.1 Rajah 6.2 / Diagram 6.2

(i) Namakan kanta yang digunakan dalam Rajah 6.2.


Name the type of lens used in Diagram 6.2. [1 markah / marks]
(ii) Lukis dan lengkapkan lintasan sinar cahaya pada Rajah 6.2.
Draw and complete the path of the light ray at Diagram 6.2. [2 markah / marks]

221
  Fizik  Tingkatan 4  Bab 6 Cahaya dan Optik

(iii) Jika keadaan seseorang yang mengalami hyperopia semakin merosot, dan memerlukan kanta yang lebih
baik, apakah pelarasan yang perlu dibuat supaya kanta pada Rajah 6.2 sesuai digunakan untuk orang yang
di atas? Berikan satu sebab bagi jawapan anda.
If the person suffer from hyperopia condition is deteriorating, and needed a better lens, what adjustment needed to make to
the lens on Diagram 6.2 to become suitable for the person above? Give a reason for your answer.
[2 markah / marks]
(b) Rajah 6.3 menunjukkan sebuah teleskop yang digunakan untuk melihat objek yang jauh seperti bintang.
Teleskop dibina dengan menggunakan dua kanta cembung. Jadual 6.1 menunjukkan lima jenis kanta dengan
kuasa berbeza.
Diagram 6.3 shows a telescope used to see further objects like stars. It is made up of two convex lenses. Table 6.1 shows five types of

.
lenses with different powers.

hd
Jenis kanta Kuasa
Type of lens Power

.B
A + 20 D
B + 10 D
C +2D

dn
D + 25 D
E +5D

iS
Rajah 6.3 / Diagram 6.3 Jadual 6.1 / Table 6.1

(i) Pilih pasangan kanta yang paling sesuai digunakan untuk membina sebuah teleskop yang berkuasa tinggi.
Terangkan jawapan anda. 5 ng
Select the most suitable pair of lenses that can be used to build a powerful telescope. Explain your answer.
[4 markah / marks]
(ii) Dengan bantuan Rajah 6.4, cadangkan dan terangkan bagaimana membina teleskop astronomi yang
la
menghasilkan imej yang lebih besar dan jelas.
With the aid of a Diagram 6.4, suggest and explain how to build astronomical telescope which produces a bigger and clear
Pe

image.
Kanta mata
BAB

Eyepiece
Kanta objek
Objective lens L = f + f
o e
n

FoFe Fe
O I1

6
ita


rb

Rajah 6.4 / Diagram 6.4


[6 markah / marks]
ne

(c) Satu objek diletakkan 20 cm di hadapan sebuah kanta cembung dengan kuasa +10 D.
An object is placed 20 cm in front of a convex lens with power +10 D
Hitungkan
Pe

Calculate
(i) panjang fokus kanta
focal length of the lens
(ii) jarak imej
the image distance
(iii) pembesaran imej.
the magnification of the image. [5 markah / marks]

222
PRAKTIS SPM 6
JAWAPAN – Bahan mestilah fleksibel supaya ia mudah bengkok
dan tidak mudah patah.
Soalan Struktur The material must be flexible so that it is easy to bend
Bahagian B without breaking.
– Jenis bahan yang digunakan mestilah lut sinar
5. (a) (i) Pantulan dalam penuh
kepada cahaya supaya cahaya boleh melaluinya.
Total Internal Reflection The type of material used must be transparent to light so

.
(ii) –
Dalam Rajah 5.1, sudut biasan, r lebih besar light can pass through it.

hd
dari sudut tuju, i. Dalam Rajah 5.2, sinar – Diameter mesti sangat kecil supaya boleh
cahaya dipantulkan balik dalam kaca dan digunakan di kawasan yang kecil.
mematuhi hukum pantulan. The diameter must be very small so that it can be uses in
In Diagram 5.1, the angle of refraction, r is greater small area.

.B
than the angle of incidence. In Diagram 5.2, all light
rays are reflected back inside the glass and obey the
(d) – Fiber optik lebih nipis dan ringan berbanding kabel
law of reflection.
kuprum.

Blok kaca semibulatan A mempunyai The fibre optic are much thinner and lighter.

dn
ketumpatan rendah berbanding blok kaca – Bilangan isyarat boleh dihantar melaluinya pada
semibulatan B. satu masa adalah tinggi.
Density of the semicircular glass A is lower than the A large number of signals can be sent through them at one
time.
semicircular glass B.

iS

Indeks biasan blok kaca semibulatan A lebih – Mereka menghantar isyarat dengan kehilangan
rendah berbanding blok kaca semibulatan B. isyarat yang kecil pada jarak yang jauh.
Refractive index of the semicircular glass A is lower They transmit signals with very little loss over great
distances.


than the semicircular glass B.
Semakin tinggi ketumpatan bahan semakin
ng

tinggi nilai indeks biasan. Bahagian C
The higher the density of substance, the higher the 6. (a) (i) Kanta cembung
la
refractive index. Convex lens
(b) – Pada cuaca panas, lapisan udara di atas (ii)
permukaan jalan adalah panas.
Pe

During hot day, the layer of air nearer the road is warmer.
– Ketumpatan udara semakin berkurang apabila
mendekati permukaan jalan.
The density of the air decrease nearer to the road surface. (iii)
Guna kanta yang lebih kuat atau kanta yang lebih
– Cahaya bergerak dari kawasan yang tebal. Supaya boleh membiaskan cahaya pada
n

berketumpatan tinggi ke kawasan berketumpatan sudut yang lebih besar.


Use high powered lens or thicker lens. So that it can
ita

rendah.
refract the light in a bigger angle.
Light travel from denser to less dense area.
– Cahaya terbias jauh dari dari normal. (b) (i) –
Teleskop terdiri daripada satu kanta yang
Light refracted away from the normal. berkuasa tinggi dan satu kanta yang bekuasa
rb

– Apabila sampai pada lapisan udara tertentu, sudut rendah.


tuju melebihi sudut gentingnya dan pantulan penuh The telescope consists of high powered lens and low
berlaku. powered lens.
ne

In certain layer of air, the incident angle exceeds the –


Kanta objektif harus mempunyai kuasa yang
critical angle and total internal reflection takes place. rendah manakala kanta mata mempunyai
– Pemerhati melihat imej langit dan awan di kuasa yang tinggi.
The objective lens has a lower power whereas
Pe

permukaan jalan raya seperti lopak air.


The observer sees the image of the sky and the clouds on eyepiece has higher power lens.
the surface of the road as a pool of water. – C dipilih sebagai kanta objektif kerana C
mempunyai kuasa paling rendah.
(c) – Fiber optik haruslah terdiri daripada teras kaca
C is chosen as the objective lens as C has the lowest
yang mana lapisan dalam mempunyai indeks power.
pembiasan yang tinggi dan dilingkari oleh lapisan – D dipilih sebagai kanta mata kerana kuasa D
luar yang mempunyai indeks pembiasan yang adalah yang tertinggi.
rendah. Hal ini membolehkan pantulan dalam D is chosen as eyepiece as the power of D is the
penuh berlaku. highest.
Optic fibre should consist of an inner core of higher
refractive index and surrounded by an outer cladding
of lower refractive index. It is to allow the total internal
reflection to accur.
  Fizik  Tingkatan 4 Jawapan

(ii) –
Sinar cahaya selari daripada objek jauh 1
(c) (i) D =
ditumpukan pada Fo untuk membentuk imej f
pertama, I1. 1
f =
Parallel light rays from a distant object converge at Fo 10
to form the first image,I1. f = 0.1 m
– Imej ini adalah nyata, songsang dan dikecilkan. f = 10 cm
This image is real, inverted and magnified. 1 1 1
– Kedudukan imej pertama, I1 di F0 menjadi (ii) = +
f u v
objek untuk kanta mata.
Position of the first image, I1 at F0 becomes the object 1 1 1
= +
for the eyepiece. 10 20 v

Kanta mata diselaraskan supaya Fo dan Fm 1 1 1 1
= – =
berada pada kedudukan yang sama v 10 20 20

.
The eyepiece is adjusted so that Fo and Fe are at the v = 20 cm

hd
same position.
v 20

Oleh kerana kedudukan I1 adalah pada Fm, (iii) M =
=
u 20
imej akhir, I2 dihasilkan pada infiniti.
As the position of I1 is at Fe, the final image, I2 is
M = 1

.B
produced at infinity.

Imej ini adalah maya, songsang dan
dibesarkan
This image is virtual, inverted and magnified

dn
iS
ng
la
Pe
n
ita
rb
ne
Pe

Potrebbero piacerti anche